Superpage
Breast

Authors: Engy Abdellatif, M.B.B.Ch., M.D., Ph.D., Elizabeth Abels, M.D., Indu Agarwal, M.D., Apeksha N. Agarwal, M.B.B.S., M.D., Di (Andy) Ai, M.D., Ph.D., Areej M. Al Nemer, M.D., Melissa Alexander, M.D., Ph.D., Afaf M. Alsharhan, M.D., Saba Anjum, M.B.B.S., Anand C. Annan, M.D., Ph.D., Maria A. Arafah, M.D., Jaya Ruth Asirvatham, M.D., Doaa Atwi, M.D., S. Emily Bachert, M.D., Gabrielle M. Baker, M.D., Harsh Batra, M.B.B.S., D.C.P., D.N.B., Samuel Bidot, M.D., Anna Biernacka, M.D., Ph.D., Luis Blanco, Jr., M.D., Susanne M. Crespo-Ramos, M.D., Rebecca Czaja, M.P.H., M.D., Carlos C. Diez Freire, M.D., Iskender Sinan Genco, M.D., Joseph Geradts, M.D., Paula S. Ginter, M.D., Evan R.J. Goyette, M.D., Anthony J. Guidi, M.D., Sabina Hajiyeva, M.D., Katie C. Hall, M.D., Lewis A. Hassell, M.D., Jing He, M.D., Yu Jing Jan Heng, Ph.D., Ali Ismail, M.B.B.S., Shabnam Jaffer, M.D., Pragya Virendrakumar Jain, M.D., Zena Jameel, M.D., Tiffany Javadi, M.D., Victoria M. Jones, M.D., Julie M. Jorns, M.D., Dia Kamel, M.D., Ph.D., Cansu Karakas, M.D., Naila Kayani, M.D., Fatima Khalid, M.B.B.S., Melissa Krystel-Whittemore, M.D., Grace Kwon, M.D., Ph.D., Sofia Lérias, M.D., Carissa LaBoy, M.D., Belinda Lategan, M.D., Melinda Lerwill, M.D., Xiaoxian (Bill) Li, M.D., Ph.D., Joshua J.X. Li, M.B.Ch.B., Daniel D. Mais, M.D., Kiran Manjee, M.D., Jonathan Marotti, M.D., Shahla Masood, M.D., Anas Mohamed, M.D., Gustavo Moreno, M.D., Samaneh A. Motanagh, M.D., Miralem Mrkonjic, M.D., Ph.D., Kristen E. Muller, D.O., Gahie Nam, M.D., Hafsa Nebbache, M.D., Nat Pernick, M.D., Mirna B. Podoll, M.D., Emil Racila, M.D., Emad Rakha, M.D., Chanchal Rana, M.D., Emily S. Reisenbichler, M.D., Marilin Rosa, M.D., Monika Roychowdhury, M.D., Thomas Sabljic, M.D., M.Sc., Ph.D., Sara Salehiazar, M.D., Mary Ann G. Sanders, M.D., Ph.D., Ruhani Sardana, M.B.B.S., Sunitha Shankaralingappa, M.B.B.S., M.D., D.M., Arundhathi Shankaralingappa, M.B.B.S., M.D., Kamaljeet Singh, M.D., Kalliopi P. Siziopikou, M.D., Ph.D., Eric Statz, M.D., Reena Tomar, M.B.B.S., M.D., Gary Tozbikian, M.D., Gary M. Tse, M.B.B.S., Gulisa Turashvili, M.D., Ph.D., Agnes Ikpoto Udoh, M.D., M.B.A., Jasmine Vickery, M.D., Jessica Wallace, P.A., A.S.C.P., Hind Warzecha, M.D., Shannon Mingo Welter, M.D., Hannah Y. Wen, M.D., Ph.D., Jessie M. Wu, M.B.Ch.B., Y. Albert Yeh, M.D., Ph.D., Hillary Zalaznick, M.D., Huina Zhang, M.D., Ph.D., Elaine Zhong, M.D., Debra L. Zynger, M.D.
Editorial Board Members: Julie M. Jorns, M.D., Ricardo R. Lastra, M.D., Kristen E. Muller, D.O., Emily S. Reisenbichler, M.D., Gary Tozbikian, M.D.
Deputy Editors-in-Chief: Genevieve M. Crane, M.D., Ph.D., Gary Tozbikian, M.D., Debra L. Zynger, M.D.
Editor-in-Chief: Debra L. Zynger, M.D.

Copyright: 2001-2024, PathologyOutlines.com, Inc.

Breast related: Jobs, Fellowships, Conferences, Cases, CME, Board Review, What's New

Editorial Board oversight: Julie M. Jorns, M.D. (last reviewed December 2022), Kristen E. Muller, D.O. (last reviewed November 2021), Gary Tozbikian, M.D. (last reviewed January 2024)
Page views in 2024 to date: 69

Abemaciclib (pending)
[Pending]

Acinic cell carcinoma
Definition / general
  • Acinic cell carcinoma is a malignant epithelial neoplasm characterized by clear and granular cells that may contain zymogen granules in the cytoplasm and shows microglandular and solid growth patterns
Essential features
  • Granular eosinophilic and basophilic cytoplasm with intracytoplasmic granules that are positive for periodic acid-Schiff with diastase (PASD)
  • Infiltrative microglandular and solid growth patterns
  • Positive immunohistochemical staining for EMA and markers of serous acinar differentiation
Terminology
  • Acinar cell carcinoma
ICD coding
  • ICD-O: 8550/3 - acinar cell carcinoma
  • ICD-11: 2C60 & XH3PG9 - carcinoma of breast, specialized type & acinar cell carcinoma
Epidemiology
Sites
Pathophysiology
Etiology
  • Unknown
Clinical features
  • Presentation is similar to invasive breast carcinoma of no special type, palpable mass (Pathology 2017;49:215)
Diagnosis
Radiology description
  • Poorly defined lump / mass with microcalcifications, although may vary in clinical and radiological appearance (Breast 2022;66:208)
Radiology images

Contributed by Miralem Mrkonjic, M.D., Ph.D.
Left breast mass with nonmass enhancement

Left breast mass with nonmass enhancement



Images hosted on other servers:
Doppler ultrasound, molybdenum target

Doppler ultrasound, molybdenum target

MRI and PET / CT

MRI and PET / CT

Prognostic factors
Case reports
Treatment
  • Breast conserving surgery or mastectomy, generally with sentinel lymph node excision or axillary lymph node dissection; with or without neoadjuvant or adjuvant chemotherapy and radiotherapy, hormonal therapy (if ER positive) (Breast 2022;66:208)
Gross description
Gross images

Images hosted on other servers:
Ill defined, slightly lobulated mass

Ill defined, slightly lobulated mass

Gray-brown rubbery tumor

Gray-brown rubbery tumor

Microscopic (histologic) description
  • Cells with serous differentiation containing zymogen granules in the cytoplasm that stain with PASD
  • Predominantly solid or microglandular growth but may be cystic, trabecular or show other growth patterns (Breast 2022;66:208)
Microscopic (histologic) images

Contributed by Miralem Mrkonjic, M.D., Ph.D.
Infiltrative growth pattern

Infiltrative growth pattern

Microglandular architecture

Microglandular architecture

Microglandular architecture and eosinophilic cytoplasm

Microglandular architecture and eosinophilic cytoplasm

Areas resembling invasive ductal carcinoma of no special type

Areas resembling
invasive ductal
carcinoma of no
special type

Prominent cytoplasmic eosinophilic granules

Prominent cytoplasmic eosinophilic granules


Background of extensive MGA and AMGA

Background of extensive MGA and AMGA

MGA in the adipose tissue

MGA in the adipose tissue

Nuclear atypia in MGA

Nuclear atypia in MGA

Absence of staining around invasive carcinoma

Absence of staining around invasive carcinoma

Invasive carcinoma is negative for ER

Invasive carcinoma is negative for ER


Invasive carcinoma is negative for HER2

Invasive carcinoma is negative for HER2

Invasive carcinoma is positive for S100

Invasive carcinoma is positive for S100

Invasive carcinoma is positive for lysozyme

Invasive carcinoma is positive for lysozyme

PASD cytoplasmic staining

PASD cytoplasmic staining

Cytology description
  • Fine needle aspiration: hypercellular, atypical cells in sheets, round to oval nuclei, single nucleoli, mild pleomorphism in size, moderately increased N:C ratio, intercellular cystic spaces, acinic pattern, cytoplasmic vacuoles and granules visible on Papanicolaou and Giemsa stains (Cytopathology 2013;24:403)
Positive stains
Negative stains
  • SMA (100% negative, 14/14 cases)
  • ER (90% negative, 59/66 cases)
  • PR (89% negative, 55/62 cases)
  • HER2 (100% negative, 54/54 cases)
  • CK20 (100% negative, 2/2 cases)
  • Reference: Breast 2022;66:208
Electron microscopy description
  • Cells with vacuolated cytoplasm containing many electron dense granules of varying size and a prominent rough endoplasmic reticulum (RER) (J Breast Cancer 2011;14:160)
Electron microscopy images

Images hosted on other servers:
Cytoplasmic electron dense granules

Cytoplasmic electron dense granules

Dilation of RER

Dilation of RER

Acinar cell granules, prominent RER

Acinar cell granules, prominent RER

Molecular / cytogenetics description
  • No pathognomonic genomic alterations have been described
  • Has a similar molecular profile to triple negative breast carcinoma and does not share the genomic rearrangement seen in salivary gland acinic cell carcinoma
  • TP53 mutations are the most consistent molecular event (Breast 2022;66:208)
  • PIK3CA and other mutations have been described in a subset of cases (Mod Pathol 2017;30:69)
  • BRCA1 gene alterations have also been described (BMC Cancer 2013;13:46, Histol Histopathol 2023;38:91)
Sample pathology report
  • Breast (right), lumpectomy:
    • Acinic cell carcinoma (see comment)
    • Comment: Sections show breast tissue involved by acinic cell carcinoma in solid nests and acini of polygonal cells with granular eosinophilic cytoplasm. These tumor cells display round, uniform nuclei with prominent nucleoli. Periodic acid-Schiff with diastase (PASD) staining demonstrates the presence of zymogen-like granules in the cytoplasm, confirming the acinar differentiation characteristic of this tumor.
Differential diagnosis
Board review style question #1

A premenopausal woman presents with a painless, palpable, slow growing mass in her left breast. Histological examination reveals cells with abundant eosinophilic and basophilic cytoplasm with large coarse PASD+ intracytoplasmic granules and centrally located nuclei with prominent nucleoli, showing an infiltrative microglandular growth pattern. What is the most likely diagnosis?

  1. Acinic cell carcinoma
  2. Angiolipoma
  3. Burkitt lymphoma
  4. Fibroadenoma
  5. Invasive lobular carcinoma
Board review style answer #1
A. Acinic cell carcinoma. These are the features of acinic cell carcinoma (see microscopic image). Answer D is incorrect because fibroadenoma is a fibroepithelial lesion, which is not infiltrative and the epithelial component does not have diffuse acinar morphology. Answer B is incorrect because angiolipoma is a mesenchymal neoplasm comprised of mature adipocytes and blood vessels. Answer E is incorrect because invasive lobular carcinoma lacks cytoplasmic granules and microglandular growth pattern. Answer C is incorrect because Burkitt lymphoma is a non-Hodgkin lymphoma growing in solid sheets with starry sky appearance.

Comment Here

Reference: Acinic cell carcinoma
Board review style question #2

Which immunostain expression would be the most helpful to support the suspected diagnosis?

  1. E-cadherin+
  2. HER2+ (3+)
  3. Lysozyme+
  4. Ki67 > 90%
  5. S100+
Board review style answer #2
C. Lysozyme+. Acinic cell carcinoma is positive for EMA and lysozyme. Answer E is incorrect because S100 is positive in acinic cell carcinoma and angiolipoma, so it would not help in differentiating between these entities. Answer D is incorrect because Burkitt lymphomas and not acinic cell carcinomas have a Ki67 index close to 100%. Answer A is incorrect because E-cadherin positivity is not specific to acinic cell carcinoma. Answer B is incorrect because acinic cell carcinomas are not HER2 amplified.

Comment Here

Reference: Acinic cell carcinoma
Board review style question #3
Which biomarker profile is typical of acinic cell carcinoma?

  1. ER+, PR+, HER2+
  2. ER+, PR+, HER2-
  3. ER+, PR-, HER2-
  4. ER-, PR-, HER2+
  5. ER-, PR-, HER2-
Board review style answer #3
E. ER-, PR-, HER2-. Acinic cell carcinomas are predominantly triple negative with rare cases showing weak ER positivity. Answer C is incorrect because only rare cases of acinic cell carcinoma have been reported to have weak ER positivity. Answers A, B and D are incorrect because acinic cell carcinoma does not share luminal, HER2 amplified or triple positive biomarker profile.

Comment Here

Reference: Acinic cell carcinoma

Acute mastitis / abscess
Definition / general
  • Infection or inflammation of breast tissue, usually bacterial, often seen in association with lactation
  • If untreated, may form abscess and fistulous tracts
Essential features
  • Infection or inflammation of breast tissue, usually bacterial, often seen in association with lactation
  • If untreated, may form abscess and fistulous tracts
  • Staphylococcus aureus most common organism
  • Microscopic examination reveals a mixed dense inflammatory infiltrate, predominantly neutrophils, in breast tissue
Terminology
  • Puerperal mastitis
ICD coding
  • ICD-10:
    • N61.0 - mastitis without abscess
    • N61.1 - abscess of the breast and nipple
    • O91.1 - abscess of the breast associated with pregnancy, the puerperium and lactation
    • O91.211 - nonpurulent mastitis associated with pregnancy, first trimester
    • O91.22 - nonpurulent mastitis associated with the puerperium
  • ICD-11:
    • GB21.Z - inflammatory disorders of breast, unspecified
    • GB21.Y - other specified inflammatory disorders of breast
    • JB45.1 - nonpurulent mastitis associated with childbirth
    • JB45.0 - abscess of breast associated with childbirth
Epidemiology
Sites
  • Subareolar (nonpuerperal)
Pathophysiology
Etiology
Clinical features
  • Erythema, swelling, firmness, breast pain, fever, malaise, decreased milk outflow
  • Abscess: well circumscribed fluctuant mass in addition to above features
  • African Americans may have higher risk of abscess formation (World J Surg 2009;33:2582)
  • Obesity (BMI > 30) and smoking risk factors for abscess (World J Surg 2009;33:2582)
  • Zuska disease (periductal mastitis): clinical scenario of recurring subareolar abscesses and sinus formation (Am J Surg 1951;81:312)
  • Fungal mastitis: rare
    • Actinomycosis may develop in nipple and form a sinus tract
    • Candida infection may interfere with breast feeding but Candida albicans is not present in milk ducts in women with clinical symptoms of ductal candidiasis (Breastfeed Med 2009;4:57)
Diagnosis
  • Based on history and clinical findings
  • Ultrasound preferred imagining modality (Radiol Bras 2020;53:405)
  • Biopsy may be warranted for suspected abscess, atypical presentation, recurrent infection or treatment failure; Gram stain and culture (aerobic and anaerobic) with sensitivities to guide antibiotic selection (rarely needed)
  • FNA can be used to drain (Am Fam Physician 2008;78:727)
Radiology description
  • Ultrasound (preferred modality): abscess shows hypoechoic lesion of purulent material, well circumscribed, macrolobulated, irregular or ill defined with septa and thick echogenic rim (Radiographics 2011;31:1683, Radiographics 2007;27 Suppl 1:S101)
  • Mammography of limited value due to nonspecific findings
Radiology images

Contributed by Kristen E. Muller, D.O.
Ultrasound hypoechoic process

Ultrasound hypoechoic process

Ultrasound irregular fluid collection

Ultrasound irregular fluid collection

Prognostic factors
  • Recurrent breast abscesses are more likely to be smokers and have mixed bacterial and anaerobic infections (World J Surg 2009;33:2582)
Case reports
Treatment
Clinical images

Contributed by Ayesha Farooq, M.B.B.S. and Julie Jorns, M.D. (Case #515)
Right breast prior to debridement Site of surgical excision and debridement

Nipple piercing related infection



Images hosted on other servers:

Mastitis and breast abscess

Lactational abscess

Gross description
  • Excisions rare; specimens usually from incision and drainage procedures and typically received in fragments
Microscopic (histologic) description
  • Mixed dense inflammatory infiltrate, predominantly neutrophils, in breast tissue
  • May obscure underlying normal breast tissue
  • Tissue necrosis may be present
  • Gram stain for microorganisms may reveal bacterial forms (gram positive cocci) associated with neutrophilic infiltrate
  • Granulation tissue and chronic inflammation with resolution
  • Lymphocytes, giant cells and granulomas are not typical (J Pathol Transl Med 2015;49:279)
Microscopic (histologic) images

Contributed by Kristen E. Muller, D.O.
Acute mastitis and abscess Acute mastitis and abscess

Acute mastitis and abscess

Lobular inflammation

Lobular inflammation

Dense inflammatory infiltrate

Dense inflammatory infiltrate

Dense neutrophilic infiltrate

Dense neutrophilic infiltrate



Contributed by Ayesha Farooq, M.B.B.S. and Julie Jorns, M.D. (Case #515)

Nipple piercing related infection

Virtual slides

Images hosted on other servers:

Breast abscess and necrosis

Cytology description
Positive stains
  • Gram stain may highlight Gram positive bacterial forms
Sample pathology report
  • Right breast, core needle biopsy:
    • Breast tissue with abundant acute inflammation and bacterial forms identified on Gram stain (see comment)
    • Comment: The histologic findings are compatible with acute mastitis and abscess formation in the appropriate clinical context. Clinical correlation and correlation with microbiology studies (if performed) is recommended.
Differential diagnosis
  • Squamous metaplasia of lactiferous ducts (SMOLD):
    • Subareolar location
    • Lactiferous ducts with squamous metaplasia
    • Multinucleated giant cells associated with keratin debris
    • Strong association with smoking
  • Fat necrosis:
    • Infarcted adipocytes, lipid laden macrophages, foreign body giant cell reaction
    • Initially acute inflammatory infiltrate may be present, chronic inflammation predominates in older lesions
    • History of surgery, prior biopsy, radiation, trauma
  • Idiopathic granulomatous mastitis:
    • Granulomatous inflammation (lymphocytes, plasma cells, histiocytes, multinucleated giant cells)
    • Lobulocentric pattern, may obliterate lobules
    • May have neutrophils forming microabscesses surrounding empty microcysts; features overlap with cystic neutrophilic granulomatous mastitis, which may have gram positive bacilli within microcysts (Corynebacterium)
  • Duct ectasia:
    • Foamy histiocytes within lumen and within wall and epithelium of ducts, periductal fibrosis
    • Inflammatory cells typically chronic; acute inflammatory infiltrate rarely present
Board review style question #1

What is the most appropriate diagnosis for this breast biopsy from a 32 year old postpartum woman?

  1. Acute mastitis
  2. Duct ectasia
  3. Fat necrosis
  4. Idiopathic granulomatous mastitis
  5. Invasive lobular carcinoma
Board review style answer #1
A. Acute mastitis

Comment Here

Reference: Acute mastitis / abscess
Board review style question #2

Which of the follow is true regarding acute mastitis?

  1. Abscess may form as a complication if left untreated
  2. Acute mastitis typically occurs in postmenopausal women
  3. Granulomas are common
  4. Most common organism isolated is Streptococcus species
Board review style answer #2
A. Abscess may form as a complication if left untreated

Comment Here

Reference: Acute mastitis / abscess
Board review style question #3
What is a common causative microorganism associated with nipple piercing related infection?

  1. Salmonella
  2. Shigella
  3. Staphylococcus
  4. Stenotrophomonas
Board review style answer #3
C. Staphylococcus

Comment Here

Reference: Acute mastitis / abscess

Adenoid cystic
Definition / general
  • Histologically similar to salivary gland counterpart
  • Triple negative and basal-like subtype of mammary carcinoma with excellent long term survival
Essential features
  • Rare subtype of breast cancer, histomorphologically identical to salivary gland counterpart
  • Comprised of epithelial and myoepithelial cells arranged in solid, cribriform or tubular arrangements and associated basement membrane material
  • Associated with MYB::NFIB fusion
ICD coding
  • ICD-10: C50.91 - malignant neoplasm of breast of unspecified site, female
Epidemiology
Diagnosis
  • Mammogram or other imaging detects a mass
  • Diagnosis made by tissue biopsy or resection, often utilizing immunohistochemistry
Radiology images

Contributed by Mark R. Wick, M.D.

Mammogram



Images hosted on other servers:

MRI

Prognostic factors
Case reports
Gross description
Microscopic (histologic) description
  • Dual population of cells (Arch Pathol Lab Med 1986;110:1045):
    • Luminal / ductal epithelial cells with variable glandular / squamous / sebaceous differentiation
    • Myoepithelial / basaloid cells
  • Cribriform, solid, tubular or trabecular architectural patterns
  • 2 types of lumens that are lined by 2 cell types
    • True glandular lumina lined by luminal / ductal epithelial cells (low molecular weight cytokeratin+, CK7+)
    • Pseudolumens containing eosinophilic cylinders comprised of basement membrane material (laminin and collagen IV positive) lined by myoepithelial / basal type cells (p63 / p40+, smooth muscle myosin+, calponin+, high molecular weight cytokeratin+, S100+)
  • Histologic variants
Microscopic (histologic) images

Contributed by Gary Tozbikian, M.D.

Extensive cribriform architecture, infiltrative growth pattern


Dual population of basaloid and epithelioid cells

Estrogen receptor

p40

CD117 / KIT

Cytology description
  • Clusters of epithelial cells oriented around solid spheres of basement membrane material
Cytology images

Contributed by Mark R. Wick, M.D.

FNAB



Images hosted on other servers:

Papanicolaou

Monomorphic tumor cells around mucoid material

Positive stains
Molecular / cytogenetics description
Sample pathology report
  • Left breast, mass 10:00, ultrasound guided core needle biopsy:
    • Adenoid cystic carcinoma, 1.2 cm in greatest length (see comment)
    • Estrogen receptor: negative (0%, no intensity)
    • Progesterone receptor: negative (0%, no intensity)
    • HER2: negative (score 0)
    • Comment: Additional immunohistochemical stains show that the tumor is positive for CD117, MYB and p40 (in basaloid cells), supporting the diagnosis.
Differential diagnosis
Board review style question #1

A 55 year old woman undergoes an excision for a 3 cm breast mass (histologic image of the tumor is shown above). Which statement is true regarding this tumor?

  1. CD117 / KIT expression is present in the luminal / ductal epithelial cell component
  2. Frequently shows regional lymph node metastases
  3. Recurrent t(12;15)(p13;q25) translocation resulting in ETV6::NTRK3 gene fusion is commonly seen
  4. Represents ~5% of all mammary cancers
  5. Triple negative for ER / PR / HER2 and has an aggressive clinical course
Board review style answer #1
A. CD117 / KIT expression is present in the luminal / ductal epithelial cell component. The excision shows an adenoid cystic carcinoma, which is a rare subtype of mammary carcinoma representing < 1% of breast cancers. Despite triple negative status, these tumors generally show excellent long term survival. Similar to the salivary gland counterparts, adenoid cystic carcinomas of the breast are characterized by t(6;9)(q22-23;p23-24) translocation resulting in a MYB::NFIB fusion gene. The t(12;15)(p13;q25) translocation resulting in ETV6::NTRK3 gene fusion (answer C) is seen in secretory carcinoma of the breast. Consistent CD117 / KIT protein expression is observed in most adenoid cystic carcinomas of the breast; however, underlying KIT gene alterations have not been identified (Histopathology 2013;62:543).

Comment Here

Reference: Adenoid cystic carcinoma
Board review style question #2
Which immunohistochemical stain finding is typically observed in adenoid cystic carcinoma of the breast?

  1. CD117 / KIT negative, estrogen receptor negative
  2. CD117 / KIT positive, estrogen receptor positive
  3. CD117 / KIT positive, HER2 positive
  4. MYB positive, estrogen receptor negative
  5. MYB positive, HER2 positive
Board review style answer #2
D. MYB positive, estrogen receptor negative. Adenoid cystic carcinoma of the breast is a biphasic tumor comprised of a ductal / epithelial cell and myoepithelial / basaloid cell components. The ductal / epithelial cells are immunoreactive for low molecular weight keratins such as CK7 and CK8/18. The myoepithelial / basaloid cell component is positive for high molecular weight keratins (CK5/6, CK14, CK17) and myoepithelial markers (p40 / p63, SMMS, calponin). Additionally, mammary adenoid cystic carcinomas generally express CD117 / KIT and MYB, and are triple negative for estrogen receptor, progesterone receptor and HER2.

Comment Here

Reference: Adenoid cystic carcinoma

Adenoid cystic-solid
Terminology
  • Also called basaloid type
Clinical features
Case reports
Gross description
  • Solitary mass, mean 4 cm
Gross images

Contributed by Dr. Semir Vranic
Microscopic (histologic) description
  • Basaloid cells with moderate to marked nuclear atypia, often with 5+ mitotic figures / 10 HPF
Microscopic (histologic) images
Scroll to see all images.


AFIP images

Solid pattern with
inconspicuous
cylindromatous
elements at arrows



Contributed by Dr. Hind Nassar, Johns Hopkins Medical Center, Maryland (USA):

Various images

p63

Actin



Contributed by Dr. Semir Vranic, University of Sarajevo, Bosnia and Herzegovina:
Missing Image Missing Image

Core biopsy


Missing Image Missing Image Missing Image Missing Image

Low power


Missing Image Missing Image Missing Image

Low power

Missing Image Missing Image

Medium power


Missing Image Missing Image Missing Image Missing Image

High power


Missing Image Missing Image Missing Image

High power

Positive stains
Negative stains
Differential diagnosis

Adenomyoepithelial adenosis
Definition / general
  • Uncommon variant of adenosis with proliferation of epithelial and myoepithelial cells
Essential features
  • Lobulocentric proliferation of both epithelial and myoepithelial components
  • Prominent myoepithelial cells, 1 - 3 layers
Terminology
ICD coding
  • ICD-10:
    • N60.2 - fibroadenosis of breast
  • ICD-11:
    • GB20.1 - fibroadenosis of breast
    • GB20.Y - other specified benign breast disease
Epidemiology
Sites
  • Lobulocentric but otherwise no specific location within the breast
Pathophysiology
  • Unknown
Etiology
  • Unknown
Clinical features
Diagnosis
  • Histologic examination of tissue with or without immunohistochemistry
Radiology description
  • No specific features
  • May show asymmetry, architectural distortion or mass with irregular margins, with or without microcalcifications
Radiology images

Images hosted on other servers:

Left breast

Hypoechoic mass

Irregular spiculated dense mass

Spiculated hypoechoic mass

Prognostic factors
  • Unknown but progression to malignancy may be associated with older age (> 50 years) and mass on imaging (Springerplus 2013;2:50)
Case reports
Treatment
  • Excision recommended due to possibility of recurrence and association with malignancy
  • Best predictors of recurrence are initial incomplete excision and close margins
Gross description
  • May form a firm mass with circumscribed or ill defined borders or may be grossly indistinguishable from surrounding fibrotic breast tissue
Microscopic (histologic) description
  • Multiple foci of haphazardly arranged ducts with eosinophilic granular secretions
  • Lined by cuboidal to columnar cells surrounded by hyperplastic myoepithelial cells
  • Resembles microglandular adenosis but with larger and more irregularly shaped glands, taller lining epithelial cells with eosinophilic granular cytoplasm and bland basal nuclei
  • May show apocrine or squamous metaplasia (Breast Care (Basel) 2008;3:427)
  • May have mild nuclear atypia
  • Rare to no mitotic activity
Microscopic (histologic) images

Contributed by Julie M. Jorns, M.D. and Kristen E. Muller, D.O.
Adenomyoepithelial adenosis Adenomyoepithelial adenosis

Adenomyoepithelial adenosis

Adenomyoepithelial adenosis Adenomyoepithelial adenosis Adenomyoepithelial adenosis

Adenomyoepithelial adenosis


Adenomyoepithelial adenosis

p63

Calponin

Calponin

Positive stains
Sample pathology report
  • Right breast, core biopsy:
    • Benign breast tissue with adenomyoepithelial adenosis
Differential diagnosis
Board review style question #1
Adenomyoepithelial adenosis Adenomyoepithelial adenosis


    The breast lesion pictured (H&E and p63 IHC, 20x) is most difficult to distinguish from which of the following lesions on core biopsy?

  1. Adenomyoepithelioma
  2. Invasive ductal carcinoma
  3. Microglandular adenosis
  4. Phyllodes tumor
Board review style answer #1
A. Adenomyoepithelioma. The pictured lesion is adenomyoepithelial adenosis, an uncommon variant of adenosis with a prominent myoepithelial component. This benign lesion may be difficult to distinguish from adenomyoepithelioma, which forms a circumscribed tumor but has similar microscopic features. Adenomyoepithelial adenosis, like other adenosis variants, may have similar features to microglandular adenosis or low grade invasive ductal carcinoma; however, these lesions lack myoepithelium, which is readily evident in adenomyoepithelial adenosis. Phyllodes tumor is a fibroepithelial neoplasm with dissimilar features, namely stromal proliferation with a resulting leaf-like growth pattern.

Comment Here

Reference: Adenomyoepithelial adenosis

Adenomyoepithelioma & malignant adenomyoepithelioma
Definition / general
  • Biphasic tumor composed of variable number of myoepithelial cells surrounding epithelial lined spaces
  • Usually expanded and prominent myoepithelial component
  • First recognized in the breast by Hamperl (Curr Top Pathol 1970;53:161)
Essential features
  • Biphasic tumor with epithelial and myoepithelial components
  • Benign to low grade malignant behavior and a propensity for recurrence
  • Either epithelial or myoepithelial component can show malignant transformation so thorough evaluation recommended
  • Wide surgical excision with appropriate margins recommended to prevent recurrence
  • Microscopy shows tubular or lobulated structures with epithelial and myoepithelial components
  • Epithelial component: cytokeratin+, EMA+, CEA+; myoepithelial component: S100+, SMA+, SMMHC+, p63+
Terminology
  • WHO (2019) divides adenomyoepithelioma into benign and malignant tumors
  • Malignant adenomyoepithelioma = adenomyoepithelioma with carcinoma, which can arise in epithelial or myoepithelial component or in both components (J Clin Pathol 2013;66:465)
ICD coding
  • Adenomyoepithelioma
    • ICD-O: 8983/0 adenomyoepithelioma
    • ICD-11: 2F30.Y & XH2V57 - other specified benign neoplasm of breast and adenomyoepithelioma, benign
  • Malignant adenomyoepitheliomas
    • ICD-O: 8562/3 epithelial-myoepithelial carcinoma
    • ICD-11: 2C6Y & XH7TL5 - malignant neoplasms of breast and adenomyoepithelioma with carcinoma
Epidemiology
Sites
Clinical features
Diagnosis
  • Well circumscribed mass, infrequently associated with tenderness and nipple discharge
  • Diagnosis on core biopsy specimen can be challenging
    • Presence of tightly aggregated glands arranged in compact nodules and prominent clear cell or spindle cell myoepithelium are clues to the diagnosis (Breast J 2004;10:522)
    • Immunohistochemical stains for myoepithelial markers are useful to highlight the myoepithelial component (Breast J 2004;10:522)
    • Excision is necessary for thorough evaluation of atypia or carcinoma arising in an adenomyoepithelioma
Radiology description
Radiology images

Contributed by Melissa Krystel-Whittemore, M.D. and Mark R. Wick, M.D.
Mammogram, irregular mass

Mammogram, irregular mass

Ultrasound, hypoechoic mass

Ultrasound, hypoechoic mass

Mammogram

Mammogram

Prognostic factors
  • Tubular type of adenomyoepithelioma, intraductal extension along periphery of lesion, incomplete excision and cytologic atypia are associated with local recurrence (Am J Surg Pathol 1991;15:554)
  • High mitotic rate, atypia, necrosis, cellular pleomorphism and infiltrative borders favor malignancy (Am J Surg Pathol 1992;16:868)
  • Malignant adenomyoepithelioma has prognosis dependent on histological subtype and grade of the malignant component
Case reports
Treatment
  • Complete wide excision with negative margins is standard treatment to prevent local recurrence
  • Mastectomy with or without axillary node dissection - only needed if malignant transformation
  • Reference: AJR Am J Roentgenol 2003;180:799
Gross description
  • Usually solitary nodule, median size: 2 cm, can be up to 8 cm (Breast J 2020;26:653)
    • Recurrent tumors usually larger
  • Sectioning reveals well circumscribed, firm, pink-white to gray-tan lesion
  • Can have focal cystic changes or necrosis
  • Malignant adenomyoepithelioma can show infiltrative borders (J Clin Pathol 2011;64:477)
Gross images

Contributed by Melissa Krystel-Whittemore, M.D., Case #418
Tan-gray mass

Tan-gray mass

Malignant adenomyoepithelioma Malignant adenomyoepithelioma

Malignant adenomyoepithelioma

Microscopic (histologic) description
  • Well circumscribed, may be encapsulated or multinodular and lobulated
  • Biphasic proliferation of epithelial and myoepithelial cells
  • Epithelial cells usually form glandular spaces; can show apocrine, sebaceous or squamous metaplasia
    • Can have papillary epithelial proliferation
  • Myoepithelial cells usually dominant and may be polygonal shaped with clear cytoplasm or spindled
  • Variants:
    • Spindle cell: spindle myoepithelial cells proliferation, epithelial lined spaces may be sparse
    • Tubular: proliferation of rounded tubules, ill defined margins
    • Lobulated: nests of myoepithelial cells surround compressed epithelial lined spaces
  • Malignant cases have infiltrative growth pattern, high mitotic rate or severe atypia which can be seen in the epithelial or myoepithelial component or in both components (Arch Pathol Lab Med 2000;124:632, Am J Surg Pathol 1992;16:868, Virchows Arch 1998;432:123)
  • Malignant transformation of epithelial component can have features of invasive carcinoma no special type, invasive lobular carcinoma, metaplastic carcinoma, including squamous cell carcinoma, spindle cell carcinoma or matrix producing carcinoma, low grade adenosquamous carcinoma or adenoid cystic carcinoma (Breast 2016;29:132, Pathol Int 2009;59:179, Virchows Arch 1995;427:243, Pathol Res Pract 2007;203:599, Virchows Arch 1998;432:123, Am J Surg Pathol 1998;22:631, Breast J 2019;25:731, Diagn Pathol 2014;9:148)
  • Malignant transformation of myoepithelial component shows features of myoepithelial carcinoma including overgrowth of myoepithelial cells, nuclear atypia and mitotic activity (Pathol Int 2006;56:211, Breast J 2007;13:203, J Clin Pathol 2011;64:477)
  • Biphasic malignant tumors (of epithelial and myoepithelial components) can be seen, with myoepithelial cells being the predominant component (Breast J 2019;25:1273)
    • Usually multilobulated or multinodular and can show a distinct transition from benign to malignant components
    • Malignant changes must be seen in both epithelial and myoepithelial cell types, specifically including increased mitotic activity
  • Malignant adenomyoepithelioma can be ER positive or negative but the carcinoma component is most commonly ER / PR / HER2 negative (Nat Commun 2018;9:1816)
Microscopic (histologic) images

Contributed by Hannah Y. Wen, M.D., Ph.D.

Circumscribed lobulated tumor

Myoepithelial cells surrounding glands

Compressed tubules

Hyaline-like basement membrane

Calponin

p63



Contributed by Fresia Pareja, M.D., Ph.D.
Biphasic epithelial and myoepithelial proliferation

Malignant adenomyo-
epithelioma

Biphasic epithelial and myoepithelial proliferation

Biphasic epithelial and myoepithelial proliferation

Biphasic epithelial and myoepithelial proliferation with nuclear atypia and mitoses

Nuclear atypia and mitoses

Biphasic epithelial and myoepithelial proliferation with fat invasion

Fat invasion

Virtual slides

Images hosted on other servers:

Adenomyoepithelioma

Cytology description
  • Moderate to highly cellular with large clusters of epithelium and myoepithelium
  • Tubular structures occasionally found
  • Myoepithelium appears as small clusters or dispersed cells with epithelioid morphology, intranuclear or intracytoplasmic vacuoles, often naked bipolar nuclei
  • Mild to moderate nuclear atypia present
  • Metachromatic fibrillary stroma occasionally found
  • No mitotic figures, no necrosis
  • Often classified incorrectly as fibroadenoma, suspicious for malignancy or malignant (Cancer 2006;108:250)
  • Malignant adenomyoepithelioma is highly cellular with neoplastic appearing cells
Positive stains
Negative stains
Electron microscopy description
  • Myoepithelial features (classic) include myofibrils with dense bodies, pinocytotic vesicles, desmosomes or tight junctions, patchy basement membrane
Molecular / cytogenetics description
Molecular / cytogenetics images

Images hosted on other servers:
<i>MYC</i> amplification

MYC amplification

Sample pathology report
  • Breast, left, excision:
    • Adenomyoepithelioma, 3.2 cm
    • Surgical margins negative for tumor
Differential diagnosis
  • Sclerosing adenosis:
    • Proliferation of epithelial glands with stromal sclerosis, which can cause architectural distortion of the glands
    • Less commonly mass forming
    • No prominent myoepithelial component
  • Intraductal papilloma:
    • Papillary lesion comprised of epithelial proliferation with fibrovascular cores
    • No prominent myoepithelial component
  • Invasive carcinoma (on core biopsy):
    • Malignant epithelial proliferation
    • Unequivocal evidence of invasion
    • Negative for myoepithelial markers (p63 / p40, SMMHC, calponin, CK5)
  • Nipple adenoma:
    • Epithelial proliferation arising in the collecting ducts of the nipple
    • No prominent myoepithelial component
  • Tubular adenoma:
    • Very well circumscribed
    • Proliferation of tubules with only single layer of myoepithelial cells surrounding the epithelial component without expansion of myoepithelial component
Board review style question #1

A 58 year old woman presents with a 2 cm nontender, firm nodule in the upper outer quadrant of her left breast. A core biopsy and subsequent resection of the lesion is performed. Representative microscopic images from the excision specimen are shown, which revealed an adenomyoepithelioma. Which of the following is true?

  1. Chemotherapy is the standard treatment of this lesion
  2. This lesion is usually HER2 positive and treated with anti-HER2 therapy
  3. This is a malignant lesion
  4. Without negative surgical margins, there is a risk of local recurrence of the lesion
Board review style answer #1
D. Without negative surgical margins, there is a risk of local recurrence of the lesion

Comment Here

Reference: Adenomyoepithelioma

Adenosis
Definition / general
  • Any hyperplastic process (see subtypes below) displaying increase in glands, typically within terminal duct lobular units
  • Common, often accompanied by fibrocystic changes
Essential features
  • Proliferation of glands, typically lobulocentric (expanding terminal duct lobular unit)
Terminology
ICD coding
  • ICD-10:
    • N60.3 - Fibrosclerosis of breast
    • N60.8 - Other benign mammary dysplasias
  • ICD-11:
    • GB20.Y - Other specified benign breast disease
    • GB20.Z - Benign breast disease, unspecified
Epidemiology
  • Wide age range, highest in third and fourth decades, paralleling fibrocystic changes
  • Premenopausal: lesions with predominant epithelial component (proliferation of acini and ducts); nodular adenosis
  • Postmenopausal: lesions with more sclerosis (sclerosing adenosis)
Sites
  • Terminal duct lobular unit; otherwise, no specific location within the breast
Clinical features
  • Considered a part of the spectrum of fibrocystic changes
  • Can present as a palpable mass if nodular adenosis / adenosis tumor
Diagnosis
  • Histologic examination of tissue with or without immunohistochemistry
Radiology description
Prognostic factors
  • With some subtypes (e.g. sclerosing adenosis), the risk of subsequent breast cancer is 1.5 - 2 times higher, as seen with proliferative fibrocystic changes
  • 2 times higher risk of breast cancer with increased Ki67 proliferation index in sclerosing adenosis or associated normal background breast tissue (Breast Cancer Res Treat 2015;151:89)
Case reports
Treatment
  • Does not require treatment unless there is cytologic atypia (rare)
Gross description
  • Variable - may be indistinguishable from surrounding benign breast tissue, ill defined fibrosis or fibrotic mass with relatively well defined borders
  • See separate adenosis topics for specific subtypes
Microscopic (histologic) description
  • Simple adenosis: proliferation of acini, with 2 layers (inner epithelial and outer myoepithelial), surrounded by basement membrane, without distortion of lobular architecture
  • Adenomyoepithelial adenosis: adenosis, with proliferation of glands (2 layers) with prominent myoepithelial cells
  • Apocrine adenosis: adenosis, with proliferation of glands (2 layers) with predominant apocrine cytology
  • Sclerosing adenosis: adenosis, with proliferation of glands (2 layers) with glandular compression and (mild to marked) distortion due to stromal fibrosis / sclerosis; entrapped secretions frequently become calcified
  • Tubular adenosis: adenosis with haphazard proliferation of branching tubules (2 layers)
  • Nodular adenosis / adenosis tumor: adenosis that is mass forming
  • Cytologic atypia is uncommon
  • See also separate topics for specific subtypes
Microscopic (histologic) images

Contributed by Julie M. Jorns, M.D.
Simple adenosis Simple adenosis

Simple adenosis

Tubular adenosis

Tubular adenosis

Nodular adenosis

Nodular adenosis

Virtual slides

Images hosted on other servers:
Breast, adenosis

Breast, adenosis

Breast, adenosis (myosin / smooth muscle stain)

Breast, adenosis (myosin / smooth muscle stain)

Cytology description
  • Cellular with small groups of benign epithelial cells with variable architecture, including sheets and individual cells and dense hyalinized stroma (Acta Cytol 2001;45:353)
Positive stains
Videos

What is adenosis?

Sample pathology report
  • Left breast, core biopsy:
    • Nodular adenosis with microcalcifications
Differential diagnosis
Board review style question #1

A 48 year old woman had calcifications detected on a screening mammogram, which showed the pictured lesion. What should be done next?

  1. Biopsy adjacent breast tissue
  2. Excision of the lesion
  3. Mammography per routine screening
  4. Neoadjuvant chemotherapy
  5. Perform breast MRI
Board review style answer #1
C. Mammography per routine screening. The pictured lesion is sclerosing adenosis, which is a benign lobular proliferation with distortion of glands by fibrosis. Microcalcifications (as seen above as dark purple intraluminal material) are frequently seen in sclerosing adenosis and are a result of calcification of entrapped secretions, making this biopsy benign and concordant with imaging findings. Therefore, no treatment or additional imaging is needed outside of routine screening.

Comment Here

Reference: Breast adenosis
Board review style question #2


Which of the following is usually true regarding simple adenosis of the breast?

  1. Clinically palpable
  2. Displays cytologic atypia
  3. Involves terminal duct lobular units
  4. Lacks myoepithelium
  5. Requires immunohistochemistry for diagnosis
Board review style answer #2
C. Involves terminal duct lobular units. Simple adenosis of the breast is a proliferation of glands with normal 2 cell layer (inner epithelial and outer myoepithelial) configuration involving terminal duct lobular units. It rarely harbors atypia and does not show distortion by fibrosis as seen in other forms of adenosis (e.g. sclerosing, tubular), making diagnosis relatively easy via H&E staining alone. It is often incidental or may less frequently show distortion or calcifications on imaging. This contrasts with nodular adenosis / adenosis tumor, which is mass forming and may be clinically palpable.

Comment Here

Reference: Breast adenosis

Amastia / aplasia / hypoplasia / athelia
Definition / general
  • Amastia: no glandular tissue (no breast bud), no nipple and no areola
  • Aplasia: nipple and areola present but no glandular tissue
  • Athelia: very rare; breast glandular tissue but no nipple or areola (Wikipedia: Athelia (Disease)); also a genus of fungi
  • Hypoplasia: small rudimentary breasts (i.e. breast underdevelopment); also called micromastia
Epidemiology
  • Uncommon
  • Occurs more often in females than males
Sites
  • Unilateral or bilateral
Etiology
Case reports
Treatment
Clinical images

Images hosted on other servers:

Amastia in 11 year old girl

Hypoplasia in 9 year old girl
(fig 1)

Becker nevus with hypoplasia

Hypoplasia of right breast



26 year old woman:

Preoperative of bilateral amastia


Amastia / aplasia / hypoplasia / athelia
Definition / general
Essential features
  • Congenital breast anomalies are rare; unilateral or bilateral
  • Athelia and amastia are associated with Poland syndrome
  • Amastia can be part of other syndromes, such as AREDYLD (acral renal ectodermal dysplasia lipoatrophic diabetes) syndrome
  • Athelia can be congenital or acquired
  • Breast reconstruction using transverse rectus abdominis muscle (TRAM) flap and nipple areola reconstruction is the treatment of choice
Terminology
ICD coding
Not applicable
Epidemiology
Sites
Pathophysiology
Not applicable
Etiology
Diagrams / tables
N/A
Clinical features
  • Patients usually present with no breast development with or without nipple areola complex in pediatric or surgery outpatient department (OPD)
Diagnosis
Laboratory
Not applicable
Radiology description
Not applicable
Radiology images
Not applicable
Prognostic factors
Not applicable
Case reports
Treatment
Clinical images

Images hosted on other servers:
Bilateral athelia

Bilateral athelia

Amastia before reconstruction

Amastia (before reconstruction)

Amastia after implant

Amastia (after implant)

Amastia after NAC reconstruction

Amastia (after NAC reconstruction)

Polythelia

Polythelia

Gross description
Not applicable
Gross images
Not applicable
Frozen section description
Not applicable
Frozen section images
Not applicable
Microscopic (histologic) description
  • Histopathology of hypoplasia is mostly normal; sometimes shows decreased gland to stroma ratio with predominance of fibrocollagenous stroma and mature adipose tissue
  • Histologically, both unilateral and bilateral hypoplastic breast tissue consists of fibrous stroma and ductal structures without acinar differentiation
  • Ducts typically resemble those in the prepubertal breast and lack lobular development (Semin Plast Surg 2013;27:42)
Microscopic (histologic) images

Contributed by Reena Tomar, M.B.B.S., M.D.
Hypoplasia Hypoplasia

Hypoplasia

Hypoplasia

Hypoplasia

Virtual slides
Not applicable
Cytology description
Not applicable
Cytology images
Not applicable
Immunofluorescence description
Not applicable
Immunofluorescence images
Not applicable
Positive stains
Not applicable
Negative stains
Not applicable
Electron microscopy description
Not applicable
Electron microscopy images
Not applicable
Molecular / cytogenetics description
Not applicable
Molecular / cytogenetics images
Not applicable
Videos
Not applicable
Sample pathology report
  • Left breast, trucut biopsy:
    • Rare benign breast ductules with fibrocollagenous stroma (clinically breast hypoplasia) (see comment)
    • Comment: A 16 year old girl presented with left side hypoplasia in surgery OPD. Trucut biopsy was done from left breast.
    • Gross: 4 trucut cores of breast swelling measuring 1 x 0.2 cm, 0.8 x 0.2 cm, 0.7 x 0.1 cm and 1.1 x 0.2 cm
    • Microscopy: Section shows few benign breast ductules with fibrocollagenous stroma. Predominance of mature adipose tissue seen. No features of atypia are seen.
Differential diagnosis
  • Turner syndrome:
    • Can present with hypoplasia
    • Presents as amenorrhea, short stature and webbed neck, which is not present in breast congenital anomalies
    • Karyotyping is 45X, whereas breast congenital anomalies karyotyping is normal 46XX
Additional references
N/A
Board review style question #1

A 16 year old normal statured girl presents to the general surgery outpatient department with no development of breast on both sides. On ultrasonography (USG) of the abdomen, uterus and bilateral ovaries are normal. Menstrual cycles are regular. Karyotyping shows 46XX. Which of the following syndromes can be associated with this condition?

  1. Androgen insensitivity syndrome
  2. Klinefelter syndrome
  3. Poland syndrome
  4. Turner syndrome
Board review style answer #1
C. Poland syndrome. Bilateral amastia is very commonly associated with Poland syndrome. Answer D is incorrect because Turner syndrome presents as short stature and amenorrhea. Answer B is incorrect because Klinefelter syndrome is 46XXY on karyotyping. Answer A is incorrect because androgen insensitivity syndrome patients have no uterus or ovaries but can have normal breast development.

Comment Here

Reference: Amastia / aplasia / hypoplasia / athelia

Amyloid
Definition / general
  • Extracellular mass-like deposition of amorphous and insoluble proteins in an abnormal fibrillary configuration
Essential features
  • Extracellular deposition of acellular eosinophilic material in fat, stoma or blood vessels creating a mass effect
  • Stains positively for Congo red; metachromatic on Wright stain
  • May be localized or part of systemic amyloidosis
  • Treatment by complete excision
Terminology
  • Amyloidosis in the breast first reported by Fernandez and Hernandez in 1973 (J Clin Pathol 2002;55:634)
  • Also called amyloidoma
Epidemiology
  • Very rare
  • Usually women ages 45 - 79 years
Sites
  • 80% occur in right breast
  • Bilateral or unilateral but only rarely is confined to breast
Pathophysiology
  • Abnormal deposition of insoluble protein in an abnormal fibrillary configuration
  • Most common types are amyloid light chain (AL) and amyloid A (AA)
  • AL is caused by immunoglobin light chains and is secondary to plasma cell dyscrasia
  • AA is reactive amyloidosis and is secondary to chronic inflammatory disease
  • In vast majority of patients, breast amyloidosis is part of a systemic AL type disease (usually kappa light chain proteins)
  • It can be associated with malignancies of the breast
Etiology
  • Depends on the type of amyloidosis:
    • AL type amyloidosis - plasma cell dyscrasia (myeloma, Waldenström macroglobulinemia)
    • AA type amyloidosis - chronic inflammatory disease (rheumatoid arthritis, reactive arthritis)
Diagrams / tables

Images hosted on other servers:

Figure 7: Flow chart for diagnosis of amyloidosis

Clinical features
  • Painless mass
  • May occur late in systemic disease
  • Complete physical examination needed to differentiate primary and secondary amyloid tumors
Laboratory
Radiology description
  • Visualized as mass on ultrasound and MRI
Radiology images

Images hosted on other servers:

Figures 1 and 2

Case reports
Treatment
  • Excisional biopsy is successful if amyloid tumor is limited to the breast
Gross description
  • 5 cm or less, firm, gray white, opalescent
Microscopic (histologic) description
  • Amorphous eosinophilic extracellular deposits in fat, stroma or vessels
  • May cause ductal atrophy, form rings around individual fat cells, have associated multinucleated giant cell reaction or osseous metaplasia (J Clin Path 2002;55:634)
Microscopic (histologic) images

Contributed by Dr. Mark R. Wick

Amyloid tumor

Thioflavin T stain

Parenchyma - Congo red

 AFIP images

Thick wavy bands of
amyloid around lobular
glands and a nodular deposit
that is virtually acellular

Amyloid occurs in
lobule (lower left) and
as separate stromal
nodule (upper right)



Images hosted on other servers:

Various images

Amyloid with osseous metaplasia

Cytology description
  • Amorphous acellular material with scattered plasma cells, lymphocytes, stromal cells and epithelial cells
  • Rare multinucleated giant cells
  • Metachromatic by modified Wright stain (Diagn Cytopathol 2003;28:325)
Cytology images

Images hosted on other servers:

Dense hyaline material and giant cells

Positive stains
  • Congo red (red orange with apple green birefringence under polarized light)
  • Metachromatic with crystal violet
Negative stains
Electron microscopy description
Differential diagnosis
  • Plasmacytoma: immature plasma cells, no amyloid
  • Stromal fibrosis: may have hyaline material but it has different histologic features from amyloid; also Congo red negative

Anatomy
Definition / general
Thelarche
  • Signifies entry into Tanner stage II of development
  • Growth is due to fat deposition, periductal connective tissue expansion and the elongation, branching and thickening of ductal system with development of terminal duct lobular units; growth may be sporadic
  • Prior to puberty, breasts of both sexes have ducts with variable branching lined by cuboidal epithelium, no lobules and no necrosis; at puberty in females, lobules develop
  • Influenced by estrogens, growth hormone and prolactin but not progesterone
  • Premature thelarche:
    • Onset prior to age 8 in girls with normal BMI in general population
    • Breast tissue resembles gynecomastia with epithelial hyperplasia in ductal system in a solid or micropapillary configuration (BMC Pediatr 2008;8:27)
    • Excision of initial subareolar disc will lead to amastia after puberty
    • Associated with increased BMI and non-Hispanic black and Mexican American ethnicity (Pediatrics 2009;123:84)
    • May be isolated or part of precocious (central) puberty if other signs of puberty are present
    • No tests currently predict risk of progression to precocious puberty in premature thelarche (J Pediatr 2010;156:466)
    • Earlier onset of thelarche and longer interval between thelarche and menarche may be associated with increased breast cancer risk (Breast Cancer Res 2014;16:R18)

    Tanner staging:

    Stage I: preadolescent

    Stage II: breast budding (thelarche)

    Stage III: enlargement of areolar diameter

    Stage IV: areola / papilla
    form secondary mound
    with separation of contours

    Stage V: mature female breasts

  • Gross anatomy
    • Breast Proper:
      • Suspended from anterior chest by ligaments of Cooper (Wikipedia: Cooper's Ligaments) attached to skin and fascia of major and minor pectoral muscles
      • Covered by skin and subcutaneous tissue anteriorly
      • Spans 2nd rib to 6th rib sacroiliac (SI), midaxillary line to medial border at edge of sternum midline, in addition to axillary tail of Spence (Wikipedia: Tail of Spence), which extends into axilla
      • Posterior / deep margin is fascia of pectoralis major muscle
      • Breast tissue may extend beyond these limits into adjacent subcutaneous tissue
        • In some patients, mastectomy removes most but not all breast epithelial elements, raising possibility of breast disease at these sites
      • Accessory breast tissue may be present along the milk line (axilla → anterior chest wall → pubis → upper thighs)
      • Glandular tissue is most abundant in upper outer quadrant of breast; as a result, half of all breast cancers occur here

    • Nipple areolar complex:
      • Located slightly medial and inferior on breast, level of 4th intercostal on nonpendulous breast
      • Cone shaped protuberance, 10 - 12 mm in average height, mean nipple diameter 11 - 13 mm, mean areolar diameter 4 cm
      • Montgomery tubercles - areolar protuberances (10 - 20), more prominent during pregnancy; composed of ducts and sebaceous apparatus

    • Lymphatic drainage:
      • Four major routes => cutaneous, axillary, internal thoracic, posterior intercostal
        • Cutaneous
          • Superficial plexus within dermis
          • Most if not all lymphatics connect to areolar lymphatic plexus (plexus of Sappey)
            • Significant for sentinel lymph node procedure, injection in area of nipple will likely identify sentinel nodes, as would injection in region of tumor
          • Cutaneous lymphatic anastomoses account for rare cases of metastases to contralateral breast in absence of distant metastases
          • Cutaneous lymphatics of inferior breast may drain to epigastric plexus → lymphatic channels of liver and intra-abdominal lymph nodes
        • Axilla
          • Majority (75%) of drainage
          • Initially through 1 - 2 sentinel nodes, first to be involved in 90% of patients with lymph node metastases
          • Three levels
            • Level I - low axilla
            • Level II - mid axilla
              • Includes Rotter nodes (interpectoral)
            • Level III
              • Apical or infraclavicular nodes
              • Metastases here portend a worse prognosis
          • Intramammary nodes
            • May be found anywhere in breast parenchyma
            • Considered as axillary lymph nodes for staging purposes
            • Only very rarely the sentinel node
        • Internal thoracic
          • < 10% of lymphatic flow from breast, may be more prominent if lymphatic drainage is obstructed / disrupted due to disease in axilla or previous treatment
          • Terminates in internal mammary nodes
            • Involvement does not alter treatment in most cases; as a result, rarely sampled when systemic therapy is planned
        • Posterior intercostal
          • Least important route of drainage
        • Supraclavicular nodes
          • Are deemed regional nodes for staging purposes
        • Metastases to all other lymph nodes, including cervical and contralateral axillary nodes, are classified as distant metastases
      • References (lymphatics): Ann Surg Oncol 2008;15:863, Hum Pathol 2001;32:178
    Breast proper
    • Epithelial and stromal components; specific lesions arise in each component or may have a dual morphology
    • Epithelium
      • Large ducts
        • Three dimensional arborizing fan shaped system with apex towards the nipple
        • Large ducts → segmental ducts → subsegmental ducts → lobules
        • Lactiferous sinuses are likely artifactual (J Anat 2005;206:525); this study changed commonly held views of anatomy (Wikipedia: Breast)
        • Duct systems are only rarely confined to a single quadrant and may overlap
        • Anastomoses may exist between large duct regions
        • Involvement of multiple duct systems by DCIS (a clonal proliferation) may be explained as follows
          • DICS arising in single large duct system may be present in overlapping quadrants
          • DCIS arising in single large duct system may involve adjacent systems through anastomotic connections
          • Separate foci of DCIS may arise in different duct systems

      • Terminal duct lobular units (TDLU)
        • "Functional unit" of breast (lacation)
        • Postulated as origin of most epithelial neoplasms (breast or myoepithelial)

      • Two types of epithelia: luminal cells and myoeptihelium
        Luminal cells ("epithelial cells"):
        • Innermost layer of bilayered ductolobular system
        • Usually express luminal keratins LMWCK, CK7, CK8, CK18, CK19
        • May express "basal keratins" HMWCK, CK5 / 6, CK14, CK17 (J Histochem Cytochem 1986;34:869)
        • Most express ER-α and or PgR in a heterogeneous pattern at any given time
        • Also expresses E-cadherin, mammaglobin and GCDFP-15
        Myoepithelium:
        • Outer layer resting on basement membrane
        • Contractile meshwork partially covering basement membrane, i.e. incomplete on cross section
        • Usually expresses "basal" keratins HMWCK, CK5 / 6, CK14, CK17
        • Also smooth muscle actin, calponin, smooth muscle myosin heavy chain, in addition to p63, S100, CD10, P-cadherin and mapsin
        • Does NOT express ER or PgR
        • Loss of myoepithelium usually considered hallmark of invasive process, with the exception of microglandular adenosis, encapsulated papillary carcinoma and solid papillary carcinoma
    Stroma
    • Interlobular
    • Intralobular
      • Surrounds acini in TDLUs
      • Compared to interlobular stroma, looser in appearance or even myxoid
      • Often small numbers of inflammatory cells
      • Lesions arising here are biphasic involving epithelial and stromal components, including fibroadenoma, phyllodes tumor
    Nipple areolar complex
    • Ramifying duct system terminating in 15 - 20 major ducts, which open onto areola
    • Covered in pigmented skin
    • Toker cells
      • Cytologically benign cells with clear / pale cytoplasm
      • Immunoprofile similar to luminal epithelial cells
      • Are most abundant adjacent to duct orifices
      • Must distinguish from mammary Paget disease (DCIS involving nipple)
    • Keratin producing squamous cells extend into proximal 1 - 2 mm of large ducts
      • Squamous metaplasia of lactiferous ducts (SMOLD) is extension of squamous epithelium beyond proximal 1 - 2 mm; associated with smoking
      • Keratin entrapment may cause epidermal inclusions and abscess formation with marked inflammatory response
    • Basement membrane of ducts and duct lobular system is contiguous with that of skin
      • Tumor cells may extend onto skin from DCIS without crossing basement membrane (Paget disease)
    • External shape / size of the breast is not predictive of internal anatomy or its lactation potential
    • References (nipple): Breast Cancer Res Treat 2007;106:171, Eur Surg Res 2006;38:545, Cancer 2004;101:1947, Aesthetic Plast Surg 2009;33:295, Am J Surg 2007;194:433, Arch Pathol Lab Med 1982;106:60
    Diagrams / tables

    Images hosted on other servers:

    Milk lines

    Breast anatomy

    Regional lymph nodes - schematic


    Radical mastectomy

    Supraclavicular fossa

    Radiograph of lymphatics

    Tracing of lymphatics

    Gross images

    Images hosted on other servers:

    Nipple orifices, some cannulated with guide wires

    Microscopic (histologic) images

    AFIP images

    Premature thelarche (9 year old girl)

    Proliferation of ducts without lobules



    Images hosted on other servers:

    Perforating lymphatic

    Instillation of
    water insoluble and
    differently colored
    dyes in orifices


    Angiosarcoma
    Definition / general
    • Malignant vascular neoplasm, which may arise in previously irradiated skin or breast tissue (secondary postradiation angiosarcoma) or in nonirradiated tissue (primary angiosarcoma)
    Essential features
    • Irregular, infiltrative anastomosing vascular channels with endothelial atypia
    • Postradiation angiosarcoma demonstrates amplification of MYC
    Terminology
    • Alternate terminology: hemangiosarcoma, lymphangiosarcoma
    ICD coding
    • ICD-O: 9120/3 - angiosarcoma, postradiation angiosarcoma
    • ICD-10: C50 - malignant neoplasm of breast
    • ICD-11: 2B56.2 & XH6264 - angiosarcoma of breast & angiosarcoma
    Epidemiology
    Sites
    • Postradiation angiosarcoma
      • Typically arises in irradiated skin and may secondarily extend into the breast parenchyma (Breast J 2020;26:458)
      • Less commonly arises in breast parenchyma
    • Primary angiosarcoma
      • Usually arises in breast parenchyma
    • Stewart-Treves syndrome
      • Cutaneous angiosarcoma of the upper limb arising in the setting of chronic lymphedema, typically occurring after mastectomy and axillary dissection (J Am Acad Dermatol 2012;67:1342)
      • May secondarily extend to the breast / chest wall
    Pathophysiology
    • Malignancy of vascular or lymphatic endothelial cells
    • Exact lineage remains unclear since endothelial cells may undergo transdifferentiation
    Etiology
    • Ionizing radiation (postradiation angiosarcoma)
    • Longstanding lymphedema (Stewart-Treves syndrome)
    • Unknown (primary angiosarcoma)
    Clinical features
    • Solitary or multiple erythematous to violaceous plaques, papules, nodules or areas of discoloration on the skin (postradiation angiosarcoma) (Am J Clin Pathol 1994;102:757)
    • Rapidly growing poorly defined mass, swelling or asymmetry of the breast parenchyma (primary angiosarcoma) (Am J Surg Pathol 2008;32:1896)
    Diagnosis
    • Histologic examination of tissue obtained by biopsy or surgical excision
    Radiology description
    • Mammography
    • Ultrasonography
      • Hyperechoic mass or areas of mixed hyper and hypoechogenicity with architectural distortion (Radiology 2007;242:725)
      • Doppler sonography shows hypervascularity
    • Magnetic resonance imaging
      • High signal intensity in T2 weighted images
      • Low grade tumors show progressive enhancement
      • High grade tumors show rapid enhancement and washout
    Radiology images

    Contributed by Melinda J. Lerwill, M.D.
    Postradiation angiosarcoma arising in the skin

    Postradiation angiosarcoma arising in the skin

    Postradiation angiosarcoma of the breast parenchyma

    Postradiation angiosarcoma of the breast parenchyma

    Prognostic factors
    Case reports
    Treatment
    • Complete surgical excision is the primary treatment modality
    • Radical resection of all previously irradiated skin may improve recurrence rate and disease specific survival in cases of postradiation angiosarcoma (Ann Surg 2017;265:814)
    • Role of chemotherapy is poorly defined but it may improve survival in patients with tumors > 5 cm (Breast Cancer Res Treat 2019;175:409)
    • Radiation therapy remains controversial, particularly in cases of postradiation sarcoma
    Clinical images

    Contributed by Mark R. Wick, M.D.
    Postradiation angiosarcoma

    Postradiation angiosarcoma

    Gross description
    • Average size of 4 - 6 cm (Breast Cancer Res Treat 2019;178:523)
    • Multifocality is more common in postradiation than in primary angiosarcoma
    • Firm to spongy hemorrhagic mass
    • Cystic necrosis can occur in higher grade tumors
    • Tumors are often larger than is grossly evident
    Gross images

    Contributed by Melinda Lerwill, M.D. and Mark R. Wick, M.D.
    Postradiation angiosarcoma

    Postradiation angiosarcoma

    Postradiation angiosarcoma

    Postradiation angiosarcoma

    Frozen section description
    Microscopic (histologic) description
    • Morphologic heterogeneity is typically present within individual tumors
    • Irregular anastomosing vascular channels dissect through the stroma
    • Infiltrative growth through normal structures (e.g., terminal duct lobular units)
    • Poorly differentiated tumors often show solid growth of spindle or epithelioid cells with limited vasoformative features
    • Necrosis and blood lakes can be seen in poorly differentiated tumors
    • Endothelial cell atypia can range from minimal to marked
    • Endothelial tufting and papillary formations are common
    • Mitotic activity increases in proportion to grade
    • Grading is based on a combination of histologic features (Table 1) (Am J Surg Pathol 1981;5:629)
      • Low grade features are more common as a dominant finding in primary angiosarcomas
      • Majority of postradiation angiosarcomas are high grade (Ann Surg Oncol 2013;20:1267, Ann Surg Oncol 2020;27:1002)
      • All histologic grades of angiosarcoma may contain low grade areas, especially at the periphery of the tumor
    Table 1. Histologic characteristics of different grades of mammary angiosarcoma
    Histologic features Low grade Intermediate grade High grade
    Endothelial cell atypia  Minimal   Moderate  Marked 
    Endothelial tufting  Minimal  Present  Prominent 
    Papillary formations  Absent  Focally present  Present 
    Solid and spindle cell foci  Absent  Absent or minimal  Present 
    Mitotic figures  Absent or rare  Present  Numerous and
    present in
    structurally
    low grade areas 
    Blood lakes (pools
    of extravasated
    erythrocytes) 
    Absent  Absent  Present 
    Necrosis  Absent  Absent  Present 
    Microscopic (histologic) images

    Contributed by Melinda Lerwill, M.D.
    Low grade primary angiosarcoma

    Low grade primary angiosarcoma

    Irregular vessels

    Irregular vessels

    Infiltrative growth

    Infiltrative growth

    Postradiation angiosarcoma Postradiation angiosarcoma

    Postradiation angiosarcoma

    High grade cytologic atypia

    High grade cytologic atypia


    Solid growth

    Solid growth

    Epithelioid cells

    Epithelioid cells

    Blood lake

    Blood lake

    ERG

    ERG

    MYC

    MYC

    Virtual slides

    Images hosted on other servers:

    High grade mammary angiosarcoma

    Cytology description
    • Variable cellularity: specimens from well differentiated tumors tend to be hypocellular and bloody, while those from poorly differentiated tumors are usually more cellular
    • Dispersed cells, cells in loose or cohesive groups or focal papillae with fibrovascular cores (Cancer Cytopathol 2016;124:659)
    • Variable cellular morphology (spindle, epithelioid or mixed)
      • Prominent epithelioid cells can mimic carcinoma
    • Oval to spindled nuclei with variable pleomorphism
    • Multiple nucleoli and bar or bullet shaped nucleoli are common; chromatin stranding may be seen (delicate radiating chromatin strands connecting the nucleolus to the nuclear membrane)
    • Vasoformative features (hemophagocytosis, cytoplasmic lumina / vacuoles with red blood cells or neutrophils, endothelial wrapping) are often present but their prominence and extent vary
    Cytology images

    Contributed by Sofia Lérias, M.D.
    Atypical spindle cells

    Atypical spindle cells

    Positive stains
    Negative stains
    Electron microscopy description
    • Groups of cells surrounded by basal lamina and closely associated with pericytes, with intercellular and intracellular lumina with or without red blood cells (Am J Surg Pathol 1998;22:683)
    • Weibel-Palade bodies may be present (Hum Pathol 1985;16;144)
    Molecular / cytogenetics description
    Videos

    Postradiation angiosarcoma

    Sample pathology report
    • Breast, mastectomy:
      • Post-radiation angiosarcoma (see comment)
      • Comment: The angiosarcoma measures 7.5 x 5 cm and involves the skin and underlying breast parenchyma. The tumor is high grade with pleomorphic spindle and epithelioid cells, a high mitotic rate (up to 30 MF / 10 HPF) and focal areas of necrosis. By immunohistochemistry, the tumor cells are positive for CD31, CD34, D2-40, ERG and MYC and negative for pancytokeratin and S100, supporting the diagnosis. Angiosarcoma extends to 0.5 cm of the superior skin margin, 1 cm from the inferior skin margin and is greater than 2 cm from the remaining margins.
    Differential diagnosis
    • Atypical vascular lesion:
      • Small vascular proliferation occurring in the dermis of irradiated skin
      • Most lesions are < 1 cm in size (and often < 0.5 cm)
      • Relatively circumscribed, often wedge shaped proliferation of thin walled vessels within the superficial to mid dermis
      • Lacks evidence of infiltration
      • No significant endothelial atypia or proliferation
      • Lacks amplification of MYC and negative or only weakly reactive for MYC by immunohistochemistry
      • Clinicopathological correlation is needed to ensure that the biopsy does not represent sampling at the edge of an angiosarcoma
    • Hemangioma:
      • Well defined
      • No destructive invasion of terminal duct lobular units
      • Flattened endothelial cells without atypia or significant mitotic activity
    • Angiomatosis:
      • Very rare benign vascular proliferation involving large areas of breast parenchyma
      • Large dilated vascular channels permeate the breast but show exquisite sparing of the terminal duct lobular units, unlike angiosarcoma
      • No endothelial atypia, stratification or notable mitotic activity
    • Carcinoma:
      • Evidence of epithelial differentiation or in situ carcinoma
      • Negative for vascular markers
    Board review style question #1
    Which of the following is true regarding postradiation angiosarcoma of the breast?

    1. Greater than 90% are low grade
    2. Incidence of 10% among patients receiving breast irradiation for breast cancer treatment
    3. Most often arises in the breast parenchyma and then secondarily involves the skin
    4. Patients have a median age of 40 years
    5. Shows strong positivity for MYC by immunohistochemistry
    Board review style answer #1
    E. Shows strong positivity for MYC by immunohistochemistry. Greater than 90% of postradiation angiosarcomas of the breast show high level amplification of MYC and corresponding strong overexpression of MYC by immunohistochemistry. Postradiation angiosarcoma develops in 0.03 - 0.14% of patients after radiation therapy for the treatment of breast cancer. Reflective of that cohort, the mean age is 70 years. Most postradiation angiosarcomas arise in the skin (from which they may secondarily involve the breast parenchyma) and the majority are of high histologic grade.

    Comment Here

    Reference: Angiosarcoma
    Board review style question #2

    A 27 year old woman with no significant prior medical history presents with an 8 cm poorly defined left breast mass. A representative histological image is shown above. What is the most likely diagnosis?

    1. Atypical vascular lesion of the skin
    2. Capillary hemangioma
    3. Low grade angiosarcoma
    4. Pseudoangiomatous stromal hyperplasia
    5. Spindle cell carcinoma with acantholytic pattern
    Board review style answer #2
    C. Low grade angiosarcoma. The illustrated lesion is a primary, low grade angiosarcoma of the breast parenchyma. It demonstrates irregularly shaped, anastomosing vascular channels that dissect through the fat and collagen and around normal glandular structures. This pattern of destructive invasion distinguishes it from benign vascular proliferations.

    Comment Here

    Reference: Angiosarcoma

    Antibody drug conjugate therapy (pending)
    [Pending]

    Apocrine
    Definition / general
    • Invasive breast carcinoma comprised of large cells with abundant eosinophilic and granular cytoplasm, enlarged nuclei and prominent nucleoli
    • Apocrine morphology in > 90% of tumor cells
    Essential features
    • Apocrine morphology in > 90% of tumor cells
    • Estrogen receptor (ER) negative, progesterone receptor (PR) negative and androgen receptor (AR) positive
    Terminology
    • Carcinoma with apocrine differentiation
    • Invasive apocrine carcinoma
    • Apocrine carcinoma
    ICD coding
    • ICD-O: 8401/3 - apocrine adenocarcinoma
    • ICD-11: 2C61 & XH4GA3 - invasive carcinoma of breast & adenocarcinoma with apocrine metaplasia
    Epidemiology
    • Rare subtype of invasive breast carcinoma, representing ~1% of invasive carcinomas
    • Mean age of diagnosis is typically older than invasive breast carcinoma, NST
    Sites
    • Breast
    • Can occur in axilla
    Etiology
    • Most carcinomas with apocrine differentiation are sporadic
    • Some carcinomas in patients with germline PTEN mutation (Cowden syndrome) display apocrine morphology (Breast Cancer Res 2010;12:R63)
    Clinical features
    Diagnosis
    • Histologic examination of involved tissue
    Radiology description
    Radiology images

    Contributed by Jonathan Marotti, M.D.
    Mammogram

    Mammogram

    Ultrasound

    Ultrasound

    Prognostic factors
    • Prognosis is based on conventional factors such as grade, tumor size and nodal status
    • Studies evaluating clinical outcomes have contradictory results (Clin Breast Cancer 2022;22:e576)
    Case reports
    Treatment
    • Surgical excision
    • Chemotherapy and radiation are dependent on stage
    • Neoadjuvant therapy may be considered
    • Anti-HER2 therapy for HER2+ tumors
    • Single agent anti-AR therapy has shown limited therapeutic benefit; clinical trials are ongoing (Clin Breast Cancer 2022;22:e576)
    Microscopic (histologic) description
    • Infiltrating tumor cells with apocrine morphology: abundant eosinophilic and granular cytoplasm, well defined cell borders, enlarged round nuclei and prominent nucleoli
    • Apocrine morphology in > 90% of tumor cells
    • Typically high grade / poorly differentiated
    • Often associated with ductal carcinoma in situ (DCIS) with apocrine features
    Microscopic (histologic) images

    Contributed by Jonathan Marotti, M.D.
    Large atypical epithelial cells

    Large atypical epithelial cells

    AR positive

    AR positive

    ER negative

    ER negative

    PR negative

    PR negative

    Large atypical epithelial cells

    Large atypical epithelial cells

    AR positive

    AR positive

    Cytology description
    • Highly cellular
    • Sheets, clusters and single cells with abundant eosinophilic cytoplasm, large nuclei with variable pleomorphism and prominent nucleoli (Acta Cytol 1996;40:247)
    • Binucleate and multinucleate forms; often many bare nuclei
    • Difficult to differentiate from benign apocrine metaplastic lesions on cytologic preparations (Diagn Cytopathol 2017;45:1156)
    Positive stains
    Negative stains
    Electron microscopy description
    Electron microscopy images

    Images hosted on other servers:

    Cytoplasm rich in organelles

    Electron dense granules

    Molecular / cytogenetics description
    • Most genetic alterations found in carcinoma with apocrine differentiation are also found in invasive breast carcinoma, NST
    • Frequent mutations of TP53 and PIK3CA / PTEN / AKT genes (Mod Pathol 2020;33:2473)
    • Gene expression profiling has defined a molecular apocrine / luminal AR signature (with or without HER2 activation)
    • Molecular apocrine / luminal AR signature does not always correlate with apocrine morphology
    Sample pathology report
    • Left breast, partial mastectomy:
      • Carcinoma with apocrine differentiation (see synoptic report and comment)
      • Comment: Sections show an infiltrating tumor comprised entirely of large epithelial cells with abundant eosinophilic and granular cytoplasm, well defined cell borders, enlarged nuclei and prominent nucleoli. The tumor cells are diffusely and strongly positive for AR and AMACR; they are negative for ER, PR and HER2. The combined cytomorphologic and immunohistochemical findings are consistent with a carcinoma with apocrine differentiation.
    Differential diagnosis
    • Invasive breast carcinoma, NST with focal apocrine differentiation:
      • Focal apocrine differentiation may be present in up to 60% of invasive ductal carcinoma, NST (J Clin Pathol 2007;60:1313)
      • Carcinoma with apocrine differentiation must have > 90% apocrine morphology
    • Invasive breast carcinoma, NST with oncocytic carcinoma pattern:
      • Tumor cells with abundant eosinophilic and granular cytoplasm with well defined cell borders and round, centrally located nuclei with prominent nucleoli
      • Positive for mitochondrial immunostains
      • Mostly positive for ER
      • Often display chromosomal gains of 11q13.1-q13.2 and 19p13
    • Granular cell tumor:
      • Infiltrating sheets, clusters or trabeculae of large, round to polygonal tumor cells with abundant, eosinophilic and granular cytoplasm and central, small and hyperchromatic nuclei
      • Positive for S100 and CD68; negative for keratins
    • Apocrine DCIS:
      • DCIS with apocrine cytology
      • Myoepithelial cell layer present
    • Apocrine adenosis / atypical apocrine adenosis:
      • Lobulocentric proliferation with apocrine cytology and distortion by stromal fibrosis / sclerosis
      • Myoepithelial cell layer present
    • Histiocytic proliferations:
      • Pale to foamy cytoplasm without nuclear atypia
      • CD68 positive
    Board review style question #1

    A 3.3 cm irregular mass was detected within the right breast of a 78 year old woman. Lumpectomy showed a tumor with histology as represented in the image throughout the tumor. What biomarker profile is concordant with this subtype of breast cancer?

    1. AR+, ER+, PR-, HER2-
    2. AR+, ER+, PR+, HER2-
    3. AR+, ER-, PR-, HER2-
    4. AR-, ER-, PR-, HER2-
    Board review style answer #1
    C. AR+, ER-, PR-, HER2-. Carcinomas with apocrine differentiation are comprised of tumor cells with apocrine cytology: large atypical epithelial cells with abundant eosinophilic and granular cytoplasm, enlarged nuclei and prominent nucleoli. Apocrine morphology must be present in > 90% of the tumor. Carcinomas with apocrine differentiation are usually triple negative (ER-, PR- and HER2-); however, HER2 amplification / overexpression has been reported in 30 to 60% of tumors. Carcinomas with apocrine differentiation demonstrate diffuse and strong AR immunoreactivity. It should be noted that focal apocrine morphology and AR immunoreactivity may be seen in up to 60% of invasive breast carcinomas, NST. Additionally, AMACR, a marker of apocrine differentiation, is positive in the majority of carcinomas with apocrine differentiation.

    Comment Here

    Reference: Apocrine carcinoma

    Apocrine adenoma
    Definition / general
    • Adenoma with apocrine cytology throughout
    • Rare adenoma that is:
      • Composed exclusively of benign apocrine cells (homogeneous)
      • Sharply demarcated from surrounding breast tissue
      • Contains only epithelial proliferative elements
      • Has minimal supportive stroma
    Essential features
    • Nodular mass composed exclusively of apocrine cells without cytological atypia
    • Sharply demarcated from adjacent breast tissue and has minimal stromal component if any
    • Considered benign, excision is curative
    • Cytological atypia, necrosis and invasive features suggest atypical hyperplasia / apocrine malignancy (DCIS / invasive), recommend careful evaluation of the entire lesion
    Terminology
    • Pure breast adenomas with apocrine differentiation were first described by Hertel et al. in 1976 (Cancer 1976;37:2891)
    Etiology
    • May represent nodular sclerosing adenosis with apocrine differentiation
    Clinical features
    • Can present as a mass
    Prognostic factors
    • Generally accepted to be benign, however, the number of cases reported is not sufficient to determine the level of risk associated (J Clin Pathol 2007;60:1313)
    Case reports
    Treatment
    • Complete excision is curative
    Microscopic (histologic) description
    • By definition, composed exclusively of benign apocrine cells (homogeneous), sharply demarcated from surrounding breast tissue, containing only epithelial proliferative elements, with minimal supportive stroma
    • Localized nodular focus of tubular, papillary and cystic apocrine metaplasia; benign glands have abundant granular eosinophilic cytoplasm, apical luminal blebs and decapitation secretion
    • May contain calcifications (Pathology 2001;33:149)
    Cytology description
    • Can be cuboidal or flattened, two distinct types seen:
      • Cytoplasm granular and strongly eosinophilic, supranuclear vacuole containing yellow brown pigment (rich in iron / hemosiderin)
        • Globoid and pale nuclei with 1 - 2 prominent nucleoli (nuclei may become hyperchromatic in flattened epithelium as in tension apocrine cysts)
      • Cytoplasm distinctly foamy with small vacuoles that may coalesce and show lipofuscin pigment in cytoplasm
        • Central nuclei with 1 - 2 prominent nucleoli
    • Recommend caution to exclude apocrine DCIS or invasive apocrine carcinoma if necrosis / atypia / mitosis present
    Positive stains
    • PAS (with diastase), EMA, CK8 / 18, AR (androgen receptor), GCDFP15, GCDFP24 (apolipoprotein D), GCDFP44 (zinc alpha2 glycoprotein)
    Negative stains
    Differential diagnosis

    Apocrine adenosis / atypical apocrine adenosis
    Definition / general
    • Defined as the presence of apocrine cytology in a recognizable terminal duct lobular unit (TDLU) associated with sclerosing adenosis (J Clin Pathol 2007;60:1313)
    • Preserved 2 cell layer (inner epithelial and outer myoepithelial cells) in which the epithelial layer has prominent apocrine features
    Essential features
    • Lobulocentric proliferation with distortion by stromal fibrosis / sclerosis and apocrine cytology
    Terminology
    • Recommended by WHO:
      • Apocrine adenosis
    • Not recommended by WHO:
      • Sclerosing adenosis with apocrine metaplasia
      • Nodular adenosis with apocrine metaplasia
    • Terminology apocrine adenosis has also been used to refer to adenomyoepithelial adenosis, a different lesion
    • Atypical apocrine adenosis (AAA): rare, defined as ≥ threefold variation in nuclear size
    • Previous (historical) names for AAA:
      • Atypical apocrine hyperplasia
      • Atypical apocrine metaplasia
    ICD coding
    • ICD-10:
      • N60.3 - fibrosclerosis of breast
      • N60.8 - other benign mammary dysplasias
    • ICD-11:
      • GB20.Y - other specified benign breast disease
      • GB20.Z - benign breast disease, unspecified
    Epidemiology
    Sites
    • Terminal duct lobular unit; otherwise, no specific location within the breast
    Pathophysiology
    • Loss of heterozygosity in several loci (1p, 3, 11q, 16 and 17q)
    • Accumulation of losses and gains if progression through apocrine DCIS and apocrine carcinoma
    Clinical features
    • Considered a part of the spectrum of fibrocystic changes
    • Often an incidental finding or detected by screening
    • Can present as a palpable mass if nodular adenosis / adenosis tumor
    Diagnosis
    • Histologic examination of tissue with or without immunohistochemistry
    Radiology description
    • Variable depending on the size / extent of breast involvement
    • If focal, may not be visualized (i.e. incidental finding on histologic examination)
    • Amorphous or pleomorphic clustered microcalcifications; architectural distortion or circumscribed to spiculated mass on mammogram
    • Irregular, noncircumscribed mass on ultrasound is associated with upgrade to malignancy on resection (J Ultrasound Med 2020;39:1517, Breast J 2017;23:569)
    Prognostic factors
    • Increased risk of cancer of 1.5 - 2 times, as seen with proliferative fibrocystic changes
    • Atypical apocrine adenosis (AAA): conflicting evidence of increased cancer risk
      • Study of 37 patients showed no increased risk for carcinoma (Arch Pathol Lab Med 2012;136:179)
      • Study of 37 patients showed the relative risk of developing carcinoma was 5.5 (95% CI 1.9 - 16), with age association - all patients developing cancer were > 60 years at diagnosis of AAA (Cancer 1996;77:2529)
      • Upgrade risk: surgical resection showed carcinoma only in patients with coexisting carcinoma present on core biopsy (N = 10) in a study of 41 patients with apocrine adenosis (N = 29) and atypical apocrine adenosis (N = 12) (Ann Diagn Pathol 2016;24:4)
    Case reports
    • 37 year old woman with atypical apocrine adenosis diagnosed via fine needle aspiration (Acta Cytol 2002;46:369)
    • 54 year old woman with atypical apocrine adenosis diagnosed via incidental discovery on coronary angiogram CT (Cureus 2020;15:e8624)
    • 66 year old woman with nodular atypical apocrine adenosis with monoclonality via restriction fragment length polymorphism (RFLP) (Histopathology 2001;38:221)
    Treatment
    • Presence of apocrine adenosis alone in a core biopsy does not require surgical excision
    • Coexisting atypia typically prompts surgical consultation, although excision is controversial based on limited prognostic data
    Gross description
    • Variable depending on extent of involvement and calcifications
    • May be indistinguishable from surrounding breast tissue
    • Multinodular, ill defined cuts with increased resistance due to fibrosis
    • Gritty due to frequent calcifications but no chalky yellow white foci or streaks as seen in fat necrosis
    • Circumscribed to ill defined white, fibrotic mass if nodular adenosis / adenosis tumor
    Microscopic (histologic) description
    • Cells with apocrine metaplasia have abundant eosinophilic cytoplasm with bright eosinophilic granules that are PAS positive
    • Apocrine cells have round nuclei and may show nuclear pleomorphism (≥ threefold in atypical apocrine adenosis); a central, round, eosinophilic nucleolus is generally seen
    • Rare multinucleation may be observed and is not considered atypical
    • Rare to no mitosis
    • No necrosis
    Microscopic (histologic) images

    Contributed by Julie M. Jorns, M.D.
    Distortion, apocrine features

    Distortion, apocrine features

    Apocrine features

    Apocrine features

    Distortion, cytologic atypia

    Distortion, cytologic atypia

    Cytologic atypia

    Cytologic atypia

    Cytology description
    • Often highly cellular
    • Cells have apocrine metaplasia with prominent nucleoli and pleomorphism, possibly resembling carcinoma but minimal hyperchromasia
    • Naked nuclei are present (Diagn Cytopathol 2007;35:296)
    Cytology images

    Contributed by Volodymyr Shponka, M.D.
    Fine needle aspiration

    Fine needle aspiration

    Negative stains
    Electron microscopy description
    • Distinct basal lamina present
    Sample pathology report
    • Left breast, core biopsy:
      • Fibrocystic changes, including apocrine adenosis with microcalcifications
    Differential diagnosis
    • Apocrine carcinoma:
      • Infiltrative growth
      • Associated desmoplasia
      • Lack of myoepithelium
    • Cancerization of lobule by apocrine DCIS:
      • Architectural atypia: expansile glandular involvement by atypical, monotonous cells
      • Other architectural patterns of DCIS are typically also present (e.g. comedo, cribriform, etc.)
      • Mitosis and necrosis frequently identified
    • Microglandular adenosis:
      • Glands are smaller, more regular
      • Lacks myoepithelial cells
    • Tubular adenosis:
      • Haphazard proliferation of elongated tubules
    Board review style question #1


    The above pictured lesion was from a breast biopsy for mammographically detected calcifications. The area of concern was successfully biopsied. What are the next steps in patient care?

    1. Excision
    2. Magnetic resonance imaging (MRI)
    3. Neoadjuvant chemotherapy
    4. Observation
    5. Rebiopsy
    Board review style answer #1
    D. Observation. The pictured lesion is apocrine adenosis with microcalcifications. There is no significant atypia present; thus, this biopsy is benign and concordant with imaging findings and the patient can be observed with routine screening. There is no need to rebiopsy, excise, do additional imaging or initiate any therapies.

    Comment Here

    Reference: Apocrine adenosis
    Board review style question #2
    What feature is present in apocrine ductal carcinoma in situ (DCIS) but is lacking in atypical apocrine adenosis (AAA)?

    1. Calcifications
    2. Cytologic atypia
    3. Epithelial expansion
    4. Myoepithelium
    5. Prominent nucleoli
    Board review style answer #2
    C. Epithelial expansion. Atypical apocrine adenosis (AAA) and apocrine ductal carcinoma in situ (DCIS) share features of apocrine cytology, including enlarged nuclei with prominent nucleoli and eosinophilic cytoplasm, as well as significant cytologic atypia and an intact myoepithelial cell layer. Both may also have calcifications; however, apocrine DCIS is differentiated from AAA by architectural atypia or epithelial expansion by clonal, monotonous cells.

    Comment Here

    Reference: Apocrine adenosis

    Apocrine metaplasia
    Definition / general
    Essential features
    • Cells with abundant eosinophilic cytoplasm and apical snouts
    • Nuclei are round with variable size, vesicular chromatin with prominent nucleoli
    • Supranuclear vacuoles or eosinophilic granules may be present
    Epidemiology
    Sites
    • Breast parenchyma ducts and lobules
    Etiology
    Clinical features
    • Grossly palpable cysts can be lined by apocrine epithelium but there are no specific clinical features per se due to the apocrine metaplasia itself
    • Can be concurrent with both benign and malignant lesions (J Clin Pathol 2007;60:1313)
    • Apocrine cysts and hyperplasia with apocrine metaplasia were more common in the breasts of American women in New York than in Japanese women in Tokyo (Hoda: Rosen's Breast Pathology, 4th Edition, 2014)
    Diagnosis
    • Histologic examination of tissue removed
    Radiology description
    • Mammographic features are nonspecific, although an enlarging lobular or micro lobulated mass may be noted
    • Heterogeneous microcalcification clusters may sometimes represent apocrine metaplasia
    • These findings can lead to needle core biopsy for definitive diagnosis (AJR Am J Roentgenol 2003;180:795)
    Prognostic factors
    • No relationship or minimal impact on the development of mammary carcinoma (Hoda: Rosen's Breast Pathology, 4th Edition, 2014)
    • Florida apocrine metaplasia with atypia may coexist with apocrine carcinoma, although most patients with short term follow up remained well
    Case reports
    Treatment
    • Specific treatment is not indicated for proliferative lesions with apocrine metaplasia
    • Surgical excision of apocrine cysts is not indicated unless there is bloody fluid, the cysts reform or there is atypia in the cytologic specimen
    • Atypical apocrine metaplasia warrants a clinical evaluation comparable to that of women with other atypical proliferative lesions, yet the precancerous significance of this abnormality remains uncertain
    Gross description
    Microscopic (histologic) description
    • Cells are enlarged with abundant eosinophilic cytoplasm and apical snouts
    • Supranuclear vacuoles or eosinophilic granules may be present
    • Nuclei are round with variable size, vesicular chromatin with prominent nucleoli
    • Can be a single or multiple layer epithelium; even papillary configurations can be seen (papillary apocrine change)
    • Apocrine cells can show a degree of nuclear variability, even in simple epithelium; this is partly a product of cross cutting of a large nucleus and partly an intrinsic property of apocrine epithelium
    • Moderate degrees of nuclear variation and atypia are ignored unless there is necrosis or significant mitotic activity (J Clin Pathol 2007;60:1313)
    • Cytologic atypia tends to be more severe in sclerosing adenosis
    • Highly complex architecture with cytologic atypia should be categorized as atypical ductal hyperplasia or ductal carcinoma in situ
    Microscopic (histologic) images

    Contributed by Carlos C. Diez Freire, M.D.

    Dilated eosinophilic glands

    Papillary projections


    Abundant cytoplasm

    Early apocrine metaplasia and p63

    Cytology description
    • Nuclei of benign apocrine cells are eccentrically located with slight variation in size; the nuclear membrane is smooth with fine granular dispersed chromatin and a single prominent nucleolus
    • Cytoplasm is abundant and dense with distinct cell borders with diffuse finely granular cytoplasm that stains pink / blue in PAP stains and purple / gray in Romanowsky stains
    • Atypical apocrine cells can include nuclear enlargement, pleomorphism, prominent nucleoli and cellular dyshesion
    • Features more concerning for carcinoma include crowded cell clusters, more pleomorphism, irregular nuclear borders, dark clumpy chromatin and increased nuclear to cytoplasmic ratio and necrosis
    • Be cautious in a diagnosis of malignancy when atypical apocrine cells occur together with benign cells or if the lesion is clinically or mammographically benign (Demay: The Art & Science of Cytopathology, 2nd Edition, 2011)
    Positive stains
    Negative stains
    Electron microscopy description
    • Abundant endoplasmic reticulum, numerous mitochondria, intermediate filaments and secretory vesicles in the cytoplasm
    Molecular / cytogenetics description
    Sample pathology report
    • Left breast, 2 o’clock, 5 cm from nipple, ultrasound guided core biopsy:
      • Papillary apocrine metaplasia and duct dilation
    Differential diagnosis
    Board review style question #1
    Apocrine metaplasia of the breast is characterized by

    1. Dilation of terminal ducts with periductal fibrosis and chronic inflammation
    2. Epithelial cells with abundant granular eosinophilic cytoplasm and prominent nucleoli
    3. Monotonous population of histiocytes with foamy cytoplasm distending the ducts
    4. Severely atypical and pleomorphic cells of mesenchymal origin
    Board review style answer #1
    B. Apocrine metaplasia of the breast is characterized by enlarged epithelial cells with abundant granular eosinophilic cytoplasm that can show apical snouting, eccentric nuclei and prominent nucleoli.

    Comment Here

    Reference: Apocrine metaplasia of breast
    Board review style question #2


    The histologic findings in the above breast images are an example of

    1. Apocrine metaplasia
    2. Intraductal papilloma
    3. Mammary duct ectasia
    4. Papillary ductal carcinoma in situ
    Board review style answer #2
    A. Apocrine metaplasia. Cells are enlarged with abundant eosinophilic cytoplasm and apical snouts. Supranuclear vacuoles or eosinophilic granules may be present. Nuclei are round with variable size, vesicular chromatin with prominent nucleoli. Can be a single or multiple layer epithelium, even papillary configurations can be seen (papillary apocrine change).

    Comment Here

    Reference: Apocrine metaplasia of breast

    Atezolizumab
    Definition / general
    • A humanized IgG1 monoclonal antibody blocking programmed death ligand 1 (PDL1)
    • Developed by Genentech, Inc.
    • Synonyms: MPDL3280A, RG7446
    Trade name
    • Tecentriq®
    Indications
    • Urothelial carcinoma locally advanced or metastatic disease fulfilling 1 of the following criteria
      • Not eligible for cisplatin based chemotherapy and tumor expresses PDL1 (PDL1 stained tumor infiltrating immune cells covering ≥ 5% of the tumor area) - FDA approved 2017
      • Not eligible for platinum based chemotherapy with any PDL1 staining status - FDA approved 2017 (Oncologist 2019;24:563)
      • Disease progression with platinum based chemotherapy - FDA approved 2016
    • Non small cell lung cancer (NSCLC) fulfilling one of the following criteria
      • First line treatment for metastatic NSCLC with high PDL1 expression (PDL1 staining ≥ 50% tumor cells or immune cells ≥ 10%) and no EGFR or ALK gene alterations - FDA approved 2020
      • In combination with chemotherapy (paclitaxel protein bound; nab-paclitaxel and carboplatin) for metastatic nonsquamous NSCLC with no EGFR or ALK gene alterations - FDA approved 2019
      • In combination with bevacizumab, paclitaxel and carboplatin for metastatic nonsquamous NSCLC with no EGFR or ALK gene alterations - FDA approved 2018
      • Metastatic NSCLC with EGFR or ALK gene alterations and disease progression when treated with platinum based chemotherapy - FDA approved 2016 (Lancet 2017;389:255)
    • Triple negative breast cancer locally advanced or metastatic with PDL1 expression of immune cells ≥ 1% of tumor area - FDA approved 2019 (N Engl J Med 2018;379:2108)
    • Small cell lung cancer advanced stage in combination with carboplatin and etoposide - FDA approved 2019 (N Engl J Med 2018;379:2220)
    • Hepatocellular carcinoma in combination with bevacizumab - FDA approved 2020 (N Engl J Med 2020;382:1894)
    • Melanoma unresectable or metastatic with BRAF V600 mutation in combination with cobimetinib and vemurafenib - FDA approved 2020 (Nat Med 2019;25:929)
    • Reference: Drugs.com: Tecentriq FDA Approval History [Accessed 18 December 2020]
    Pathophysiology
    • PD1 receptor expressed on activated T cells, B cells, macrophages and NK cells (Nat Rev Immunol 2018;18:153)
    • PDL1 expressed on antigen presenting cells and many types of cancer cells (Nature 2016;534:402)
    • Binding of PD1 to PDL1 or B7H1 can suppress T cell proliferation, cytokine production and cytolytic activity
    • Aberrant expression of PDL1 on cancer cells through interaction with PD1 receptor blocks antitumor immunity and escape immune surveillance (Nature 2016;534:402)
    • Atezolizumab, a PDL1 inhibitor, blocks the binding of PD1 with PDL1, resulting in prevention of inhibition of T cell immune response and reactivation of T cell mediated antitumor cytolytic activity (Clin Cancer Res 2017;23:1886)
    Diagrams / tables

    Images hosted on other servers:
    Atezolizumab blocks PDL1 on tumor cells

    Atezolizumab blocks PDL1 on tumor cells

    PDL1 inhibits T cell dependent cytotoxicity through a STAT3 / caspase-7–dependent signaling pathway in cancer cells

    Inhibition of PDL1 through STAT3 / caspase 7 signaling

    Crystal structures of the interaction of PDL1 with atezolizumab antigen binding fragment

    (a,c) Crystals of PDL1 with atezolizumab

    Clinical information
    • Immune mediated diseases associated with treatment with atezolizumab
    • Pneumonitis: withhold or discontinue based on severity (Chest 2017;152:271)
    • Hepatitis: monitor liver function, withhold or discontinue
    • Colitis: withhold or discontinue based on severity (JAMA Oncol 2018;4:1721)
    • Endocrinopathies (JAMA Oncol 2018;4:173):
      • Hypophysitis: withhold based on severity of hypophysitis
      • Hyperthyroidism: monitor thyroid function
      • Adrenal insufficiency: withhold based on severity
      • Type 1 diabetes mellitus: withhold based on severity
    • Infections: withhold for severe infection (Respir Med 2020;161:105853)
    • Infusion related reactions: slow the rate of infusion or discontinue
    • Embryo fetal toxicity: advise females of reproductive potential
    • Pharmacokinetics (Clin Pharmacol Ther 2017;102:305):
      • Dose range: 1 - 20 mg/kg, including a dose of 1,200 mg every 3 weeks
      • Clearance: 0.20 L/day
      • Volume of distribution: 6.91 L
      • Terminal half life: 27 days
      • Steady state: achieved after 6 - 9 weeks (2 - 3 cycles)
    • Tecentriq treatment costs approximately $13,860 per month (Drugs.com: What Is the Cost of Tecentriq? [Accessed 17 December 2020])
    Uses by pathologists
    • VENTANA PDL1 SP142 Assay (Roche Diagnostics): FDA approved companion diagnostic to atezolizumab; not substitutable with other PDL1 assays (22C3, SP263) (Appl Immunohistochem Mol Morphol 2019;27:92)
    • Tumor cell (TC) staining: membranous, linear, partial to complete circumferential staining pattern at any intensity (Mod Pathol 2019;32:929, Arch Pathol Lab Med 2018;142:982)
    • Immune cell (IC), including lymphocyte, neutrophil and macrophage staining: punctate, incomplete, membranous staining pattern at any intensity; exclude intravascular immune cells and cells in necrotic area (Mod Pathol 2019;32:929, Arch Pathol Lab Med 2018;142:982)
    • Total tumor area: the area covered by tumor cells with intratumoral and peritumoral stroma
    • Urothelial carcinoma: evaluate immune cells only and disregard tumor cells
      • Positive: presence of staining at any intensity in immune cells ≥ 5% of tumor area
      • Negative: absence of any staining in immune cells < 5% of tumor area (see PDL1 SP142) (Am J Surg Pathol 2018;42:1059)
    • NSCLC: evaluate tumor cells and then evaluate immune cells (if tumor cells negative)
      • Positive: presence of staining at any intensity in tumor cells ≥ 50%
      • Negative: staining at any intensity in tumor cells < 50% (proceed to immune cell count)
      • Positive: presence of staining at any intensity in immune cells ≥ 10% of tumor area
      • Negative: staining at any intensity in immune cells < 10% of tumor area (Histopathology 2018;72:449)
    • Triple negative breast carcinoma: evaluate immune cells only and disregard tumor cells
      • Positive: presence of staining at any intensity in immune cells ≥ 1% of tumor area
      • Negative: staining at any intensity in immune cells < 1% of tumor area (see PDL1 SP142)
    Side effects
    • Tecentriq as a single agent (reported in > 20% of patients)
    • Tecentriq in combination with bevacizumab, paclitaxel and carboplatin (reported in > 20% of patients)
      • Arthralgia, asthenia, alopecia, constipation, decreased appetite, diarrhea, hypertension, nausea, peripheral neuropathy (N Engl J Med 2018;378:2288)
    • Tecentriq in combination with paclitaxel protein bound (reported in > 20% of patients)
      • Alopecia, anemia, constipation, cough, decreased appetite, diarrhea, fatigue, headache, nausea, neutropenia, peripheral neuropathies, vomiting (N Engl J Med 2018;379:2108)
    • Others: hyponatremia, pneumonia, hyperkalemia, thrombocytopenia, death
    • Reference: FDA: Tecentriq (atezolizumab) [Accessed 18 December 2020]
    Drug administration
    • For metastatic urothelial carcinoma:
      • Tecentriq 1,200 mg IV over 60 min for 3 weeks
    • For treatment of nonsquamous non small cell lung cancer:
      • Tecentriq 1,200 mg IV over 60 minutes, followed by bevacizumab, paclitaxel and carboplatin every 3 weeks for a maximum of 4 - 6 cycles
      • Following completion of chemotherapy, Tecentriq 1,200 mg IV, followed by bevacizumab every 3 weeks
    • For metastatic triple negative breast cancer:
      • Tecentriq 840 mg IV over 60 minutes, followed by 100 mg/m² paclitaxel
      • Every 28 days, Tecentriq is administered on days 1 and 15 and paclitaxel is administered on days 1, 8 and 15
    • Reference: PDR: Atezolizumab - Drug Summary [Accessed 17 December 2020]
    Molecular theory
    • PDL1 expressed on tumor cells, through binding to PD1, inhibits T cell immune response
    • PDL1 may suppress IFN mediated cytotoxicity through a STAT3 / caspase 7 dependent signaling pathway (Sci Adv 2020;6:2712)
    • The RMLDVEKC motif of PDL1 is essential to hinder interferon toxicity, while the DTSSK motif blocks this function (Sci Adv 2020;6:2712)
    • PDL1 protects tumor cells from T cell dependent cytotoxicity
    • Crystal structures of interaction of PDL1 with atezolizumab antigen binding fragment through the immunoglobulin superfamily V set domain of PDL1 (Sci Rep 2017;7:5532)
    Board review style question #1
    Which of the following proteins is blocked by atezolizumab?

    1. PD1
    2. PDE1
    3. PDGF
    4. PDL1
    Board review style answer #1
    D. PDL1. Atezolizumab is a humanized IgG1 monoclonal antibody that blocks programmed death ligand 1 (PDL1). Answers A - C are incorrect because PD1, PDGF and PDE1 are not blocked by atezolizumab.

    Comment Here

    Reference: Atezolizumab
    Board review style question #2
    Which of the following is true about the PDL1 SP142 assay?

    1. Evaluates percentage of tumor cells stained positive for PDL1 and disregards tumor infiltrating immune cells in non-small cell lung cancer
    2. Evaluates percentage of tumor cells stained positive for PDL1 and evaluates tumor infiltrating immune cells if tumor cells are negative in non-small cell lung cancer
    3. Evaluates percentage of tumor cells stained positive for PDL1 in triple negative breast carcinoma
    4. Evaluates percentage of tumor cells stained positive for PDL1 in urothelial carcinoma
    Board review style answer #2
    B. Evaluates percentage of tumor cells stained positive for PDL1 and evaluates tumor infiltrating immune cells if tumor cells are negative in non-small cell lung cancer. Answers C and D are incorrect because PDL1 SP142 evaluates immune cells only and disregards tumor cells in both urothelial carcinoma and triple negative breast carcinoma. Answer A is incorrect because in non-small cell lung cancer, PDL1 SP142 evaluates tumor cells and then evaluates immune cells if tumor cells are negative.

    Comment Here

    Reference: Atezolizumab

    Atypical ductal hyperplasia
    Definition / general
    • Intraductal clonal epithelial cell proliferation with similar histologic features to (but insufficient involvement or volume for the diagnosis of) low grade ductal carcinoma in situ (DCIS)
    Essential features
    • Intraductal clonal epithelial cell proliferation with cytologic and morphologic features similar to low grade ductal carcinoma in situ
    • Differentiated from low grade ductal carcinoma in situ by size (≤ 2 mm) or space occupied by clonal cells (≤ 2 spaces or only portion of involved space)
    • Rigid bridges, polarization around lumen and estrogen receptor (ER) (diffuse, strong) positivity
    Terminology
    • B3 lesion (uncertain malignant potential)
    • Atypical intraductal hyperplasia (AIDH)
    • Ductal intraepithelial neoplasia (DIN) 1B
    • Mammary intraepithelial neoplasia (MIN)
    • Atypical hyperplasia is not recommended by WHO unless ductal versus lobular differentiation cannot be discerned
    ICD coding
    • ICD-10:
      • N60.8 - other benign mammary dysplasias
      • N60.9 - unspecified benign mammary dysplasia
    • ICD-11: GB20.Y - other specified benign breast disease
    Epidemiology
    • F > M (rare incidental finding in gynecomastia)
    • Commonly presents in fourth decade
    • 5 - 20% of breast biopsies (StatPearls: Atypical Breast Hyperplasia [Accessed 19 November 2019])
    • Reduction of postmenopausal hormonal therapy linked to decreased rates (Cancer Epidemiol Biomarkers Prev 2009;18:2822)
    • 4 - 5x risk of developing ductal carcinoma in situ within 5 years (Cancer Prev Res (Phila) 2014;7:211)
    • Absolute risk of breast cancer is approximately 1% per year for at least 25 years, with a mean latency period of 8 - 12 years after initial diagnosis
    • Recent radiological pathological concordance series have shown that biopsy diagnosed atypical ductal hyperplasia (ADH) has an upgrade rate of 10 - 20% to DCIS or invasive carcinoma
    Sites
    Pathophysiology
    • Low grade model of progression from normal breast / benign proliferative breast disease to atypical ductal hyperplasia (Histopathology 2010;57:171)
    Etiology
    Radiology description
    • No specific diagnostic features differentiate atypia from malignant lesions (AJR Am J Roentgenol 2014;202:1389)
    • Incidentally discovered on routine screening mammograms (3.3 per 10,000)
    • Often identified in breast biopsies performed as a result of calcifications on imaging (Acad Radiol 2019;26:893)
    • May be seen as nonmass enhancement on breast MRI
    Prognostic factors
    Case reports
    Treatment
    • Most biopsy proven lesions undergo excision with some variation, as directed by
      • Imaging findings (e.g., small versus larger extent of calcifications)
      • Biopsy type / size: excision rate 60% if diagnosed by vacuum assisted biopsy
      • Patient / surgeon preference
    • Tamoxifen reduced the risk of subsequent cancer from 21 to 7.5% (Breast Cancer Res 2018;20:39)
    Gross description
    • Usually difficult to detect grossly
    • Associated calcifications may feel gritty
    Microscopic (histologic) description
    • Histologic sections show well defined, monomorphic cells with evenly spaced, small rounded nuclei
    • Variable patterns:
      • Solid
      • Cribriform (polarization around lumina)
      • Micropapillary (club shaped)
    • Unicentric or multicentric
    • Uniform thickness of rigid bridges or arcades (differential feature from usual ductal hyperplasia)
    • Intraluminal microcalcifications are often seen
    • Rarely can be mixed with usual ductal hyperplasia or other proliferations within the same lobule
    • Differentiation from low grade ductal carcinoma in situ is controversial and shows interobserver variability with low agreement rate on biopsy (40 - 60%), especially because the majority of the distinction is based on the following (generally accepted) quantitative features:
    • Conservative approach is suggested on biopsy; definitive diagnosis should be upon surgical excision if the lesion is borderline between atypical ductal hyperplasia and low grade ductal carcinoma in situ
    • If nuclear atypia warrants a diagnosis of intermediate or high grade ductal carcinoma in situ, atypical ductal hyperplasia should not be diagnosed based on size
      • Intermediate or high grade ductal carcinoma in situ should be diagnosed regardless of size or extent
    Microscopic (histologic) images

    Contributed by Julie M. Jorns, M.D.

    Cribriform

    Small foci with cribriforming and monotony

    Polarization


    Bridges with uniform thickness

    Cribriform and solid

    Solid growth


    Micropapillary

    Short papillae

    Partial gland involvement

    Virtual slides

    Images hosted on other servers:

    Atypical ductal
    hyperplasia with
    microcalcifications

    Positive stains
    Negative stains
    Sample pathology report
    • Left breast, 1 o'clock, stereotactic core biopsy:
      • Focal (0.1 cm) atypical ductal hyperplasia (ADH) with microcalcifications
    Differential diagnosis
    • Usual ductal hyperplasia:
      • Nonclonal cell proliferation
      • Cells are not uniform; they show different sizes and shapes with overlapping / streaming architecture
      • Cells not oriented around lumen
      • Bridges stretch with central attenuation and lumina are flattened / slit-like
      • Positive mosaic or patchwork pattern of CK5/6 staining
    • Low grade ductal carcinoma in situ:
      • Same cytologic and morphologic characteristics of atypical ductal hyperplasia but ≥ 2 mm in size, involving the entire lobular space and extending to > 2 lobules
    Board review style question #1

    A breast biopsy was performed for calcifications. The above image is most characteristic of which pattern of atypical ductal hyperplasia?

    1. Cribriform
    2. Flat
    3. Micropapillary
    4. Solid
    Board review style answer #1
    C. Micropapillary

    While specifying the pattern of atypical ductal hyperplasia is not clinically important or necessary in the pathology report, recognizing key features and patterns is important in making the diagnosis. Atypical ductal hyperplasia is defined by having some but not all features of low grade ductal carcinoma in situ, with low volume or size (≤ 2 spaces or including only a portion of the involved space or ≤ 2 mm in overall size). Common patterns of atypical ductal hyperplasia include cribriform, micropapillary and solid. Flat patterns are more in keeping with the diagnosis of flat epithelial atypia. The example above has short, club shaped micropapillae, characteristic of the micropapillary pattern of atypical ductal hyperplasia.

    Comment Here

    Reference: Atypical ductal hyperplasia
    Board review style question #2

    What is the most useful immunohistochemical stain in the differential diagnosis of usual ductal hyperplasia and atypical ductal hyperplasia?

    1. Calponin
    2. CK5/6
    3. CK7
    4. CK20
    5. p63
    Board review style answer #2
    B. CK5/6

    Usual ductal hyperplasia and atypical ductal hyperplasia of the breast may sometimes be difficult to distinguish morphologically. Thus, use of CK5/6 immunohistochemical staining may be helpful in the differential diagnosis, as usual ductal hyperplasia is positive (in a mosaic or patchwork pattern) and atypical ductal hyperplasia is negative for CK5/6. Both usual ductal hyperplasia and atypical ductal hyperplasia are typically positive for CK7 and negative for CK20 (breast ductal epithelium immunophenotype). Myoepithelial markers p63 and calponin show positive myoepithelial staining in both usual ductal hyperplasia and atypical ductal hyperplasia.

    Comment Here

    Reference: Atypical ductal hyperplasia

    Atypical lobular hyperplasia
    Definition / general
    • Clonal proliferation of discohesive epithelial cells arising in terminal duct lobular units
    • Similar histologic features to lobular carcinoma in situ (LCIS) but of insufficient quantity
    Essential features
    • Clonal proliferation of discohesive cells identical to lobular carcinoma in situ but in smaller quantity
    • Typical dysfunction / loss of E-cadherin
    • Generally considered incidental finding (clinically / radiologically occult)
    • 4 - 5 times increased risk of (bilateral risk but higher risk ipsilateral) invasive breast carcinoma compared with general population
    • Does not necessarily require excision if isolated finding
    Terminology
    • Lobular intraepithelial neoplasia 1 (LIN1)
    ICD coding
    • ICD-10: N60.8 - other benign mammary dysplasias
    • ICD-10: N60.9 - unspecified benign mammary dysplasia
    Epidemiology
    Sites
    • Bilateral breasts with no specific location
    Pathophysiology and etiology
    Clinical features
    • Frequently associated with columnar cell lesions and flat epithelial atypia; less commonly associated with low grade invasive carcinomas which may have mammographically detectable calcifications, density or mass targeted on biopsy (Am J Surg Pathol 1998;22:1521Am J Surg Pathol 2007;31:417)
    • Does not form a palpable mass
    • 19% develop invasive cancer at mean 15 years after diagnosis (4 - 5 times usual risk), 42% are special subtypes with good prognosis (Cancer 2006;107:1227)
    Diagnosis
    • Core biopsy
    • Incidental after mammography, ultrasound or MRI
    Radiology description
    • Classically considered to be an incidental finding with no radiologic correlates
    • Calcifications, if associated with atypical lobular hyperplasia in biopsy or non-mass enhancement on MRI, may be considered concordant after biopsy is read (Breast Dis 2016;36:5)
    Radiology images

    Contributed by Azadeh Khayyat, M.D. and Julie M. Jorns, M.D. (Case #533)
    Mammogram

    Mammogram


    Prognostic factors
    Case reports
    Treatment
    • Controversial; some surgeons excise, others get lifelong screening / follow up screening intervals
    • Growing evidence to avoid excision for well sampled lesions (Ann Surg Oncol 2017;24:2848)
    • Selective estrogen receptor modulators (SERMs) and aromatase inhibitors (AIs) can lower risk of subsequent invasive carcinoma in patients with atypical hyperplasias (atypical lobular hyperplasia or atypical ductal hyperplasia not separated) (J Natl Compr Canc Netw 2018;16:1391)
    Gross description
    • No specific gross findings
    • May be associated with calcifications but typically due to other coexisting lesions
    Microscopic (histologic) description
    • Solid, discohesive proliferation of cells that are monomorphic, small, have pale pink cytoplasm, uniform oval nuclei and indistinct nuclei
    • Some cells have plasmacytoid or signet ring appearance (intracytoplasmic vacuoles)
    • Do not form arcades, lumens or papillary projections
    • Criteria of Page et al. (Schnitt: Biopsy Interpretation of the Breast, 3rd Edition, 2017):
      • Distends 50% or more acini within a lobule (so resembles lobular carcinoma in situ) but not uniformly present throughout entire lobule OR
      • Involves all acini in a terminal duct lobular unit (TDLU) and so resembles lobular carcinoma in situ but does not distend the acini (i.e. caliber of the involved acini is similar to that of uninvolved acini)
    • Can involve ducts; alteration occurs around the duct as outpouchings producing a cloverleaf pattern
    • Lacks intracytoplasmic mucin
    • May be no / minimal inflammatory response
    Microscopic (histologic) images

    Contributed by Eric Statz, M.D., Julie M. Jorns, M.D. and Azadeh Khayyat, M.D. (Case #533)

    ALH in fibroadenoma

    ALH in fat

    Plasmacytoid ALH

    Duct with ALH


    Lobule expanded by ALH

    ALH full lobule

    Loss of E-cadherin

    Loss of E-cadherin
 ALH / LCIS


    Ductal carcinoma in situ (DCIS), solid type, with microcalcifications, confined to a fibroadenoma Ductal carcinoma in situ (DCIS), solid type, with microcalcifications, confined to a fibroadenoma Ductal carcinoma in situ (DCIS), solid type, with microcalcifications, confined to a fibroadenoma Ductal carcinoma in situ (DCIS), solid type, with microcalcifications, confined to a fibroadenoma Ductal carcinoma in situ (DCIS), solid type, with microcalcifications, confined to a fibroadenoma

    Ductal carcinoma in situ (DCIS), solid type, with microcalcifications, confined to a fibroadenoma; lobular neoplasia (atypical lobular hyperplasia / lobular carcinoma in situ), with involvement of a fibroadenoma

    Cytology description
    • Loosely cohesive cell clusters composed of uniform cells with occasional intracytoplasmic lumina, minimal nuclear atypia but frequent eccentric nuclei
    Positive stains
    Negative stains
    Electron microscopy description
    • Intracytoplasmic lumina, microvilli with secretory droplets; basement membrane and myoepithelial cells are present
    Molecular / cytogenetics description
    Sample pathology report
    • Left breast, 12 o'clock, core biopsy:
      • Atypical lobular neoplasia (ALH) (see comment)
      • Comment: An immunohistochemical stain for E-cadherin is negative in the atypical focus, supporting the diagnosis of ALH. Controls are appropriate.
    Differential diagnosis
    Board review style question #1

      A lesion was biopsied with immunohistochemistry pictured above. It is best diagnosed as

    1. Atypical apocrine adenosis (AAA)
    2. Atypical ductal hyperplasia (ADH)
    3. Atypical lobular hyperplasia (ALH)
    4. Flat epithelial atypia (FEA)
    Board review style answer #1
    C. Atypical lobular hyperplasia (ALH)

    Explanation: The above images show an in situ proliferation of small to moderate sized, monotonous epithelial cells with mild expansion of involved glands. There are a few cells with intracytoplasmic vacuoles as well as variably prominent cell borders. Thus, the differential diagnosis includes atypical ductal hyperplasia (ADH) with a solid growth pattern and atypical lobular hyperplasia (ALH). Atypical apocrine adenosis (AAA) has larger cells with apocrine cytology including large, round nuclei with prominent nucleoli and abundant eosinophilic cytoplasmic. Flat epithelial atypia (FEA) has dilated glands with atypical low columnar to cuboidal epithelium with high nuclear cytoplasmic ratio and lack of orientation toward lumina. Characteristic features of AAA and FEA are lacking in the pictured lesion. In the differential of ALH and solid ADH immunohistochemistry can be very helpful, with ALH showing lack or attenuation of E-cadherin and cytoplasmic staining via p120 as shown above. ADH in contrast has strong, diffuse E-cadherin staining and membranous staining via p120.

    Comment Here

    Reference: Atypical lobular hyperplasia (ALH)
    Board review style question #2
      Atypical lobular hyperplasia (ALH) seen on core biopsy most frequently correlates to the following feature via breast imaging

    1. Asymmetry
    2. Calcifications
    3. Mass
    4. No specific finding
    Board review style answer #2
    D. No specific finding

    Explanation: Atypical lobular hyperplasia (ALH) is typically not identified via standard imaging modalities. Occasionally ALH has calcifications, although if a core biopsy that has ALH has an indication of calcifications, these are more frequently identified in other lesions in the biopsy such as fibrocystic change or other atypias (e.g. flat epithelial atypia or atypical ductal hyperplasia). ALH is not typically identified as a mass or asymmetry on breast imaging.

    Comment Here

    Reference: Atypical lobular hyperplasia (ALH)

    Atypical vascular proliferation
    Definition / general
    • Atypical vascular proliferations on irradiated skin, often small and multiple
    Essential features
    • Superficial dermal based growth of narrow vascular spaces
    • Develops after external radiation for breast carcinoma
    • Typically involves the skin overlying breast
    • 2 subtypes: lymphatic type and vascular type
    • Differentiated from angiosarcoma by lack of multilayering, mitosis, marked cytologic atypia, blood lakes and absence of extension into subcutaneous tissue
    Terminology
    • Atypical vascular lesion term coined by Fineberg and Rosen in 1994 (Am J Clin Pathol 1994;102:757)
    • Acceptable terminology: atypical vascular proliferation
    ICD coding
    • ICD-O: 9126/0 - atypical vascular lesion
    • ICD-11: XH8KN7 - atypical vascular lesion
    Epidemiology
    Sites
    • Skin of breast or anterior chest wall exposed to previous radiation
    Pathophysiology
    • Secondary to irradiation but precise pathogenesis is unknown
    Etiology
    Clinical features
    Diagnosis
    • Requires correlation of clinical history and physical examination with microscopic features
    • Complete excision of the lesion is required for definitive diagnosis
    Prognostic factors
    Case reports
    • 41 and 61 year old women with small vascular lesions of the breast diagnosed by MRI guided vacuum assisted biopsy (Radiol Case Rep 2022;17:2492)
    • 43 year old woman presented with 29 atypical vascular lesions following breast conserving surgery and radiation (Cutis 2019;103:E22)
    • 49 year old woman presented with postradiation atypical vascular proliferation mimicking angiosarcoma 8 months following breast conserving therapy for breast carcinoma (J Clin Aesthet Dermatol 2011;4:47)
    Treatment
    Clinical images

    Contributed by Anna Purdy, M.S.N.
    Upper outer quadrant erythema

    Upper outer quadrant erythema

    Axillary papules

    Axillary papules



    Images hosted on other servers:
    Missing Image

    Postmastectomy

    Missing Image

    Left breast is erythematous with sclerotic tissue changes

    Missing Image

    Papules and nodules around the mastectomy scar

    Gross description
    • ≥ 1 circumscribed papules, bluish purple nodules, small vesicles or erythematous plaques, median of 0.5 cm
    • Tends to be well demarcated, symmetric and wedge shaped (Semin Diagn Pathol 2017;34:410)
    Microscopic (histologic) description
    • Relatively well circumscribed lesion with a somewhat wedge shaped outline, limited to the dermis and not extending into subcutis
    • 2 histologic subtypes with overlapping histologic features (Am J Surg Pathol 2008;32:943):
      • Lymphatic subtype:
        • More common subtype
        • Irregularly shaped, thin walled vascular channels with a branching and anastomosing growth
        • Lymphatic valve-like structures may be evident
        • Vessels lined by single layer of endothelial cells without atypia
        • Hobnailing and nuclear hyperchromasia may be seen
      • Vascular subtype:
        • Thin capillary channels lined by flattened to hobnail endothelium, lacking lobular growth pattern
    • Individual lesions may show composite features of both subtypes
    Microscopic (histologic) images

    Contributed by Indu Agarwal, M.D., Sharlene See, M.D. and Julie M. Jorns, M.D.
    Wedge shaped outline

    Wedge shaped outline

    Flat to plump endothelial cells

    Flat to plump endothelial cells

    Vascular channels

    Vascular channels

    Excision specimen

    Excision specimen


    Papule

    Papule

    Skin punch biopsy

    Skin punch biopsy

    Skin punch biopsy

    Skin punch biopsy (CD31)

    Skin punch biopsy

    Skin punch biopsy (c-MYC)

    Positive stains
    Negative stains
    Molecular / cytogenetics description
    • Unknown
    Sample pathology report
    • Left breast skin, punch biopsy:
      • Atypical vascular lesion (see comment)
      • Comment: The lesion is comprised of a vascular proliferation dissecting the collagen in the superficial dermis. There is no significant nuclear pleomorphism or mitotic activity. An immunostain performed for c-MYC is negative in the lesion. Morphologic and immunohistochemical findings support the diagnosis.
    Differential diagnosis
    • Angiosarcoma, well differentiated:
      • Features of angiosarcoma which are lacking in atypical vascular lesions:
        • Large (median size is 7.5 cm) infiltrative lesion with destruction of adnexa and extension into subcutaneous tissues, multilayering of endothelial cells, prominent nucleoli, mitoses, blood lakes and solid areas
        • Unlike secondary angiosarcomas, atypical vascular lesions lack MYC amplification (by FISH) and MYC overexpression by immunohistochemistry
      • However, angiosarcoma may be part of morphologic continuum with atypical vascular lesions and may need additional biopsies to distinguish these entities (J Am Acad Dermatol 2007;57:126)
    • Hemangioma of skin:
      • No history of radiation
      • Lesions consist of dilated blood vessels in skin, filled with blood
      • Vessels have rounded contours and are nonanastomosing
    • Benign lymphangioendothelioma (Am J Surg Pathol 2000;24:1047):
      • Rare lesion with no history of radiation
      • Head and neck are more common sites, followed by limbs and trunk
      • Skin lesions may be macular or papular and pigmented
      • Anastomosing dilated vessels lined by flattened endothelium are present in superficial dermis; spaces may be empty or contain blood
      • Cytologic atypia and mitoses are absent
    Board review style question #1

    A brownish papular lesion of skin on the breast is excised. Histopathologic examination shows that it is comprised of a wedge shaped, relatively well circumscribed vascular proliferation, dissecting the collagen in the superficial dermis. No significant nuclear pleomorphism or mitotic activity is noted. What is the most important information you need to make the diagnosis?

    1. Family history of cancer
    2. History of cancers of other sites
    3. History of chemotherapy
    4. History of radiation to the site
    Board review style answer #1
    D. History of radiation to the site. Atypical vascular lesions only occur in previously irradiated areas of the breast. The histologic description matches that of an atypical vascular lesion but to make a confident diagnosis, prior history of irradiation and histologic examination of the entire lesion is essential.

    Comment Here

    Reference: Atypical vascular proliferation

    Axillary lymph nodes
    Definition / general
    • Extent of axillary lymph node involvement is important for breast cancer stage, clinical management and prognosis
    Essential features
    • Evaluation of axillary lymph nodes for pathologic staging of invasive breast cancer requires sentinel lymph node biopsy with or without axillary lymph node dissection
    • Axillary lymph node dissection should contain 10 or more lymph nodes
    • Primary goal when histologically evaluating axillary lymph nodes is to identify all macrometastases
    Terminology
    • Macrometastases: > 2.0 mm
    • Micrometastases: > 0.2 mm to 2.0 mm, or > 200 cells in a single histologic cross section
    • Isolated tumor cells (ITCs): ≤ 0.2 mm or ≤ 200 cells in a single histologic cross section
    ICD coding
    • ICD-10: C77.3 - axillary sentinel lymph node
      • Effective in 2019 edition of ICD-10-CM, starting October 1, 2018
    • ICD-11: 2D60.3 - malignant neoplasm metastasis in axillary lymph nodes
    • CPT codes (surgical pathology) for axillary lymph node:
      • 88305 - axillary tail / axilla, lymph node biopsy
      • 88307 - lymph node, sentinel
      • 88307 - axillary tail / axilla, lymph nodes regional resection
    Clinical features
    • Axillary lymph node dissection typically removes level I and level II lymph nodes
    • Lymph node dissection of level I and level II of the axilla should contain 10 or more lymph nodes
    • If level III lymph nodes are surgically removed, the surgeon should separately identify them for staging purposes
    • Regional lymph nodes:
      • Axillary (ipsilateral), subdivided as follows:
        • Level I (low axilla): lateral to the lateral border of pectoralis minor muscle
        • Level II (mid axilla): between medial and lateral borders of pectoralis minor muscle, plus the interpectoral (Rotter) lymph nodes
        • Level III (apical axilla or infraclavicular nodes): medial to the medial margin of the pectoralis minor muscle and inferior to the clavicle
      • Internal mammary (ipsilateral): lymph nodes in the intercostal spaces along the edge of the sternum in the endothoracic fascia
      • Supraclavicular (ipsilateral)
    • Intramammary nodes are present within breast tissue and are most commonly found in the upper outer quadrant and may rarely be sentinel lymph nodes; these nodes are included with axillary nodes for AJCC N classification
    • Side effects of axillary nodal dissection include lymphedema, shoulder restriction, numbness, weakness and pain syndromes (Cancer J 2008;14:216)
    Diagnosis
    • Lymph nodes are thinly sliced at 2 mm intervals (to detect all macrometastasis)
      • All lymph node tissue is submitted for histologic evaluation
      • If grossly positive then a representative section (including any possible areas of extranodal extension) is adequate
    • Additional evaluation of SLNs to detect occult metastasis, including multiple levels and cytokeratin immunohistochemistry, offers no clinical benefit (N Engl J Med 2011;364:412)
    • Intraoperative evaluation of sentinel lymph nodes can be avoided in most patients who are clinically node negative and are undergoing breast conserving surgery (Arch Pathol Lab Med 2016;140:830)
    Radiology description
    • Axillary ultrasound (US) is the primary modality for evaluating axillary lymph node status prior to surgery
      • Cortical thickening, hilar effacement and nonhilar cortical blood flow are more important than size criteria in the identification of metastases (Radiographics 2013;33:1589)
      • Preoperative axillary US and fine needle aspiration (FNA) cytology are routine at many breast units, with a sensitivity of 56% (confidence interval: 47 - 64%) and specificity of 90% (84 - 93%) for US alone, and 76% (61 - 87%) and 100% (65 - 100%) when combined with FNA cytology (J BUON 2011;16:454)
    • Standard breast MRI is comparable to dedicated axillary US in breast cancer patients; subsequent axillary US can be done on those with suspicious nodal findings (Eur J Radiol 2016;85:2288)
    • CT and radionuclide imaging play a lesser role in imaging the axilla (Radiographics 2013;33:1589)
    Prognostic factors
    • Presence of axillary lymph node metastases is the most important prognostic factor for disease free and overall survival in the absence of distant metastasis and is important for determining treatment for breast cancer
    • Macroscopic extranodal extension presents clinically as palpable matted lymph nodes and is a risk factor for local regional recurrence
    • Microscopic extranodal extension measuring > 2 mm is associated with greater axillary nodal burden (Ann Surg Oncol 2014;21:2897)
    • Tumor deposits and intravascular tumor emboli in extranodal axillary fat are associated with greater axillary nodal burden (Ann Surg Oncol 2020;27:3585)
    • Presence of treatment effect (fibrosis with or without macrophages or hemosiderin) after neoadjuvant chemotherapy is an important prognostic factor
    Case reports
    Treatment
    • Sentinel lymph node biopsy (SLNB) at time of breast surgery for early stage breast cancer with clinically negative axillary nodes (J Clin Oncol 2017;35:561):
      • If no sentinel lymph node (SLN) metastasis, then no axillary lymph node dissection (ALND)
      • If 1 or 2 SLNs with metastases are present and patient is undergoing breast conserving surgery with whole breast radiotherapy, then no ALND
      • If SLN metastases are present and patient is undergoing mastectomy, then ALND should be offered
    • ALND can be omitted in patients who undergo mastectomy for early stage clinically node negative breast cancer with 1 or 2 SLN metastases, if treated with axillary radiotherapy (Pract Radiat Oncol 2016;6:e219)
    • ALND can be avoided in patients with early stage breast cancer and only micrometastasis in the SLNs (Lancet Oncol 2013;14:297)
    • SLNB may be considered optional for elderly patients, particularly if axillary staging will not affect selection of adjuvant systemic or radiotherapy
    • ALND at time of breast surgery, if clinically palpable lymph nodes or ≥ 3 suspicious nodes by imaging with axillary nodal metastasis confirmed on core biopsy or FNA; SLNB may be considered if clinically node negative after neoadjuvant therapy
    Frozen section description
    • Axillary SLN are identified by the surgeon by determining uptake of radioisotope technetium 99 sulfur colloid, methylene blue dye or both
    • Intraoperative frozen section or imprint cytology may be performed on the SLN to determine need for axillary lymph node dissection (World J Surg Oncol 2008;6:69, Eur J Surg Oncol 2009;35:16)
    • Frozen section (FS) had an overall mean sensitivity of 73% (macrometastasis: 94%; micrometastasis 40%) and mean specificity of 100% (meta-analysis of 47 studies, Cancer 2011;117:250)
    • SLN FS may be safely omitted in patients who meet ACOSOG Z11 criteria (Clin Breast Cancer 2018;18:276, J Clin Pathol 2016;146:57)
    • False negative rate of intraoperative frozen section, post neoadjuvant chemotherapy, was 5.4% in a single institution study of 711 cases (Am J Surg Pathol 2019;43:1377)
    • Atypical cases on FS are usually negative on permanent sections (Mod Pathol 2005;18:58)
    • In most cases, if metastases are present, the SLN will be involved; in rare cases, only nonsentinel nodes contain metastases
    • Metastasis to nonsentinel lymph nodes can occur if the true SLN is completely replaced by tumor (and therefore is not detected by radioactive tracer or dye), if there is unusual lymphatic drainage or if there is failure of the technique to identify the node
    Microscopic (histologic) description
    • Size of a metastatic deposit consists of a contiguous focus of tumor cells, including any intervening desmoplasia (for post neoadjuvant treatment, do not include intervening fibrous tumor bed for deposit size)
    • Tumor cells of metastatic lobular carcinoma are often dispersed and not contiguous
      • 200 cell cutoff helps to determine if dispersed cells in a lymph node should be classified as isolated tumor cells
    • Extranodal extension (ENE) is included in the size of the metastatic deposit
    • ENE is defined as carcinoma invading into fat outside of a lymph node, either via invasion through the capsule of the node or invasion into fat outside the node at the hilum
      • Invasion into fat located within the contours of the lymph node is not ENE
      • Carcinoma located within lymphatics or blood vessels in axillary fat is not ENE
    • ENE is reported as present or not present; If present, the amount of ENE can be reported as ≤ 2 mm or > 2 mm or the measurement of the largest dimension
    • Nodular tumor deposits in axillary fat without evidence of residual lymph node are counted as positive axillary lymph nodes
    • pN: see staging topic
    • (sn) modifier for pN stage is used when 5 or fewer lymph nodes are sampled / evaluated
    • Intramammary lymph nodes are included in the axillary lymph node count
    • Metastasis to an ipsilateral level III axillary lymph node is staged as pN3a
    • Metastasis to a contralateral axillary lymph node is staged as pM1
    Microscopic (histologic) images

    Contributed by Mary Ann G. Sanders, M.D., Ph.D.
    Benign squamous epithelial inclusion

    Benign squamous epithelial inclusion

    Benign glandular epithelial inclusion

    Benign glandular epithelial inclusion

    PAX8

    PAX8

    Treatment effect

    Treatment effect


    Pankeratin

    Pankeratin

    Matted lymph nodes

    Matted lymph nodes

    Extranodal extension

    Extranodal extension

    Cytology description
    • FNA smears of positive axillary lymph node are characterized by crowded, disorganized groups of cells with enlarged nuclei, nuclear pleomorphism, irregular nuclear membranes and intracytoplasmic vacuoles with or without mucin
    • Necrosis or mucin in the background and single intact malignant cells may be present
    • Imprint cytology had an overall sensitivity of 63% (macrometastasis: 81%, micrometastasis: 22%) and specificity of 99% (meta analysis of 31 studies) (Br J Surg 2005;92:1068)
    • Factors that can affect the outcome of FNA are the size of the metastasis in the lymph node, experience of the person performing image guided FNA and evaluation of the sample for adequacy by a pathologist (at the time of FNA)
    Positive stains
    Molecular / cytogenetics description
    • Some studies have reported usefulness of molecular assays, usually reverse transcriptase polymerase chain reaction (RT-PCR) (J Clin Oncol 2008;26:3338)
      • False positive and false negative results can occur with RT-PCR
      • Significance of a positive RT-PCR result for a histologically negative lymph node is unknown (J Clin Oncol 2008;26:3338)
      • Current AJCC cancer staging manual (8th edition) includes positive molecular findings by RT-PCR; no isolated tumor cell clusters detected as pN0 (mol+)
    Sample pathology report
    • Right axillary lymph nodes, dissection:
      • Metastatic carcinoma is present in 3 of 10 lymph nodes, two macrometastases and one micrometastasis, (3/10)
      • The largest metastatic deposit measures 8 mm
      • Extranodal extension is present (measuring 3 mm)
    Differential diagnosis
    Board review style question #1

    The gross image provided is a right axillary sentinel lymph node received from the OR of a patient with stage 1 breast cancer undergoing lumpectomy and sentinel lymph node biopsy. Recommendations provided by the College of American Pathologists (CAP) for evaluating axillary lymph nodes in breast cancer include which of the following?

    1. Evaluate 3 H&E levels for sentinel lymph nodes
    2. Submit all lymph node tissue if grossly positive
    3. Thinly slice each lymph node at 2 mm intervals
    4. Tumor deposits in axillary tissue are counted as extranodal extension
    5. Use cytokeratin immunohistochemistry on all negative lymph nodes
    Board review style answer #1
    C. Thinly slice each lymph node at 2 mm intervals. Per the CAP guidelines, sampling of lymph nodes must be adequate to detect all macrometastasis (> 2 mm deposit). Therefore, all lymph nodes must be thinly sliced at 2 mm intervals. All tissue from grossly negative lymph nodes must be submitted for histologic evaluation. For grossly positive lymph nodes, a representative section including areas suspicious for extranodal extension is adequate. Additional evaluation, including multiple H&E levels and cytokeratin immunohistochemistry, increases detection of isolated tumor cells and micrometastasis (occult metastasis), however this provides minimal clinical benefit. Therefore, the CAP recommends evaluating 1 H&E level from the surface of the tissue block when lymph node tissue has been sliced at 2 mm intervals. Tumor deposits in axillary tissue are counted as positive axillary lymph nodes (lymph node replaced by tumor).

    Comment Here

    Reference: Axillary lymph nodes
    Board review style question #2
    A 50 year old woman underwent core needle biopsy of a 10 mm right breast mass detected on ultrasound and diagnosed as invasive ductal carcinoma, grade 2, that was estrogen receptor positive, progesterone receptor positive and HER2 negative. Right axillary ultrasound showed normal appearing lymph nodes. Breast MRI showed no additional lesions. There was no evidence of distant metastasis. What is the appropriate surgical management for this patient?

    1. Double mastectomy
    2. Lumpectomy
    3. Lumpectomy with axillary lymph node dissection
    4. Lumpectomy with sentinel lymph node biopsy
    5. No surgery
    Board review style answer #2
    D. Lumpectomy with sentinel lymph node biopsy. This patient has early stage breast cancer with clinically negative axillary lymph nodes; therefore, she will need sentinel lymph node biopsy to stage the axilla and to determine further axillary management and adjuvant systemic therapy and radiotherapy.

    Comment Here

    Reference: Axillary lymph nodes

    Benign breast implant related changes
    Definition / general
    • Saline and silicone breast implants are suitable for use in augmentation mammaplasty and breast reconstruction following mastectomy
    • Soft tissue surrounding breast implants can display various benign histological findings over time, including dense fibrosis (capsule), chronic inflammation, granulomatous reaction, calcification and synovial metaplasia (Aesthetic Plast Surg 2010;34:481)
    • Implant associated reactions vary based on the implant composition, duration and mechanical forces along the capsule and if violated / ruptured (Biomaterials 2003;24:1101)
    Essential features
    • Capsules can form around breast implants over time, especially silicone type implants
    • Chronic movement and gliding of the implant in its cavity produces benign changes along the implant capsule interface with associated clinical and histological findings (Acta Cytol 2014;58:511)
    • Implant related changes include dense fibrosis, chronic inflammation / granulomatous reaction, calcification and synovial metaplasia
    • Leaking or rupture of silicone implant leads to silicone granuloma, which consists of amorphous / crystalline material with a foreign body giant cell reaction
    • Synovial metaplasia can be noted, which consists of epithelioid cells lining the inner capsular surface with a well developed reticulin network histologically indistinguishable from normal joint synovium
    Terminology
    • Synovial metaplasia is also referred to as synovial-like reaction or synovial-like metaplasia
    Epidemiology
    • Occurs in smooth and textured silicone surfaced breast implants as well as in saline filled and polyurethane coated implants (Plast Reconstr Surg 1997;100:1558, Biomaterials 2003;24:1101)
    • Risk of implant related changes increases with prolonged implant duration
    • Textured surface breast implants with a duration of 0 - 5 years exhibit the highest incidence of synovial metaplasia (Biomaterials 2003;24:1101)
    • Risk of rupture increases with duration, with a minimum of 15% estimated to rupture between the third and tenth year after implantation in a cohort followed by MRI (Arch Surg 2003;138:801)
    Sites
    • Breast implant capsule interface
    Pathophysiology
    • Inflammatory changes
      • Immune recognition of breast implants as foreign results in inflammatory response, leading to the formation of scar (capsule) around the implant, which may contract and cause the implant to feel hard or change shape
    • Thick, fibrous capsule is formed as a host reaction to foreign material (implant) or possibly as a form of repair
    • Breast implant related changes can lead to other complications, such as implant rupture or displacement and breast implant associated anaplastic large cell lymphoma (BIA-ALCL) in the context of textured implants (Plast Reconstr Surg 2017;140:645)
    • Synovial metaplasia
      • Persistent movement of the implant induces a local cellular response along the inner capsular surface that evolves into an organized synovial-like structure resembling normal joint synovium (Plast Reconstr Surg 1995;96:1747)
    Clinical features
    • Usually delayed onset of symptoms
      • Pain with or without swelling
      • Distortion / capsular contracture
      • Excessive firmness of breast
      • Infection
    • Reference: Plast Reconstr Surg 1997;100:1558
    Diagnosis
    Radiology description
    • Capsular thickening with or without peri-implant fluid on breast ultrasound / MRI (Clin Imaging 2020;59:144)
    • MRI has high accuracy in diagnosing silicone implant rupture (Eur Radiol 2014;24:1167)
    • Ruptured implant: asymmetric circumferential increased density can be seen on mammography
    Radiology images

    Contributed by Di Ai, M.D., Ph.D., Kristen E. Muller, D.O. and Samantha A. Stephen, D.O. (Case #530)
    Chest Xray

    Chest Xray

    Case reports
    • 25 year old woman with peri-implant fluid collection mimicking implant rupture / silicone leak, 1 year after bilateral breast augmentation (J Med Case Rep 2008;2:277)
    • 53 year old woman undergoing surgery for capsular contracture of textured, silicone breast implants (Diagn Cytopathol 2018;46:769)
    • 55 year old HIV+ transgender woman with sudden onset breast skin thickening and swelling, 9 years after bilateral saline breast implant mammoplasty (Clin Imaging 2020;59:144)
    • 66 year old woman with a history of invasive lobular carcinoma and bilateral mastectomy with breast implant reconstruction developed rib pain (Case of the month #530)
    Treatment
    Gross description
    • Silicon implant rupture
      • Implant ruptures may be intracapsular or extracapsular
        • An intracapsular rupture occurs when the shell of the implant ruptures but the formed fibrous capsule remains intact; silicone does not extravasate freely
        • An extracapsular rupture occurs when the formed capsule ruptures, which can lead to a change in the implant contour
    Gross images

    Contributed by Di Ai, M.D., Ph.D.
    Ruptured silicone breast implants

    Ruptured silicone breast implants

    Microscopic (histologic) description
    • Thickened fibrosis (capsule) with varying degrees of chronic inflammation, fat necrosis, foreign body giant cell reaction, hemosiderin laden macrophages, calcifications
    • Synovial metaplasia
      • Consists of a layer that is 1 - 7 cells thick, lined by epithelioid cells arranged in a palisading pattern and embedded in a well developed reticulin network
      • Cells are perpendicularly oriented with round to oval fibroblast-like nuclei, inconspicuous nucleoli and abundant eosinophilic cytoplasm
      • No atypia
    • Silicone granuloma
      • Multiple cystic spaces and vacuoles partially filled with silicone granules and surrounded by fibrosis with accompanying local foreign body giant cell reaction, foamy histiocytes and lymphocytes (J Clin Pathol 1998;51:493)
      • Usually secondary to extracapsular implant rupture
      • Silicone gel leakage may be seen even in the absence of implant rupture
    • References: Biomaterials 2003;24:1101, Aesthetic Plast Surg 2010;34:481
    Microscopic (histologic) images

    Contributed by Di Ai, M.D., Ph.D., Julie M. Jorns, M.D., Kristen E. Muller, D.O. and Samantha A. Stephen, D.O. (Case #530)
    Synovial metaplasia Synovial metaplasia

    Synovial metaplasia

    Synovial metaplasia Synovial metaplasia Synovial metaplasia

    Synovial metaplasia


    Synovial metaplasia CD68

    Synovial metaplasia CD68

    Synovial metaplasia vimentin

    Synovial metaplasia vimentin

    Synovial metaplasia AE1 / AE3

    Synovial metaplasia AE1 / AE3

    Silicone leakage

    Silicone leakage

    Silicone granuloma

    Silicone granuloma

    Silicone leakage

    Silicone leakage


    Virtual slides

    Images hosted on other servers:

    Breast, silicone granuloma

    Positive stains
    Sample pathology report
    • Right breast tissue, capsulectomy:
      • Dense fibroconnective tissue with chronic inflammation, fat necrosis, foreign body giant cells and amorphous foreign materials (see comment)
      • Comment: Foreign materials are amorphous, transparent and nonpolarized, consistent with silicone leakage.

    • Right breast capsule, excision:
      • Benign fibrous capsule with chronic (lymphohistiocytic) inflammation and giant cell reaction to silicone
    Differential diagnosis
    • Breast implant associated fibromatosis (J Surg Case Rep 2018;2018:rjy249):
      • Can be locally aggressive, with high rates of recurrence despite adequate excision, while lacking metastatic potential
      • Develop from fibroblasts or myofibroblasts, in association with the capsule surrounding a silicone breast implant
      • Long fascicles of spindle cells with no / minimal cytologic atypia and no mitotic figures
      • Focal prominent stromal collagen
      • Nuclear staining for beta catenin, while negative for CD34 and p63
    • Squamous cell carcinoma (Ann Plast Surg 1992;29:425):
      • Poorly differentiated squamous cell carcinoma is rarely seen
      • The histologic features of lining of squamous epithelium around a breast implant include: pleomorphic and hyperchromatic nuclei, single cell dyskeratosis, atypical mitotic figures and infiltration into the stroma around the capsule
      • Immunostaining positive for cytokeratin
      • Squamous metaplasia is bland and lacks the histologic features mentioned above
    • Implant associated anaplastic large cell lymphoma (Clin Imaging 2020;59:144):
      • Tumor lymphocytes have significant atypia (anaplastic) with horseshoe shaped nuclei, CD30 positive
    • High grade sarcoma:
      • High grade sarcoma, such as liposarcoma and angiosarcoma, has been reported to be associated with breast implant; also, breast implant associated fibrosarcoma was reported in 35% in a rat model (Plast Reconstr Surg Glob Open 2013;1:e11)
      • Histologic features include high grade nuclear atypia, atypical mitotic figures and necrosis
    Additional references
    Board review style question #1

    A 44 year old woman with history of breast implants underwent capsulectomy. Which of the following markers is positive in the lining epithelium of the inner capsular surface (denoted by blue arrows in the image shown above)?

    1. AE1 / AE3
    2. CD3
    3. CD68
    4. EMA
    5. Factor VIII
    Board review style answer #1
    C. CD68. The lining cells exhibit an immunohistochemical profile similar to synovial cells, including positive staining for CD68 and vimentin. Answers A, B, D and E are incorrect because staining for cytokeratins, CD3, EMA and factor VIII are negative.

    Comment Here

    Reference: Benign breast implant related changes
    Board review style question #2
    Which of the following statements is true about acellular dermal matrix?

    1. Comprised of avascular and acellular collagen fibers after implantation
    2. Never clinically palpable
    3. Not commonly used in breast reconstructive surgery
    4. Shows hypercellular stroma with intermixed breast ducts and lobules
    Board review style answer #2
    A. Comprised of avascular and acellular collagen fibers after implantation. The histologic features of acellular matrix vary based on the stage of tissue remodeling. After implantation, the tissue is comprised of acellular and avascular disorganized collagen fibers. Eventually, the acellular matrix undergoes neovascularization with the production of small capillaries and proliferation of fibroblasts around the collagen fibers. Once acellular dermal matrix undergoes tissue remodeling and incorporation, the tissue may resemble mature dense fibrous tissue (see Case #530 for more information).

    Comment Here

    Reference: Benign breast implant related changes

    Biopsy marking devices
    Definition / general
    • Clip placement, indicating the location of a percutaneous breast core biopsy, is utilized in many settings:
      • Correlate lesion location across different imaging modalities (i.e. ultrasound, mammogram, tomosynthesis, MRI)
      • Follow up progression of benign lesions over time
      • Indicate tumor location in patients receiving neoadjuvant chemotherapy
      • Assist surgeons in localizing lesions or extent of disease in nonpalpable lesions, confirming excision of the appropriate region
      • Guide the pathologist to identify the area of interest on gross specimen examination
      • Different clips help distinguish separate lesions
    • Markers are most commonly metallic, in various shapes, generally 2 - 3 mm in size, placed with or without accompanying packing / embedding material such as collagen, hydrogel or starch
    Essential features
    • Biopsy marker placement is an important step in any breast biopsy procedure
    • Clips of varying material and shapes are placed to help distinguish different biopsied sites / lesions
    • Clips may migrate away from the biopsy site or be incorrectly positioned; microscopic features of biopsy site changes must therefore still be identified in excision specimens
    • Accompanying packing material placed with marker clips may make biopsy sites more readily detectable radiographically and often increase homeostasis immediately following the biopsy procedure
    Clinical features
    • Routine biopsy clip placement is associated with a higher rate of negative margins for wire localization excisions (Am J Surg 2005;190:618)
    • Advantages to packing material with marker placement include decreased clip displacement, increased homeostasis, better identification of clip site soon after biopsy procedure
    • Depending on clip design, some markers are delivered into the biopsy cavity through the biopsy probe while others are delivered manually by image guidance
    • Clip migration away from the biopsied site is a well recognized complication and occurs for various reasons (Radiographics 2004;24:147), including breast decompression (aka the accordion effect, J Radiol 2009;90:31) or displacement due to hematoma formation following the biopsy procedure; migration may be reduced in clips placed with an embedding material (Eur Radiol 2016;26:866)
    • FDA reports of allergic reactions to metallic biopsy markers have been reported (FDA: Johnson and Johnson, Ethicon Division Titanium Breast Clip [Accessed 14 December 2017]) but are rare
    • Although markers with collagen embedding material are not placed in people with allergies to beef products, collagen or collagen products, there are no reports of allergic reactions in the current literature
    Radiology images

    Contributed by Emily S. Reisenbichler, M.D.

    Biopsied area with marker placement

    Biopsy clips with localization wires

    M shaped and top hat clips



    Images hosted on other servers:

    Lost biopsy marking clips (arrows)


    Case reports
    Clinical images

    Images hosted on other servers:

    Devices and radiographs (various images)

    Gross images

    Images hosted on other servers:

    Biopsy clips may be much smaller than standard surgical clips

    Microscopic (histologic) description
    • Gel / polymer pellets:
      • Initially hypocellular fibrotic reaction around empty spaces (processing dissolves the polymer), then multinucleated giant cell reaction with eosinophilic material in marker core (Am J Surg Pathol 2005;29:814)
      • May also retain blue / grey material, resembling suture with multinucleated giant cell reaction to material (see microscopic images)
    • Collagen plugs:
      • Eosinophilic, hyalinized and acellular material with lymphocytic and eosinophilic infiltrate that gradually penetrates into the core
      • No prominent multinucleated giant cell reaction
      • May resemble amyloidosis (see microscopic images)
    Microscopic (histologic) images

    Contributed by Emily S. Reisenbichler, M.D.

    Acellular hyalinized collagen plug

    Gel material and central hemorrhage

    Gel material resembling suture material

    Prominent multinucleated giant cell reaction

    Differential diagnosis
    Board review style question #1
    Which of the following is not a use for biopsy marker placement?

    1. Assist the pathologist in gross identification of the targeted lesion in the surgical excision specimen
    2. Identification of a nonpalpable breast lesion (such as calcifications, asymmetry or distortion) for excision
    3. Identification of a palpable lesion in a patient with a metal allergy
    4. To follow the appearance and possible progression of a benign biopsied lesion over time
    5. Tumor localization in a patient undergoing neoadjuvant chemotherapy prior to surgery
    Board review style answer #1
    C. Although there are almost no contraindications for placing a biopsy marker, there are rare reports of allergic reaction to metallic clips. Forgoing clip placement in this situation may be considered, particularly if the lesion is palpable and has not been entirely removed by the biopsy procedure. For most cases, a marker clip is necessary and helpful to surgeons, oncologist and pathologists.

    Comment Here

    Reference: Breast biopsy marking devices

    BRCA associated carcinoma
    Definition / general
    • Group of inherited breast cancers attributed to mutations in the breast cancer associated genes: BRCA1 and BRCA2
    • Associated with increased risk of breast and ovarian cancers in women with BRCA germline mutations
    Essential features
    • Presence of BRCA1 / 2 genes germline mutation
    • Responsible for 25 - 30% of familial breast cancers and ~2% of all breast cancers
    • Differs from sporadic breast carcinoma genotypically and phenotypically
    • Occurs at a younger age (< 50 years) with high predilection for contralateral breast involvement
    • Often well demarcated with pushing margins and medullary-like features
    • Usually high grade (poorly differentiated) and triple negative
    Terminology
    • BRCA1 related breast cancer or BRCA1 germline mutation related breast cancer
    • BRCA1 / 2 associated hereditary breast and ovarian cancer syndrome
    Epidemiology
    Sites
    • Breast, ovarian, pancreatic, prostate and possibly other cancers can be associated with BRCA1 / 2 associated hereditary breast and ovarian cancer syndrome
    Pathophysiology
    Etiology
    Diagrams / tables

    Images hosted on other servers:
    Breast cancer predisposition genes

    Breast cancer predisposition genes

    Clinical features
    Diagnosis
    Radiology description
    Radiology images

    Images hosted on other servers:
    Mammogram and USG findings

    Mammogram and USG findings

    Circumscribed hypoechoic mass with <i>BRCA1</i> mutation

    Circumscribed hypoechoic mass with BRCA1 mutation

    Irregular obscured isodense mass

    Irregular obscured isodense mass

    Hypoechoic, irregular shape, calcifications in <i>BRCA2</i>

    Hypoechoic, irregular shape, calcifications in BRCA2

    Prognostic factors
    • Prognostic significance of BRCA1 / 2 mutational status on breast cancer survival is still debatable (Breast Cancer Res Treat 2010;119:13)
    • Majority of studies have failed to demonstrate a significant overall survival difference between BRCA associated breast cancer and sporadic breast cancer (Breast Cancer Res Treat 2010;119:13)
    • High risk of contralateral breast cancers (10 year risk ranging from 20 to 40%) (Breast Cancer Res Treat 2010;119:13)
    • Metastases to the CNS occur frequently in women with BRCA1 / BRCA2 germline mutations, especially with BRCA2 mutation (Cancer 2020;126:271)
    • Negative factors for overall survival may be infiltration of axillary lymph nodes, negative steroid receptor status and increased size of the primary tumor (Oncol Lett 2019;17:1986)
    Case reports
    Treatment
    • BRCA mutation carriers
    • BRCA mutation associated breast cancer (Onco Targets Ther 2022;15:815)
      • Total mastectomy is preferred over breast conserving surgery
      • Responsive to platinum based therapy
      • PARP inhibitors are effective in adjuvant, neoadjuvant and metastatic settings
    Clinical images

    Images hosted on other servers:
    Mass in <i>BRCA</i> positive man

    Mass in BRCA positive man

    Gross description
    • Usually presents as well circumscribed / well demarcated fleshy mass
    Microscopic (histologic) description
    Microscopic (histologic) images

    Contributed by Kristen E. Muller, D.O. and Chanchal Rana, M.D.
    Well demarcated pushing borders

    Well demarcated pushing borders

    High grade nuclear features

    High grade nuclear features

    Lymphoplasmacytic infiltration

    Lymphoplasmacytic
    infiltration

    Syncytial growth with necrosis

    Syncytial growth with necrosis


    ER negative

    ER negative

    PR negative

    PR negative

    HER2 neu negative

    HER2 neu negative

    Ki67

    Ki67

    p53 expression

    p53 expression

    Cytology description
    • No specific cytology findings
    • Corresponds to the histological type
    Positive stains
    Negative stains
    Molecular / cytogenetics description
    Molecular / cytogenetics images

    Images hosted on other servers:
    <i>BRCA</i> pedigree

    BRCA pedigree

    LGRs <i>BRCA1</i>

    LGRs BRCA1

    LGRs <i>BRCA2</i>

    LGRs BRCA2

    <i>MYC</i> amplification

    MYC amplification


    Sample pathology report
    • Right breast mass, mastectomy:
      • Invasive breast carcinoma of no special type with medullary pattern (see synoptic report)
      • Grade III
      • Triple negative (ER / PR / HER2 neu negative)
      • Comment: In view of the presence of young age, medullary pattern and triple negativity, there is a possibility of BRCA associated breast carcinoma; genetic analysis may be advised if clinically indicated.
    Differential diagnosis
    Board review style question #1

    Which statement about BRCA1 positive breast cancer, with histological features shown in the image above, is true?

    1. Expression of ER, PR and HER2 neu on immunohistochemistry
    2. High probability of the development of contralateral breast cancer in the future
    3. Low chance of association with ovarian cancer
    4. Patient is likely to be > 60 years of age
    Board review style answer #1
    B. High probability of the development of contralateral breast cancer in the future. BRCA associated breast cancer has high rates of contralateral breast cancer. Answer D is incorrect because BRCA associated breast cancer develops at a younger age, usually < 50 years. Answer A is incorrect because BRCA associated breast cancer is mostly triple negative. Answer C is incorrect because patients with BRCA associated breast cancers have a high chance of developing other cancers, especially ovarian cancer in women.

    Comment here

    Reference: BRCA1 associated breast carcinoma
    Board review style question #2
    Which of the following is the most sensitive radiological investigation to diagnose as well as follow up a patient with BRCA1 associated breast cancer?

    1. Dynamic contrast enhanced MRI
    2. Mammography
    3. Next generation sequencing
    4. Ultrasound
    Board review style answer #2
    A. Dynamic contrast enhanced MRI. Dynamic contrast enhanced MRI is the most sensitive screening modality, with a sensitivity of 71 - 94%. Answers B and D are incorrect because ultrasound and mammography are limited by the masking effect of dense breast tissue in young women, demarcated nature and lower incidence of ductal carcinoma in situ BRCA associated breast cancer. Answer C is incorrect because next generation sequencing is a molecular technique.

    Comment here

    Reference: BRCA1 associated breast carcinoma

    Breast implant associated anaplastic large cell lymphoma
    Definition / general
    Essential features
    • Strongly positive CD30 pleomorphic lymphocytes in the peri-implant seroma or capsular tissue
    Terminology
    • Breast implant associated anaplastic large cell lymphoma is recognized as BIA-ALCL
    • Synonymous to seroma associated anaplastic large cell lymphoma
    ICD coding
    • ICD-O: 9715/3 - mucosal associated lymphoid tissue lymphoma (MALT)
    Epidemiology
    Sites
    • Peri-implant seroma or capsular tissue (Ann Oncol 2016;27:306)
    • Locoregional lymph nodes may be involved
    Pathophysiology
    • Although breast implants are associated with an increased risk of developing BIA-ALCL, the pathogenesis is still unknown
    • Almost all documented BIA-ALCL cases have been associated with a textured device
    • Genetic susceptibility may play a role
    • BIA-ALCL is characterized by a triple negative genetic subtype and activation of the JAK-STAT3 pathway (Aesthet Surg J 2019;39:S14)
    • Other contributing factors may include:
    Etiology
    • Theoretically can be caused by active components in manufactured silicone, such as residual vinyl groups
    • Possibly a result of chronic antigenic stimulation (Aesthet Surg J 2016;36:773)
    • It is hypothesized that manufactured silicone contains several components that are potentially biologically active, such as residual vinyl groups, which can access the circulation via lymphatics leading to a foreign body carcinogenesis (Plast Reconstr Surg 2007;120:94S)
    • Another hypothesis is that breast implant associated anaplastic large cell lymphoma results from chronic bacterial antigen stimulation, sustained T cell proliferation and subsequent genetic events in the capsular tissues and the surrounding seromas (Aesthet Surg J 2016;36:773)
    Diagrams / tables

    Images hosted on other servers:

    Anaplastic large cell
    lymphoma cells in
    seroma and within
    fibrous capsule

    Clinical features
    Diagnosis
    • Pleomorphic, neoplastic, strongly positive CD30 lymphocytes present on cytology, lining the breast capsule may show capsular invasion
    Radiology description
    • Ultrasonography can show a fluid collection between the breast implant and the capsule
    • MRI can show effusions, masses or capsular enhancement with ruptured implants
    • T2 weighted axial MRI can show a peri-implant collection around the subpectoral saline implant (Breast J 2019;25:69)
    • PET / CT: fluorodeoxyglucose (18F) avidity on positron emission tomography
    Radiology images

    Contributed by Andrii Puzyrenko, M.D., Ph.D. and Julie Jorns, M.D. (Case #512)
    CT scan

    CT scan

    Prognostic factors
    • 2 pathological subtypes, associated with 2 clinical pictures:
      • Effusion around the implant corresponding to a localized disease confined to the capsule
      • Palpable mass massively invading the capsule and adjacent tissues corresponding to an invasive disease
    • Generally considered an indolent T cell lymphoma but the mass forming variant requires more intensive therapeutic approaches (Ann Oncol 2016;27:306)
    • Median survival rate is 12 years
    • Those associated with tumor cells forming solid masses have adverse prognosis
    Case reports
    Treatment
    • In localized disease, complete capsulectomy without chemotherapy is curative
    • Adjuvant chemotherapy may be required for more invasive disease
    • Brentuximab may be used as second line treatment (Eur J Surg Oncol 2017;43:1393)
    Clinical images

    Images hosted on other servers:

    Clinical presentation / intraoperative appearance

    Gross description
    • Arises in the fibrous capsule rather than the breast parenchyma or skin (JPRAS O 2015;6:1)
    • Usually has fluid around the implant
    Gross images

    Images hosted on other servers:

    Breast capsule

    Microscopic (histologic) description
    • Neoplastic cells line the fibrous capsule
    • Neoplastic lymphoid cells have large malignant anaplastic nuclei, coarse and occasionally fine granular to pale chromatin with prominent nucleoli (Arch Pathol Lab Med 2014;138:842)
    • Tumor cells show eccentric, horseshoe or kidney shaped nuclei referred to as hallmark cells
    • Mitotic figures are conspicuous
    • Cells have abundant cytoplasm
    • Rarely, the malignant cells form a distinct tumor mass (Breast J 2019;25:69)
    • Malignant cells are seen in a background containing fibrinoid material and inflammatory cells
    • When forming a solid mass,
      • Can show multinodular appearance
      • Neoplastic cells are arranged in sheets
      • Can show extensive sclerosis and fibrosis
      • Can show areas of extensive coagulative necrosis
    Microscopic (histologic) images

    Contributed by Dia Kamel, M.D., Ph.D.

    Breast capsule

    Pleomorphic cells


    CD30

    EMA

    ALK1

    CD4




    Contributed by Andrii Puzyrenko, M.D., Ph.D. and Julie Jorns, M.D. (Case #512)

    Capsule

    Aspirated fluid

    Aspirated fluid

    Cytology description
    Cytology images

    Contributed by Dia Kamel, M.D., Ph.D.

    Cell block

    Positive stains
    Negative stains
    • Negative for anaplastic lymphoma kinase 1 (ALK1)
    • Frequent loss of T cell markers
    Flow cytometry description
    • Identification of CD30+ atypical, large cells in peri-implant effusions with relatively increased CD40 expression may help in confirming histological diagnosis (Cytometry B Clin Cytom 2015;88:58)
    Molecular / cytogenetics description
    • Molecular abnormalities are similar to those seen in ALK- systemic anaplastic large cell lymphoma
    • Genomic characterization shows JAK / STAT activation and MYC / TP53 dysregulation
    • Majority of cases show clonal rearrangement of the TR genes
    • Despite molecular similarities, the clinical behavior is different as systemic anaplastic large cell lymphoma has aggressive behavior in comparison to breast implant associated anaplastic large cell lymphoma (Plast Reconstr Surg 2019;143:59S, Aesthet Surg J 2019;39:S14)
    Molecular / cytogenetics images

    Images hosted on other servers:

    2D: PCR studies

    Cytogenetic studies

    Videos

    Update on plastic surgery of anaplastic large cell lymphoma

    Sample pathology report
    • Clinical details: Suspected breast implant associated anaplastic large cell lymphoma
    • Specimen: Right and left breast capsules
    • Macroscopy:
      • Right breast capsule: Capsule weighs 38 g and measures 120 x 39 x 5
      • Left breast capsule: Capsule weighs 35 g and measures 110 x 40 x 5
    • Microscopy:
      • Right breast capsule:
        • Sections of the capsule show hyalinized tissue with focal chronic inflammatory cell infiltrate and foreign body giant cell reaction to the implant material. There are occasional atypical cells of hallmark type, seen within the inner capsule surface and in one focus; the atypical lymphoid cells appear to superficially infiltrate the fibrous capsule. Immunohistochemical stains highlight the atypical lymphoid cells which show positive immunoreactivity to CD3, CD43 and CD30. On the other hand, ALK1 is negative. CD20 decorates the reactive B lymphocytes and CD68 stains macrophages.
        • The features together with the immunohistochemical staining profile are in keeping with anaplastic large cell lymphoma that is breast implant associated and involving the right breast capsule. As the atypical lymphoid cells are seen in the inner surface of the capsule, this is compatible with the effusion variant which carries a better prognosis. However, clinical and pathological correlation and followup are recommended.
      • Left breast capsule:
        • Sections show hyalinized fibrous tissue with focal chronic inflammation and foreign body giant reaction to implant material. There is no evidence of infiltration by atypical lymphoid cells in the examined sections. This is confirmed by negative immunohistochemical staining for CD30.
    • Diagnosis:
      • Right breast capsule: Breast implant associated anaplastic large cell lymphoma
      • Left breast capsule: Chronic inflammation
    • Comment: On verbal communication with the consultant radiologist, it was stated that the implant capsule is not significantly thickened and therefore this is most likely of effusion variant type.
    Differential diagnosis
    Board review style question #1

    The hallmark cell of breast implant associated anaplastic large cell lymphoma stains strongly positive for which of the following?

    1. CD20
    2. CD23
    3. CD30
    4. CD68
    5. S100
    Board review style answer #1
    C. CD30. Breast implant associated anaplastic large cell lymphoma is characterized by abnormal growth of pleomorphic lymphocytes within the peri-implant seroma or capsular tissue. Answers A and D are incorrect because CD20 stains the reactive B lymphocytes and CD68 highlights histiocytic and monocytic cells / macrophages, which aren't the hallmark cells for this condition. Answers B and E are incorrect because CD23 and S100 are not useful for characterizing this condition.

    Comment Here

    Reference: Breast implant associated anaplastic large cell lymphoma
    Board review style question #2
    Which of the following is best for management of patients with early / localized breast implant associated anaplastic large cell lymphoma?

    1. Capsulectomy only
    2. Capsulectomy with radiotherapy and chemotherapy
    3. Chemotherapy only
    4. Radiotherapy and chemotherapy
    5. Radiotherapy only
    Board review style answer #2
    A. Capsulectomy only. In early, localized disease, complete casulectomy without chemotherapy is curative. Answers B - E are incorrect because in early disease, radiotherapy and chemotherapy are not needed. Adjuvant chemotherapy may be required for more invasive disease.

    Comment Here

    Reference: Breast implant associated anaplastic large cell lymphoma

    Classic
    Definition / general
    • Invasive breast carcinoma with loss of cellular adhesion, characteristically arranged in discohesive or single file patterns
    Essential features
    • Special subtype of invasive breast carcinoma characterized by discohesive tumor cells arranged in single files or as individual single cells
    • Shows 16q loss (CDH1 gene located at 16q22.1 encodes E-cadherin, integral in formation of adherens junction responsible for cell adhesion)
    • Loss of E-cadherin expression on immunohistochemistry helpful but not required for diagnosis
    Terminology
    • Invasive lobular carcinoma, classic type
    ICD coding
    • ICD-O: 8520/3 - lobular carcinoma, NOS
    • ICD-11: 2C61.1&XH2XR3 - invasive lobular carcinoma of breast & lobular carcinoma, NOS
    Epidemiology
    Sites
    • Breast
    • Can occur in axilla accessory breast tissue
    Etiology
    • Invasiveness and loss of cellular cohesion due to abnormalities of the adherens complex, formed by cadherins and catenins (Biochim Biophys Acta 2008;1778:660)
    • Lobular carcinoma in situ is a risk factor and nonobligate precursor of invasive lobular carcinoma
    Clinical features
    Diagnosis
    • Mainly histological or radiological
    Radiology description
    • Ultrasound (Radiographics 2009;29:165)
      • Hypoechoic mass with spiculated or ill defined margins and posterior acoustic shadowing
    • Mammography (Radiographics 2009;29:165)
      • Does not consistently present as a mass (44% - 65%)
      • Less commonly manifests as architectural distortion
      • Microcalcification infrequent
    • Magnetic resonance imaging findings similar (Radiographics 2009;29:165)
      • Can aid in identifying multifocal or residual disease
      • Preoperative breast MRI may improve surgical planning (Breast J 2016;22:143)
    Radiology images

    Contributed by Dr. Mark R. Wick

    Mammogram



    Images hosted on other servers:

    Irregular mass (mammography), hypoechoic area (ultrasound)

    Prognostic factors
    • Older age, larger tumor size and metastatic axillary lymph node involvement are independent risk factors for survival and recurrence (Cancer 2008;113:1511)
    • Higher histological grade is associated with poorer outcome (Breast Cancer Res Treat 2008;111:121, Histopathology 2015;66:409)
      • Independently predicts shorter disease specific survival and disease free interval
      • Associated with higher stage and hormone receptor negativity
    • Classic type more favorable than solid, pleomorphic and other types (Cancer 2008;113:1511)
      • Lower number of lymph node metastases and risk of recurrence
    • E-cadherin negative lobular carcinomas have a higher disease specific mortality than E-cadherin positive counterparts (Histopathology 2015;66:409)
    • High Ki67 index associated with risk of distant metastasis (Cancer 2008;113:1511)
    • Better initial outcome but worse long term (> 10 years after diagnosis) survival compared with invasive ductal carcinoma (Eur J Cancer 2008;44:73)
    Case reports
    Treatment
    Clinical images

    Images hosted on other servers:

    Carcinoma en cuirasse

    Gross description
    • Can form a discrete irregular mass (BMJ Case Rep 2016;2016:bcr2016215665)
    • Frequently ill defined borders, due to lack of sclerotic stromal response
    • Not uncommonly grossly unidentifiable or is grossly underestimated, only slightly firm on palpation
    Gross images

    Images hosted on other servers:

    Anal metastasis

    Multiple foci with irregular margins

    Frozen section description
    • Discohesive tumor cells may be difficult to identify on frozen sections, particularly when cellularity is low
    • Single tumor cells can be mistaken as inflammatory cells (histiocytes and lymphocytes) and vice versa
    • Ill defined nature of lesion increases chance of margins sent for frozen section being involved by tumor microscopically
    Frozen section images

    Contributed by Emily S. Reisenbichler, M.D.
    Invasive lobular carcinoma

    Invasive lobular carcinoma

    Microscopic (histologic) description
    • Tumor cells arranged in single files, cords and single cells (Breast Cancer Res 2015;17:12)
      • Can be arranged concentrically around normal ducts, giving a targetoid appearance
    • Tumor cells discohesive, small, monomorphic and lacking marked atypia
      • Round or notched ovoid nuclei, usually grade 1 or 2 nuclear score
      • Scant cytoplasm, occasional with intracytoplasmic lumen
      • Mitosis infrequent
    • Desmoplastic reaction and necrosis uncommon
    • Requires high index of suspicion for metastasis
    • Other (nonclassic) patterns of lobular carcinoma
      • Solid:
        • Sheets or large nests of tumor cells
      • Alveolar:
        • Clusters and aggregates of ≥ 20 cells
      • Tubulolobular carcinoma:
        • Tumor cells arranged in small round tubules mixed with classical lobular carcinoma
      • Pleomorphic:
        • Markedly pleomorphic (nuclear size > 4 times lymphocyte or nuclear pleomorphism equivalent to high grade ductal carcinoma in situ)
        • Higher rate of hormone receptor negativity, HER2 and p53 immunohistochemistry positivity
      • Histiocytoid:
        • Foamy cytoplasm resembling histiocytes
      • Apocrine:
        • Abundant eosinophilic granular cytoplasm
      • Signet ring
    Microscopic (histologic) images

    Contributed by Joshua J.X. Li, M.B.Ch.B., Gary M. Tse, M.B.B.S and Kristen E. Muller, D.O.
    Tumor cells in single file

    Tumor cells in single file

    Low grade nuclei

    Low grade nuclei

    Adjacent in situ component

    Adjacent in situ component

    E-cadherin loss E-cadherin loss

    E-cadherin loss


    Intracytoplasmic vacuoles

    Intracytoplasmic vacuoles

    Luminal A subtype Luminal A subtype Luminal A subtype Luminal A subtype

    Luminal A subtype


    Alveolar pattern

    Alveolar pattern

    Signet ring pattern

    Signet ring pattern

    Solid pattern

    Solid pattern

    Virtual slides

    Images hosted on other servers:

    Core biopsy

    In situ and invasive tumor

    Gastrointestinal metastasis

    Cytology description
    • Cellularity can be low
    • Tumor cells arranged in chains, single files or as single cells
      • Tumor cells arranged in small chains are helpful clues
    • Nuclear atypia mostly mild
      • Nuclei round to oval and eccentric
    • Cytoplasm scanty with a high nuclear / cytoplasmic ratio
      • Occasional intracytoplasmic vacuolations may be seen
    • Cytologic features can resemble mesothelial cells in effusion fluid (Diagn Cytopathol 2012;40:311)
    Cytology images

    Contributed by Joshua J.X. Li, M.B.Ch.B. and Gary M. Tse, M.B.B.S
    Discohesive tumor cells

    Discohesive tumor cells

    Mild atypia

    Mild atypia

    Loose chains of cells

    Loose chains of cells

    Intracytoplasmic vacuolation

    Positive stains
    Negative stains
    Molecular / cytogenetics description
    Sample pathology report
    • Left breast, mastectomy:
      • Invasive lobular carcinoma (see comment)
      • Comment: Sections show breast tissue with malignant cells in a diffuse and discohesive pattern, with some forming single files and small clusters. Targetoid pattern is noted. The tumor cells show mild nuclear pleomorphism with rare mitoses, corresponding to a Nottingham histological grade of I. No lymphovascular permeation is seen. The tumor measures ___cm in maximal dimension. The remaining breast tissue and nipple are unremarkable. All the resection margins are clear, with a minimal clearance of ___cm at the ___ margin. Immunohistochemical staining shows tumor cells are E-cadherin negative. The features are those of an infiltrating lobular carcinoma, classic type.
    Differential diagnosis
    • Primary disease
      • Mixed invasive breast carcinoma and invasive lobular carcinoma:
      • Lobular carcinoma in situ involving sclerosing adenosis:
        • Retained lobular architecture
        • Myoepithelial cells present and can be demonstrated by immunohistochemistry
      • Inflammatory infiltrates and lymphoma:
        • Lacks background lobular carcinoma in situ
        • Lymphoid cells may be more hyperchromatic and have less cytoplasm
        • No intracellular cytoplasmic vacuolation
        • Cytokeratin negative and lymphocyte markers positive
      • Melanoma:
        • Lacks background lobular carcinoma in situ
        • Can display marked nuclear atypia
        • Cytokeratin negative and melanocytic markers positive
      • Myofibroblastoma, epithelioid variant:
        • Well circumscribed borders
        • Lacks background lobular carcinoma in situ
        • Presence of spindle cell component
        • Cytokeratin negative
    • Metastatic disease
    Board review style question #1
    Which of the following germline mutations is associated with an increased risk of developing lobular carcinoma of the breast?

    1. APC
    2. BRCA1
    3. BRCA2
    4. CDH1
    5. TP53
    Board review style answer #1
    D. CDH1. CDH1 encodes E-cadherin and germline mutation results in hereditary diffuse gastric cancer syndrome. No increased risk of developing lobular carcinoma of the breast is reported for germline BRCA1 and BRCA2 mutations. Germline TP53 and APC mutations result in Li-Fraumeni syndrome and familial adenomatous polyposis, which are not associated with lobular carcinoma of the breast. (Cancer Epidemiol Biomarkers Prev 2012;21:134, Breast J 2019;25:16)

    Comment Here

    Reference: Classic lobular carcinoma
    Board review style question #2
    Which of the following features is more commonly seen in classic lobular carcinoma compared with pleomorphic lobular carcinoma?

    1. Formation of invasive tubular structures
    2. Grade 3 (markedly pleomorphic) nuclear features
    3. HER2 positivity
    4. Hormone receptor positivity
    5. TP53 mutation
    Board review style answer #2
    D. Hormone receptor positivity. Compared with classic lobular carcinoma, pleomorphic lobular carcinoma is more likely to be HER2 positive, hormone negative and TP53 mutated. Marked nuclear pleomorphism is a required feature of pleomorphic lobular carcinoma. Neither classic nor pleomorphic lobular carcinoma feature glandular formation. (Cell Oncol (Dordr) 2012;35:111)

    Comment Here

    Reference: Classic lobular carcinoma

    Collagenous spherulosis
    Definition / general
    Essential features
    • Benign proliferation with intraluminal pink basement membrane material (spherule) surrounded by myoepithelial cells
    Terminology
    • Collagenous spherulosis
    • Simple spherulosis
    • Mucinous spherulosis
    • Adenoid cystic hyperplasia (not recommended due to confusion with adenoid cystic carcinoma)
    ICD coding
      ICD-10:
    • N60.8 - other benign mammary dysplasias
    • N60.9 - unspecified benign mammary dysplasia

      ICD11:
    • GB20.Y - other specified benign breast disease
    • GB20.Z - benign breast disease, unspecified
    Epidemiology
    Sites
    • No specific site in the breast
    Pathophysiology
    • Unclear
    • Accumulation of basement membrane material in spaces
    Etiology
    • Unknown
    Clinical features
    Diagnosis
    • Based on histologic examination of removed tissue
    Radiology description
    • May present as radiologic or rarely as architectural distortion, density or mass (Pathology 2017;49:181)
    Prognostic factors
    Case reports
    Treatment
    • No treatment required
    Gross description
    • No specific gross features (usually not discernable from background breast tissue)
    Microscopic (histologic) description
    • Round spaces filled with acellular pink basement membrane material (spherules) (Pathology 2017;49:181)
    • Spherules measure 20 - 100 μm in diameter (Hoda: Rosen's Diagnosis of Breast Pathology by Needle Core Biopsy, 4th Edition, 2017)
    • Center of the spherules may be transparent but thin rim of basement membrane material always present
    • Radiating stellate fibrils or microcalcifications may be seen in the spherules
    • Spherules surrounded by small oval / spindle myoepithelial cells with interspersed cuboidal luminal cells
    • Myoepithelial cells are uniformly present at the periphery of involved glands
    • Majority associated with benign proliferative lesions, mostly intraductal papilloma
    Microscopic (histologic) images

    Contributed by Iskender Sinan Genco, M.D. and Sabina Hajiyeva, M.D.
    Cribriform architecture

    Cribriform architecture

    Multiple ducts

    Multiple ducts

    2 different sized ducts

    2 different sized ducts

    Cribriform architecture

    Cribriform architecture

    Partial involvement Partial involvement

    Partial involvement


    Adjacent apocrine metaplasia

    Adjacent apocrine metaplasia

    Columnar cell changes

    Columnar cell changes

    Smooth muscle myosin heavy chain

    Smooth muscle myosin heavy chain

    CD10

    CD10

    KIT / CD117

    KIT / CD117

    In situ carcinoma

    In situ carcinoma


    ER

    ER

    CK5/6

    CK5/6

    Adjacent invasive carcinoma

    Adjacent invasive carcinoma

    Smooth muscle actin (SMA)

    Smooth muscle actin

    Smooth muscle myosin heavy chain (SMMHC)

    Smooth muscle myosin heavy chain

    Calponin

    Calponin


    p63

    p63

    PAS PAS

    PAS

    Cytology description
    Cytology images

    Contributed by Mark R. Wick, M.D.
    FNA

    FNA

    Negative stains
    Electron microscopy description
    Videos

    Breast calcifications

    Sample pathology report
    • Breast, left, 3 o'clock, stereotactic core biopsy:
      • Lobular carcinoma in situ (LCIS) involving collagenous spherulosis, with microcalcifications (see comment)
      • Comment: Immunohistochemical stain for E-cadherin is negative in LCIS, supporting the diagnosis. Controls are appropriate.
    Differential diagnosis
  • Atypical ductal hyperplasia:
    • Some, but not all, features of low grade ductal carcinoma in situ (see above)
    • Negative for CK5/6
  • Board review style question #1

    A premenopausal woman had a stereotactic breast core biopsy for calcifications. Microscopic examination revealed fibrocystic changes with microcalcifications as well as the microscopic finding shown above (H&E). Regarding this entity, which of the following statements is true?

    1. Comprised of a clonal (neoplastic) proliferation
    2. Epithelial cells interspersed between spaces are negative for ER
    3. Flattened (myoepithelial) cells around spaces are positive for CD10
    4. Luminal epithelial cells are positive for KIT / CD117
    5. Surgical excision should be performed
    Board review style answer #1
    C. The flattened (myoepithelial) cells around spaces are positive for CD10. The picture shows slightly expanded ducts with spaces filled with eosinophilic material. The spaces are surrounded by small oval / spindle myoepithelial cells with interspersed cuboidal luminal cells, features consistent with collagenous spherulosis. Collagenous spherulosis is a benign finding and does not require treatment. Adenoid cystic carcinoma is the main differential diagnosis which shows MYB-NFIB fusion gene, is negative for ER and PR, positive for KIT / CD117 and should be surgically excised.

    Comment Here

    Reference: Collagenous spherulosis
    Board review style question #2
    Collagenous spherulosis Collagenous spherulosis Collagenous spherulosis


    A 34 year old woman was diagnosed with invasive ductal carcinoma on core biopsy and subsequently underwent surgical excision. One of the surgical specimen sections showed the microscopic lesion with cribriform architecture (middle) adjacent to invasive carcinoma. Based on the provided H&N, SMMH, and PAS stains, what is the correct diagnosis?

    1. Atypical ductal hyperplasia
    2. Collagenous spherulosis
    3. Ductal carcinoma in situ
    4. Lobular carcinoma in situ involving ductal carcinoma in situ
    5. Usual ductal hyperplasia
    Board review style answer #2
    B. Collagenous spherulosis

    The H&E slide shows invasive carcinoma (upper), benign breast tissue (right lower), and a duct with cribriform architecture (middle). SMMH highlights the myoepithelial cells around the cribriform spaces and the periphery of the involved duct, whereas invasive carcinoma is negative for SMMH. This lesion with cribriform architecture represents collagenous spherulosis. PAS stain highlights the basement membrane material within the cribriform spaces produced by the surrounding myoepithelial cells. The other options may also show cribriform architecture but do not have myoepithelial cells surrounding the cribriform spaces. PAS stain helps confirm the diagnosis of collagenous spherulosis, but does not entirely rule out the other options.

    Comment Here

    Reference: Collagenous spherulosis

    Columnar cell change / hyperplasia
    Definition / general
    • Enlarged terminal duct lobular units (TDLUs) with irregularly dilated acini lined by columnar epithelial cells
    • Intraluminal secretions and calcifications common
    Essential features
    Terminology
    • WHO acceptable:
      • Columnar cell change
      • Columnar cell hyperplasia
    • Not recommended by WHO:
      • Blunt duct adenosis
      • Columnar alteration of lobules
      • Columnar metaplasia
      • Hyperplastic unfolded lobules
      • Hyperplastic enlarged lobular units
      • Enlarged lobular units with columnar alteration
    • Lesions with cytologic atypia are flat epithelial atypia (FEA)
    ICD coding
    • ICD-10:
      • N60.9 - unspecified benign mammary dysplasia
      • N60.89 - other benign mammary dysplasias of unspecified breast
    • ICD-11:
      • GB20.Y - other specified benign breast disease
    Epidemiology
    • Columnar cell lesions (columnar cell change / hyperplasia and flat epithelial atypia) seen in as high as 42% of biopsies for calcifications (Am J Surg Pathol 1998;22:1521)
    Sites
    • Breast terminal duct lobular units
    Pathophysiology
    • Might represent the earliest nonobligate precursor in low grade breast neoplasia pathway and a marker for potential presence of atypia
    Etiology
    • Unknown
    Clinical features
    • No specific clinical features
    • Most often detected on screening mammography when associated with microcalcifications
    Diagnosis
    • Image guided needle core or surgical excisional biopsy with tissue histologic examination
    • Diagnosis is reproducible with available histologic diagnostic criteria (Mod Pathol 2006;19:172)
    Radiology description
    Radiology images

    Contributed by Jonathan Marotti, M.D.

    Calcifications associated with columnar cell change / hyperplasia

    Prognostic factors
    • Slight increased risk for breast cancer development (relative risk ~1.5) but risk with columnar cell change / hyperplasia not independent of risk associated with concurrent proliferative lesions (Breast Cancer Res 2010;12:R61, Cancer 2008;113:2415)
    • Can be associated with other lesions on the low grade breast neoplasia pathway (flat epithelial atypia, atypical ductal hyperplasia, low grade ductal carcinoma in situ, lobular neoplasia) (Surg Pathol Clin 2018;11:177)
    Case reports
    • 43 year old woman with columnar cell hyperplasia and mucocele-like lesion (J Med Case Rep 2008;2:138)
    • 2 cases (41 and 49 year old Japanese women) in which epithelial detachment artifact of columnar cell lesions was thought to result in pseudomicropapillae mimicking atypia (Virchows Arch 2013;463:93)
    Treatment
    • Excision not required when columnar cell change / hyperplasia without atypia is the most significant pathology in core biopsy specimens
    • No further management required when columnar cell lesions are the most significant pathology in excision specimens
    • Reference: Ann Surg 2012;255:259
    Gross description
    • No specific gross features
    Microscopic (histologic) description
    • Columnar cell change:
      • Terminal duct lobular units with irregular, variably dilated acini
      • Acini lined by 1 - 2 cell layers
      • Lining cells have uniform, ovoid to elongated nuclei oriented perpendicular to basement membrane
      • Apical snouts are frequently present
      • Luminal secretions and calcifications may be present
    • Columnar cell hyperplasia:
      • Terminal duct lobular units with irregular, variably dilated acini
      • Acini lined by stratified cells (more than 2 cell layers); may form tufts or mounds
      • Lining cells have uniform, ovoid to elongated nuclei that may appear crowded and overlap
      • Apical snouts often present
      • Luminal secretions and calcifications may be present
    • Columnar cell change / hyperplasia with cytologic atypia (high nuclear cytoplasmic ratio, irregular nuclear chromatin, variably prominent nucleoli) can be classified as flat epithelial atypia
    • Round nuclei with variably prominent nucleoli and loss of orientation perpendicular to basement membrane is flat epithelial atypia
    • Rigid bars, arcades and true micropapillations is atypical ductal hyperplasia or low grade ductal carcinoma in situ
    • Reference: Adv Anat Pathol 2003;10:113
    Microscopic (histologic) images

    Contributed by Jonathan Marotti, M.D.

    Columnar cell change

    Dilated TDLU

    Flocculent secretions

    Columnar cells

     
    Columnar cell hyperplasia

    Irregular TDLU

    Calcifications

    Tufts / mounds

    Cytology description
    • Range of atypia; cannot reliably differentiate from papillary neoplasms or well differentiated adenocarcinoma
    • Flat sheets of cells with distinct cell borders, finely granular cytoplasm, enlarged nuclei and few myoepithelial cells (Diagn Cytopathol 2007;35:73)
    • 3 dimensional clusters of polygonal epithelial cells mixed with myoepithelial cells; palisading columnar cells peripherally (Diagn Cytopathol 2004;31:370)
    Positive stains
    Negative stains
    Molecular / cytogenetics description
    Sample pathology report
    • Right breast, needle core biopsy:
      • Columnar cell change and columnar cell hyperplasia with associated microcalcifications
    Differential diagnosis
    • Cysts:
      • Attenuated, cuboidal or apocrine epithelium (noncolumnar)
    • Apocrine metaplasia:
      • Abundant, granular eosinophilic cytoplasm
      • Round nuclei with prominent nucleoli
      • Prominent apical snouting not typically seen
    • Cystic hypersecretory hyperplasia:
      • Cystic structures containing dense eosinophilic colloid-like material
      • Epithelium can demonstrate spectrum of alterations
    • Flat epithelial atypia:
      • Dilated terminal duct lobular units with round contours
      • Low grade, monomorphic, round to ovoid nuclei with variably prominent nucleoli and increased nuclear cytoplasmic ratio
      • Nuclei lose perpendicular orientation to basement membrane
    • Atypical ductal hyperplasia (ADH) and low grade ductal carcinoma in situ:
      • Architectural features of atypical ductal hyperplasia and low grade ductal carcinoma in situ (arcades, bridges, papillations, round fenestrations) are present
    • Ductal carcinoma in situ, clinging type:
      • Marked cytologic atypia (typically high grade)
    Board review style question #1

    What is the appropriate clinical management for the above lesion when identified on core biopsy?

    1. Follow up breast MRI
    2. No additional action required
    3. Surgical excision with negative margins
    4. Tamoxifen
    Board review style answer #1
    B. No additional action required. The image demonstrates the histologic features of columnar cell hyperplasia: stratified columnar epithelial cells forming tufts, exaggerated apical snouts and luminal calcifications. If columnar cell hyperplasia is the most significant alteration in the core biopsy sample and assuming adequate radiology and pathology correlation, no further clinical management is required. Surgical excision with negative margins is typically appropriate for ductal carcinoma in situ and invasive carcinomas. Tamoxifen is reserved for patients with higher risk lesions. Breast MRI is not indicated for columnar cell change / hyperplasia.

    Comment Here

    Reference: Columnar cell change / hyperplasia
    Board review style question #2
    Columnar cell change / hyperplasia is often diagnosed because of

    1. Calcifications seen on mammography
    2. Large irregular mass seen on ultrasound
    3. New palpable breast mass
    4. Nipple discharge
    Board review style answer #2
    A. Calcifications seen on mammography. Columnar cell change / hyperplasia is often biopsied following identification of calcifications on mammography. Columnar cell change / hyperplasia can also be an incidental finding in core biopsies or surgical excisions.

    Comment Here

    Reference: Columnar cell change / hyperplasia

    Core biopsy
    Definition / general
    Core biopsy versus fine needle aspiration biopsy
    Core biopsy is favored over fine needle aspiration because core biopsy:
    Guidance modalities

    MRI directed breast excision:

    Definition / general:
    Indications:
    Clinical features:
    Technical details
    False negatives
    • Radiologic - histologic correlation: must determine if histologic results provide a sufficient explanation for the imaging features - if not, lesion may not have been adequately sampled
    • Most false negatives are due to radiologic - histologic discordance and are discovered immediately (Eur Radiol 2010;20:782, Eur J Cancer 2010;46:1835)
    • Follow up imaging is recommended, even in patients with concordant benign findings (Radiographics 2007;27:79)
    Complications
    Reporting system of UK National Health Service Screening Programme
    • B1: normal tissue / inadequate sample (comment on microcalcifications and specimen adequacy)
    • B2: benign lesion (specify)
    • B3: uncertain malignant potential (includes radial scar, some papillary lesions, ADH, lobular neoplasia)
    • B4: suspicious of malignancy (suggestive but not diagnostic due to scanty material or artifacts)
    • B5: malignant; specify if invasive or not, if possible; indicate grade of DCIS
    • References: J Clin Pathol 2004;57:897
    Diagrams / tables

    Images hosted on other servers:

    Mammotome

    Microscopic (histologic) images

    Images hosted on other servers:

    Displaced epithelial cells; #2 - intracystic papillary carcinoma

    Detached clusters
    of ductal carcinoma
    in situ from
    pre-existing ducts


    Cribriform
    Definition / general
    • Low grade invasive breast carcinoma microscopically characterized by a predominant cribriform pattern
    • By WHO classification, a specific subtype of invasive breast carcinoma
    • Variants:
      • Pure: cribriform architecture is > 90% of the tumor
      • Mixed: tumor composed of 10 - 90% of another morphological type (other than tubular carcinoma)
    Essential features
    • Invasive breast carcinoma with > 90% of tumor composed of cribriform islands of malignant epithelial cells
    • Low grade nuclei and sparse mitosis (grade 1)
    • ER+ and HER2-
    Terminology
    • Invasive cribriform carcinoma (ICC)
    ICD coding
    • ICD-O: 8201/3 - cribriform carcinoma, NOS
    • ICD-10: C50.9 - malignant neoplasm of breast of unspecified site
    • ICD-11: 2C60 & XH1YZ3 - carcinoma of breast, specialized type & cribriform carcinoma, NOS
    Epidemiology
    Sites
    • Breast
    • Axilla - accessory breast tissue
    Pathophysiology
    • Luminal A molecular pathway / subtype of breast cancer (Cell Genom 2021:1;100067)
    • Expresses hormone receptors and lacks HER2 overexpression
    Etiology
    • Unclear
    Clinical features
    Diagnosis
    • Breast imaging including mammogram and ultrasound of breast
    • Cytologic or histologic examination of involved tissue
    Radiology description
    • Mammogram: density, asymmetry, spiculated mass, microcalcifications
    • Ultrasound: mass
    • MRI: enhancing focus or mass
    Radiology images

    Images hosted on other servers:

    Mammogram: 2.5 cm tumor, fuzzy border

    Prognostic factors
    • Pure ICC: excellent; usually less lymph node metastasis and lower stage
    • Mixed ICC: prognosis poorer than pure but better than invasive ductal carcinoma of no special type (IDC, NST)
    • Lymph node / distant metastasis is rare (BMC Cancer 2021;21:168)
    • ICC: more favorable prognosis than invasive breast carcinoma with less cribriform pattern and low grade IDC (J Clin Exp Pathol 2013;6:445)
    Case reports
    Treatment
    • Primary surgical excision
    • Postsurgical local radiation therapy (typically in the setting breast conserving therapy only for this special subtype of breast carcinoma)
    • Endocrine therapy
    Gross description
    Microscopic (histologic) description
    • Irregular cribriform growth pattern > 90% of cells
    • Nuclear grade I pure ICC: > 90% cribriform pattern
    • Mixed: 10 - 90% other morphological type, other than tubular carcinoma
    • Low grade: Nottingham grade 1 tumors
    • Invasive islands or nests of malignant cells with round or angulated contours and well defined cribriform spaces formed by arches of epithelial cells; this gives a sieve-like appearance
    • Tumor cells: small, mild / moderate pleomorphism, no nuclear atypia
    • No / sparse mitotic activity
    • Desmoplastic stroma
    • Mucin positive secretion, microcalcifications in lumen
    • Osteoclastic giant cells may be seen
    • Myoepithelial cells absent
    • Associated with cribriform ductal carcinoma in situ (DCIS)
      Microscopic (histologic) images

      Contributed by Sunitha Shankaralingappa, M.B.B.S, M.D., D.M.
      Invasive cribriform carcinoma

      Epithelial cells in cribriform pattern

      Pure ICC

      Pure ICC

      Low grade, cribriform pattern, mucin

      Low grade, cribriform pattern, mucin

      Low grade nuclei

      Low grade nuclei

      Cribriform carcinoma with cribriform DCIS

      Cribriform carcinoma with cribriform DCIS


      Cribriform DCIS with invasive component

      Cribriform DCIS with invasive component

      Cribriform DCIS

      Cribriform DCIS

      IHC- ER strongly positive

      IHC: ER strongly positive

      IHC- PR strongly positive

      IHC: PR strongly positive

      IHC- HER2 negative

      IHC: HER2 negative

      Virtual slides

      Images hosted on other servers:

      Cribriform (> 90%), grade I, desmoplastic stroma

      Cytology description
      • Cytologic features and limitations
        • 3D clusters and cohesive sheets of bland ductal cells, cribriform pattern (Acta Cytol 2001;45:593)
        • Naked bipolar nuclei and myoepithelial cells are absent
        • Definitive diagnosis on cytology is rare; core needle biopsy is preferred
      Cytology images

      Images hosted on other servers:

      Large sheets, cribriform architecture

      Positive stains
      Negative stains
      • HER2
      • Myoepithelial markers: distinguish ICC from cribriform DCIS
      Electron microscopy description
      Sample pathology report
      • Breast, right, modified radical mastectomy
        • Invasive cribriform carcinoma, grade 1, measuring 1.5 cm
        • Immunohistochemical biomarker results: estrogen receptor positive (90%, strong intensity), progesterone receptor positive (50%, moderate intensity), HER2 negative (score 0).
      Differential diagnosis
      Board review style question #1

      A 62 year old woman underwent a core needle biopsy for a lump, 1.2 cm in diameter, detected on mammogram. The histopathology pattern is shown above. Which statement is true regarding this tumor?

      1. Has an increased chance of distant metastasis
      2. Lesion is high grade
      3. Strong and diffuse positivity for ER
      4. Strongly positive (3+) for HER2 by IHC
      Board review style answer #1
      C. Strong and diffuse positivity for ER. This biopsy shows cribriform carcinoma. This type of breast carcinoma is seen in elderly, postmenopausal women and is most often detected on mammogram. It is strongly positive for ER, usually positive for PR and negative for HER2. It is a low grade tumor (Nottingham grade I) with a good prognosis and lower frequency of lymph node and distant metastasis.

      Comment Here

      Reference: Cribriform carcinoma
      Board review style question #2
      Which of the following is one of the diagnostic criteria for invasive cribriform carcinoma of the breast?

      1. < 50% component of tubular carcinoma
      2. ER, PR, HER2 negative
      3. At least 90% of tumor composed of cribriform islands with low grade nuclei
      4. Presence of myoepithelial cells on IHC
      Board review style answer #2
      C. Nuclear grade I in at least 90% of cells. Invasive cribriform carcinoma (ICC) is an infiltrating breast carcinoma with predominantly cribriform pattern. It is pure when > 90% of tumor is of cribriform pattern. It is termed mixed ICC when the other invasive component constitutes 10 - 90%. It is a low grade tumor with grade I nuclei and sparse mitotic activity. It is differentiated from cribriform DCIS by absence of myoepithelium. ICC is a luminal A molecular subtype of breast carcinoma and expresses hormone receptors and lacks HER2 overexpression.

      Comment Here

      Reference: Cribriform carcinoma

      Cystic hypersecretory carcinoma
      Definition / general
      • Not part of WHO breast classification
      • First described in 1984 (Am J Surg Pathol 1984;8:31)
      • Very rare (< 100 cases reported)
      • DCIS or hyperplasia is more common
      Clinical features
      • Usually low grade for several years but may metastasize
      Case reports
      Gross description
      • Numerous cysts with mucoid or gelatinous secretions
      Gross images

      Contributed by Dr. Okechukwu C. Okafor

      Mastectomy with 12 cm multicystic tumor



      Images hosted on other servers:

      Numerous cysts with a gelatinous secretion

      Microscopic (histologic) description
      • Cystic dilation of ducts containing colloid-like eosinophilic material that often retracts from epithelium
      • Epithelium focally has micropapillary DCIS
      • Also invasion of surrounding stroma by nests of carcinoma, which may be high grade, usually without hypersecretory characteristics
      • Extravasation of cyst material into stroma is not invasion
      Microscopic (histologic) images

      AFIP images

      Poorly differenti-
      ated carcinoma
      invades stroma
      next to cyst

      Low power shows
      prominent cysts with
      no apparent ducts
      containing carcinoma

      Associated micropapillary DCIS, invasion elsewhere

      Associated micropapillary DCIS with no
      evident secretion in tumor cells, which have
      a hobnail appearance, nuclei are relatively
      clear with small, discrete nucleoli


      Associated micropap-
      illary DCIS with sparse
      secretion that is re-
      tracted from epithelium

      Cysts lined by flat cuboidal epithelium contain
      homogeneous secretions, these cysts are
      nonspecific - they can be found in cystic
      hypersecretory hyperplasia or carcinoma


      Axillary nodal
      metastases, with
      some cells exhib-
      iting clear nuclei

      Note transition in cyst epithelium with plaque of tumor cells
      in bottom half, micropapillary pattern is obscured where carcinoma
      nearly fills ducts, but traces of retracted secretion remain
      (arrows), clear nuclei are also evident, even at this magnification



      Contributed by Dr. Okechukwu C. Okafor

      12 cm tumor



      Images hosted on other servers:

      Various images

      Cytology description
      Differential diagnosis

      Cystic hypersecretory hyperplasia
      Definition / general
      • Cystically dilated ducts of various sizes with colloid-like material
      • Ducts are lined by flat, orderly, bland columnar epithelial cells
      • Not included in WHO classification of breast lesions
      Terminology
      • Cystic hypersecretory carcinoma: lesions that resemble cystic hypersecretory hyperplasia but with proliferative atypical epithelium
      Epidemiology
      • Rare
      Etiology
      • Unknown
      Clinical features
      • Palpable mass or occasionally asymptomatic with mammographic abnormality
      • Benign lesion with good prognosis when not associated with atypia or carcinoma
      Case reports
      Treatment
      Gross description
      • Resembles juvenile papillomatosis
      • Large, ill defined, firm to rubbery, spongy mass of fibrous tissue containing multiple small cysts
      • Also abundant thick, sticky mucin within the cysts; resembles thyroid colloid
      • May coexist with atypia or DCIS of cystic hypersecretory type, so sample generously
      Microscopic (histologic) description
      • Cystically dilated ducts of various sizes with colloid-like material, often with parallel fracture lines, retraction halo and overlapping due to processing
      • Ducts are lined by flat, orderly, columnar epithelial cells with eosinophilic cytoplasm; nuclei are round / oval, vesicular, bland
      • Atypical features are epithelial crowding, enlarged nuclei lacking normal polarization, hyperchromasia and rare mitotic figures
      • Can be associated with pregnancy-like (pseudolactational) hyperplasia (Am J Surg Pathol 2000;24:1670)
      Microscopic (histologic) images

      AFIP images

      Cysts lined by flat
      cuboidal epithelium
      contain eosinophilic
      material

      Atypia - focal atypical epithelium between cysts


       Case #35

      Cysts lined by flat cuboidal epithelium contain eosinophilic material

      Intraluminal secretions show
      fine, parallel cracks and
      shrinkage of the cyst content
      with peripheral scalloping

      Cytology description
      • Secretory material (same as histology) with isolated or clusters of epithelial cells
      Positive stains
      • PR in a small subset
      Negative stains
      Differential diagnosis
      • Cystic hypersecretory DCIS

      Cystic neutrophilic granulomatous mastitis
      Definition / general
      • Inflammatory process of the breast usually due to Corynebacterium species
      Essential features
      • Most patients are women, parous or currently pregnant
      • Characteristic morphologic features include granulomas comprised of epithelioid histiocytes and giant cells with central lipid vacuoles containing gram positive bacteria and rimmed by neutrophils
      • Association with Corynebacterium species, especially Corynebacterium kroppenstedtii, although microbiologic evidence can be difficult to obtain
      • Prolonged antibiotic therapy required for complete resolution
      Terminology
      • Not recommended: granulomatous mastitis, idiopathic granulomatous mastitis, granulomatous lobular mastitis, idiopathic lobular granulomatous mastitis
      Epidemiology
      • Rare, with at least 141 cases reported since 2002 (J Clin Pathol 2020;73:445)
      • Female patients, parous or currently pregnant
      • Mean age 35 years (range 19 - 57)
      Sites
      Etiology
      Clinical features
      • Most commonly painful breast mass, nipple inversion or sinus formation (J Clin Pathol 2020;73:445)
      • Nipple discharge, skin erythema and abscess
      • Fever and leukocytosis
      • Axillary lymphadenopathy
      Diagnosis
      • Microscopic examination and Gram stain with or without microbiologic culture
      Laboratory
      Radiology description
      Radiology images

      Images hosted on other servers:

      Palpable breast mass

      Prognostic factors
      • Prolonged (weeks to months) antibiotic therapy required for complete resolution
      • May recur with inadequate therapy
      Case reports
      Treatment
      • Observation
      • Antibiotics:
        • Empirical antimicrobial therapy may be started prior to histologic diagnosis
        • Lipophilic antibiotics such as doxycycline, trimethoprim sulfamethoxazole, clarithromycin and rifampicin may be more effective (J Clin Microbiol 2015;53:2895)
        • Antibiotic susceptibility test should be performed
      • Steroids
      • Surgery including incision and drainage, excision and even mastectomy
      • Combination of the above therapies
      Gross description
      Microscopic (histologic) description
      • Presence of 2 - 3 of the following features (J Clin Pathol 2020;73:445):
        • Lobulocentric mixed inflammatory infiltrate composed of lymphocytes, neutrophils and scattered multinucleated giant cells
        • Round to oval cystic spaces (lipid vacuoles) rimmed by neutrophils (microabscesses)
        • Nonnecrotizing granulomas
      • Combined with either:
        • Presence of coryneform gram positive bacilli (rod shaped bacteria arranged in V shaped forms or palisades like Chinese characters) within lipid vacuoles or
        • Positive Corynebacteria culture or molecular testing
      Microscopic (histologic) images

      Contributed by Jessie M. Wu, M.B.Ch.B. and Gulisa Turashvili, M.D., Ph.D.
      Mixed inflammation Mixed inflammation

      Mixed inflammation

      Lipid vacuoles

      Lipid vacuoles

      Gram stain

      Gram stain



      Contributed by Ankur Sangoi, M.D. (Case #462)
      Mixed inflammation Mixed inflammation

      Mixed inflammation

      Lipid vacuoles Lipid vacuoles

      Lipid vacuoles

      Gram stain Gram stain

      Gram stain

      Cytology description
      • Cytologic diagnosis may be difficult
      • Limited by significant cytomorphologic overlap with granulomatous mastitis and other inflammatory breast lesions
      • Features suggestive of cystic neutrophilic granulomatous mastitis on ThinPrep include cystic spaces, inflammatory cells such as neutrophils with aggregates of epithelioid histiocytes, multinucleated giant cells, lymphocytes and plasma cells and lack of necrosis (Diagn Cytopathol 2018;46:966)
      Positive stains
      • CD68: positive in histiocytes
      • Gram positive coryneform bacteria within lipid vacuoles:
        • Rod shaped bacteria arranged in V shaped forms or palisades like Chinese characters
        • Bacteria may not be identified in all lipid vacuoles
        • Thick section Gram stain performed at 6 µm (vs 4 µm) improves both detection rate and ease of identification of gram positive bacilli (Am J Clin Pathol 2020;153:593)
      Negative stains
      Molecular / cytogenetics description
      Sample pathology report
      • Right breast, core biopsy:
        • Cystic neutrophilic granulomatous mastitis (see comment)
        • Comment: A tissue gram stain highlights gram positive bacilli, supporting the diagnosis. Controls are appropriate.
        • Microscopic description (optional): Sections show a perilobular mixed inflammatory infiltrate composed of central lipid vacuoles rimmed by neutrophils and an outer cuff of epithelioid histiocytes. Some of the lipid vacuoles contain sparse, rod shaped, gram positive bacilli. The surrounding inflammatory infiltrate contains multinucleated giant cells, lymphocytes and neutrophils. The morphology combined with the presence of gram positive bacilli on Gram stain is consistent with cystic neutrophilic granulomatous mastitis.
      Differential diagnosis
      Board review style question #1

      A 35 year old woman presents with a right breast mass. Core biopsy shows empty appearing cystic spaces rimmed by neutrophils and an outer cuff of epithelioid histiocytes. Some vacuoles contain sparse, rod shaped bacilli. Which organism is most commonly associated with this type of granulomatous mastitis?

      1. Corynebacterium diphtheriae
      2. Corynebacterium kroppenstedtii
      3. Mycobacterium tuberculosis
      4. Staphylococcus aureus
      5. Staphylococcus epidermidis
      Board review style answer #1
      B. Corynebacterium kroppenstedtii. Cystic neutrophilic granulomatous mastitis is most commonly associated with Corynebacterium kroppenstedtii.

      Comment Here

      Reference: Cystic neutrophilic granulomatous mastitis
      Board review style question #2
      Cystic neutrophilic granulomatous mastitis most commonly affects which of the following patient population?

      1. Female smokers
      2. Parous women
      3. Postmenopausal women
      4. Women living in endemic areas of tuberculosis
      5. Women with a history of Corynebacterium diphtheriae infection
      Board review style answer #2
      B. Parous women. Cystic neutrophilic granulomatous mastitis most commonly affects women who are currently pregnant or have been pregnant.

      Comment Here

      Reference: Cystic neutrophilic granulomatous mastitis
      Board review style question #3
      Which of the following statements is true regarding cystic neutrophilic granulomatous mastitis?

      1. Diagnosis may require matrix assisted laser desorption / ionization time of flight mass spectrometry
      2. Gram positive bacilli are identified in nearly all cases
      3. Resolution typically takes days
      4. Usually presents as a painless palpable mass in postmenopausal women
      Board review style answer #3
      A. Diagnosis may require matrix assisted laser desorption / ionization time of flight mass spectrometry. Diagnosis of cystic neutrophilic granulomatous mastitis may require culture, 16S rRNA gene sequencing or matrix assisted laser desorption / ionization time of flight mass spectrometry.

      Comment Here

      Reference: Cystic neutrophilic granulomatous mastitis

      Cytology
      Definition / general
      • Refers mainly to fine needle aspiration cytology (FNA) and also includes cytologic examination of imprint, nipple discharge and ductal lavage
      • Although largely replaced by core needle biopsy, FNA can increase the accuracy rate if both tests are combined, minimizing false negative results of core needle biopsy related to sampling issues, as more needle passes are performed in FNA (Diagn Cytopathol 2016;44:578)
      • Nowadays, it is used as part of the triple approach (along with clinical and radiologic findings) to confirm a benign diagnosis, as a conjugate to core needle biopsy to enhance the accuracy rate, solo for palpable breast lesions in one step clinic, for lymph node assessment and for lesions difficult to access by core needle biopsy, like those near the chest wall
      • Breast FNA is a safe, reliable, accurate, fast, cost saving, almost complication free method that can be performed without anesthesia
      • Along with core needle biopsy, these tests replaced frozen sections and surgical biopsies for diagnosing breast lesions
      • Breast FNA cytology doesn't give a definite diagnosis name to the lesion most of the time and also can't differentiate atypical ductal hyperplasia from low grade cancer or high grade in situ carcinoma from invasive cancer
      Essential features
      • Breast FNA can't differentiate atypical duct hyperplasia from low grade ductal carcinoma in situ or high grade ductal carcinoma in situ from invasive carcinoma
      • It is a screening test; it doesn't usually provide a specific diagnosis name to the lesion but enhances the sensitivity of cancer detection if combined with core needle biopsy
      • Biomarkers and other ancillary studies can be performed on cell block
      • False negative result is frequent in special types of carcinoma, like lobular and tubular, and false positive diagnoses are seen with changes associated with lactation and therapy
      CPT coding
      • 10004: FNA without imaging guidance, each additional lesion
      • 10005: FNA including ultrasound guidance, first lesion
      • 10006: FNA including ultrasound guidance, each additional lesion
      • 10007: FNA including fluoroscopic guidance, first lesion
      • 10008: FNA including fluoroscopic guidance, each additional lesion
      • 10009: FNA including CT guidance, first lesion
      • 10010: FNA including CT guidance, each additional lesion
      • 10011: FNA including MRI guidance, first lesion
      • 10012: FNA including MRI guidance, each additional lesion
      • 10021: FNA without imaging guidance, first lesion only
      • 19000: Puncture aspiration of breast cyst
      • 19001: Puncture aspiration of breast cyst, each additional cyst (used with 19000)
      Sites
      • Breast, lymph node
      Ancillary testing
      • Ancillary studies include biomarker testing in carcinoma cases, flow cytometry in lymphoma cases and molecular studies
      • Biomarker testing on breast FNA
        • Useful for patients who will undergo neoadjuvant therapy if core needle biopsy is not available
        • Also, the American Society of Clinical Oncology / College of American Pathologists (ASCO / CAP) recommends evaluating the biomarkers on FNA obtained from foci of metastasis and local recurrence after treatment
        • Same ASCO / CAP criteria that is used for tissue can be applied to cytology specimens, including the cold ischemia time (ideally less than an hour), as well as fixation time (6 - 72 hours) (College of American Pathologists: CAP QCDR Measure Formalin Fixation Time [Accessed 19 August 2021])
        • Recommendation of CAP is to report the percentage of positive nuclei for ER and PR; minimum of 100 cells should be counted (Arch Pathol Lab Med 2010;134:907)
        • FISH is more reliable than IHC for HER2 / neu
        • HER2 gene amplification can be reliably detected in cytology specimen when air fixed or fixed with FISH fixative
        • More interobserver discordance rate than the corresponding biomarker testing in histologic samples is attributed to the admixed inflammatory cells and mesothelial cells in body cavities; adding GATA3 or GCDFP-15 is recommended to enhance the accurate detection of tumor cells (Acta Cytol 2018;62:288)
        • It is impossible to distinguish the positivity of markers in the invasive component from the in situ component
      Procedure (FNA)
      • Ideally performed by a trained radiologist under ultrasound or stereotactic guidance
      • Direct smearing is the most common preparation method in FNA
      • Both alcohol fixed and air dried are used
      • Alternatives include cytospin preparation, thin layer preparation, cell block
      Limitations / disadvantages of FNA
      • Requires training, subjective in interpretation; the accuracy is influenced by both the quality of smear and the expertise of the reader
      • Can't differentiate atypical duct hyperplasia from low grade ductal carcinoma in situ
      • Can't differentiate high grade ductal carcinoma in situ from invasive carcinoma
      • Doesn’t give a definite diagnosis name to the lesion most of the time
      • Equivocal, false positive, false negative results
      Breast cytologic studies other than FNA
      • Cell block:
        • Enhances the utility of FNA
        • Ancillary studies including biomarkers can be done
        • Diagnostic architectural features can be displayed
        • Returned cell block from a Papanicolaou stained smear is also useful in difficult cases to interpret, e.g. if the morphology is obscured by blood (Diagn Cytopathol 2016;44:505)
      • Nipple discharge cytology:
        • Nipple discharge fluid commonly reflects hormonopathy, like dysthyroidism and hyperprolactinemia
        • Papilloma is the most common breast lesion that presents with nipple discharge
        • It has a weak association with cancer; increased if the discharge is bloody or associated with a mass
        • Nipple discharge cytology has a low sensitivity (Ann Surg Oncol 2020 Nov 9 [Epub ahead of print], Diagnosis (Berl) 2020 Jun 26 [Epub ahead of print])
        • Negative discharge: acellular or hypocellular, composed mainly of proteinaceous fluid and foamy cells; benign ductal epithelial or apocrine cells can be seen with no atypical features
      • Ductal lavage cytology:
        • Has a questionable utility in risk stratification of getting breast cancer in women with contralateral breast cancer or BRCA1 or BRCA2 mutation
      • Imprints:
        • Intraoperative imprint cytology of sentinel lymph node: limited sensitivity for micrometastasis
        • Intraoperative imprint cytology for diagnosing breast carcinoma and evaluating margins: useful in combination with frozen section or alone
        • Intraoperative imprint cytology to predict occult nipple involvement before performing nipple sparing mastectomy (Breast Cancer Res Treat 2015;153:557)
        • Touch imprint cytology of core needle biopsy: to complement core needle biopsy, providing immediate cytologic diagnosis to the lesion (Geburtshilfe Frauenheilkd 2016;76:59)
      Case reports
      Breast FNA C1 - C5 classification
      • In U.K. and Europe, numerical system of 5 points classification is used (C1 to C5)
      • In U.S. and Canada, same system but without the numerical designation
      • 5 categories: inadequate, benign, atypia favor benign, suspicious probably malignant and malignant
      • Inadequate / C1:
        • Smear is not interpretable or is undiagnostic
        • Common causes include: low cellularity, obscured morphology due to blood, crushing artifact, poor fixation and degenerated cells
        • No definite criteria for adequacy in breast FNA cytology
        • MD Anderson Cancer Center Group proposed that 4 - 6 well visualized cell groups with 10 or more cells / flat sheets as an adequacy criteria (Cancer 1997;81:3)
        • Samples that lack epithelial cells are considered inadequate in general
        • If the lesion is suspected to be lipoma or fat necrosis, the absence of epithelial clusters doesn't indicate inadequacy
      • Benign / C2:
        • Benign epithelial clusters are seen with no atypia or malignant features
        • Cellularity is usually not high; cells are seen in flat sheets, depicting fine chromatin, smooth nuclear membrane, low nuclear cytoplasmic (N/C) ratio, no pleomorphism
        • Fatty fragments are commonly seen
        • Bare nuclei of myoepithelial cells are easily seen
        • Histiocytes and apocrine cells can be found
        • In duct ectasia: inspissated material within macrophages and multinucleated giant cells
      • Atypia / C3:
        • Similar to the findings of benign category but with higher cellularity, slight crowdedness, 3 dimensional groups, nuclear enlargement and pleomorphism
        • Common lesions encountered in this category: proliferative fibrocystic change with or without atypia, papillary lesions, lobular lesions including atypical lobular hyperplasia / lobular carcinoma in situ and some special types of breast carcinoma commonly missed in FNA, like lobular, tubular and cribriform
      • Suspicious / C4:
        • Highly atypical findings but insufficient to be called malignant
        • Commonly encountered reasons: hypocellular or poorly preserved smear, the findings are focally seen in a benign background or the degree of atypia is more than the previously described in C3 category but not sufficient for C5 category
        • Common lesions encountered in this category: atypical duct hyperplasia, low grade ductal carcinoma in situ, invasive carcinoma that wasn't adequately sampled and special types of carcinoma like lobular, tubular and cribriform
      • Malignant / C5:
        • Cellularity is usually high and the cytologic features are malignant (high N/C ratio, irregular nuclear contour, hyperchromasia, conspicuous nucleoli)
        • Malignant cells are seen singly or in loose clusters
        • Necrotic debris might be seen in background
        • Some special types of carcinoma can be specifically recognized or suspected, like mucinous, medullary-like, papillary
      Breast FNA Masood and Modified Masood classification
      • Suggested scoring systems: Masood followed by the more reproducible Modified Masood Scoring System (Acta Cytol 2019;63:233)
      • It is based on 6 morphologic features:

      Cell arrangement Cell pleomorphism Myoepithelial cells Anisonucleosis Nucleoli Chromatin clumping Score
      Monolayer No Many No No No 1
      Nuclear overlapping Mild Moderate Mild Micronucleoli Rare 2
      Clustering Moderate Few Moderate Rare macronucleoli Occasional 3
      Discohesion Marked No Marked Mainly macronucleoli Marked 4

      • Breast lesions are classified into 4 categories according to the given score:

      Breast lesion Total score in Masood system Total score in Modified Masood system
      Nonproliferative breast disease 6 - 10 6 - 8
      Proliferative breast disease with no atypia 11 - 14 9 - 14
      Proliferative breast disease with atypia 15 - 18 15 - 18
      Malignancy 19 - 24 19 - 24
      Cytology description of benign lesions
      • Fibrocystic change:
        • Cellularity is variable, commonly moderately cellular but can be paucicellular if fibrotic
        • Epithelial cells are arranged in cohesive flat sheets, polarized to give honeycomb pattern
        • Myoepithelial cells naked nuclei are seen in background and overlying the epithelial sheets
        • With or without apocrine and foam cells, fat and stroma
      • Fibroadenoma:
        • Moderately to highly cellular smear
        • Sheets of cohesive epithelial cells that branch, giving the antler horn pattern
        • Myoepithelial cells naked nuclei are numerous
        • Stromal fragments can be seen
        • Apocrine and foam cells: less likely than fibrocystic change
        • If hypercellular stromal fragment is seen with atypia, the lesion is likely to be phyllodes tumor
      • Adenomyoepithelioma:
        • Moderately to highly cellular smear
        • More chance to show crowded groups or 3 dimensional clusters than fibroadenoma
        • Epithelial cells and myoepithelial cells are seen in variable proportions
        • Myoepithelial cells can be seen as naked nuclei as in other lesions or take the form of comma shaped dark nuclei, spindle, epithelioid or plasmacytoid
        • Epithelioid cells with intranuclear inclusion and intracytoplasmic vacuoles gives the characteristic soap bubble appearance (Cancer 2006;108:250)
      • Papillary lesions (intraductal papilloma, etc):
        • Difficult to segregate benign papilloma from well differentiated carcinoma in FNA
        • Cellular smear, branching epithelial sheets like fibroadenoma or 3 dimensional clusters with or without stromal cores
        • Columnar cells favor papilloma over fibroadenoma (Cancer 2003;99:156)
        • Uniformity of cells favor malignancy and polymorphism favor benign papilloma
      • Gynecomastia:
        • Spectrum of pictures from overtly benign to fibroadenoma-like changes to worrisome morphology like nuclear molding, atypia, conspicuous nucleoli and inconspicuous myoepithelial cell component
      Cytology description of malignant lesions
      • Invasive breast carcinoma of no special type:
        • Cellular smear
        • Less cohesiveness with or without single epithelial cells scattered in background
        • Epithelial sheets are crowded with 3 dimensional or syncytial pattern; might form acini
        • No or minimal myoepithelial cells
        • With or without tumor diathesis
      • Lobular carcinoma:
        • Can be missed as false negative due to the low cellularity and limited atypia
        • Some cases resemble invasive carcinoma of no special type
        • Cellularity is frequently low to moderate
        • Cells are uniform, mildly atypical, dispersed in singles, linear cords or loose clusters
        • Myoepithelial cells are absent
        • Signet ring cells can be seen
      Grading of malignant smears
      • Not routinely done
      • Mainly depends on nuclear grading due to the difficulty of assessing mitoses and tubular formation
      • Robinson and McKee proposed a grading system that correlates well with the histologic grading, depending on 6 parameters: dissociation, nuclear size and margin, nucleoli, chromatin and uniformity of cells (Acta Cytol 1995;39:1257)
      • Another proposed prognostic system is based on 3 characteristics: nuclear grade, architecture (discohesion) and background features (bare atypical epithelial cell nuclei); it is claimed to correlate well with histologic grading, lymph node status, positivity for HER2 / neu, p53 and Ki67 (Diagn Cytopathol 2006;34:542)
      Cytology images

      Contributed by Areej M. Al Nemer, M.D.
      Benign epithelial clusters (C2)

      Benign epithelial clusters (C2)

      Granulomatous inflammation (C2) Granulomatous inflammation (C2) Granulomatous inflammation (C2)

      Granulomatous inflammation (C2)

      Fibrocystic change (C2) Fibrocystic change (C2)

      Fibrocystic change (C2)


      Lactational changes Lactational changes Lactational changes Lactational changes Lactational changes Lactational changes

      Lactational changes (C2, false positive diagnosis is a possible pitfall)


      Lactational changes

      Lactational changes (C2)

      Fibroadenoma (C2) Fibroadenoma (C2) Fibroadenoma (C2) Fibroadenoma (C2) Fibroadenoma (C2)

      Fibroadenoma (C2, 5 images)


      Phyllodes tumor (C2 / C3) Phyllodes tumor (C2 / C3) Phyllodes tumor (C2 / C3) Phyllodes tumor (C2 / C3) Phyllodes tumor (C2 / C3) Phyllodes tumor (C2 / C3)

      Phyllodes tumor (C2 / C3)


      Phyllodes tumor (C2 / C3)

      Phyllodes tumor (C2 / C3)

      Suspicious nipple discharge

      Suspicious nipple
      discharge (C4):
      invasive & in situ
      apocrine carcinoma

      Suspicious nipple discharge

      Suspicious nipple discharge (C4): juvenile papillomatosis

      Malignancy (C5) Malignant (C5)

      Malignant (C5): invasive carcinoma of no special type

      Malignant (C5)

      Malignant (C5): invasive lobular carcinoma


      Malignant DCIS (C5)

      Malignant DCIS (C5)

      Sample pathology report
      • Should state the category, followed by detailed explanation of the findings that justify the specified category
        • If positive for malignancy, it is recommended to comment on the nuclear grade
      • Examples:
        • Left breast, FNA:
          • Inadequate (C1)
          • The smear shows only blood and rare degenerated cells.
        • Right breast, FNA:
          • Malignant (C5)
          • Markedly hypercellular smear with single epithelial cells admixed with cohesive and discohesive clusters of malignant cells, depicting enlarged, highly pleomorphic nuclei, coarse chromatin, some prominent nucleoli, nuclear molding and occasional mitoses. Nuclear grade 3/3.
      Differential diagnosis
      • False positive diagnosis:
        • Poor fixation:
          • Cellular enlargement and irregular chromatin
          • Crush artifacts: hyperchromatic dissociated cells
          • Ultrasound gel: contaminating the aspirate resembling necrotic debris
        • Fibroadenoma:
          • Cellular smear, mild pleomorphism, some discohesion, irregular clusters
          • Hint: myoepithelial cells are easily seen
        • Apocrine lesions:
          • Might show atypia and discohesion
          • Hint: no necrosis, admixture with benign ductal cells and macrophages, low N/C ratio
        • Lactational changes / adenoma:
          • Cellular smear, mild pleomorphism, enlarged hyperchromatic nuclei, prominent nucleoli, discohesion with fragile granular / vacuolated cytoplasm
          • Myoepithelial cells are there but less conspicuous
          • Background is dirty, with secretions that can be confused with necrosis by the nonexpert
          • Clinical history of pregnancy, lactation or hormonal intake is helpful
        • Proliferative fibrocystic change:
          • Cellular smear with mild atypia
          • Myoepithelial cells are plentiful
        • Seroma:
          • Cystic lesion that might follow breast conserving surgery
          • Smear is usually hypocellular with a few degenerated cells
          • Might show atypia with high N/C ratio and squamous metaplasia
        • Treatment induced cellular change:
          • Commonly hypocellular smear with scattered atypical, maybe enlarged epithelial cells
          • Might show nuclear enlargement with prominent nucleolus, chromatin clearing, nuclear and cytoplasmic vacuolation
          • Clinical data of prior therapy is crucial for accurate diagnosis
        • Intramammary lymph node:
          • Some activated lymphocytes might resemble lobular carcinoma
          • Smear will show nodal polymorphic population of lymphocytes admixed with tangible body macrophages
          • CD45+ and cytokeratin-
        • Axillary lymph node:
          • Sampling through apocrine glands, presence of benign intranodal epithelial inclusions, activated lymphocytes mimicking lobular carcinoma, granulomas, metastatic melanoma or carcinoma from other sites can cause false positive diagnosis
          • Immunohistochemistry is helpful to resolve any ambiguity
      • False negative diagnosis:
      Board review style question #1

      Nipple discharge cytologic examination of a 45 year old woman revealed the picture shown above. What is your diagnosis?

      1. Atypia
      2. Inadequate
      3. Malignant
      4. Negative
      5. Suspicious
      Board review style answer #1
      D. Negative. In nipple fluid cytology, the presence of epithelial cells is not mandatory to call it adequate.

      Comment Here

      Reference: Cytology
      Board review style question #2
      Which of the following is correct regarding breast biomarker testing on cytologic specimen?

      1. Any positivity for hormonal receptors is counted as positive results
      2. Exact percentage of positivity is less accurate than the corresponding tissue assessment
      3. For HER2 / neu assessment, only immunohistochemistry can be performed on cytologic specimens
      4. It has similar interobserver discordance rate as in the tissue testing
      Board review style answer #2
      B. Exact percentage of positivity is less accurate than the corresponding tissue assessment. It is impossible to differentiate the malignant cells of the in situ component from the invasive component. For hormonal receptor positivity, the same cutoff point of 1% is used as in tissue.

      Comment Here

      Reference: Cytology

      DCIS
      Definition / general
      • Ductal carcinoma in situ (DCIS) is a neoplastic proliferation of mammary ductal epithelial cells confined to the ductal-lobular system without evidence of invasion through the basement membrane into the surrounding stroma (Arch Pathol Lab Med 2009;133:15)
      • Is a nonobligate precursor lesion of invasive breast cancer (Breast Cancer Res Treat 2010;123:757, Cancer 2005;103:2481)
      • Encompasses a heterogeneous group of lesions in terms of histomorphology, underlying genetic alterations, biomarker expression profile and biologic potential for progression to invasive carcinoma
      Essential features
      • Neoplastic proliferation of mammary ductal epithelial cells that is confined within ducts and lobules
      • Nonobligate precursor of invasive breast disease
      • Heterogenous entity with wide range of histologic appearances, growth patterns, genetic abnormalities and clinical behavior
      Terminology
      • Ductal carcinoma in situ (DCIS)
      • Intraductal carcinoma
      • Ductal intraepithelial neoplasia: terminology never widely adopted and removed from the 4th Edition WHO classification of mammary neoplasms (International Agency for Research on Cancer: WHO Classification of Tumours of the Breast, 4th Edition, 2012)
      • Mammary intraepithelial neoplasia: a general term to include both ductal and lobular intraepithelial neoplasia but never adopted, terminology should not be used (Am J Surg Pathol 1991;15:209, Hum Pathol 2002;33:620, Am J Surg Pathol 1980;4:241)
      • Extensive DCIS / extensive intraductal component: terminology should only be used to describe DCIS that is associated with invasive carcinoma in which the DCIS component constitutes > 25% of the entire area of invasive carcinoma and DCIS extends beyond the invasive carcinoma into the surrounding breast tissue (Pathol Annu 1988;23:1)
      • Multicentric DCIS: no uniform definition, generally refers to DCIS that involves multiple breast quadrants and different distinct regions of the breast
      ICD coding
      • ICD-O: 8500/2 - intraductal carcinoma, noninfiltrating, NOS
      • ICD-10: D05.1 - intraductal carcinoma in situ of breast
      Epidemiology
      Sites
      • Breast
      • Can occur in axillary breast tissue
      Clinical features
      Diagnosis
      • Calcifications or distortion on imaging raises concern and need for obtaining tissue
      • Diagnosis is made by histologic examination of tissue obtained via needle core biopsy, lumpectomy or mastectomy
      Radiology description
      Radiology images

      Contributed by Azadeh Khayyat, M.D. and Julie M. Jorns, M.D. (Case #533)
      Mammogram

      Mammogram



      Images hosted on other servers:

      Microcalcifications detected by mammography and tomosynthesis

      Microcalcifications detected by mammography


      Irregular mass via ultrasound

      MRI of an irregular mass

      MRI of a linear enhancement

      Prognostic factors
      Case reports
      Treatment
      Gross description
      • Usually no gross lesion, but high grade DCIS may present as firm gritty mass with multiple yellow punctate flecks on the cut surface representing comedonecrosis
      • Serial sequential sampling with mammographic correlation is useful to estimate DCIS size (Arch Pathol Lab Med 2009;133:31, Arch Pathol Lab Med 2009;133:26)
      Microscopic (histologic) description
      • DCIS traditionally classified based on architectural growth pattern (no clinical relevance other than comedo pattern):
        • Cribriform:
          • Fenestrated proliferation with multiple, round, rigid extracellular lumens with punched out appearance
          • Neoplastic cells are frequently evenly distributed equidistant and polarized with long axis of cell perpendicular to the central lumen
          • Trabecular bars comprised of rigid rows of cells with long axes perpendicular or at least not parallel to the long axis of the bar
          • Roman bridges comprised of curvilinear trabecular bars connecting two portions of the epithelial lining
        • Micropapillary:
          • Papillary fronds and tufts lacking fibrovascular cores projecting into duct lumen
          • Papillae often have club shaped cells comprising the micropapillae are uniform in appearance
          • Tips of fronds may fuse, forming bridges and arcades
        • Papillary:
          • Papillary fronds containing prominent fibrovascular septa projecting into duct lumen, papillary cores generally lack myoepithelial cell layer
        • Solid:
          • Lumen of ducts or lobules filled with sheets of cohesive cells
          • Cells are evenly spaced especially in low or intermediate grade DCIS
        • Flat or clinging:
          • 1 - 2 layers of generally high grade malignant cells lining a gland with a large empty lumen
        • Comedo:
          • Central expansile necrosis containing cellular debris, generally associated with high grade DCIS, frequently associated with coarse microcalcifications
      • No universally accepted grading system for DCIS but more recently endorsed classification systems stratify DCIS by nuclear grade (low, intermediate, high) and presence or absence of necrosis (Lancet 1995;345:1154, Hum Pathol 1998;29:1056, Cancer 1997;80:1798, Surg Clin North Am 1990;70:853)
      • Nuclear grade:
        • Low grade:
          • Monotonous, round nuclei with smooth contours, small size nuclei (size of normal ductal epithelial cell or 1 - 1.5 diameter of normal red blood cell)
          • Diffuse fine chromatin, absent or indistinct nucleoli, no or rare mitotic figures, necrosis is uncommon but does not preclude the diagnosis of low grade DCIS
        • Intermediate grade:
          • Moderate pleomorphism, mild to moderate variability in nuclear size
          • Variably coarse chromatin, occasional nucleoli, infrequent mitoses, features in between low and high grade DCIS
          • Loss of monotony can mimic usual ductal hyperplasia
        • High grade:
          • Prominent pleomorphism, large size nuclei (> 2.5 size of normal ductal epithelial cell)
          • Vesicular chromatin with irregular distribution, prominent nucleoli, frequent mitoses, comedo necrosis frequent but not required
      • Necrosis:
      • Myoepithelial cell layer surrounding ducts spaces containing DCIS is intact, but may be attenuated especially in high grade DCIS
      • Associated stromal reaction (chronic inflammatory infiltrate, fibrosis / sclerosis) may be prominent in areas surrounding DCIS, especially in high grade DCIS and does not indicate invasion
      • Cancerization of lobules, extension of DCIS into acini of terminal duct lobular unit
      • Variants include apocrine, cystic hypersecretory, squamous, spindle cell, clear cell, signet ring cell, mucinous, small cell, others
      Microscopic (histologic) images

      Contributed by Gary Tozbikian, M.D., Azadeh Khayyat, M.D. and Julie M. Jorns, M.D. (Case #533)

      Low grade, cribriform

      Intermediate grade, cribriform


      Intermediate grade, papillary and cribriform

      Intermediate grade, papillary

      Intermediate grade, solid

      Intermediate grade, micropapillary


      High grade, flat

      High grade, solid

      High grade, solid and cribriform


      High grade, solid and cribriform

      Ductal carcinoma in situ (DCIS), solid type, with microcalcifications, confined to a fibroadenoma Ductal carcinoma in situ (DCIS), solid type, with microcalcifications, confined to a fibroadenoma Ductal carcinoma in situ (DCIS), solid type, with microcalcifications, confined to a fibroadenoma Ductal carcinoma in situ (DCIS), solid type, with microcalcifications, confined to a fibroadenoma Ductal carcinoma in situ (DCIS), solid type, with microcalcifications, confined to a fibroadenoma

      Ductal carcinoma in situ (DCIS), solid type, with microcalcifications, confined to a fibroadenoma; lobular neoplasia (atypical lobular hyperplasia / lobular carcinoma in situ), with involvement of a fibroadenoma

      Virtual slides

      Images hosted on other servers:

      Intermediate grade, solid, comedo necrosis

      Cytology description
      Cytology images

      Images hosted on other servers:

      Low grade

      High grade

      Positive stains
      Negative stains
      Molecular / cytogenetics description
      • Low grade DCIS and high grade DCIS are genetically distinct
      • Low grade has frequent chromosomal losses at 16q and 17p and gains at 1q
      • High grade has frequent losses at 8p, 11q, 13q and 14q, and gains at 5p, 8q and 17q (J Pathol 2005;205:248)
      • High grade DCIS has similar molecular profile as invasive breast cancer (Cancer Res 2015;75:3980)
      Sample pathology report
      • Right breast, 10:00 calcifications with ribbon clip, stereotactic core needle biopsy:
        • Ductal carcinoma in situ (DCIS), intermediate nuclear grade, solid and cribriform patterns with comedo necrosis and microcalcifications
          • Immunohistochemical biomarker results: Estrogen receptor positive (90%, strong intensity), Progesterone receptor positive (40%, moderate intensity)
      Differential diagnosis
      • Usual type ductal hyperplasia:
        • Heterogenous proliferation of cells with variable size/shape, that are irregularly distributed with cohesive or syncytial growth, haphazard orientation of nuclei, nuclear overlap, nonrigid peripheral slit-like fenestrations with nuclei parallel to the lumina
        • Heterogeneity can mimic intermediate grade DCIS
        • Variable or mosaic pattern of expression of CK5/6 and CK5 and estrogen receptor
      • Atypical ductal hyperplasia:
        • Architectural and cytologic features suggestive of but not diagnostic for a diagnosis of low grade DCIS, partial involvement of ducts, limited quantitative extent
      • Lobular carcinoma in situ:
        • Low grade discohesive cells, often feathery clear space between cells, with solid growth pattern, intracytoplasmic vacuoles, lack of polarization around luminal spaces
        • Reduced or absent membranous expression of E-cadherin, cytoplasmic reactivity for p120
      • Invasive cribriform carcinoma:
      • Adenoid cystic carcinoma:
      • Collagenous spherulosis:
        • Pseudolumens containing nodules of flocculent blue-grey basement membrane enclosed by cuticle of basement membrane and surrounded by stellate myoepithelial cells
        • Lacks true cribriform spaces with ductal epithelial cells polarized around true lumens
        • Can be involved by atypical lobular hyperplasia or lobular carcinoma in situ
        • Myoepithelial markers (p40 / p63 / SMMS / calponin / CK5/6 and CK5) highlight myoepithelial cell component
      • Lymphovascular invasion:
        • Surrounding space is lined by endothelial cells, lymphatic spaces often adjacent to blood vessels
        • Lack circumscription by most myoepithelial markers (p40 / p63 / CK5/6 and CK5), circumscribed by endothelial markers (CD31, ERG) and also circumscribed by SMMS
      • Gynecomastia:
        • Micropapillae have tapered (nonclubbed) configuration, features of usual type ductal hyperplasia (cellular heterogeneity, haphazard, nonuniform distribution)
      • Solid papillary carcinoma:
        • Solid growth pattern punctuated by delicate fibrovascular cores, perivascular pseudorosettes, neuroendocrine features, extracellular mucin
        • Lack circumscription by myoepithelial markers (p40 / p63 / SMMS / calponin / CK5/6 and CK5) at periphery of nodules
      Board review style question #1
        Which of the following statements regarding DCIS is correct?

      1. Adjuvant anti endocrine therapy reduces the risk of local recurrence in patients with DCIS who have undergone breast conserving surgery with radiation therapy
      2. DCIS is generally a clinically and mammographically occult lesion
      3. Low grade DCIS is a nonobligate precursor lesion to high grade DCIS
      4. Recommended standard biomarkers to evaluate and report in patients with DCIS include estrogen receptor, progesterone receptor, and HER2
      5. Sentinel lymph node sampling is not indicated in patients with DCIS only, as the disease is noninvasive and there is no risk of metastatic involvement of axillary lymph nodes
      Board review style answer #1
      A. DCIS is a nonobligate precursor lesion to invasive breast cancer. Molecular studies have demonstrated that low and high grade DCIS are genetically distinct biologic entities. DCIS most often presents mammographically as microcalcifications. Per ASCO/CAP guidelines, currently the only recommended biomarker for DCIS is estrogen receptor, while testing for progesterone receptor is optional. Unlike for invasive breast carcinoma, HER2 is not assessed in patients with DCIS only as anti HER2 therapy is not indicated. Sentinel lymph node sampling has a role in subset of DCIS only patients, the reported positive rate is 0.4 - 13.7% (mostly isolated tumor cell deposits and micrometastases) presumably due to missed or undersampled invasive disease. In the National Surgical Adjuvant Breast and Bowel Project (NSABP) B-24 trial, tamoxifen reduced the likelihood of ipsilateral recurrence at five years from 9 - 6%, and also reduced the risk of a tumor in the contralateral breast. (J Clin Oncol 2014;32:1365, Lancet 1999;353:1993)

      Comment Here

      Reference: DCIS - general
      Board review style question #2

        A 55 year old woman undergoes a stereotactic core needle biopsy for mammographically detected microcalcifications. Which statement is true regarding the grade and expected immunohistochemical staining pattern of this lesion?

      1. Negative for circumferential calponin and p63
      2. Positive (Score 3+) for HER2 by immunohistochemistry
      3. Reduced or absent membranous reactivity for E-cadherin
      4. Strongly, diffusely positive for estrogen receptor
      5. The lesion is high grade
      Board review style answer #2
      D. The biopsy shows low grade DCIS with a cribriform pattern and associated microcalcifications. Loss of E-cadherin expression is seen lobular neoplasia. Myoepithelial markers such as calponin and p63 will show retained myoepithelial cell layer surrounding foci of DCIS. Assessment of HER2 status in DCIS is not indicated per current guidelines, however HER2 overexpression is more often observed in high grade DCIS. Low grade DCIS typically shows strong estrogen receptor expression.

      Comment Here

      Reference: DCIS - general

      Duct ectasia
      Definition / general
      • Mammary duct ectasia is a disorder of the extralobular ducts
      • Is characterized by different degrees of periductal inflammation, periductal fibrosis and duct dilatation
      • Foamy histiocytes are typically present within the luminal secretions and infiltrate the wall of the involved ducts
      Essential features
      • Foamy histiocytes within luminal secretions and within wall and epithelium of involved ducts
      • Periductal inflammation, can be plasma cell heavy
      • Periductal fibrosis in late stages
      Terminology
      • Also called mastitis obliterans and comedo mastitis, although these terms have not been as widely used as duct ectasia
      ICD coding
      • ICD-10: N60.49 - mammary duct ectasia of unspecified breast
      Epidemiology
      • Usually women in perimenopausal and postmenopausal age
      • It can occur in neonates
      Sites
      • Extralobular ducts; retro areolar, can be bilateral
      Etiology
      • Not well known
      • Some authors believe that involution seen in postmenopausal women is the primary pathologic process (gland atrophy → stasis of secretions → leakage of lipid rich material through duct wall → periductal inflammation)
      • Other authors believe that periductal inflammation is the inciting process that is followed by duct sclerosis, obliteration and finally ectasia
      • Stasis caused by duct obstruction is considered by some to be the inciting cause of duct inflammation with further obstruction, stasis and dilatation as observed in cases of squamous metaplasia of lactiferous ducts (SMOLD)
      • Smoking, phenothiazine treatment and hyperprolactinemia have been postulated as possible causes (Hoda: Rosen's Breast Pathology, 4th Edition, 2014)
      Clinical features
      Case reports
      Treatment
      • Circumareolar incision and removal of the affected duct and fistula tract, if present, is the treatment of choice
      Gross description
      Microscopic (histologic) description
      • Dilation of ducts with foamy histiocytes within the luminal secretions and infiltrating the wall of the involved ducts
      • Periductal inflammation with lymphocytes, plasma cells, neutrophils and histiocytes; these can become multinucleated in cases of duct rupture due to reaction to spilled secretions in the stroma
      • Periductal fibrosis with obliteration of the duct’s lumen can also be seen (garland sign)
      Microscopic (histologic) images

      Contributed by Carlos C. Diez Freire, M.D.
      Missing Image Missing Image Missing Image Missing Image

      Dilated ducts with foamy macrophages


      Missing Image Missing Image Missing Image

      Dilated ducts with foamy macrophages

      Missing Image

      Garland sign

      Cytology description
      • Thick cheesy secretion composed of amorphous material and cellular debris that resembles necrosis
      • Amorphous material may have a crack artifact as seen in watery thyroid colloid
      • Small crowded groups or sheets of ductal cells with reactive atypia may be present
      • The inflammatory component has plasma cells (can be abundant), neutrophils and foamy macrophages that can have pigment
      • Giant cells, calcifications and cholesterol clefts can also be present (DeMay: Art and Science of Cytopathology, Volume 2, Superficial Aspiration Cytology, 2nd Edition, 2012)
      Positive stains
      • CD68 highlights luminal and subepithelial histiocytes
      Sample pathology report
      • Left breast, 2 o’clock, 5 cm from nipple, ultrasound guided core biopsy:
        • Duct ectasia
      Differential diagnosis
      • Breast abscess:
        • Presence of severe acute inflammatory infiltrate
        • Often associated with lactation
      • Ductal carcinoma in situ:
        • Neoplastic epithelial cells confined to the mammary ductal lobular unit
        • Can have expansive comedo type necrosis
      Board review style question #1

      Which of the following is true about the breast entity depicted above?

      1. Increases the risk of invasive ductal carcinoma
      2. Is always a unilateral process
      3. Most commonly affects the extralobular ducts in the retroareolar region
      4. Most frequently affects men and children
      Board review style answer #1
      C. Most commonly affects the extralobular ducts in the retroareolar region. The photo depicts duct ectasia. Answer A is incorrect because a history of duct ecstasia does not increase risk for breast carcinoma. Answer B is incorrect because duct ecstasia can be bilateral. Answer D is incorrect because duct ecstasia is most common in perimenopausal and postmenopausal women.


      Comment Here

      Reference: Duct ectasia of breast
      Board review style question #2
      Mammary duct ectasia is characterized by which of the following microscopic findings?

      1. Epithelial proliferation that fills and distends the terminal duct lobular units
      2. Infiltrating ductal proliferation with desmoplastic stromal response
      3. Periductal inflammation and dilation of ducts, with foamy histiocytes within the luminal secretions and infiltrating the wall of ducts
      4. Proliferation of both stromal and glandular elements
      Board review style answer #2
      C. Periductal inflammation and dilation of ducts, with foamy histiocytes within the luminal secretions and infiltrating the wall of ducts with periductal inflammation. Duct estasia is characterized by different degrees of periductal inflammation, periductal fibrosis and duct dilation, as well as the presence of foamy histiocytes within luminal secretions and the periductal epithelium. Answers A, B and D are incorrect because duct ectasia is a benign nonproliferative disease.

      Comment Here

      Reference: Duct ectasia of breast

      Ductal adenoma
      Definition / general
      • Benign tumor composed of distorted glands in a sclerotic stroma surrounded by a fibrous capsule
      • First described by Azzopardi and Salm in 1984 (J Pathol 1984;144:15)
      Essential features
      • Rare, benign adenomatous lesion of medium and small sized ducts in the peripheral breast
      • Well circumscribed, single (occasionally multinodular), solid proliferation of tubules set within a fibrous stroma and surrounded by a thickened fibroelastotic wall
      • Considered a highly sclerotic variant of intraductal papilloma with obliterated papillary growth
      • May show calcifications, hemorrhage, infarction, squamous or apocrine metaplasia, epithelial hyperplasia, myxoid or chondroid change, central scar or pseudoinfiltrative pattern (exclude invasion by showing intact myoepithelial cells; e.g., p63, SMMHC)
      • Excision is curative
      Terminology
      • Not recommended: sclerosing papilloma
      ICD coding
      • ICD-O: 8503/0 - duct adenoma, NOS
      • ICD-10
        • D24 - benign neoplasm of breast
        • D24.1 - benign neoplasm of right breast
        • D24.2 - benign neoplasm of left breast
        • D24.9 - benign neoplasm of unspecified breast
      • ICD-11: 2F30.2 & XH4LZ4 - intraductal papilloma of breast & intraductal papilloma
      Epidemiology
      Sites
      Pathophysiology
      • Most likely, it originates from intraductal papilloma in small and medium sized ducts (J Pathol 1984;144:15, Hum Pathol 1989;20:903)
      • Due to the stromal repair process, papilloma undergoes sclerosis, resulting in loss of the arborizing papillary architecture
      • Another theory implicates a direct expansion of hyperplastic processes such as sclerosing adenosis into small and medium sized ducts or coexistent intraductal papilloma (Hum Pathol 1989;20:903)
      • Stromal degenerative changes may occur in the center, similar to radial scar
      Etiology
      Clinical features
      Diagnosis
      • May resemble carcinoma on clinical examination, imaging, needle aspiration cytology and core biopsy (J Pathol 1984;144:15, Breast Cancer 2006;13:354)
      • Excision is often recommended due to histological and radiological mimicry of carcinoma
      Radiology description
      • Imaging features may suggest papilloma, fibroadenoma or mimic carcinoma (Breast Cancer 2006;13:354)
      • Mammography shows well to ill defined mass with or without calcification
      • Ultrasonography shows a well defined, round hypoechoic nodule with shadowing and posterior enhancement
      • On contrast enhanced MRI, margins are smooth despite histologic pseudoinvasion (Breast Cancer 2016;23:597)
      Radiology images

      Contributed by Thomas Lienhoop, M.D.
      Screening mammogram Screening mammogram

      Screening mammogram

      Diagnostic ultrasound Diagnostic ultrasound

      Diagnostic ultrasound

      Prognostic factors
      • Benign, with no increased risk for recurrence or malignant transformation after complete excision (Hum Pathol 1989;20:903)
      • 1 case reported in the literature associated with malignant transformation to apocrine and epithelial myoepithelial carcinoma (Malays J Pathol 2015;37:281)
      Case reports
      Treatment
      Gross description
      • Ranges in size from 0.5 to 5.0 cm (average: 0.85 cm) (J Pathol 1984;144:15)
      • Solid white nodule with lobulated and granular cut surface and central gray softening
      • Gritty texture and elastic streaks mimic carcinoma
      • Solitary or occasionally multiple nodules in close proximity
      • Multinodular tumors due to the involvement of different parts of the same ductal system (Hum Pathol 1989;20:903)
      Microscopic (histologic) description
      • Well circumscribed, intraductal, solitary or multinodular (rare) lesion, surrounded by a thickened concentric fibroelastotic wall (J Pathol 1984;144:15)
      • Adenomatous portion has solid architecture and consists of proliferating tubules set in a fibrous stroma
      • No arborizing papillary fronds (as seen in intraductal papilloma) or myoepithelial hyperplasia (as seen in adenomyoepithelioma)
      • Glands range from round to oval, elongated to branched
      • Epithelial and myoepithelial cell types are present (the latter may be inconspicuous)
      • Ductal cells are cuboidal, columnar or spindled, with no atypia and only rare mitoses
      • Cystic gland dilatation, epithelial hyperplasia, apocrine or squamous metaplasia, eosinophilic secretions or laminated calcifications may be present (Hum Pathol 1989;20:903)
      • Stroma is usually fibrous, modest in amount and may show myxoid change, prominent hyalinization and rarely cartilaginous metaplasia (Pathol Int 2002;52:239)
      • Compression of glands similar to sclerosing adenosis is frequently seen (Hum Pathol 1989;20:903)
      • Distortion and entrapment of glands in the sclerotic wall may result in a pseudoinfiltrative pattern
      • Glandular proliferation may protrude into surrounding tissue, pushing beyond ductal confines (pushing borders) (J Pathol 1984;144:15)
      • Duct wall may contain dystrophic calcifications; periductal chronic inflammation may be seen (Hum Pathol 1989;20:903)
      • In stellate form, central fibroelastosis with entrapped epithelial tubules and peripheral dilated glandular elements resembles radial scar (J Pathol 1984;144:15)
      • Secondary changes such as hemorrhage or infarction may occur (Breast Cancer 2006;13:354)
      Microscopic (histologic) images

      Contributed by Susanne M. Crespo-Ramos, M.D., Anna Biernacka, M.D., Ph.D. and AFIP
      Epithelial cell nests and glands

      Epithelial cell nests and glands

      Compressed glands

      Compressed glands

      Well circumscribed adenomatous nodule

      Well circumscribed adenomatous nodule

      Duct wall / capsule

      Duct wall / capsule

      Duct distention and pseudoinvasion

      Duct distention and pseudoinvasion

      Pseudoinfiltrative and pushing borders

      Pseudoinfiltrative and pushing borders


      Glands and stroma

      Glands and stroma

      Epithelial and myoepithelial cells

      Epithelial and myoepithelial cells

      Eosinophilic secretions

      Eosinophilic secretions

      Usual ductal hyperplasia

      Usual ductal hyperplasia

      Missing Image

      Central fibrosis, peripheral bland ducts

      Missing Image

      Nodular lesion, primarily spindle cells


      Missing Image

      Spindle cells with residual ducts

      Missing Image

      Spindled myoepithelial cells around peripheral duct

      Missing Image

      Ducts lined by apocrine cells

      Smooth muscle myosin

      Smooth muscle myosin

      p63

      p63

      Cytokeratin 5/6

      Cytokeratin 5/6

      Cytology description
      • Highly cellular aspirates (Diagn Cytopathol 1994;10:143, Diagn Cytopathol 1995;13:252)
      • Numerous branching flat sheets of regularly spaced cohesive ductal cells
      • Large fragments of purple stroma in tight association with epithelial cells, forming finger-like hyaline structures or globules between cells
      • Background of single cells, naked nuclei, histiocytes and apocrine cells
      • Nuclei are round / oval with bland chromatin
      • Cytoplasm with small punched out vacuoles
      Positive stains
      Negative stains
      Molecular / cytogenetics description
      • 9 cases of ductal adenoma that have been studied using next generation sequencing showed recurrent missense mutations affecting AKT1, GNAS and PIK3CA genes (Genes Chromosomes Cancer 2017;56:11)
      • Given the high frequency of AKT1 mutations in breast papillomas, these data support the notion that ductal adenomas are closely related to or are, in fact, intraductal papillomas that have undergone sclerosis (Mod Pathol 2010;23:27)
      Sample pathology report
      • Right breast, excisional biopsy:
        • Benign intraductal glandular proliferation with sclerotic stroma consistent with ductal adenoma (see comment)
        • Comment: The specimen shows a solid nodule with a prominent component of glands and stromal sclerosis, surrounded by a thickened hyalinized wall. Immunohistochemical studies show a heterogenous staining pattern for cytokeratin 5/6 and estrogen receptor and focally attenuated yet retained myoepithelial cells highlighted by smooth myosin heavy chain and p63. The overall findings are consistent with ductal adenoma, which is a benign tumor with features similar to sclerosing intraductal papilloma.
      Differential diagnosis
      • Intraductal papilloma:
        • Proliferative epithelial cells with fibrovascular cores (papillary architecture)
        • Can arise from small to medium sized peripheral ducts but also from larger subareolar ducts
      • Tubular adenoma:
        • Closely packed round and oval tubules with sparse intervening stroma
        • Histogenetically related to the pericanalicular subset of fibroadenomas
      • Fibroadenoma:
        • Biphasic tumor; proliferation of both glandular epithelial and stromal components
      Board review style question #1

      A 65 year old woman had a right breast lumpectomy showing a well circumscribed firm nodule with a lobulated and granular cut surface. Histologic features are shown in the image. Which of the following is the correct diagnosis?

      1. Ductal adenoma
      2. Fibroadenoma
      3. Intraductal papilloma
      4. Invasive ductal carcinoma
      5. Tubular adenoma
      Board review style answer #1
      A. Ductal adenoma. Ductal adenoma is a benign, well circumscribed adenomatous lesion in a duct surrounded by a densely thickened fibroelastotic wall. It has a solid architecture with an intermixture of tubules and sclerotic stroma and lacks arborizing papillary fronds. Answer C is incorrect because intraductal papillomas display arborizing papillary fronds. Answer D is incorrect because although gland distortion by sclerosis may mimic an infiltrative pattern, the myoepithelium is retained in this specimen. Answers B and E are incorrect because fibroadenomas and tubular adenomas arise from terminal duct lobular units rather than duct lumens and lack a prominent capsule. They are histogenetically related biphasic lesions, with the stromal component dominating fibroadenoma and florid adenosis-like epithelial proliferation dominating tubular adenoma.

      Comment Here

      Reference: Ductal adenoma

      Encapsulated papillary carcinoma
      Definition / general
      • Expansile papillary tumor, occurring in postmenopausal women, with low or intermediate grade nuclei surrounded by a fibrous capsule that lacks myoepithelial cells along the papillae and at the periphery or shows focal peripheral myoepithelial staining
      Essential features
      • Papillary tumor comprised of delicate papillary fronds with fibrovascular cores lined by cuboidal to columnar epithelial cells with low to intermediate grade atypia, often within a cystically dilated duct surrounded by a thick fibrous capsule
      • Lacks myoepithelial cells along the papillae and around the periphery of the tumor or only focal peripheral myoepithelial staining
      • Staged as ductal carcinoma in situ (DCIS) (i.e., Tis) in the absence of invasive carcinoma, due to indolent behavior and excellent prognosis
      • Encapsulated papillary carcinoma (EPC) with high grade cytologic atypia and increased mitotic activity are rare; they are best classified as high grade invasive breast carcinoma with features of EPC and staged as an invasive carcinoma
      Terminology
      • Intracystic papillary carcinoma (not recommended)
      • Encysted papillary carcinoma
      ICD coding
      • ICD-O:
        • 8504/2 - noninfiltrating intracystic carcinoma
        • 8504/3 - intracystic carcinoma, NOS
      • ICD-11:
        • 2E65.Y & XH9XV2 - other specified carcinoma in situ of breast & noninfiltrating intracystic carcinoma
        • 2C6Y & XH0GT6 - other specified malignant neoplasms of breast & encapsulated papillary carcinoma with invasion
      Sites
      Pathophysiology
      Etiology
      • Unknown
      Diagrams / tables

      Images hosted on other servers:

      Myoepithelium and hormone receptors in EPC

      Clinical features
      Diagnosis
      • Histologic examination of tissue removed by biopsy or surgical excision
      Radiology description
      • No distinct imaging findings can distinguish between subtypes of papillary carcinomas (AJR Am J Roentgenol 2012;198:264, Front Surg 2022;8:743881)
      • Imaging does not distinguish encapsulated papillary carcinoma (EPC) with and without invasion beyond the capsule (Breast J 2019;25:539, Breast J 2021;27:209)
      • Mammography:
        • Circumscribed to lobulated, round to oval mass
        • May have irregular, angulated or multilobulated borders
        • Calcifications uncommon
      • Ultrasound:
        • Hypoechoic solid mass, sometimes with cystic component
        • Color Doppler often highlights prominent vascular supply
      • Magnetic resonance imaging (MRI):
        • Well defined, round to oval, heterogeneous mass with varying signal intensity depending on cystic fluid content
      Radiology images

      Contributed by Mariel Molina Nunez, M.D. and Julie Jorns, M.D. (Case #518)
      Mammogram of the left breast mass

      Mammogram of the left breast mass

      Ultrasound view of the complex mass

      Ultrasound view of the complex mass



      Images hosted on other servers:

      Mammographic, sonographic and MRI findings


      Mammographic findings

      Mammographic findings

      Prognostic factors
      Case reports
      Treatment
      Gross description
      Gross images

      Images hosted on other servers:

      Papillary tumor within cystic space

      Solid tumor within cystic duct

      High grade EPC with hemorrhage

      Microscopic (histologic) description
      Microscopic (histologic) images

      Contributed by Kristen E. Muller, D.O. , Mariel Molina Nunez, M.D. and Julie Jorns, M.D. (Case #518)
      Encapsulated papillary carcinoma with invasion

      Encapsulated papillary carcinoma with invasion

      Synaptophysin

      Synaptophysin

      EPC, circumscribed with fibrous capsule EPC, circumscribed with fibrous capsule

      EPC, circumscribed with fibrous capsule

      Low grade nuclear atypia Low grade nuclear atypia

      Low grade nuclear atypia


      EPC with invasion EPC with invasion

      EPC with invasion

      EPC with DCIS and invasion

      EPC with DCIS and invasion

      High grade EPC High grade EPC

      High grade EPC

      Estrogen receptor (ER)

      Estrogen receptor (ER)


      Absent myoepithelium Absent myoepithelium

      Absent myoepithelium

      Progesterone receptor (PR)

      Progesterone receptor (PR)

      Thick fibrous capsule encircles mass

      Thick fibrous capsule encircles mass

      Surrounding encapsulation and cystic space

      Surrounding encapsulation and cystic space

      Fibrovascular cores within  epithelial proliferation

      Fibrovascular cores within epithelial proliferation


      Loss of E-cadherin in the lesion

      Loss of E-cadherin in the lesion

      Virtual slides

      Images hosted on other servers:

      Encapsulated papillary carcinoma

      Cytology description
      • No formal cytologic criteria exist to distinguish EPC from other benign papillary lesions (J Clin Pathol 2008;61:945)
        • Sensitivity and specificity for specific diagnoses is low
        • If papillary lesion is suspected on FNA, surgical biopsy is necessary for further classification
      • General findings in fine needle aspirates of EPC (Breast J 2006;12:237, J Clin Pathol 2008;61:945)
        • High cellularity, presence of cell balls, presence of slender papillae, mild cytologic atypia
      Positive stains
      Negative stains
      Molecular / cytogenetics description
      Videos

      Papillary lesions of the breast

      Sample pathology report
      • Breast, right, partial mastectomy:
        • Encapsulated papillary carcinoma (see comment)
        • Comment: Histologic sections show a circumscribed papillary tumor surrounded by a thick fibrous capsule. Cytologically, the tumor cells are columnar and cuboidal with low grade nuclear atypia. The lesion lacks myoepithelium throughout the fibrovascular cores and around the periphery. Conventional invasive carcinoma is not seen infiltrating into or beyond the fibrous capsule. In the absence of an invasive carcinomatous component, encapsulated papillary carcinomas are staged as in situ (Tis); however, some believe that they may actually be a very good prognosis subtype of invasive carcinoma due to the lack of myoepithelium.
      • Breast, right, core needle biopsy (when entire lesion is not removed or cannot be visualized):
        • Low grade papillary carcinoma (see comment)
        • Comment: The biopsy contains fragments of an atypical lesion with papillary architecture and low grade monomorphic nuclear atypia. p63 and calponin immunostains are negative for myoepithelium throughout the lesion and around the periphery; however, the periphery of the lesion is not completely visualized in the biopsy. The findings raise the possibility of a low grade encapsulated papillary carcinoma. Papillary ductal carcinoma in situ is also in the differential. A conventional invasive carcinoma is not present in this biopsy. Definitive classification is deferred to the surgical specimen.
      Differential diagnosis
      Board review style question #1

      What is the expected pattern of myoepithelial cell distribution in encapsulated papillary carcinoma of the breast?

      1. Absent both centrally within the lesion and along the periphery
      2. Absent centrally within the lesion but diffusely present along the periphery
      3. Present both centrally within the lesion and along the periphery
      4. Present only focally within the center of the lesion but diffusely present along the periphery
      Board review style answer #1
      A. Absent both centrally within the lesion and along the periphery. Encapsulated papillary carcinomas characteristically lack myoepithelial cells both within and along the edge of the lesion. The remaining answer choices apply to other papillary lesions of the breast; a benign intraductal papilloma has myoepithelial cells lining the entirety of the proliferation both centrally and peripherally, whereas a papilloma involved by DCIS and papillary DCIS will show focal to complete absence of myoepithelium centrally (corresponding to the regions of atypia) with retention peripherally.

      Comment Here

      Reference: Encapsulated papillary carcinoma
      Board review style question #2
      A 72 year old woman undergoes partial mastectomy for a 1.8 cm papillary lesion in the retroareolar tissue of her left breast. A diagnosis of encapsulated papillary carcinoma is made upon finding absent myoepithelium within the papillae and around the periphery. How should this lesion be staged, according to the AJCC TNM classification system?

      1. T0
      2. Tis
      3. T1
      4. T2
      Board review style answer #2
      B. Tis. Encapsulated papillary carcinoma is staged as an in situ lesion given its similar clinical behavior to ductal carcinoma in situ. If conventional invasive carcinoma is found, infiltrating beyond the fibrous capsule, the lesion should be staged according to the size of the invasive component only; the size of the EPC should not be included for staging.

      Comment Here

      Reference: Encapsulated papillary carcinoma
      Board review style question #3
      In which group is encapsulated papillary carcinoma of the breast most commonly seen?

      1. Childhood (≤ 18 years)
      2. Lactating / pregnant
      3. Men
      4. Postmenopausal women
      Board review style answer #3
      D. Postmenopausal women. Encapsulated papillary carcinoma (EPC) of the breast is an uncommon breast cancer that is classified as in situ, is typically low to intermediate grade and most frequently occurs in elderly, postmenopausal women rather than in younger women. EPCs constitute a slightly higher proportion of breast cancers in male patients as compared to female patients. However, male breast cancer is uncommon and is most frequently invasive ductal carcinoma (comprising about 85% of male breast cancers).

      Comment Here

      Reference: Encapsulated papillary carcinoma

      Fat necrosis
      Definition / general
      • Nonneoplastic inflammatory breast lesion characterized by necrotic adipose tissue with foamy histiocytes, chronic inflammatory cells and multinucleated giant cells
      Essential features
      • Nonneoplastic inflammatory reaction that is commonly secondary to adipose tissue injury
      • May simulate carcinoma both clinically and radiologically
      • There are different radiographical and histological stages which vary based on time from inciting event
      ICD coding
      • ICD-10: N64.1 - fat necrosis of breast
      • ICD-11: GB23.2 - fat necrosis of breast
      Epidemiology
      • 0.6% of breast excisions
      • Age range: 37 - 68 years (mean age: ~50 years) (Breast 2006;15:313)
      • Most described in perimenopausal women after trauma, surgery or biopsy
      Sites
      • Any part of the breast may be affected
      • After trauma, typically occurs at site of injury and is commonly detected in periareolar and subareolar region; it is often superficial in location since these sites are most vulnerable to trauma (Lipids Health Dis 2019;18:158)
      • In patients with no history of trauma, lesion is most often seen in upper outer quadrant of breast
      Pathophysiology
      • Many cases are due to previous biopsy procedures (such as fine needle aspiration or needle core biopsy), surgical procedure or radiation therapy (Plast Reconstr Surg 2014;134:1202)
      • May result from incidental trauma
      Etiology
      • Trauma:
      • Iatrogenic injury:
        • Fine needle aspiration, core needle biopsy, mastectomy or lumpectomy of breast, autologous fat injection, reduction mammoplasty, breast implants (Breast Cancer 2019;26:125)
      • Radiation therapy:
        • Due to postradiation vascular damage with subsequent ischemia, including external beam, MammoSite, iridium implantation, intraoperative radiation
      • Other rare causes:
        • Anticoagulant treatment (warfarin [Coumadin]), polyarteritis nodosa, Weber-Christian disease, granulomatous angiopanniculitis, lupus mastitis, injection of methylene blue dye
      • Unknown cause in ~50% of patients
      Clinical features
      • Typically present with a painless superficial solitary mass which may mimic cancer, especially in patients who have undergone breast conserving surgery and radiation therapy (Lipids Health Dis 2019;18:139)
      • Firm and relatively circumscribed on palpation, may span 2 cm in diameter (average)
      • Occasionally can be associated with skin changes: bruising and tenderness, tethering or dimpling of the overlying skin if the lesion is superficially located, nipple retraction
      Diagnosis
      • Palpation of breast mass, radiographical findings and correlation with clinical history of trauma or breast procedures (Lipids Health Dis 2019;18:139)
      • Core needle biopsy is frequently performed if there are masses or calcifications
      • Tissue diagnosis needs to correlate with imaging findings and must be used to distinguish from fat necrosis next to another lesion
      Radiology description
      • Many cases can be identified by radiographic appearance (e.g., forming mammographic masses or mammographic calcifications)
      • Manifestations on mammography, ultrasonography and MRI depend upon its stage of evolution; features can be variable and may be indistinguishable from carcinoma (Radiol Res Pract 2015:613139)
      • Time from inciting event may be useful in correlating radiographic findings (Br J Radiol 2018;91:20180213)
      • Mammography usually reveals ill defined irregular mass with or without calcifications or focal asymmetries around central area of necrotic adipose tissue
      • May contain punctate, irregular or coarse pleomorphic calcifications
      • Sonography demonstrates a discrete mass which may be well circumscribed or ill defined
        • Sonographic appearance is variable and is usually associated with distortion of the parenchymal architecture
      • Less frequently, the lesion may appear as a lipid (oil) cyst which consists of a circumscribed, round or oval radiolucent oil filled cyst that is partly calcified
        • Oil cysts typically show posterior acoustic shadowing by sonography
      • Eggshell calcifications are late findings with characteristic appearance around central area of necrotic adipose tissue
      • MRI demonstrates a wide spectrum of findings
        • Necrotic fat usually shows low signal intensity on T1 weighted MRI
        • Fibrosis may be seen as architectural distortion
      Radiology images

      Images hosted on other servers:

      Contraction
      and
      microcalcification

      Fat necrosis within surgical site

      Prognostic factors
      Case reports
      Treatment
      • Biopsy the lesion to confirm the diagnosis if clinical / radiological features can not distinguish from cancer
      • Follow up with imaging and clinical examination to detect occult carcinoma that might be masked by coexisting fat necrosis
      • Surgical excision is usually not necessary after the diagnosis has been established, only for painful or aesthetically undesirable lesions (StatPearls: Breast Fat Necrosis [Accessed 1 May 2020])
      Clinical images

      Images hosted on other servers:

      Skin and fat necrosis due to methylene blue

      Gross description
      • Varies based on time from injury
        • Early lesion has the appearance of hemorrhage in indurated fat (Radiol Res Pract 2015:613139)
          • Cut surface of the lesion has a variegated, yellow-gray appearance with focally reddish color
        • After several weeks, it appears as a firm yellow-gray nodule / mass with a demarcated border
          • Mass may retract surrounding parenchyma, cystic degeneration may develop in the center, showing a cavity containing oily fluid secondary to necrotic fat with calcification in the cystic walls
        • At a later stage, the lesion may show a dense scar that may appear like a mass
      Gross images

      Contributed by Elizabeth Abels, M.D.

      Fat necrosis within biopsy site

      Frozen section description
      • Frozen sections are difficult to obtain due to high fat content of specimen
      Microscopic (histologic) description
      • Histiocytes are relatively uniform in size with round nuclei, fine intracytoplasmic vacuoles and no indentation of the nucleus
      • Varies based on time from injury
        • Early stage:
          • Necrosis of adipocytes (fine vacuolization, diminished size and dropout)
          • Focal hemorrhage in fat (AJR Am J Roentgenol 2009;192:815)
          • Cystic degeneration may result in cavity with or without calcifications in cyst walls
          • Formation of demarcated distinct yellow-gray and focally reddish mass over several weeks
        • Intermediate stage:
          • Mixed inflammatory cell infiltrate of histiocytes, lymphocytes, plasma cells, sometimes with eosinophils
          • Lipid laden foamy histiocytes and multinucleated foreign body type giant cells are due to phagocytosis of necrotic adipocytes
          • Hemosiderin deposition
          • Early fibrosis with hypercellular areas of fibroblastic proliferation
        • Late stage:
          • Varying degrees of fibrosis and calcifications may remain for years
          • Fibrosis contract into a scar and may result in irregular mass
          • Loculated necrotic fat within a cyst surrounded by dense fibrous tissue (oil cyst) with dense calcification in the walls of cysts (eggshell calcifications)
          • Reactive squamous metaplasia may develop in the epithelium of ducts and lobules in the area of lesion
      Microscopic (histologic) images

      Contributed by Jing He, M.D.
      Disruption of adipocytes

      Disruption of adipocytes

      Mixed inflammation

      Mixed inflammation

      Mixed inflammatory cells

      Mixed inflammatory cells

      Hemorrhage and hemosiderin

      Hemorrhage and hemosiderin

      Hemosiderin and giant cells

      Hemosiderin and giant cells


      Multinucleated giant cells

      Multinucleated giant cells

      Lipid laden histiocytes and hemorrhage

      Lipid laden histiocytes and hemorrhage

      Lipid laden histiocytes Lipid laden histiocytes

      Lipid laden histiocytes

      Mass formation

      Mass formation


      Necrotic adipocytes

      Necrotic adipocytes

      Cystic degeneration Cystic degeneration

      Cystic degeneration

      Early fibrosis

      Early fibrosis

      Oil cyst

      Oil cyst


      Late stage fibrosis

      Late stage fibrosis

      Eggshell calcifications

      Eggshell calcifications

      CD68 CD68

      CD68

      AE1 / AE3

      AE1 / AE3

      Cytology description
      • Characterized by lipid laden histiocytes (lipophages), multinucleated giant cells and degenerating adipocytes (AJR Am J Roentgenol 2009;192:815)
      • Lipid laden macrophages demonstrate abundant foamy cytoplasm with variably sized clear lipid vacuoles and small, bland nuclei
      • Histiocytes are relatively uniform in size with round nuclei, fine intracytoplasmic vacuoles and no indentation of the nucleus
      • Degenerating adipocytes are characterized by aggregates of adipocytes that have lost their nuclei and have finely granular cytoplasm instead of the clear cytoplasm that is characteristic of lipid
      • Reactive and reparative changes involving fibroblasts and histiocytes can result in cells with large pleomorphic nuclei and prominent nucleoli
      • Background may show scattered inflammatory cells, hemosiderin laden macrophages, globules and acellular lipid
      • Scant clusters of ductal epithelial cells are usually cytologically bland but may exhibit cytologic atypia with pleomorphic nuclei and prominent nucleoli
      Cytology images

      Images hosted on other servers:

      Stages of fat necrosis

      Positive stains
      Negative stains
      Videos

      Breast traumatic fat necrosis

      Breast inflammatory conditions

      Sample pathology report
      • Breast, right, retroareolar, core biopsy:
        • Fat necrosis
      Differential diagnosis
      • Breast carcinoma:
        • May have abundant foamy cytoplasm and small bland nuclei that can mimic histiocytes
        • Usually demonstrates more nuclear atypia
        • Cytokeratin+, CD68-
      • Lipomatous neoplasm:
        • Vacuolated cytoplasm and hyperchromatic, indented or often scalloped nuclei
        • S100+, CD34+
      • Granular cell tumor:
        • Infiltrating tumor cells with abundant eosinophilic granular cytoplasm
        • Minimal to no inflammatory infiltrate
        • S100+, CD68 variable
      • Amyloidosis (Pathol Res Pract 2019;215:152699):
        • Can present as mass or calcifications with amyloid surrounding adipocytes
        • Usually associated with systemic amyloidosis and hematologic disorder such as plasmacytoma
        • No chronic inflammatory cell infiltrate
        • Amyloid deposits within blood vessel walls and causes thickening of basement membranes and vascular walls
        • Congo red apple green birefringence under polarized microscopy
      • Erdheim-Chester disease:
        • Infrequent xanthomatous form of non Langerhans cell histiocytosis
        • Usually synchronous nonmammary subcutaneous nodules, osseous and orbital lesions
        • Infiltration of foamy histiocytes, Touton type giant cells, plasma cells and epithelioid granuloma
        • CD68+, factor XIIIa (strong)+, S100 variable, CD1a-
      Board review style question #1

        A 65 year old woman has a clinical history of right breast carcinoma treated with local resection and radiation therapy. Her mammogram showed an ill defined irregular mass with calcifications within the right breast. A stereotactic core biopsy shows microcalcifications within the lesion. Which of the following is true about this lesion?

      1. Continue routine surveillance for this patient
      2. Negative CD68 and positive AE1 / AE3 are characteristic of this lesion
      3. The patient has recurrent breast carcinoma
      4. The patient requires surgical excision of this lesion followed by chemotherapy
      Board review style answer #1
      A. Continue routine surveillance for this patient. Fat necrosis of the breast is a possible sequela of breast tissue trauma or surgical procedure. This patient likely developed fat necrosis after her initial resection and radiation therapy. When the diagnosis is unclear from radiology, biopsy is necessary to rule out cancer. The biopsy section shows foamy lipid laden histiocytes, mixed inflammatory cells infiltrate and disruption of adipocytes, associated with microcalcification. Positive CD68 and negative AE1 / AE3 can help to distinguish fat necrosis from recurrent carcinoma of breast. Fat necrosis is a nonneoplastic inflammatory process and usually surgical treatment is not required.

      Comment Here

      Reference: Fat necrosis of breast
      Board review style question #2
        A 45 year old woman developed a palpable mass in the subareolar area of the left breast. Mammogram shows a cystic lesion with eggshell calcifications in the cyst wall. She reports a recent history of a car accident in which she had minor injuries and was wearing her seatbelt. Which of the following is the most likely sequela of the lesion?

      1. The cyst will often become infected and require surgical drainage
      2. The lesion will be replaced by varying degrees of fibrosis
      3. The lesion will eventually progress to invasive carcinoma if left untreated
      4. The lesion will grow and compress surrounding tissue
      Board review style answer #2
      B. The lesion will be replaced by varying degrees of fibrosis. Fat necrosis has three stages of evolution. The first stage is characterized by disruption of adipocytes following an inciting event. This can lead to local cyst formation with or without calcifications. In the intermediate stage, inflammatory cells phagocytose the necrotic adipocytes and fibroblasts proliferate. Eventually the fibroblasts replace the necrotic areas with varying degrees of fibrosis. Fat necrosis is a nonneoplastic inflammatory lesion that usually resolve or regress over time.

      Comment Here

      Reference: Fat necrosis of breast

      Fibroadenoma
      Definition / general
      • Benign biphasic tumor composed of a proliferation of both glandular epithelial and stromal components of the terminal duct lobular unit
      Essential features
      • Most common breast tumor in adolescent and young women
      • Benign biphasic tumor comprised of glandular epithelium and specialized interlobular stroma of the terminal ductal lobular unit (Pathol Annu 1994;29:1)
      • Can show a spectrum of histologic appearances; generally uniform in stromal cellularity and distribution of glandular and stromal elements within a given lesion (an important distinction from phyllodes tumor)
      • Fibroadenomas with hypercellular stroma and prominent intracanalicular pattern can show morphologic overlap with benign phyllodes tumors, especially in needle biopsy specimens
      Terminology
      • Fibroadenoma, usual type fibroadenoma, adult type fibroadenoma
      • Myxoid fibroadenoma
      • Juvenile fibroadenoma
      • Complex fibroadenoma
      ICD coding
      • ICD-10: D24 - benign neoplasm of breast
      Epidemiology
      Sites
      • Breast
      • Can occur in axilla accessory breast tissue
      Etiology
      Clinical features
      • Often presents as painless, firm, mobile, slow growing mass
      • Usually solitary, can be multiple and bilateral
      • Usually less than 3 cm in diameter but may grow to large size (AJR Am J Roentgenol 2008;190:214)
      Diagnosis
      • Histologic examination of involved tissue
      Radiology description
      Radiology images

      Contributed by Mark R. Wick, M.D., Azadeh Khayyat, M.D. and Julie M. Jorns, M.D. (Case #533)
      Mammogram

      Mammogram



      Images hosted on other servers:

      Juvenile fibroadenoma

      Case reports
      Treatment
      • Management depends on patient risk factors and patient preference
      • Conservative management with close clinical followup, especially if concordant radiology findings (Br J Surg 1987;74:857)
      • Local surgical excision, especially if symptomatic (Am J Surg Pathol 1985;9:730)
      • If atypia / neoplasia is found within a fibroadenoma, the surgical and systemic therapeutic management is specific and appropriate to the primary atypical / neoplastic lesion
      Gross description
      • Firm, well circumscribed, ovoid mass with bosselated surface, lobulations bulge above the cut surface, slit-like spaces
      • May have mucoid or fibrotic appearance; can be calcified
      Gross images

      Contributed by Gary Tozbikian, M.D.

      Well circumscribed tumor with bulging cut surface

      Microscopic (histologic) description
      • Well circumscribed, unencapsulated
      • Biphasic tumor, proliferation of both glandular and stromal elements
      • Glandular component
        • 2 recognized growth patterns (of no clinical significance, both patterns may occur within a single lesion)
          • Intracanalicular: glands are compressed into linear branching structures by proliferating stroma
          • Pericanalicular: glands retain open lumens but are separated by expanded stroma
        • Glandular elements have intact myoepithelial cell layer
        • Often associated with usual type ductal hyperplasia, apocrine metaplasia, cyst formation or squamous metaplasia
        • Rare mitotic activity can be observed in the glandular component, has no clinical significance
      • Stromal component
        • Generally uniform cellularity within a given lesion
        • Collagen and bland spindle shaped stromal cells with ovoid or elongated nuclei
        • Usually no mitotic activity; rare mitotic activity may be present in young or pregnant patients (Breast J 2017;23:182)
        • No stromal atypia
        • Stroma may show myxoid change or hyalinization
        • May show multinucleated giants cells (Am J Surg Pathol 1986;10:823, Arch Pathol Lab Med 1994;118:912)
        • Rarely benign heterologous stromal elements (adipose, smooth muscle, osteochondroid metaplasia)
        • Fibroadenomas may be involved by mammary neoplasia (e.g. invasive breast carcinoma, ductal carcinoma in situ, lobular carcinoma in situ) and atypical epithelial proliferations (e.g. atypical ductal hyperplasia, atypical lobular hyperplasia) often as a result of spread from an adjacent lesion
      • Variants
        • Myxoid fibroadenoma
          • Similar structure but with prominent myxoid stromal change composed of abundant pale, blue-gray extracellular matrix material
        • Complex fibroadenoma
          • Cysts > 3 mm, sclerosing adenosis, epithelial microcalcifications or papillary apocrine metaplasia (N Engl J Med 1994;331:10)
        • Cellular fibroadenoma
          • Diffuse stromal hypercellularity
        • Juvenile fibroadenoma
          • Increased stromal cellularity
          • Increased epithelial hyperplasia with gynecomastoid-like micropapillary projections
          • Fascicular stromal arrangement
          • Pericanalicular growth pattern
          • May show rapid growth and large size (Am J Surg Pathol 1985;9:730)
      Microscopic (histologic) images

      Contributed by Gary Tozbikian, M.D., Debra Zynger, M.D., Azadeh Khayyat, M.D. and Julie M. Jorns, M.D. (Case #533)

      Fibroadenoma

      Fibroadenoma

      Complex fibroadenoma



      Myxoid fibroadenoma

      Juvenile fibroadenoma



      Cellular fibroadenoma

      Fibroadenoma with ALH



      Fibroadenoma with atypical ductal hyperplasia

      Fibroadenoma with atypical ductal hyperplasia



      Multinucleated stromal cells


      Ductal carcinoma in situ (DCIS), solid type, with microcalcifications, confined to a fibroadenoma Ductal carcinoma in situ (DCIS), solid type, with microcalcifications, confined to a fibroadenoma Ductal carcinoma in situ (DCIS), solid type, with microcalcifications, confined to a fibroadenoma Ductal carcinoma in situ (DCIS), solid type, with microcalcifications, confined to a fibroadenoma Ductal carcinoma in situ (DCIS), solid type, with microcalcifications, confined to a fibroadenoma

      Ductal carcinoma in situ (DCIS), solid type, with microcalcifications, confined to a fibroadenoma; lobular neoplasia (atypical lobular hyperplasia / lobular carcinoma in situ), with involvement of a fibroadenoma

      Virtual slides

      Contributed by Andrey Bychkov, M.D., Ph.D.

      Fibroadenoma

      Cytology description
      Cytology images

      Contributed by Areej M. Al Nemer, M.D.
      Branching sheets

      Branching sheets

      Bare nuclei

      Bare nuclei

      Bland stromal fragments Bland stromal fragments Bland stromal fragments

      Bland stromal fragments



      Images hosted on other servers:
      Diff-Quik

      Diff-Quik

      Pap

      Pap

      Negative stains
      Molecular / cytogenetics description
      Videos

      Fibroadenoma

      Sample pathology report
      • Breast, right, 2:00 zone 2, ultrasound guided core biopsy:
        • Fibroadenoma
      Differential diagnosis
      Board review style question #1


      A 13 year old girl presents with a firm, palpable 3.5 cm breast mass. She undergoes an ultrasound guided biopsy and subsequent local excision showing a fibroepithelial lesion with circumscribed borders, uniformly increased stromal cellularity, glandular proliferation with pericanalicular growth pattern and prominent gynecomastoid-like usual type ductal hyperplasia. Rare mitotic activity (< 1/10 high power fields) is identified. Stromal cytologic atypia is not present. What is the correct diagnosis?

      1. Fibroadenoma
      2. Juvenile fibroadenoma
      3. Myxoid fibroadenoma
      4. Phyllodes tumor (benign)
      5. Phyllodes tumor (borderline)
      Board review style answer #1
      B. Juvenile fibroadenoma. The clinical presentation and histologic features are characteristic of a juvenile fibroadenoma.

      Comment Here

      Reference: Fibroadenoma
      Board review style question #2

      A 25 year old woman presents with a 3 cm breast mass. An ultrasound guided biopsy shows a fibroepithelial lesion. Which histologic feature is consistent with a diagnosis of fibroadenoma?

      1. Circumscribed borders
      2. Marked stromal cytologic atypia
      3. Stromal condensation around glandular structures
      4. Stromal mitotic activity (7 - 8/10 high power fields)
      5. Stromal overgrowth
      Board review style answer #2
      A. Circumscribed borders. Fibroadenomas generally show noninfiltrative, circumscribed borders. The histologic features in answer choices B, C, D and E are observed in phyllodes tumors.

      Comment Here

      Reference: Fibroadenoma
      Board review style question #3

      Which of the following is true about the breast lesion shown above?

      1. Displays infiltrative margins
      2. Most common benign tumor arising in the breast
      3. Occurs primarily in postmenopausal women
      4. Often has a brisk mitotic rate
      5. Stromal overgrowth is a key finding
      Board review style answer #3
      B. Most common benign tumor arising in the breast. This is a fibroadenoma.

      Comment Here

      Reference: Fibroadenoma
      Board review style question #4
      Fibroadenomas harbor epithelial atypia or malignancy at approximately what rate?

      1. 0.2%
      2. 2%
      3. 20%
      4. 50%
      Board review style answer #4
      B. 2%. The classification and behavior of fibroepithelial tumors of the breast including fibroadenoma and phyllodes tumor is driven by the stroma. However, epithelial changes may be seen and are thought to occur by chance or are driven by unrelated risk factor(s). Thus, epithelial atypia and malignancy is uncommon, with a recent large study by Krishnamurthy et al. showing a rate of ~2% (Breast Dis 2019;38:97). See Case #533 for more information.

      Comment Here

      Reference: Fibroadenoma

      Fibroadenomatoid change
      Definition / general
      • Benign, often incidental finding in a background of fibrocystic changes
      • Lesion with features resembling a fibroadenoma but lacking sharp circumscription
      Essential features
      • Proliferation of intralobular stroma with the formation of stromal nodularity that often appears to blend in with the surrounding breast tissue
      • Morphologically reminiscent of a fibroadenoma but not forming a well circumscribed mass
      • May represent an incipient fibroadenoma
      • Often multifocal and surrounded by fibrocystic changes
      Terminology
      • Also known as fibroadenomatous change, fibroadenomatoid hyperplasia, fibroadenomatoid mastopathy, fibroadenomatosis, sclerosing lobular hyperplasia
      • Less frequently referred to as mixed lesion, reflecting the common association of fibroadenomatoid change and fibrocystic changes
      ICD coding
      • ICD-10: N60.2 - fibroadenosis of breast
      Epidemiology
      Sites
      Pathophysiology
      • Proliferation of intralobular stroma
      • May represent a stage in the evolution of a fibroadenoma, produced by the coalescence of fibroadenomatoid nodules
      • Reference: Hum Pathol 1984;15:336
      Etiology
      • Unknown; possibly due to or influenced by reproductive hormones, similar to fibroadenoma
      Diagrams / tables
      No information provided
      Clinical features
      • May be detected by imaging modalities (see Radiology description below) as a palpable mass or incidentally in breast tissue sampled for a different lesion
      Diagnosis
      Laboratory
      No information provided
      Radiology description
      Radiology images

      Images hosted on other servers:
      Well circumscribed nodule

      Well circumscribed nodule

      Suspicious granular microcalcifications

      Suspicious granular microcalcifications

      Prognostic factors
      • Widely considered to have no increased risk of malignancy
      • Single study reported co-occurrence of fibroadenomatoid change and HER2 negative invasive breast carcinoma (PLoS One 2015;10:e0129500)
      Case reports
      Treatment
      • No treatment required
      Clinical images
      No information provided
      Gross description
      • Lesion appears dense and poorly circumscribed on cut section
      • Difficult to distinguish from fibrocystic changes by gross examination
      • Reference: Malays J Pathol 1991;13:101
      Gross images
      No information provided
      Frozen section description
      No information provided
      Frozen section images
      No information provided
      Microscopic (histologic) description
      Microscopic (histologic) images

      Contributed by Melissa Alexander, M.D., Ph.D.
      Edges blending with surrounding Edges blending with surrounding

      Edges blending with surrounding

      Vague stromal nodularity

      Vague stromal nodularity

      Poorly circumscribed

      Poorly circumscribed

      Background fibrocystic change

      Background fibrocystic change

      Multiple nodules

      Multiple nodules


      Multiple nodules

      Multiple nodules

      Loose stroma versus dense hyalinized

      Loose stroma versus dense hyalinized

      Dense stroma

      Dense stroma

      Loose versus dense nodules

      Loose versus dense nodules

      Loose stroma, compressed glands

      Loose stroma, compressed glands

      Fibroadenomatoid nodule, compressed glands

      Fibroadenomatoid nodule, compressed glands

      Virtual slides
      No information provided
      Cytology description
      • Elements similar to fibroadenoma, including stromal fragments; however, lacking other elements of fibroadenoma, including branching antler horn epithelial clusters (Cytojournal 2006;3:8, Acta Cytol 2001;45:765)
      Cytology images
      No information provided
      Immunofluorescence description
      No information provided
      Immunofluorescence images
      No information provided
      Positive stains
      No information provided
      Negative stains
      No information provided
      Electron microscopy description
      No information provided
      Electron microscopy images
      No information provided
      Molecular / cytogenetics description
      No information provided
      Molecular / cytogenetics images
      No information provided
      Videos
      No information provided
      Sample pathology report
      • Breast, right, core biopsy:
        • Fibroadenomatoid change associated with calcifications (see comment)
        • Comment: Correlation with specimen radiograph was performed.
      Differential diagnosis
      • Fibroadenoma:
        • Well circumscribed mass comprised of stromal and epithelial components sharply demarcated from the surrounding breast parenchyma
      Board review style question #1

      A poorly circumscribed nodular focus was identified in a breast needle core biopsy, shown in the image above. Which of the following is the best diagnosis for this histologic finding?

      1. Apocrine metaplasia
      2. Duct ectasia
      3. Columnar cell change
      4. Fibroadenomatoid change
      5. Microcalcifications
      Board review style answer #1
      D. Fibroadenomatoid change

      Comment Here

      Reference: Fibroadenomatoid change
      Board review style question #2
      Which of the following is true regarding fibroadenomatoid change?

      1. Arises from interlobular stroma
      2. It is considered a precursor lesion to invasive carcinoma
      3. Often presents in a background of fibrocystic changes
      4. Sharply circumscribed lesion on cut section
      5. Treatment is surgical excision
      Board review style answer #2
      C. Often presents in a background of fibrocystic changes

      Comment Here

      Reference: Fibroadenomatoid change

      Fibroadenomatoid change
      Definition / general
      • Benign, often incidental finding in a background of fibrocystic changes
      • Lesion with features resembling a fibroadenoma but lacking sharp circumscription
      Essential features
      • Proliferation of intralobular stroma with the formation of stromal nodularity that often appears to blend in with the surrounding breast tissue
      • Morphologically reminiscent of a fibroadenoma but not forming a well circumscribed mass
      • May represent an incipient fibroadenoma
      • Often multifocal and surrounded by fibrocystic changes
      Terminology
      • Also known as fibroadenomatous change, fibroadenomatoid hyperplasia, fibroadenomatoid mastopathy, fibroadenomatosis, sclerosing lobular hyperplasia
      • Less frequently referred to as mixed lesion, reflecting the common association of fibroadenomatoid change and fibrocystic changes
      ICD coding
      • ICD-10: N60.2 - fibroadenosis of breast
      Epidemiology
      Sites
      Pathophysiology
      • Proliferation of intralobular stroma
      • May represent a stage in the evolution of a fibroadenoma, produced by the coalescence of fibroadenomatoid nodules
      • Reference: Hum Pathol 1984;15:336
      Etiology
      • Unknown; possibly due to or influenced by reproductive hormones, similar to fibroadenoma
      Clinical features
      • May be detected by imaging modalities (see Radiology below), as a palpable mass or incidentally in breast tissue sampled for a different lesion
      Diagnosis
      Radiology description
      • Hypoechoic mass
      • Asymmetric increased density
      • Suspicious granular clustered microcalcifications (more frequent in women > 50 years old)
      • No suspicious findings in patient with palpable mass
      • Imaging findings in one study of fibroadenomatoid hyperplasia showed 53% were well defined mass
        • 33% had normal mammographic findings
        • 13% had an asymmetric density
        • 1% with mammographic calcifications
      • References: AJR Am J Roentgenol 1995;165:291, AJR Am J Roentgenol 1998;171:1331, Hum Pathol 1984;15:336
      Radiology images

      Images hosted on other servers:
      Well circumscribed nodule

      Well circumscribed nodule

      Suspicious granular microcalcifications

      Suspicious granular microcalcifications

      Prognostic factors
      • No increased risk of malignancy
      Case reports
      Treatment
      • No treatment required
      Gross description
      • Lesion appears dense and poorly circumscribed on cut section
      • Difficult to distinguish from fibrocystic changes by gross examination
      • Reference: Malays J Pathol 1991;13:101
      Microscopic (histologic) description
      Microscopic (histologic) images

      Contributed by Melissa Alexander, M.D., Ph.D.
      Edges blending with surrounding Edges blending with surrounding

      Edges blending with surrounding

      Vague stromal nodularity

      Vague stromal nodularity

      Poorly circumscribed

      Poorly circumscribed

      Background fibrocystic change

      Background fibrocystic change

      Multiple nodules

      Multiple nodules


      Multiple nodules

      Multiple nodules

      Loose stroma versus dense hyalinized

      Loose stroma versus dense hyalinized

      Dense stroma

      Dense stroma

      Loose versus dense nodules

      Loose versus dense nodules

      Loose stroma, compressed glands

      Loose stroma, compressed glands

      Fibroadenomatoid nodule, compressed glands

      Fibroadenomatoid nodule, compressed glands

      Cytology description
      • Elements similar to fibroadenoma, including stromal fragments; however, lacking other elements of fibroadenoma including branching antler horn epithelial clusters (Cytojournal 2006;3:8, Acta Cytol 2001;45:765)
      Sample pathology report
      • Breast, right, core biopsy:
        • Fibroadenomatoid change associated with calcifications (see comment)
        • Comment: Correlation with specimen radiograph was performed.
      Differential diagnosis
      • Fibroadenoma:
        • Well circumscribed mass comprised of stromal and epithelial components sharply demarcated from the surrounding breast parenchyma
      Board review style question #1

      A poorly circumscribed nodular focus was identified in a breast needle core biopsy, shown in the image. The best diagnosis for this histologic finding is which of the following?

      1. Apocrine metaplasia
      2. Duct ectasia
      3. Columnar cell change
      4. Fibroadenomatoid change
      5. Microcalcifications
      Board review style answer #1
      D. Fibroadenomatoid change

      Comment Here

      Reference: Fibroadenomatoid change

      Fibromatosis
      Definition / general
      • Low grade infiltrative spindle cell neoplasm composed of fibroblasts and myofibroblasts
      Essential features
      • Rare low grade neoplasm with infiltrative pattern
      • Bland spindle cells arranged in long intersecting fascicles within a collagenous stroma
      • Characterized by alterations in WNT / beta catenin pathway
      • Does not metastasize but can locally recur after surgical excision
      Terminology
      • Desmoid tumor, aggressive fibromatosis, desmoid type fibromatosis, extra-abdominal desmoid
      ICD coding
      • ICD-O: 8821/1 - desmoid type fibromatosis
      • ICD-11: 2F75 & XH13Z3 - neoplasms of uncertain behavior of breast & aggressive fibromatosis
      Epidemiology
      Sites
      • May be primary to the breast or be secondary extension into the breast from the chest wall
      Pathophysiology
      • Up to 95% harbor CTNNB1 activating mutation (sporadic); APC gene inactivating mutation responsible for remainder of cases (hereditary) (Cancers (Basel) 2020;12:1851)
      • Through the WNT / beta catenin pathway, these mutations motivate cells to proliferate while destabilizing beta catenin that then accumulates within the nucleus (J Clin Oncol 2018;36:202)
      Etiology
      Clinical features
      • Slow growing, often self detected firm mass
      Diagnosis
      • May arise as palpable mass that appears as abnormality on mammography
      • Rarely presents with breast pain, nipple discharge and skin retraction (Breast Care (Basel) 2021;16:77)
      Radiology description
      • Imaging may show well circumscribed mass or poorly defined / spiculated mass that can be mistaken for malignant process (Breast Care (Basel) 2021;16:77)
      Radiology images

      Contributed by Carissa LaBoy, M.D.
      Mammogram of right breast

      Mammogram of right breast

      Ultrasound of right breast

      Ultrasound of right breast

      Mammogram of breast mass

      Mammogram of breast mass

      Ultrasound of left breast

      Ultrasound of left breast

      Prognostic factors
      Case reports
      Treatment
      Gross description
      • Well to poorly circumscribed mass of variable size (from < 1 cm to > 10 cm), with white-gray whorled or trabeculated cut surface
      Gross images

      Images hosted on other servers:

      White trabeculated cut surface

      Missing Image

      Chest wall mass

      Frozen section description
      • Bland spindle cells seen on intraoperative frozen section would prompt benign diagnosis
      Microscopic (histologic) description
      • Long intersecting fascicles composed of bland spindle cells with indistinct borders, hyperchromatic nuclei with occasional nucleoli and eosinophilic cytoplasm
      • Cells infiltrate normal ducts and lobules, adipose tissue and skeletal muscle, thus mimicking invasive process
      • Mitosis is rare
      • Background stroma has thickened collagen resembling keloid
      • Older lesions may calcify
      • Lymphoid aggregates may be seen at the periphery of the lesion
      • Reference: Pathologica 2019;111:344
      Microscopic (histologic) images

      Contributed by Carissa LaBoy, M.D.
      Infiltrative breast mass

      Infiltrative breast mass

      Cells entrapping normal ducts

      Cells entrapping normal ducts

      Cells in collagenous stroma

      Cells in collagenous stroma

      Bland spindle cells

      Bland spindle cells

      Positive margin on excision

      Positive margin on excision


      Beta catenin positivity

      Beta catenin positivity

      Beta catenin nuclear positivity

      Beta catenin nuclear positivity

      CD34 negative spindle cells

      CD34 negative spindle cells

      Pancytokeratin negative spindle cells

      Pancytokeratin negative spindle cells

      Cytology description
      • Fine needle aspiration is difficult and often yields fibrous nondiagnostic material
      • When successful, fine needle aspiration shows bland spindle cells with hyperchromatic nuclei and wispy cytoplasm within abundant stroma (Cancer 2007;111:166)
      Cytology images

      Images hosted on other servers:

      Fine needle aspiration of fibromatosis

      Fine needle aspiration of fibromatosis

      Positive stains
      Molecular / cytogenetics description
      Videos

      Review of fibromatosis

      Sample pathology report
      • Left breast, needle localized lumpectomy:
        • Fibromatosis, measuring 2.8 cm in greatest dimension
        • Margins negative with tumor > 0.5 cm from all margins
      Differential diagnosis
      • Scar:
        • Focally infiltrative lesion composed of bland spindle cells with associated histiocytes, foreign body giant cell reaction and lymphocytes
        • History of prior procedure
      • Nodular fasciitis:
        • Focally infiltrative lesion composed of spindle cells forming short fascicles or storiform pattern with increased mitotic activity
        • Does not display nuclear beta catenin staining
      • Dermatofibrosarcoma protuberans:
        • Focally infiltrative lesion with spindle cells in storiform growth pattern and low mitotic activity
      • Low grade fibromatosis-like metaplastic carcinoma:
        • Infiltrative lesion composed of spindle to epithelioid cells arranged in small clusters with low mitotic activity
        • Cytokeratin and p63 positive
      • Low grade myofibroblastic sarcoma:
        • Focally infiltrative lesion composed of spindle cells arranged in fascicles
        • Mild to moderate nuclear pleomorphism and brisk mitosis
      • Myofibroblastoma:
        • Spindle cells arranged in short, disorganized fascicles with intersecting thick collagen bands resembling keloid; do not typically entrap normal ducts or lobules
      • Benign phyllodes tumor:
        • Well circumscribed mass with hyperplastic epithelium within a hypercellular stroma composed of spindle cells with mild atypia and low mitotic activity
      Board review style question #1

      What is the pattern of beta catenin immunohistochemical staining for fibromatosis pictured above?

      1. Cytoplasmic
      2. Cytoplasmic and membranous
      3. Membranous
      4. Nuclear
      Board review style answer #1
      D. Nuclear. The pathophysiology of fibromatosis involves an activating CTNNB1 mutation in sporadic cases and an APC gene inactivating mutation in hereditary cases. Both types of mutations activate the WNT / beta catenin pathway, motivating cells to proliferate while degrading beta catenin, which then accumulates within the nucleus. This in turn yields a nuclear pattern of staining by immunohistochemistry.

      Comment Here

      Reference: Fibromatosis
      Board review style question #2
      Which of the following is correct regarding the histologic features and clinical behavior of breast fibromatosis?

      1. Chemotherapy is the preferred treatment due to high mitotic activity
      2. Metastasis is a common feature of this tumor
      3. Necrosis and pleomorphism make this a high grade tumor
      4. Tumor has a high frequency of local recurrence
      Board review style answer #2
      D. Tumor has a high frequency of local recurrence. Breast fibromatosis is a low grade tumor consisting of bland spindle cells with rare or no mitotic activity and no metastatic potential. However, due to its infiltrative nature, this tumor has a tendency to locally recur after surgical excision.

      Comment Here

      Reference: Fibromatosis

      Fibromatosis-like
      Definition / general
      • Variant of metaplastic carcinoma with low grade, bland spindle cells histologically resembling fibromatosis
      • May arise de novo or in association with papillary or sclerosing lesions
      • High propensity for local recurrence but lymph node and distant metastases are rare
      • May represent a distinct subtype as these tumors do not share similar genomic alterations with other types of metaplastic carcinomas
      Essential features
      • Variant of metaplastic carcinoma composed of cytologically bland spindle cells resembling fibromatosis within abundant stromal collagen
      • Immunohistochemical stains needed to establish epithelial differentiation (cytokeratins, p63) since in situ and conventional type invasive breast carcinoma are typically absent
      Terminology
      • Low grade fibromatosis-like metaplastic carcinoma
      • Fibromatosis-like spindle cell metaplastic carcinoma
      ICD coding
      • ICD-O: 8575/3 - metaplastic carcinoma, NOS
      • ICD-11: 2C6Y & XH0RD4 - metaplastic carcinoma of breast & metaplastic carcinoma, NOS
      Epidemiology
      Sites
      • Breast, unilateral
      Pathophysiology
      • Unknown; may arise in association with papillary or sclerosing lesions
      Etiology
      • Unknown
      Clinical features
      Diagnosis
      • Core needle biopsy or surgical excision
      Radiology description
      • Variable appearance: round, oval, lobular or irregularly shaped; densely solid, heterogeneous or cystic parenchyma; well circumscribed or irregular / spiculated shaped (Arch Pathol Lab Med 2015;139:552)
      • Calcifications uncommon
      Radiology images

      Images hosted on other servers:

      Mammogram, microlobulated high density mass

      Ultrasound, hypoechoic mass

      MRI, irregular mass with heterogeneous enhancement

      Prognostic factors
      Case reports
      Treatment
      Gross description
      Microscopic (histologic) description
      Microscopic (histologic) images

      Contributed by Kristen E. Muller, D.O. and Julie M. Jorns, M.D.
      Fibromatosis-like metaplastic carcinoma

      Fibromatosis-like metaplastic carcinoma

      Squamous and glandular elements

      Squamous and glandular elements

      Fascicular arrangement

      Fascicular arrangement

      Bland spindle cells

      Bland spindle cells

      Round cells with nucleoli

      Round cells with nucleoli

      Focal squamous and glandular elements

      Focal squamous and glandular elements


      Infiltrative border Infiltrative border

      Infiltrative border

      p63

      p63

      CK MNF 116

      CK MNF 116

      Virtual slides

      Images hosted on other servers:

      Breast fibromatosis-like metaplastic carcinoma

      Cytology description
      • Low cellularity smear with clusters of round to oval and spindle shape cells with low grade, bland nuclear features (Acta Cytol 2010;54:712)
      Positive stains
      Electron microscopy description
      • Tumor cells show fibroblastic, myoepithelial and epithelial features with peripheral villous processes with a focal basal lamina and intercellular junctions (World J Surg Oncol 2007;5:24)
      Electron microscopy images

      Images hosted on other servers:
      Fibroblastic, myoepithelial and epithelial features

      Fibroblastic, myoepithelial and epithelial features

      Molecular / cytogenetics description
      Sample pathology report
      • Breast, right, partial mastectomy:
        • Fibromatosis-like spindle cell metaplastic carcinoma (see comment and synoptic report for additional details and staging)
        • Comment: H&E sections show an infiltrative spindle cell tumor arranged in long sweeping fascicles. The lesional cells lack significant nuclear atypia and mitotic activity is scarce. Conventional in situ (DCIS) and invasive carcinoma are not identified. Immunohistochemical staining shows that the tumor cells are positive for multiple cytokeratins (CK5, CK MNF 116, CK AE1 / AE3) and p63. The morphology and immunoprofile are consistent with fibromatosis-like metaplastic carcinoma.
      Differential diagnosis
      Board review style question #1

      The lesion depicted in the image was taken from a 67 year old woman with a 2 cm mass. Which immunohistochemical panel supports a diagnosis of fibromatosis-like metaplastic carcinoma?

      1. ER positive, CD34 negative, beta catenin cytoplasmic expression
      2. ER positive, CD34 negative, beta catenin nuclear expression
      3. ER negative, CK AE1 / AE3 negative, beta catenin cytoplasmic expression
      4. ER negative, CK AE1 / AE3 positive, beta catenin cytoplasmic expression
      Board review style answer #1
      D. ER negative, CK AE1 / AE3 positive, beta catenin cytoplasmic expression. Fibromatosis-like metaplastic carcinoma shows the following immunoprofile: cytokeratin positive (typically high molecular weight and broad spectrum keratins), p63 positive, ER, PR and HER2 negative and are CD34 negative. A distinguishing factor in differentiating this lesion with desmoid type fibromatosis is the lack of nuclear beta catenin expression that is characteristic of fibromatosis.

      Comment Here

      Reference: Fibromatosis-like metaplastic carcinoma
      Board review style question #2

      The lesion depicted in the image was taken from a 72 year old woman with a 3 cm mass. The lesional cells showed scattered expression of CK5/6 and p63. Focal squamous differentiation was seen, which comprised approximately 2% of the tumor. Which statement is true regarding this lesion?

      1. These tumors typically lack cytologic atypia and conspicuous mitoses
      2. This variant is an unfavorable pattern of metaplastic breast carcinoma with frequent lymph node metastases
      3. This variant is commonly associated with conventional invasive ductal carcinoma or ductal carcinoma in situ
      4. This variant shares similar molecular alterations with other variants of metaplastic breast carcinoma
      Board review style answer #2
      A. These tumors typically lack cytologic atypia and conspicuous mitoses. The photo and vignette describe a fibromatosis-like metaplastic breast carcinoma. These tumors can histologically mimic desmoid type fibromatosis and are composed of bland spindle cells arranged in long sweeping fascicles with infrequent mitotic activity. Focal glandular and squamous elements can be present; however, conventional invasive carcinoma and ductal carcinoma in situ are rarely present. Compared to other variants of metaplastic breast carcinomas, fibromatosis-like metaplastic carcinoma portends a more favorable prognosis with infrequent lymph node and distant metastases. These tumors show low genomic instability and do not share copy number alterations with other metaplastic carcinoma types.

      Comment Here

      Reference: Fibromatosis-like metaplastic carcinoma

      Flat epithelial atypia
      Definition / general
      • Introduced by World Health Organization (WHO) Working Group on Pathology and Genetics of Tumours of the Breast to encompass columnar cell lesions with low grade / monomorphic cytologic atypia that lack architectural features of atypical ductal hyperplasia (ADH) or low grade ductal carcinoma in situ (LG DCIS)
      Essential features
      • Atypical epithelial proliferation with one to several layers of disorganized, monotonous, low columnar to cuboidal cells without architectural atypia
      Terminology
      • WHO acceptable:
        • Flat epithelial atypia
        • Also columnar alteration with atypia (columnar cell change with atypia or columnar cell hyperplasia with atypia)
      • Not recommended by WHO:
        • Low grade clinging carcinoma (monomorphous type)
        • Atypical cystic lobules
        • Atypical lobules type A
      • Other (historical):
        • Flat ductal intraepithelial neoplasia grade 1A (DIN1A)
        • Atypical columnar alternation with prominent apical spouts and secretions
      ICD coding
      • ICD-10: N60.9 - unspecified benign mammary dysplasia
      • ICD-10: N60.89 - other benign mammary dysplasias
      Epidemiology
      Sites
      • Breast terminal duct lobular units (TDLUs)
      Etiology
      Clinical features
      Diagnosis
      • Imaging guided (frequently stereotactic due to mammographically detected calcifications) needle core or surgical excisional biopsy with tissue histologic examination
      Radiology description
      Radiology images

      Contributed by Julie M. Jorns, M.D. and Shadie Majidi, M.D.

      Calcifications corresponding to FEA

      Calcifications, craniocaudal view

      Calcifications, mediolateral view

      Prognostic factors
      Case reports
      • 47 year old woman with sonographic features of flat epithelial atypia manifesting as a nonmass-like lesion (Am J Case Rep 2019;20:340)
      Treatment
      Gross description
      • No discernible lesion (typically a microscopic finding only)
      Microscopic (histologic) description
      • Atypia that does not fulfill criteria of atypical ductal hyperplasia or low grade ductal carcinoma in situ due to flat growth pattern (i.e. cytologic atypia and lack of architectural atypia)
      • Distention of terminal duct lobular unit with dilated glands
      • Glands usually more blue than normal terminal duct lobular units at low power
      • Frequent secretions and calcifications within cystically dilated glands
      • Atypical epithelial proliferation with lack of regular orientation perpendicular to basement membrane
      • One to several layers of low columnar or cuboidal cells with low grade cytologic atypia resembling low grade ductal carcinoma in situ
      • Increased nuclear:cytoplasmic ratio (imparts blue low power appearance)
      • Prominent apical tufting (snouts)
      • Nuclei are usually round with variably prominent nucleoli
      • Apocrine features such as eosinophilic cytoplasm may be present
      Microscopic (histologic) images

      Contributed by Julie M. Jorns, M.D.

      Bluer appearance

      Luminal secretions

      Lack of polarity

      Microcalcifications


      Apocrine features

      Apocrine
      features and
      luminal secretions

      Prominent nucleoli

      Cytology description
      • Flat sheets of atypical cells with enlarged nuclei, distinct cell borders; indistinguishable from well differentiated in situ and invasive carcinoma (Diagn Cytopathol 2007;35:73)
      Positive stains
      Negative stains
      Molecular / cytogenetics description
      • Phosphatidylinositol 3 kinase catalytic subunit (P13KCA) mutations frequently observed similar to other breast epithelial proliferations (Mod Pathol 2014;27:740)
      Sample pathology report
      • Right breast, core biopsy:
        • Flat epithelial atypia (FEA) with microcalcifications
      Differential diagnosis
      Board review style question #1



      Acceptable WHO (5th edition) terminology for the pictured lesion (H&E, 10x) includes

      1. Atypical cystic lobules
      2. Atypical lobules type A
      3. Flat epithelial atypia
      4. Low grade clinging carcinoma (monomorphous type)
      Board review style answer #1
      C. Flat epithelial atypia

      The pictured lesion has classic features of flat epithelial atypia (FEA), at low power showing a terminal duct lobular unit (TDLU) with dilated, blue glands owing to increased nuclear:cytoplasmic ratios, which at high power show loss of orientation toward lumina and prominent apical tufting or snouts. Other current WHO accepted names include columnar cell alteration (change / hyperplasia) with atypia. Atypical cystic lobules, atypical lobules type A and low grade clinging carcinoma (monomorphous type) are terms that are not recommended for use by the WHO.

      Comment Here

      Reference: Flat epithelial atypia of breast
      Board review style question #2
      Flat epithelial atypia most commonly has the corresponding imaging feature of

      1. Calcifications on mammography
      2. Enhancing mass on magnetic resonance imaging (MRI)
      3. Filling defect on ductogram (galactogram)
      4. Mass on ultrasound
      Board review style answer #2
      A. Calcifications on mammography

      Flat epithelial atypia (FEA) is typically diagnosed by stereotactic biopsy of mammographically detected intraluminal calcifications. Less frequently, it constitutes an incidental finding on biopsy or excision. It does not typically present as mass on imaging nor a filling defect on ductogram or galactogram.

      Comment Here

      Reference: Flat epithelial atypia of breast

      Foreign body reaction
      Definition / general
      • Foreign body giant cell inflammation that usually is localized and occurs 1 - 2 weeks after trauma, surgery or radiation
      Terminology
      Clinical features
      • Clinically resembles carcinoma due to skin retraction
      Case reports
      Due to injections:
      Gross description
      • Localized to subcutaneous tissue
      • May be nodular, fibrotic, orange brown (due to hemosiderin)
      • Up to 1 cm
      Microscopic (histologic) description
      • Not encapsulated
      • Fibrosis, foreign body giant cells, chronic inflammatory cells, fat necrosis, calcification, cholesterol clefts (needle-like empty spaces, due to substances dissolved during processing), hemosiderin
      • May see a foreign body
      Microscopic (histologic) images

      Contributed by Dr. Saroona Haroon, The Aga Khan University Hospital (Pakistan)

      Cholesterol granuloma in 56 year old woman with 4 month history of hard immobile mass of left breast; no history of pus or blood discharge



      Images hosted on other servers:

      Asteroid body

      Granulation tissue

      Videos

      Foreign body reaction


      Frozen section procedure
      Definition / general
      • A frozen section (cryosection) is a pathological laboratory technique used for rapid microscopic analysis / diagnosis of a specimen / disease
      • Usually used with oncologic surgery
      • Rapid diagnosis can guide intra-operative patient management
      Indications
      • Use frozen section to
        • Provide rapid gross or microscopic diagnosis to identify an unknown pathologic process, identify extent of disease / evaluate margins, identify metastases or simply identify a tissue
        • Process tissue to provide appropriate and accurate diagnosis, prognosis and to adhere to research and special study protocols
        • Confirm that pathological tissue is present for diagnosis on permanent sections
      • Do not use if
        • Frozen section diagnosis has no immediate implications for decision making
        • Tissue is needed for permanent processing (is unique or small or requires extensive study for diagnosis)
      • Consider not freezing tissue if
        • Frozen section is known to produce severe artifacts that hinder proper interpretation
        • Tissue is heavily ossified / calcified
        • Risk of serious infection (HIV, TB, hepatitis B or C)
        • Tissue is fatty
      Tissue type
      • Tissue should be received fresh, otherwise it will not stay on slide
      • At time of receipt of tissue, decide whether to obtain smears or touch preps and whether to freeze all or part of it
        • Touch preps and smears are often performed on lymph nodes suspicious for lymphoma
        • Some primary small lesions should not be entirely submitted for frozen section
        • There is debate on whether sentinel nodes should be entirely or representatively submitted for frozen section
      • Fixed tissue:
        • There are special slides to keep tissue affixed to slide
        • To freeze fixed tissue, make sure it has been preserved in formalin and not alcoholic fixatives like Carnoy's, because tissue fixed in alcohol is harder to freeze
        • Avoid freezing tissue fixed with heavy metal salts such as B5 and Helly's (Zenker’s formal solution), which can denature proteins and shrink the tissue
        • Avoid hard tissues like bone and cartilage that require decalcification
        • Avoid tissues with a lot of fat
        • Avoid tissues from patients with known TB or other infection (if absolutely necessary, wear appropriate protection)
        • Avoid freezing tissue that will be needed to make a permanent diagnosis
      Freezing tissue
      • OCT (optimal cutting temperature) or similar embedding media like TBS or Cryogel should be placed on an appropriate sized chuck that has been precooled in a cryostat
      • The chuck should be clean
      • A toothbrush is useful to remove tissue and OCT
      • Dipping the chuck in methanol removes ice crystals
      • Place the chuck into a -20 to -15 degree (optimal) cryostat; note that the OCT media should not be frozen completely
      • It is better to have a semisolid consistency; this will alleviate tissue artifact
      • Tissue size should be no greater than 3mm - 5mm in greatest dimension (thinner specimens have shorter freezing time and minimal ice crystal artifact formation)
      • The smaller the tissue, the more even and thorough the freeze
      • Place the tissue on the semisolid chuck and add more media rapidly over the tissue, covering it entirely but avoiding overflow
      • Place chuck quickly back into the cryostat
      • Apply heat sink or CO2 aerosol (optional) to rapidly freeze or use "quick freeze" option on cryostat
        • Histobath: being phased out
        • Cryowells: useful in keeping all tissue on an even plane; also helpful in eliminating loss of smaller tissues that are frozen with larger ones, although recommended to not freeze different sizes together
        • Aerosol sprays: often canned CO2 (but may aerosolize infectious diseases)
        • Liquid nitrogen
        • Isopentane based workflow (Virchows Arch 2008;452:305)
      Cryosectioning
      1. Once the chuck is in position, there should be a manual or an automatic advance option to move the block close to the cutting blade

        Tissue embedded within OCT

      2. Fully face the tissue by using a trim setting on your cryostat; if you do not have this setting, then an advance button should be available, which should be pressed each time before one full revolution of the instrument's wheel
      3. If wells are used to freeze the blocks, then the tissue should be on an even plane and the tissue will be faced faster
      4. To polish the tissue, avoid advancing the cryostat or deselect the trim setting on the cryostat and turn 10 - 15 times
      5. As you cut the tissue, anchor the tissue to prevent folding or curling; this can be done with an anti roll bar (a plastic plate attached to cryostat) or by using a precooled paintbrush with stiff bristles and a wide gripping surface
      6. The brush should be held like a pen with your left hand at an angle
      7. You can rest your fifth finger on the stage for stabilization
      8. Cutting the brushes' bristles at an angle can aid in the brush meeting the tissue flat over its length because you will hold it at an angle
      9. Turn the wheel with your right hand in a continuous motion without stopping; avoid speeding up or slowing down
      10. Avoid stopping the wheel at the beginning of the section, slowly grabbing the tissue and then resuming wheel revolutions; this can cause artifacts such as variation in section thickness and tissue folding
      11. Move the brush as the chuck moves towards the blade; your brush should move down in pace with the chuck

             

        Riding the block: as the block descends
        toward the brush, the brush keeps pace
        with the block by gently resting on the
        bottom 2 - 3 mm of the block

      12. You can rest your brush softly on the very bottom of your chuck avoiding tissue contact
      13. Pull the brush away easily as the chuck meets the blade

          

        Catching the curl: as the block meets the blade and
        the section begins its curl, the brush leaves the block
        while catching the curling edge of the section;
        then the brush jumps off the block with the curl

      14. The downward motion of the brush allows you to keep a continuous motion as you take your section

              

        Pull over the blanket: the brush holding the curl
        pulls the section horizontally over the stage,
        like pulling the blanket over yourself,
        without pressing the tissue to the stage

      15. A glass slide is gently laid upon the tissue section

        Gently touch the section to the slide; avoid stretching or folding the section by keeping a steady hand, and keep the transverse axis of the slide parallel to the section

      16. The tissue section should melt onto the slide
      17. Prepared slides should immediately go into formal alcohol, 95% alcohol (methanol/ethanol) or formalin while awaiting the stain line; if you delay this step, drying artifact will occur
      18. You can take a deeper level after approximately 20 turns (multiple levels may be needed for breast or prostate biopsies)
      19. Optimal cutting thickness is 4 - 7 microns for sectioning and 20 - 40 microns for trimming
      Staining slides
      • Keep all stains and solutions fresh and well maintained
      • Dip slide in reagents in this order for H&E staining:
        • After obtaining frozen section, IMMEDIATELY fix in 95% ethanol (even 15 seconds of delay can cause significant artifact)
        • Formal alcohol, formalin or 95% alcohol: 45 - 60 seconds
        • Water: 5 - 7 seconds
        • Hematoxylin: 60 seconds
        • Lithium carbonate or 0.2 % aqueous ammonia (Bluing): 15 - 20 seconds
        • Eosin: 20 - 60 seconds
        • 95% alcohol: 10 seconds
        • 100% alcohol: 10 seconds
        • Xylene, toluene, limonene derivatives and Clearite: 10 seconds
        • Then add mounting media for cover slipping
      Troubleshooting
      • Ice crystal artifacts
        • Due to slow freezing of tissue
        • Solution: Freeze fast (flash / snap); the faster the freeze, the smaller the ice crystals, the less tissue damage (best freezing method is arguably liquid nitrogen)
        • Smaller tissues yield less artifact - optimally tissue should be 0.5 x 0.5 x 0.3 cm or less
        • Never freeze fragments larger than the diameter of the chuck
        • Avoid freezing fat around tissue
        • Blot the outer surface of the tissue dry with gauze before making your block
      • Knife artifact
        • A nicked cutting blade will produce a split / tear in your section
        • Solution: change your blade every few cases; some institutions use a new blade for each case
      • Overfreezing
        • Can cause section to have holes
        • Solution: polish block with a couple extra turns of the blade to create friction and warm up block by pressing on it with your finger (5 - 10 seconds)
      • Underfreezing
        • Underfreezing can be troublesome for fatty tissue
        • Solution: add heat sink to block or select rapid freeze setting on your cryostat (if available)
      • Staining issues
        • Dirty "stain line" can cause floaters (extraneous foreign tissue) to adhere to slides; overly diluted stains and alcohols can diminish slide quality
        • Poor staining hinders frozen section diagnoses, as nuclear detail is compromised
        • Solutions: (a) maintain a clean stain line by frequent solution changes; (b) follow recommended staining times; (c) don't rush
        • Note: brain tissue may stain best in eosin for 60+ seconds
        • Water: should be changed after each frozen section
        • Alcohols and stains: change at least weekly, alcohols may need to be changed more frequently depending on work load
      • Fatty tissue
        • Includes lymph nodes, breast, skin; may be too soft to cut
        • Solution: maintain an extremely cold cutting temperature (-20C)
        • Firm lymph nodes, spleen, brain and liver cut better at -10C; tissue may shatter if sectioning is performed at lower temps
      • Air bubbles
        • May be trapped under cover slips, which can cause the underlying tissue to dry out
        • Solution: make sure an appropriate amount of resin (2 drops) is applied; gently move air bubbles off the slide with finger or tweezers; do not press on the slide too hard or it will break
      • Overly thick sections
        • May cause tissue to fall off slide
        • Solution: reduce the cryostat's sectioning thickness
      Microscopic (histologic) images

      Images hosted on other servers:

      Left: ice crystals in edematous stroma by frozen section, right: H&E

      Nuclear ice crystals (particularly a problem with thinner sections): left - lung adenocarcinoma, right - uterine sarcoma


      Glioma

      Videos

      Brush technique

      Embedding small specimens

      Speed embedding


      Frozen sections
      Definition / general
      • Click here for the frozen section procedure topic
      • Historically used for incisional biopsies for primary diagnosis in the breast (Pathologica 2011;103:325); now core biopsy is utilized
      • Currently used most frequently for axillary sentinel lymph node evaluation to determine need for further axillary dissection (Breast 2017;34:S64) and to evaluate nipple margins in nipple sparing mastectomies (Breast J 2016;22:18)
      • Select institutions utilize intraoperative margin assessment by frozen section for breast conserving surgery (Ann Surg Oncol 2017 Jan 5 [Epub ahead of print])
      Essential features
      • Frozen section introduces histologic artifacts: (a) alteration of architectural features makes benign lesions such as sclerosing adenosis more closely mimic invasive carcinoma; (b) artifactual clefting / spaces around tumor cells simulates lymphovascular invasion (Arch Pathol Lab Med 2005;129:1565)
      • Use of frozen section for sentinel lymph node assessment has declined following American College of Surgeons Oncology Group Z0011 trial findings (Arch Pathol Lab Med 2016;140:830, Am J Clin Pathol 2016;146:57)
      • Frozen section has been shown to be highly sensitive (86%) and specific (96%) in a meta-analysis of intraoperative breast margin assessment (Ann Surg 2017;265:300)
      • Frozen section use should be considered for margin assessment if reoperation rates at an institution are > 15% (Ann Surg Oncol 2017 Jan 5 [Epub ahead of print])
      Clinical features
      • Use of frozen section to intraoperatively assess breast margins has reduced reoperative rates at selected institutions by 12.3% (Eur J Surg Oncol 2017;43:1273)
      • Frozen section use on margins for invasive lobular carcinomas or for those with extensively close margins may not significantly lower subsequent breast operations (Eur J Surg Oncol 2017;43:1273)
      • Frozen assessment of margins for DCIS has shown mixed results in the effect on reoperative rates (Breast Cancer (Auckl) 2016;10:205, Ann Surg Oncol 2016;23:2788)
      • Frozen section of lumpectomy margins offers potential cost saving to patients and payers (Oncol Pract 2016;12:e413)
      • Frozen section assessment of nipple margins for nipple sparing mastectomy is highly specific but only moderately sensitive for the detection of carcinoma in subareolar tissue, allowing nipple resection at the time of mastectomy (Breast J 2016;22:18)
      Microscopic (histologic) description
      • Diagnostic errors in frozen section of breast tissue include sampling errors or proliferative lesions mimicking in situ (usual ductal hyperplasia, papilloma) and invasive carcinoma (sclerosing adenosis) (Tumori 1999;85:15)
      Microscopic (histologic) images

      Contributed by Emily S. Reisenbichler, M.D.
      Missing Image

      Dermal lymphovascular invasion

      Missing Image

      Invasive lobular carcinoma

      Missing Image Missing Image

      Invasive mammary carcinoma


      Missing Image

      Nipple with chronic inflammation

      Missing Image

      Sclerosing adenosis

      Missing Image

      Ductal carcinoma in situ



      Images hosted on other servers:

      Infiltrating ductal carcinoma

      Differential diagnosis
      Additional references
      Board review style question #1
        Which of the following is not currently a typical use of intraoperative frozen section in breast cancer surgical management?

      1. Axillary sentinel lymph node evaluation.
      2. Breast margin evaluation in breast conserving surgery.
      3. Nipple margin evaluation in nipple sparing mastectomy.
      4. Primary diagnosis of a breast mass for same day surgical management.
      Board review style answer #1
      D. Primary diagnosis of a breast mass for same day surgical management.

      In the past, breast carcinoma could be diagnosed intraoperatively, allowing surgical management to take place at the time of the primary biopsy. In the current age of neoadjuvant therapy, testing breast carcinomas for estrogen / progesterone receptor and HER2 status prior to definitive surgical management is the standard of care.

      Comment Here

      Reference: Frozen sections

      Galactocele
      Definition / general
      Essential features
      Terminology
      ICD coding
      • ICD-10: N64.89 - other specified disorders of breast
      Epidemiology
      Sites
      Pathophysiology
      Etiology
      Clinical features
      Diagnosis
      Laboratory
      Radiology description
      Radiology images

      Images hosted on other servers:
      Missing Image

      Cyst with circumscribed rounded margins

      Missing Image

      Oval shaped mass with hypoechoic rim

      Prognostic factors
      Case reports
      Treatment
      Gross description
      • Circumscribed to oval lesions usually ranging in size from 1 - 6 cm; cut section showing cyst filled with thin yellow fluid
      Gross images

      Contributed by Nasir Ud Din, M.B.B.S.

      Oval multicystic lesion with well circumscribed borders

      Microscopic (histologic) description
      • Multiple variable sized anastomosing cysts that are lined by cuboidal or flat epithelial cells
      • Lining epithelium exhibits regular nuclei with cytoplasmic vacuolations due to lipid accumulation
      • Apocrine metaplasia may be seen
      • Intact cysts are surrounded by fibrous wall of varying thickness
      • Little to no inflammatory reaction, unless leakage of cyst contents occurs
      • Fat necrosis can be present (Hoda: Rosen's Breast Pathology, 5th Edition, 2020)
      Microscopic (histologic) images

      Contributed by Saba Anjum, M.B.B.S.

      Multiple variable sized anastomosing cysts

      Cystic lesion lined by cuboidal epithelium

      Proteinaceous material


      Cyst lined by cuboidal epithelium

      Galactocele with lactational change and mild fibrosis

      Cytology description
      • Thick, chalky white material with a gritty sensation during aspiration; well defined purple crystals, which show positive birefringence
      • Granular, amorphous, proteinaceous material in the background admixed with frothy appearing lipid micelles
      • Absence of distinct epithelial fragments, bipolar nuclei and fibrous stroma (J Cytol 2020;37:149, StatPearls: Galactocele [Accessed 21 September 2022])
      Cytology images

      Contributed by Dhiraj B. Nikumbh, M.D.

      27 year old woman with 1.5 cm nontender breast nodule and fine needle aspiration cytology



      Images hosted on other servers:

      Dark purple crystals in a proteinaceous background

      Sample pathology report
      • Breast, excision:
        • Galactocele (see comment)
        • Comment: It is a benign condition with no recurrence.
        • Microscopic description: Breast tissue exhibiting dilated anastomosing channels lined by cuboidal epithelium with secretory activity. Adjacent tissue shows secretory change and fibroinflammatory reaction with foamy macrophages. No evidence of granulomatous or neoplastic processes are seen.
      Differential diagnosis
      Board review style question #1

      34 year old lactating mother presented with the history of a palpable lump in the left breast. Breast ultrasound examination showed a circumscribed cystic lesion. Histology is shown in the picture above. What is the diagnosis?

      1. Fibroadenoma
      2. Galactocele
      3. Hamartoma
      4. Invasive breast carcinoma of no special type
      5. Lactating adenoma
      Board review style answer #1
      B. Galactocele

      Comment Here

      Reference: Galactocele
      Board review style question #2
      Galactocele formation is related to which of the following conditions?

      1. Age of the mother
      2. Duct obstruction
      3. Estrogen effect
      4. Family history
      5. Number of pregnancies
      Board review style answer #2
      B. Duct obstruction

      Comment Here

      Reference: Galactocele

      Granular cell tumor
      Definition / general
      • Tumor whose cells have granular eosinophilic cytoplasm and bland small nuclei
      Essential features
      • Uncommon benign breast tumor that may present as a mass lesion
      • Imaging shows ill defined spiculated lesion mimicking malignancy
      • Microscopy shows sheets / cords of polygonal cells with abundant eosinophilic cytoplasm, round nuclei and prominent nucleoli; no mitosis or atypia seen
      • Treated by complete excision
      Terminology
      Epidemiology
      Sites
      • Usually inner quadrant of breast
      Etiology
      • Appears to derive from Schwann cells of peripheral nerves (at all sites)
      Clinical features
      • Resembles invasive carcinoma clinically but almost always benign
      • Xray: suggestive of malignancy due to apparent infiltration
      • Painless lump most commonly in upper middle and medial quadrants
      Radiology description
      • On mammography appears as an ill defined spiculated lesion mimicking malignancy
      • On ultrasound appears as solid, hypoechoic mass with posterior shadow
      Radiology images

      Contributed by Mark R. Wick, M.D.
      Missing Image

      Mammogram spot film



      Images hosted on other servers:
      Missing Image

      Fig 2, case 1:
      ultrasonography
      revealed a tumor
      7 x 8 mm in size

      Missing Image

      Mammograph of the breast

      Prognostic factors
      • Most are benign and reportedly malignant cases are rare, occurring in only 1% or 2% of cases
      • Metastasis to liver, lung, bone and axillary lymph nodes are reported with malignant granular cell tumor (J Ultrasound Med 2011;30:1295)
      Case reports
      Treatment
      • Local excision, local recurrence reported with incomplete excision
      Gross description
      • Firm, homogenous, gray white and yellow, usually 3 cm or less and ill defined
      Gross images

      Contributed by Mark R. Wick, M.D. and AFIP
      Missing Image

      Dense
      homogenous
      tumor with
      infiltrating margins

      Missing Image

      Various images



      Images hosted on other servers:

      Firm tumor with irregular borders

      Missing Image

      Granular cell tumor in right breast

      Missing Image

      52 year old woman with 1 cm nodule

      Microscopic (histologic) description
      • Infiltrating sheets / cords of polygonal bland cells with well defined cell borders, abundant eosinophilic granular cytoplasm and round / oval nuclei with prominent nucleoli
      • Collagenous stroma
      • May be close to small nerve bundles and have infiltrative margins; occasional multinucleation
      • Overlying epithelium may show pseudoepitheliomatous hyperplasia
      • Rare mitotic figures, no / mild atypia
      • Histologic criteria for malignancy (proposed by Fanburg-Smith et al., Am J Surg Pathol 1998;22:779):
        • Spindling
        • Necrosis
        • Vesicular nuclei with large nucleoli
        • High N/C ratio
        • Nuclear pleomorphism
        • Increased mitotic activity
      • 1 or 2 of 6 criteria are considered atypical and 3 or more are considered to be associated with malignant behavior (Breast J 2004;10:528, Arch Pathol Lab Med 2004;128:771), but metastasis remains the only unequivocal sign of malignancy (Virchows Arch 2016;468:527)
      Microscopic (histologic) images

      Contributed by Emily S. Reisenbichler, M.D., Mark R. Wick, M.D., Dr. Oleksandr Grygoruk, AFIP and @ThatGlassTho on Twitter
      Missing Image

      Infiltration into fat

      Missing Image

      S100+ tumor cells and myoepithelial cells

      Missing Image

      Granular cell tumor

      Missing Image

      Various images


      Various images

      Missing Image

      S100

      Granular cell tumor Granular cell tumor Granular cell tumor

      Granular cell tumor


      Missing Image Missing Image Missing Image Missing Image Missing Image

      9 year girl with a 2 cm granular cell tumor near the left nipple, unchanged in size over past 3 years


      Missing Image Missing Image Missing Image Missing Image Missing Image

      9 year girl with a 2 cm granular cell tumor near the left nipple, unchanged in size over past 3 years



      Images hosted on other servers:
      Missing Image

      Core biopsy

      Abundant eosinophilic, granular cytoplasm

      Missing Image

      Beneath squamous epithelium (not necessarily breast): H&E and S100


      Missing Image

      Compact nests of polygonal cells

      Missing Image

      Cells contained granular eosinophilic cytoplasm

      Missing Image

      IHC study showed reactivity for S100 protein

      Missing Image

      H&E staining of granular cells

      Cytology description
      • Highly cellular, large cohesive groups and single cells mixed with connective tissue
      • Cells have ill defined, abundant granular cytoplasm and bland small nuclei with prominent nucleoli
      • No mitotic figures, no necrosis (Diagn Cytopathol 2007;35:725)
      Cytology images

      Contributed by Mark R. Wick, M.D.
      Missing Image

      FNAB

      Positive stains
      Electron microscopy description
      • Myelin figures, lysosomes
      Differential diagnosis
      Board review style question #1
        A 45 year old woman presents with a breast mass. Excisional biopsy shows a granular cell tumor. Which one of the following is not in the differential diagnosis based on morphologic findings?

      1. Apocrine carcinoma
      2. Melanoma
      3. Metastatic renal cell carcinoma
      4. Tubular carcinoma
      Board review style answer #1
      D. Tubular carcinomas form angulated tubular structures composed of mildly atypical ductal cells whereas granular cell tumors have sheets of large cells with abundant eosinophilic granular cytoplasm.

      Comment here

      Reference: Granular cell tumor

      Grossing & features to report
      Definition / general
      • This summary describes how to sample breast resection specimens with documented, suspected or potential malignancy
      • Essential clinical history:
        • Indication
        • Prior biopsy findings
        • Imaging findings
        • Presence of prior biopsy clips / localization devices
        • Treatment history
      • Multicolor inks recommended for oriented specimens; sample inked margins with perpendicular, not tangential / en face, sections to provide optimal clinical information (Cancer 1997;79:1568, Arch Pathol Lab Med 2018;142:496)
      • Inking may be performed by pathologists or surgeons; the latter may be more accurate (Ann Surg Oncol 2009;16:285, World J Surg Oncol 2010;8:4)
      • Orange / yellow inks may be difficult to distinguish microscopically; apply to opposite ends
      • Blot specimen dry, apply ink(s) and blot dry again prior to sectioning to prevent ink artifact; consider mordant (i.e., acetic acid) to optimize ink adherence
      • Slice specimens as thinly as possible; palpate each section for lesions that may not be visible
      • Change gloves if soiled to prevent ink artifact
      • Diagrams or photographs that provide orientation / section keys facilitate accurate reporting
      • Maintain specimen orientation following sampling in case additional sections required
      • Ensure cold ischemic time < 1 hour and total fixation time 6 - 72 hours
      Essential features
      • Know indication for procedure, results of prior procedures, presence of biopsy clips
      • Carefully ink specimens and blot dry to avoid inking artifact
      • Sample all lesions and all margins (perpendicular, not tangential / en face, margins)
      • For pure ductal carcinoma in situ lesions, submit all or as much fibrous tissue as practical to assess for invasion
      • Consider utilizing specimen diagrams or photographs to illustrate orientation / section key
      Excisions with palpable masses
      • Gross exam:
        • Measure, weigh specimen and describe orienting sutures
        • Ink specimen as above; if fragmented, ink each fragment
        • Describe / measure all lesions, including color / consistency / tumor border (i.e., well circumscribed, irregular, stellate); note presence of necrosis / hemorrhage
        • If multiple lesions, measure the distance between each
        • Measure distance between lesions and closest inked margins
        • Note biopsy site changes / fat necrosis
        • See below for discussion of postneoadjuvant specimens
      • Sections:
        • Suspected benign lesions: 1 section per cm (2 sections per cassette)
        • Suspected malignant lesions: submit entire lesion up to 4 cassettes
        • Provide a full face section(s) of the tumor across the longest axis of the tumor where possible, using adjoining ink to indicate contiguous sections (Ann Diagn Pathol 2011;15:291)
        • If the longest axis of the tumor is perpendicular to the plane of sectioning, tumor size can be calculated by multiplying the average slice thickness by the number of consecutive slices involved (Int J Surg Pathol 2021;29:39, Arch Pathol Lab Med 2009;133:15)
        • Sample all grossly heterogeneous areas, including areas of necrosis
        • If no discrete lesion, submit up to 12 cassettes of fibrous tissue as a screening step
        • Sample intervening tissue between lesions
        • Margins: document / sample closest margins; include lesion in section if possible
        • Sample remaining margins with at least 2 sections (1 cassette) each
        • Submit section of skin if present
        • If specimen or lesion not entirely submitted, estimate proportion sampled
        • For pure ductal carcinoma in situ, the College of American Pathologists recommends "when practical, the entire specimen should be submitted in a sequential fashion for histologic examination" (CAP: Protocol for the Examination of Resection Specimens from Patients With Ductal Carcinoma In Situ (DCIS) of the Breast [Accessed 18 March 2021])
      Excisions for lesions localized with imaging studies
      • Gross exam:
        • Specimen imaging typically performed
          • Findings regarding targeted lesions, calcifications, biopsy clips and localization devices should be reconciled with gross exam
          • Clips may be present and not grossly identified
        • Lesions may or may not be grossly identified
        • Same instructions for grossing specimens with palpable lesions should be followed
        • Various localization devices are utilized; be familiar with devices at your institution:
          • Localizing wires
          • Radioactive seeds
          • Tags using radiofrequency, infrared or electromagnetic waves
        • Radiation safety officers should approve radioactive seed processing protocols
        • See below for discussion of postneoadjuvant specimens
      • Sections:
        • Visible lesions should be sampled in the same manner as palpable masses
        • If no lesions grossly identified, use imaging studies, clips and localizing devices to guide sampling so that the targeted lesions are entirely submitted
        • Submit up to 12 cassettes of fibrous tissue as a screening step
        • Sample margins and skin in the same manner as palpable masses
      Cavity / shave margins performed simultaneously with primary excisions
      • To decrease need for subsequent re-excisions, surgeons often perform multiple cavity / shave margins simultaneously with primary excisions
      • Gross exam:
        • Primary excision may or may not be oriented
        • If oriented, primary excision should be processed similarly to oriented primary excisions without cavity / shave margins
        • If unoriented, primary excision can be inked black and processed similarly to unoriented primary excisions without cavity / shave margins; some labs do not ink unoriented primary excisions and report margins only on separate cavity / shave margins
        • Cavity / shave margins should be measured in 3 dimensions
        • True margin surface of cavity / shave margins should be inked with care to avoid artifactual ink seeping onto the nonmargin surface; some labs ink the nonmargin surface a different color to assist orientation microscopically
      • Sections:
        • Entire specimen can typically be submitted in < 10 cassettes
      Re-excision specimens
      • Subsequent procedures performed to obtain negative margins; may be global, surrounding the entire excision site or targeted, excising 1 or more specific margins
      • Gross exam:
        • True margins should be inked similarly to cavity / shave margins, avoiding artifactual ink seeping onto nonmargin surface
        • Describe grossly evident or palpable lesions and biopsy cavity contents
        • Measure range of thickness of cavity walls
      • Sections:
        • Global re-excisions:
          • If re-excision is for invasive carcinoma, representative sections generally suffice
          • If re-excision is for pure ductal carcinoma in situ, it is important to exclude invasive carcinoma; all parenchymal / fibrous tissue should be submitted if practical
          • Submit all gross lesions and at least 4 sections from the biopsy cavity with closest margins
          • Submit at least 2 sections from remaining margins (2 sections per cassette)
          • If not completely submitted, include an estimate of proportion submitted
          • Sample attached skin, if present
        • Targeted re-excisions:
          • Entire specimen can usually be submitted in < 10 cassettes
          • For larger re-excisions, at least 1 section per 1 cm should be submitted
          • For pure ductal carcinoma in situ lesions, all parenchymal / fibrous tissue should be submitted if practical
      Mastectomy specimens
      • Removal of most breast tissue to treat malignancy or to reduce risk of cancer (prophylactic)
      • Mastectomy types:
        • Simple: remove breast, skin and nipple / areola, typically without axillary nodes
        • Modified radical: simple mastectomy and axillary dissection
        • Radical: includes pectoralis muscle (rarely performed)
        • Skin sparing: remove breast and nipple / areola leaving skin intact
        • Subcutaneous: remove breast leaving skin and nipple intact; typically for benign indications
        • Nipple sparing: remove breast leaving skin and nipple intact; typically for patients with cancer or at high risk; often includes biopsy of nipple duct margin
      • Indications:
        • Local recurrence after prior radiation therapy
        • Locally advanced tumor
        • Multiple cancers not amenable to excision
        • Inflammatory cancer
        • Occult primary cancer in patients with positive axillary node
        • High risk for subsequent cancer (often positive for BRCA1 / BRCA2)
      • Gross exam:
        • Know indication, prior procedures, presence of biopsy clips and other clinical questions (i.e., residual calcifications)
        • Intraoperative imaging often not performed; may need to obtain imaging to facilitate gross exam and document lesion / clip removal, particularly if no prior excision
        • Weigh / measure specimen; describe orienting sutures
        • Note presence / absence of skin, nipple / areola, skeletal muscle, axillary nodes; measure / describe each
        • Describe skin, nipple / areolar lesions (retraction, ulceration, scars, etc.); palpate skin, as skin tumor satellites may not be visible
        • Describe surgical defects (i.e., exposed biopsy cavity, etc.)
        • Ink posterior / fascial aspect; the significance of soft tissue margins other than deep / fascial margin is unknown
        • Examine lateral aspect for nodes; may occasionally be found in simple mastectomies
        • Serially section fresh specimen from posterior anterior, without cutting through skin
        • Estimate proportion of fatty to parenchymal / fibrous tissue
        • Describe all lesions present similarly to excision protocol, including quadrants, distance between lesions and distance between lesions and margins
        • Discrepancies between clinical and gross findings should be reconciled with surgeon
        • See below for discussion of postneoadjuvant specimens
      • Sections:
        • Follow protocol for excision specimens regarding sampling lesion(s) and biopsy site(s)
        • Section perpendicular deep / fascial margin closest to lesion(s)
        • Sample uninvolved quadrants (2 sections / 1 cassette each)
        • Sample skeletal muscle (1 perpendicular section closest to tumor or biopsy site)
        • Sample nipple / areola; if no lesions, 1 central perpendicular section; if lesion or suspected Paget disease, more extensive sampling is recommended
        • Sample skin lesions, including scars (1 section)
        • For nipple sparing mastectomies, nipple / areolar margin insertion site on anterior surface should be inked / sampled (2 perpendicular sections, 1 cassette)
        • Sample all lymph nodes, as detailed below
        • For prophylactic mastectomies, sample all gross lesions as above; if no lesions, submit 2 sections per quadrant and sample nipple / areola and deep / fascial margins, as above
      Postneoadjuvant specimens
      • Neoadjuvant chemo or endocrine therapy may be used to shrink tumors prior to definitive excision or mastectomy
      • Assessment of treatment response in the breast / nodes provides useful prognostic information
      • Review imaging reports describing tumor location, local extent, pre / posttherapy characteristics and presence of biopsy clip(s)
      • Gross exam:
        • If significant treatment response, may be difficult to identify tumor bed
        • Coordinated gross / microscopic evaluation required to define tumor bed in at least 2 dimensions (Surg Pathol 2018;11:213)
        • For multiple discrete primary tumors, tumor bed estimates should be provided for each
      • Sections:
        • In addition to standard sections for excision / mastectomy specimens, submit sections spanning largest span of tumor bed to assess overall tumor cellularity
        • Amount of sampling is dictated by pretreatment tumor size and presence of visible tumor bed / gross residual tumor
        • Recommended minimum of 1 block per cm of pretreatment tumor size (Int J Surg Pathol 2021;29:39)
      Sentinel lymph node excisions
      • Gross exam:
        • Quantitate discrete nodes / node candidates
        • Comment on blue dye, if present
        • If previously biopsied, document clips (specimen imaging may be required)
        • Slice each node as thinly as possible, 2 mm or thinner; comment on size, color, consistency and presence / absence of grossly evident tumor or scarring
        • If tumor evident, comment on tumor border / evidence of extranodal extension
      • Sections:
        • Entirely submit each node
        • Provide detailed slide key regarding number of nodes in each cassette
      Axillary dissection specimens
      • Gross exam:
        • Quantitate discrete nodes / node candidates
        • If nodes previously biopsied, document clips (specimen imaging may be required)
        • Section / describe each node as described above
        • Multiple positive nodes may be fused / matted; estimate number of fused nodes, if possible
        • Full axillary dissections should yield 10 or more nodes; if < 10 nodes, consider utilizing clearing solutions (i.e., Carnoy fixative) or sample surrounding adipose tissue
      • Sections:
        • Entirely submit each node
        • Large grossly positive nodes may be representatively sampled; include any irregular tumor borders to assess for extranodal extension
      Microdochectomy specimens
      • Usually small excisions performed to evaluate patients with nipple discharge (with or without abnormal ductogram)
      • Gross exam:
        • If oriented, maintain orientation; ink using the protocol above
        • If dilated duct grossly evident, serially section perpendicular to the duct, as thinly as possible; examine for intraductal lesions
      • Sections:
        • Entirely submit
      Microdochectomy images

      Images hosted on other servers:

      Insertion of ductoscope

      Selective lactiferous duct sample

      Nipple resections
      • Usually performed for Paget disease of the nipple
      • Gross exam:
        • Measure in 3 dimensions; describe orienting sutures
        • If oriented, apply multiple inks to maintain orientation, similarly to skin excisions
        • Describe color and consistency of skin surface; measure and describe lesions and distance to inked margins
      • Sections:
        • Serially section perpendicular to skin surface
        • Entirely submit in a manner allowing assessment of all perpendicular lateral and deep margins
      Specimens from oncoplastic procedures
      • Procedures involve removal of tumor simultaneously with plastic surgery techniques designed to obtain cosmetically optimal results
      • Typically results in large, irregular and difficult to orient specimens
      • Gross exam:
        • Communicate with the surgeon to identify appropriate margins to sample
        • Surgeon may submit multiple separate shave margins to ensure optimal margin evaluation; process similarly to targeted re-excision specimens, as above
        • Due to specimen complexity, utilize diagrams / photographs with orientation / section key
      • Sections:
        • Sample all lesions using excision protocol, above
        • Sample all relevant perpendicular inked margins indicated by surgeon
      Grossing implant specimens with suspicion of implant associated lymphoma
      • Implants (intact or ruptured) are typically sent to pathology for gross exam, often with attached or separate peri-implant capsular tissue that should be sampled for histologic evaluation
      • Implant associated anaplastic large cell lymphoma is a rare entity most commonly associated with textured implants (silicone or saline), usually associated with a peri-implant effusion and rarely associated with a grossly evident mass (Plast Reconstr Surg 2019;143:65S)
        • Lymphoma may arise many decades following implant placement
      • If lymphoma is clinically suspected, determine if there is associated effusion; if so, fluid should be aspirated / sent for cytologic evaluation and flow cytometry; could be performed by surgeon intraoperatively or by pathologist if effusion is resected intact
      • Gross exam:
        • All specimens should be photographed, including implant manufacturer information
        • Weigh specimen and measure in 3 dimensions
        • Describe the surface of the implant (adherent tissue / capsule, smooth or textured surface, intact or disrupted, color of effusion) and implant contents
      • Sections:
        • If effusion received intact, aspirate and send for fluid for cytology and flow cytometry
        • Extensively sample tissue surrounding the effusion; may require sampling material scraped from implant surface
        • Sample adherent tissue and peri-implant capsule (multiple sections for each of 6 margins if possible)
      Reduction mammoplasty specimens
      • Typically includes unoriented breast tissue and skin
      • Tissue weights often important for billing purposes
      • Unsuspected atypical and malignant lesions more commonly discovered in older patients (Arch Pathol Lab Med 2017;141:1523)
      • Gross exam:
        • Report weights provided by surgeon as these may differ from lab weights due to blood / fluid loss and fixation
        • Measure specimens in aggregate
        • Describe skin surfaces (lesions, etc.)
        • If oriented, ink specimen / maintain orientation when sectioning
        • Serially section as thinly as possible; inspect and palpate slices for lesions
      • Sections:
        • Sample all grossly evident breast and skin lesions
        • In no lesions, submit sections of fibrous parenchyma:
          • Women < 40: 4 sections in 2 cassettes
          • Women ≥ 40 or known risk factors: 8 sections in 4 cassettes
      Specimens removed for gynecomastia
      • Subcutaneous mastectomies, often bilateral and unoriented
      • Gross exam:
        • Weigh, measure and ink each specimen
        • Serially section as thinly as possible, examining for visible and palpable lesions
      • Sections:
        • Submit all lesions, with nearest inked margins
        • If no lesions, submit 4 sections (2 cassettes) of each specimen focusing on fibrous parenchyma
      Gender reassignment specimens (top surgery)
      • There is lack of evidence for specific guidelines at this time
      Features to report
      • Click here for recommendations from the College of American Pathologists based on the specimen type
      Gross images

      Contributed by Anthony J. Guidi, M.D.
      Uninked specimen

      Uninked specimen

      Inked specimen

      Inked specimen

      Sectioned perpendicular to margins

      Sectioned perpendicular to margins

      Board review style question #1
      Regarding breast excision specimens for ductal carcinoma in situ, the College of American Pathologists recommends which of the following?

      1. Measuring the specimen and sampling 1 section per cm, focusing on fibrous / parenchymal tissue
      2. Sampling up to 10 sections as a screening step
      3. Serially sectioning the specimen and submitting every other slice
      4. Serially sectioning the specimen and submitting 1 section per slice
      5. Submitting the entire specimen in a sequential fashion, when practical
      Board review style answer #1
      E. Submitting the entire specimen in a sequential fashion, when practical. If this is not possible, at least the entire region of the targeted lesion should be examined microscopically (CAP: Protocol for the Examination of Resection Specimens from Patients With Ductal Carcinoma In Situ (DCIS) of the Breast [Accessed 14 February 2022]).

      Comment Here

      Reference: Breast - Grossing & features to report
      Board review style question #2
      Which statement pertaining to breast specimen processing is true?

      1. Biopsy clips may be present in excision or mastectomy specimens by specimen imaging but not grossly evident
      2. For breast cancer specimens, tangential / en face margins provide similar clinically relevant information as margins taken perpendicular to inked surfaces
      3. It is rarely necessary to correlate the gross examination of breast specimens with specimen imaging studies for specimens excised for image detected lesions
      4. Knowledge of the indications for mastectomy is rarely required to optimally process these specimens
      Board review style answer #2
      A. Biopsy clips may be present in excision or mastectomy specimens by specimen imaging but not grossly evident

      Comment Here

      Reference: Breast - Grossing & features to report

      Gynecomastia
      Definition / general
      • Benign enlargement of the male breast, typically presenting as a palpable subareolar mass
      • Histologically identical gynecomastoid hyperplasia can be seen in female breast
      Essential features
      • Benign ductal and stromal proliferation with characteristic stromal cuffing, typically with 3 layered ductal lining and pseudoangiomatous stromal hyperplasia
      ICD coding
      • ICD-10: N62 - hypertrophy of breast
      • ICD-11: GB22 - hypertrophy of breast
      Epidemiology
      • Trimodal age distribution:
        • Infancy and neonatal period
        • Puberty (peak incidence age 13 - 14)
        • Older males age 50 - 80 (likely due to increased adiposity, decreased testosterone, medications)
      • Estimated lifetime prevalence of 32 - 65% of men, depending on method of assessment (Indian J Endocrinol Metab 2014;18:150)
      Sites
      Etiology
      Clinical features
      Diagnosis
      • Histologic changes are necessary for the diagnosis; important to exclude malignancy
      Laboratory
      • Usually not needed, except to identify underlying causes
        • Estrogen to testosterone ratio (E2/TTE), follicle stimulating hormone (FSH), luteinizing hormone (LH), prolactin, thyroid stimulating hormone (TSH), hCG, AFP, dehydroepiandrosterone (DHEA), cortisol, karyotyping
      Radiology description
      • 3 patterns of mammographic findings (Radiographics 1999;19:559, Andrology 2021 May 25 [Epub ahead of print])
        • Nodular: 72%, corresponds to florid histologic phase; fan shape radiating from nipple into surrounding fat
        • Dendritic: 18%, corresponds to fibrous histologic phase; flame shaped opacity with radiating projections penetrate surrounding fat / upper outer quadrant
        • Diffuse: ~10%, typically in transgender females undergoing gender affirming hormonal therapy; resembles dense female breast but heterogeneous and without Cooper ligaments
      • Imaging useful to exclude pseudogynecomastia and carcinoma
      Radiology images

      Contributed by Elaine Zhong, M.D.
      Mammogram Mammogram

      Mammogram

      Prognostic factors
      • May recur if underlying cause is not addressed
      • No strong evidence of increased risk of breast carcinoma
      Case reports
      Treatment
      • Some regress spontaneously within 2 years and do not require treatment
      • Sudden or symptomatic gynecomastia may be treated medically
      • Identify and treat any underlying causes
      • Excision in adult patients to rule out malignancy and for cosmesis; however, recurrence is common if the underlying cause is not identified and treated
      Clinical images

      Images hosted on other servers:

      Physiologic gynecomastia in puberty

      Spironolactone induced

      Gross description
      • Soft, rubbery to firm gray-white subareolar mass
      • Can be ill defined subareolar induration
      Microscopic (histologic) description
      • Increased number of ducts with characteristic 3 layer epithelium (Am J Surg Pathol 2012;36:762):
        • Luminal epithelium expressing CK5/6, CK14
        • Intermediate cuboidal to columnar epithelium expressing ER, PR, AR
        • Outer myoepithelium expressing CK5/6 and CK14
      • 3 patterns of proliferative changes, without correlation to age or duration of lesion (Am J Clin Pathol 1972;57:431, J Plast Surg Hand Surg 2018;52:166)
        • Florid: irregular branching ducts, mild to moderate epithelial hyperplasia, cellular / myxoid / edematous stroma cuffing around ducts; can have papillary or cribriform architecture, myoepithelial hyperplasia
        • Intermediate: mixture of florid and fibrous patterns
        • Fibrous: quiescent epithelium, hyalinized hypocellular stroma
      • Other findings can include pseudoangiomatous stromal hyperplasia (PASH), apocrine or squamous metaplasia
      • Atypical ductal hyperplasia (ADH) in 0.4 - 5.4% (Ann Plast Surg 2015;74:163, Histopathology 2015;66:398)
      Microscopic (histologic) images

      Contributed by Elaine Zhong, M.D.
      Florid pattern Florid pattern Florid pattern Florid pattern

      Florid pattern

      3 layer epithelium

      3 layer epithelium


      Fibrous pattern Fibrous pattern

      Fibrous pattern

      ADH in gynecomastia

      ADH in gynecomastia

      DCIS in gynecomastia

      DCIS in gynecomastia

      Virtual slides

      Images hosted on other servers:

      Excision from 33 year old man

      Biopsy from 81 year old man

      Cytology description
      • Biphasic epithelial and stromal fragments in a background of scattered single bipolar nuclei, similar to fibroadenoma
      • Spindle cells, apocrine cells, foamy macrophages can be present
      Cytology images

      Images hosted on other servers:

      Clusters of bland, cohesive epithelial cells

      Positive stains
      Sample pathology report
      • Breast, retroareolar; core biopsy:
        • Gynecomastia, (florid, intermediate or fibrotic) stage.
      Differential diagnosis
      • Male breast cancer:
        • Histology identical to carcinoma of the female breast
      • Atypical ductal hyperplasia:
        • Monomorphic intraductal cells in rigid cribriform, micropapillary or solid formations
      • Myofibroblastoma:
        • Mass forming spindle cell proliferation that may entrap normal glandular elements
        • Variable morphology and cellularity
      • Mammary hamartoma:
        • Circumscribed mass
        • Additional mesenchymal elements (smooth muscle, adipose tissue, cartilage, etc.) may be present
      • Fibroadenoma:
        • Circumscribed mass
        • Uniform stroma without significant periductal cuffing
        • Intracanalicular or pericanalicular pattern of glandular elements
      Board review style question #1
      Which of the following is the most common etiology in the development of gynecomastia?

      1. Cirrhosis
      2. Estrogen exposure in utero
      3. Hyperthyroidism
      4. Idiopathic
      5. Klinefelter syndrome
      Board review style answer #1
      D. Idiopathic

      Comment here

      Reference: Gynecomastia
      Board review style question #2

      What is the immunohistochemical profile of the intermediate epithelial cells of gynecomastia?

      1. ER/PR/AR+, CK5/6+
      2. ER/PR/AR+, CK5/6-
      3. ER/PR/AR-, CK5/6+
      4. ER/PR/AR-, CK5/6-
      Board review style answer #2
      B. ER/PR/AR+, CK5/6-

      Comment Here

      Reference: Gynecomastia

      Hamartoma
      Definition / general
      • Well circumscribed, often encapsulated mass composed of normal but disorganized components of breast tissue
      Essential features
      • Uncommon breast mass lesion that contains normal breast elements in varying proportions
      • Clinically presents as painless lump or detected via breast imaging
      • Microscopy shows well circumscribed mass with normal breast components (e.g., mammary glands, adipose tissue and fibrous stroma)
      • Definitive diagnosis on core needle biopsy requires correlation with imaging
      Terminology
      • First described by Hogeman and Ostberg in 1968; term first used by Arrigoni in 1971 (Sem Hop 1976;52:1405)
      • Acceptable by WHO: adenolipoma, chondrolipoma, myoid hamartoma
      • Not recommended by WHO: fibroadenolipoma, adenolipofibroma
      Epidemiology
      • Uncommon; < 5% of all benign breast masses (Histopathology 2022;80:304)
      • Predominantly women in fifth decade but can be seen at any age
      Sites
      • No specific location in the breast
      Etiology
      • Largely unknown; unclear whether a malformation or neoplasm
      • Mostly sporadic; rarely associated with Cowden syndrome (multiple hamartomas)
      Clinical features
      • Presents as painless breast mass or detected by screening mammography or other breast imaging modalities
      Diagnosis
      • Mammographic or sonographic abnormality
      • Often cannot be definitively diagnosed on core needle biopsy; excisional biopsy is more definitive
      Radiology description
      Radiology images

      Images hosted on other servers:
      Mammographic findings

      Mammographic findings

      Prognostic factors
      • Rarely recurs; reported recurrences represent incomplete excisions (Aust J Gen Pract 2019;48:275)
      • No increase in malignancy (same malignant potential as normal breast tissue)
      Case reports
      Treatment
      • Surgical excision is curative but not necessary
      Gross description
      • Well circumscribed, round to oval, range 1 - 14 cm; may resemble normal breast tissue, lipoma or fibroadenoma
      Gross images

      Images hosted on other servers:

      Cut section of mammary hamartoma

      Microscopic (histologic) description
      Microscopic (histologic) images

      Contributed by Indu Agarwal, M.D.

      Mammary hamartoma

      Components of mammary hamartoma Components of mammary hamartoma

      Components of mammary hamartoma

      Myoid hamartoma Myoid hamartoma

      Myoid hamartoma


      Myoid hamartoma, immunostain desmin

      Myoid hamartoma, immunostain desmin

      Myoid hamartoma, immunostain SMA

      Myoid hamartoma, immunostain SMA

      Cytology description
      • Moderately cellular, bland epithelial and stromal components
      • Diagnosis limited as findings are not specific (J Clin Pathol 2002;55:951)
      Positive stains
      Molecular / cytogenetics description
      Sample pathology report
      • Left breast, mass at 11 o'clock, ultrasound guided core needle biopsy:
        • Consistent with mammary hamartoma (see comment)
        • Comment: A well circumscribed lesion is seen comprised of benign breast elements arranged in a disorganized manner. In light of the known breast mass seen on imaging, findings are consistent with a mammary hamartoma.
      Differential diagnosis
      • Fibroadenomas:
        • In fibroadenoma, the epithelial elements often show more proliferative changes (e.g., usual type ductal hyperplasia, apocrine metaplasia)
        • Fibroadenomas only rarely include adipose tissue
      • Phyllodes tumor:
        • Hypercellular stromal component, stromal atypia, frond formation, may show infiltrative borders
      Board review style question #1

      A 45 year old woman presents for screening mammogram and a multilobulated 1.5 cm mass is detected. Ultrasound confirms the findings and a guided core needle biopsy is performed, which shows normal breast ducts, lobules, fibrous stroma and adipose tissue. The radiologist confirms that the lesion was biopsied. What is the most likely possibility?

      1. Angiolipoma
      2. Breast hamartoma
      3. Myofibroblastoma
      4. Radiologist missed the lesion
      Board review style answer #1
      B. Breast hamartoma. The mass lesion by imaging and the histologic finding of normal breast components with somewhat disorganized architecture support this diagnosis. Also, since the radiologist was confident that the lesion was biopsied, the possibility that the lesion was missed is excluded.

      Comment here

      Reference: Hamartoma

      Hemangioma
      Definition / general
      • Hemangioma:
        • Benign vascular lesions, microscopically well circumscribed and lacking endothelial cell atypia (Semin Diagn Pathol 2017;34:410)
        • Origination from large nonneoplastic "feeding" vessels, may be seen at the periphery of the lesion
        • The following categories are recognized:
          • Capillary (composed of compact, lobular, collections of small blood vessels)
          • Cavernous (dilated vessels filled with erythrocytes)
          • Venous (featuring vascular structures containing muscular walls of varying thickness) (Am J Surg Pathol 1985;9:659)
          • Perilobular (≤ 2 mm, capillary hemangiomas, intra or interlobular location)
      • Angiomatosis:
        • Often clinically present as large mass (9 - 22 cm)
        • Histologically benign vascular proliferation affecting a large segment of the breast
        • Lacks circumscription of hemangioma and can involve the subcutaneous tissue and skin (Am J Surg Pathol 1985;9:652, Cancer 1988;62:2392)
      Essential features
      • Proliferation of well differentiated vessels of varying sizes
      • Angiomatosis shows diffuse growth of blood vessels of varying sizes that surrounds ducts / lobules but without invasion of the intralobular stroma
      • No complex anastomotic channels, cytologic atypia, cellular multilayering, solid areas, mitotic activity, necrosis or hemorrhage (blood lakes), features differentiating from angiosarcoma
      Terminology
      • Acceptable: angioma
      ICD coding
      • ICD-O: 9120/0 - hemangioma, NOS
      • ICD-11: 2F30.Y & XH5AW4 - other specified benign neoplasm of breast and hemangioma, NOS
      Epidemiology
      Sites
      • Mammary parenchyma, subcutaneous tissue or dermis
      Pathophysiology
      • Unknown
      Etiology
      • Nonneoplastic vascular malformations
      Diagrams / tables

      Images hosted on other servers:
      Missing Image

      Normal arterial and venous anatomy of the breast

      Clinical features
      Diagnosis
      • Biopsy or excision
      Radiology description
      • Mostly nonpalpable, found on imaging (MRI or mammography)
      • No pathognomonic features, may show mammographic density or mass on mammogram or ultrasound, with oval or lobular shape and well circumscribed or microlobulated margins (AJR Am J Roentgenol 2008;191:W17)
      Radiology images

      Contributed by Indu Agarwal, M.D.
      Cavernous hemangioma ultrasound

      Cavernous hemangioma, ultrasound

      Cavernous hemangioma mammographic

      Cavernous hemangioma, mammography

      Capillary hemangioma ultrasound

      Capillary hemangioma, ultrasound

      Prognostic factors
      • Can recur; no metastasis reported
      Case reports
      Treatment
      Clinical images

      Contributed by Mark R. Wick, M.D.

      Capillary hemangioma, breast skin



      Images hosted on other servers:

      Diffuse dermal angiomatosis

      Gross description
      • Larger lesions: circumscribed red or dark-brown spongy lesion
      • Small lesions may not be appreciated grossly
      • Angiomatosis presents as a large, red, hemorrhagic, cystic, spongy mass lesion
      Gross images

      AFIP images
      Missing Image

      Angiomatosis composed of vessels / breast parenchyma



      Images hosted on other servers:
      Missing Image

      Soft tissue mass with smooth external surface

      Missing Image

      Cut surface shows congestion and slit-like areas

      Microscopic (histologic) description
      • Hemangioma:
        • Proliferation of well differentiated blood vessels of varying sizes, generally small (< 2 cm), usually nonanastomosing
        • Largely well circumscribed
        • Cytologic atypia, hemorrhage, mitotic activity and necrosis are absent
        • Within interlobular stroma, except in perilobular hemangioma which involves intralobular stroma
        • Perilobular hemangioma: usually incidental microscopic lesions, ≤ 2 mm, conglomerate of small, thin walled capillary vessels in and around a lobule (Am J Surg Pathol 1985;9:491)
      • Angiomatosis:
        • Proliferation of vascular channels with wide variation in caliber, diffusely growing throughout the breast with some anastomosis
        • Unencapsulated with infiltrative borders
        • No invasion of intralobular stroma
        • May extend to overlying skin (Clin Breast Cancer 2016;16:e7)
        • Absence of destructive invasion, solid areas, hemorrhage and necrosis, differentiating it from angiosarcoma
        • Low mitotic rate, Ki67 proliferation index < 2%
      Microscopic (histologic) images

      Contributed by Indu Agarwal, M.D. and AFIP images
      Cavernous hemangioma of breast Cavernous hemangioma of breast

      Cavernous hemangioma of breast

      Capillary hemangioma Capillary hemangioma

      Capillary hemangioma

      Perilobular hemangioma Perilobular hemangioma

      Perilobular hemangioma


      Complex hemangioma

      Complex hemangioma

      Angiomatosis

      Positive stains
      Negative stains
      Videos

      Histopathology breast, soft tissue - hemangioma

      Sample pathology report
      • Left breast, MRI guided core needle biopsy:
        • Breast tissue with perilobular hemangioma
      Differential diagnosis
      • Angiolipoma:
        • Has intermixed adipose tissue, clusters of capillary sized vessels, some with microthrombi
      • Angiosarcoma:
        • Has solid areas, mitotic activity, infiltrative pattern of growth and complex anastomosing vascular channels
      • Atypical vascular lesion:
        • Often past history of radiation
        • Anastomosing thin walled vessels, unlike well formed vessels of hemangioma
      Board review style question #1

      A 58 year old woman had a 4.0 cm breast mass with overlying erythematous skin patch. A stereotactic biopsy showed the histology above. What is the best treatment recommendation for this patient?

      1. Excision with wide margins
      2. Lumpectomy with hormone therapy
      3. Lumpectomy with radiation
      4. No further treatment needed
      Board review style answer #1
      A. Excision with wide margins. This is an angiosarcoma with a predominantly solid appearance and the cells demonstrate nuclear atypia, unlike a hemangioma, which lacks these features. Wide excision, preferably mastectomy, should be performed.

      Comment here

      Reference: Hemangioma
      Board review style question #2
      A 35 year old woman presents with a 10 cm palpable breast mass. Core needle biopsy shows the lesion occupying the entire biopsy tissue and consisting of large, thin walled vessels lined by bland endothelial cells. The lesion surrounds but does not invade the normal breast lobular units. No cytologic atypia, necrosis, solid areas or mitotic activity is seen. The Ki67 proliferation index is 1%. What is the most likely diagnosis?

      1. Angiomatosis
      2. Angiosarcoma
      3. Cavernous hemangioma
      4. Lymphangioma
      Board review style answer #2
      A. Angiomatosis. This is a case of angiomatosis of breast. The lesion is large and consists of large, thin walled vessels, which lack atypia, mitosis and other malignant cytologic features, and do not invade the normal breast lobular stroma.

      Comment here

      Reference: Hemangioma

      HER2 low breast cancer
      Definition / general
      • HER2 low breast cancer is defined as either HER2 IHC score of 1+ or 2+ / ISH not amplified
      Essential features
      • HER2 low breast cancer is defined as either HER2 IHC score of 1+ or 2+ / ISH not amplified
      • Heterogenous group that includes hormone receptor positive and triple negative breast cancers
      • Per 2023 American Society of Clinical Oncology (ASCO) / College of American Pathologists (CAP) guidelines, interpretation for HER2 low is not recommended in the final pathology report at this time
      Terminology
      • HER2 negative by IHC (score 1+)
      • HER2 equivocal by IHC (score 2+) and HER2 ISH not amplified
      Clinical features
      • Nearly half (~55 - 60%) of breast cancer patients will fit in HER2 low category
      • Heterogenous group that includes hormone receptor positive and triple negative breast cancers
      • Reference: Nat Med 2022 Jun 16 [Epub ahead of print]
      Interpretation
      • 2023 ASCO / CAP update on HER2 IHC and FISH interpretation still follows the 2018 ASCO / CAP guidelines (J Clin Oncol 2023;41:3867)
      • HER2 IHC with a score of 0 or 1+ is still interpreted as negative, while score of 2+ is reported as equivocal with a reflex FISH test (please see also HER2 stain topic)
      • However, the 2023 ASCO / CAP update recommends addition of a footnote to HER2 IHC interpretation report to address HER2 low results and the potential eligibility / benefit for these patients with the new class of ADC drugs (see Sample pathology report below for ASCO / CAP recommended footnote)
      • See 2023 ASCO / CAP update on HER2 testing in breast cancer (Arch Pathol Lab Med 2023;147:993)
      • See 2018 ASCO / CAP update on HER2 testing in breast cancer (J Clin Oncol 2018;36:2105)
      Uses by pathologists
      • HER2 IHC positive (score 3+)
      • HER2 IHC equivocal (score 2+) with reflex to HER2 ISH
      • HER2 IHC negative (score 0 or 1+)
      • Per 2023 ASCO / CAP guidelines, interpretation for HER2 low is not recommended at this time
      • Reference: Nat Med 2022 Jun 16 [Epub ahead of print]
      Prognostic factors
      Treatment
      • HER2 targeted monoclonal antibodies have not shown benefit in HER2 low breast cancer
      • New class of HER2 monoclonal antibodies conjugated with a chemotherapy drug has been developed recently (antibody drug complex, ADC)
      • Bystander effect / killing of ADC: HER2 antibodies target the covalently linked cytotoxic drugs to HER2 expressing cancer cells and kills them; these cytotoxic drugs are then released into extracellular space that are then uptaken by surrounding non-HER2 expressing cancer cells
      • ADCs have shown benefit in HER2 low breast cancer in phase I / II clinical trials (Nat Med 2023;29:2110, J Clin Oncol 2020;38:1887)
      • Phase 3 DESTINY-Breast04 trial shows increased median progression free survival (PFS) in patients receiving trastuzumab deruxtecan with advanced breast cancer (either unresectable or in a metastatic disease setting) (N Engl J Med 2022;387:9)
      • DESTINY-Breast04 trial also shows the risk of disease progression / death is 50% lower in treatment group compared to control arm
      Microscopic (histologic) images

      Contributed by Anand C. Annan, M.D., Ph.D., Doaa Atwi, M.D. and Lewis A. Hassell, M.D.
      Invasive ductal carcinoma, Nottingham grade 2

      Invasive ductal carcinoma, Nottingham grade 2

      HER2 score 2+

      HER2 score 2+

      Invasive ductal carcinoma, grade 2

      Invasive ductal carcinoma, grade 2

      HER2 score 2+

      HER2 score 2+

      HER2 IHC score 1+ HER2 IHC score 1+

      HER2 IHC score 1+


      Invasive ductal carcinoma with cord and tubule formation Invasive ductal carcinoma with cord and tubule formation

      Invasive ductal carcinoma with cord and tubule formation

      HER2 IHC score 1+ HER2 IHC score 1+ HER2 IHC score 1+

      HER2 IHC score 1+

      Molecular / cytogenetics images

      Contributed by Terry Dunn, Ph.D.
      Nonamplified HER2 FISH

      Nonamplified HER2 FISH

      Sample pathology report
      • Breast, left, ultrasound guided core needle biopsy:
        • Invasive breast carcinoma of no special type (ductal), grade 2
          • Estrogen receptor: positive (90%, strong intensity)
          • Progesterone receptor: positive (70%, intermediate intensity)
          • HER2 by IHC: negative (score 1+) (see comment)
        • Comment: Patients with breast cancers that are HER2 IHC 3+ or IHC 2+ / ISH amplified may be eligible for several therapies that disrupt HER2 signaling pathways. Invasive breast cancers that test HER2 negative (IHC 0, 1+ or 2+ / ISH not amplified) are more specifically considered HER2 negative for protein overexpression / gene amplification despite having some level of protein expression. Patients with breast cancers that are HER2 IHC 1+ or IHC 2+ / ISH not amplified may be eligible for treatment with a cytotoxic drug regimen that targets nonamplified / nonoverexpressed levels of HER2 (IHC 0 results do not currently result in eligibility).
      Board review style question #1

      According to the current 2023 update of American Society of Clinical Oncology (ASCO) / College of American Pathologists (CAP) guidelines for HER2 interpretation in breast cancer, how should the IHC result (see image above) be interpreted in a pathology report?

      1. HER2 negative (score 0)
      2. HER2 negative (score 1+)
      3. HER2 low positive (score 1+)
      4. HER2 positive (score 3+)
      Board review style answer #1
      B. HER2 negative (score 1+). The above IHC for HER2 shows faint / incomplete staining for HER2 in > 10% of tumor cells and is therefore interpreted with a score of 1+. Answer D is incorrect because as per the 2023 ASCO / CAP update, this IHC result should still be HER2 negative. Answer A is incorrect because tumors demonstrating HER2 IHC score 0 expression show no staining or ≤ 10% incomplete, faint / barely perceptible staining. Answer C is incorrect because although this is considered HER2 low positive, ASCO / CAP guidelines do not recommend using this terminology in the pathology report at this time. See 2023 ASCO / CAP update on HER2 interpretation (J Clin Oncol 2023;41:3867).

      Comment Here

      Reference: HER2 low breast cancer
      Board review style question #2
      What is the current understanding of the prognostic implication of HER2 low positive breast cancer patients?

      1. HER2 low breast cancer patients with an IHC score of 2+ have a better prognosis that those with an IHC score of 1+
      2. There is no definite evidence of distinct prognostic implications
      3. They have a better prognosis than HER2 negative (score 0) patients
      4. They have a poorer prognosis than HER2 negative (score 0) patients
      Board review style answer #2
      B. There is no definite evidence of distinct prognostic implications. Answers A, C and D are incorrect because although there is some evidence in the literature that shows poorer prognoses for HER2 low patients compared to HER2 negative (score 0) patients, there is no distinct prognostic or predictive implications for HER2 low status.

      Comment Here

      Reference: HER2 low breast cancer

      Histologic grading
      Definition / general
      • Most commonly used system is the Nottingham combined histologic grade (Elston-Ellis modification of Scarff-Bloom-Richardson grading system) based on (a) tubule and gland formation, (b) nuclear pleomorphism, (c) number of mitotic figures in most active areas (Histopathology 1991;19:403, Br J Cancer 1957;11:359)
      • Tumor should be graded based on representative portion of tumor (Arch Pathol Lab Med 1983;107:411)
      Essential features
      • Nottingham modification of Bloom-Richardson system, based on (a) tumor tubule formation, (b) nuclear pleomorphism and (c) number of mitotic figures in most active areas, with each given a score of 1, 2 or 3
      • Scoring is done as follows
        • 3 - 5 points: well differentiated (grade I)
        • 6 - 7 points: moderately differentiated (grade II)
        • 8 - 9 points: poorly differentiated (grade III)
      • Histologic grade is a strong predictor of survival

      Tumor tubule or gland formation
      • 1 point: > 75% of tumor
      • 2 points: 10 - 75% of tumor
      • 3 points: < 10% of tumor

      • Notes
        • Overall appearance of the tumor must be taken into consideration in scoring tubule formation
        • Tubules must have clear central lumina to be counted

      Nuclear pleomorphism
      • 1 point: minimal nuclear variation in size and shape (< 1.5 times the size of benign epithelial cell nuclei), even chromatin, inapparent to inconspicuous nucleoli
      • 2 points: moderate nuclear variation in size and shape (1.5 - 2 times the size of benign epithelial cell nuclei), typically visible but small nucleoli
      • 3 points: marked nuclear variation in size and shape (> 2 times the size of benign epithelial cell nuclei), often prominent nucleoli

      • Notes
        • Evaluate areas with greatest atypia
        • For infiltrating lobular carcinoma, more emphasis has been placed on nuclear pleomorphism since tubule formation and mitotic activity show little variation, with proposal of a 2 tiered (nuclear grade 1 versus 2 / 3) system by some (Ann Diagn Pathol 2009;13:223)

      Number of mitotic figures in most active area, counting 10 high power fields
      (a) (b) (c)
      1 point 0 - 5 0 - 9 0 - 11
      2 points 6 - 11 10 - 19 12 - 22
      3 points 11+ 20+ 23+

      • (a) Nikon or Labophot 40x objective or comparable with field diameter of 0.44 mm
      • (b) Leitz or Ortholux 25x objective or comparable with field diameter of 0.59 mm
      • (c) Leitz or Diaplan 40x objective or comparable with field diameter of 0.63 mm

      • Notes
        • Count mitotic figures at peripheral leading edge of the tumor in most mitotically active area; count 10 high power fields in the same area but not necessarily contiguous; select fields with as much tumor as possible; avoid poorly preserved areas
        • Hyperchromatic, karyorrhectic or apoptotic nuclei should be excluded; only clearly identifiable mitotic figures should be counted
        • Quick scan mitotic impression is less accurate (Hum Pathol 2008;39:584)

      Scoring
      • 3 - 5 points: well differentiated (grade I)
      • 6 - 7 points: moderately differentiated (grade II)
      • 8 - 9 points: poorly differentiated (grade III)

      Feature Subcategory Score
       1  Tumor tubule or gland formation > 75% of tumor 1
      10 - 75% of tumor 2
      < 10% of tumor 3
       2  Nuclear pleomorphism Minimal variation in nuclear size and shape 1
      Moderate variation in nuclear size and shape 2
      Marked variation in nuclear size and shape 3
       3  Mitotic count
      (per 10 high power fields)
      See table above
      (dependent on microscope field area)
      1
      2
      3
      Total score Grade I (well differentiated) 3 - 5
      Grade II (moderately differentiated) 6 or 7
      Grade III (poorly differentiated) 8 or 9
      Epidemiology
      • In the U.S., Black people overall have higher grade tumors than White people (Cancer 2003;98:908)
      Diagrams / tables

      Images hosted on other servers:
      Breast cancer grade

      Breast cancer grade

      Prognostic factors
      • Histologic grade is an independent predictor of both breast cancer specific survival (BCSS) and disease free survival (DFS) in operable breast cancer (Mod Pathol 2005;18:1067)

      Criticisms of Nottingham-Bloom-Richardson grading system
      Challenges of Nottingham-Bloom-Richardson grading system
      • Small foci of carcinoma and crush or other artifacts on core biopsy may not allow adequate assessment of mitotic activity
        • Spot count (number of mitoses in a single high power field) has been suggested as a surrogate and may be the only way to estimate mitotic activity in cases with limited tumor (Cancer 1989;64:1914)
      • Tumors with 3 + 3 + 1 pattern (i.e., < 10% tubules, marked nuclear pleomorphism but few mitotic figures) may have low mitotic count due to failure to fix tumor immediately after excision, which allows tumor cells to complete cell division and leads to reduction in observed mitotic figures; Ki67 may serve as surrogate for mitotic figure counting in these cases (Oncologist 2008;13:477)

      Possible steps to overcome limitations
      • Interobserver reproducibility of histologic grade can be improved by better distinction between various categories for nuclear pleomorphism (Am J Clin Pathol 1995;103:195)
      • Correlation with a proliferation marker such as Ki67 can help facilitate reproducibility and add complementary information to the modified Bloom-Richardson grading system; it can be used to further categorize grade II tumors into 2 prognostic subgroups with prognosis equivalent to grades I and III, respectively (J Cancer Res Ther 2018;14:537, Mod Pathol 2005;18:1067)
      • Counting 20 - 30 rather than 10 high power fields to generate the mitotic score has been suggested to allow for lowering the cutoffs for mitotic index (Mod Pathol 2005;18:1067)
      Microscopic (histologic) images

      Contributed by Julie M. Jorns, M.D.
      Grade 1 invasive lobular carcinoma

      Grade 1 invasive lobular carcinoma

      Grade 1 invasive ductal carcinoma

      Grade 1 invasive ductal carcinoma

      Grade 2 invasive ductal carcinoma Grade 2 invasive ductal carcinoma

      Grade 2 invasive ductal carcinoma

      Grade 3 invasive ductal carcinoma Grade 3 invasive ductal carcinoma

      Grade 3 invasive ductal carcinoma

      Videos

      Breast ductal carcinoma (breast cancer)

      Breast cancer grading and staging

      Sample pathology report
      • Right breast, core biopsy:
        • Invasive ductal carcinoma, preliminary Nottingham (modified Bloom-Richardson) grade 1 (tubules - 1, nuclear pleomorphism - 2, mitotic activity - 1)
      Board review style question #1

      A 52 year old woman has a breast biopsy showing the histology pictured above (H&E, 20x). A subsequent resection shows similar histology throughout the tumor. What is the Nottingham grade of this invasive breast carcinoma?

      1. Grade 1 (tubules - 1, nuclear pleomorphism - 1, mitotic activity - 1)
      2. Grade 1 (tubules - 1, nuclear pleomorphism - 2, mitotic activity - 1)
      3. Grade 1 (tubules - 3, nuclear pleomorphism - 1, mitotic activity - 1)
      4. Grade 2 (tubules - 3, nuclear pleomorphism - 2, mitotic activity - 1)
      5. Grade 2 (tubules - 3, nuclear pleomorphism - 2, mitotic activity - 2)
      Board review style answer #1
      C. Grade 1 (tubules - 3, nuclear pleomorphism - 1, mitotic activity - 1). The tumor is an invasive lobular carcinoma, which consists of infiltrative small cells (score 1 - similar in size to adjacent benign glandular epithelial cells) with minimal pleomorphism (score 1) arranged in linear files with no tubule formation (score 3), resulting in a Nottingham combined score of 5, for overall grade of 1 or a well differentiated carcinoma. Invasive lobular carcinomas have little variation in tubule formation (there is typically none but tubules may occasionally be seen) and mitotic activity, which is usually low. Thus, the Nottingham grade is often reliant on the degree of nuclear pleomorphism, which is typically minimal (score 1), resulting in an overall grade of 1 or moderate (score 2), resulting in an overall grade of 2. The other answers are incorrect as the tumor does not show any significant tubule formation (indicated in answers A and B), pleomorphism (answers B, D and E) or mitotic activity (answer E).

      Comment Here

      Reference: Breast - Histologic grading
      Board review style question #2
      When evaluating Nottingham grade of invasive breast carcinoma, what is typically the best area of the tumor to assess mitotic score?

      1. Central portion
      2. Entire tumor
      3. Peripheral leading edge
      4. Region with greatest tubules
      Board review style answer #2
      C. Peripheral leading edge. Answer B is incorrect because while the entire tumor should be examined for histologic features, the peripheral leading edge is typically most mitotically active and this area should be focused on while generating the mitotic score component in Nottingham grading. Answer A is incorrect because the central portion of a breast cancer may have central fibrosis or necrosis and may be less well preserved in resection specimens, potentially resulting in falsely lower mitotic activity than indicative of the tumor biology. Answer D is incorrect because areas with higher tubule formation are more likely to be well differentiated areas of a heterogeneous tumor with lower mitotic activity and not representative of the most aggressive portion of the tumor.

      Comment Here

      Reference: Breast - Histologic grading

      Histology
      Definition / general
      • Mammary glands located on the upper anterior thorax that produce and secrete milk in females
      Essential features
      • Adult female breast consists of branching ducts and acini; the acini group together to form lobules
      • Ductal - lobular system: composed of ducts and acini lined by a dual inner (luminal) epithelial cell layer and outer (basal) myoepithelial cell layer, resting on a basement membrane, enveloped by stroma
      • Exhibits physiologic histomorphologic changes during menstrual cycle, pregnancy / lactation, menopause, exogenous hormone therapy
      Embryology
      • Considered an epidermal gland, as it develops as diverticula of epidermis into dermis
      • Mammary gland development begins at week 5
      • Week 5:
        • Ectodermal thickenings called mammary ridges develop along ventral embryo, extending from axilla to medial thigh
        • Ridges eventually disappear, except at fourth intercostal space where mammary gland develops
      • Week 15 - 20:
        • Mesenchyme condenses around epithelial stalk under testosterone influence
        • Solid epithelial columns develop within mesenchyme
      • Week 20 - 32:
        • Differentiation of mesenchyme into fat within collagenous stroma
        • Stroma of reticular dermis forms suspensory ligaments of Cooper, attaching breast to skin and subdividing breast into lobes
        • Areola formed by ectoderm
      • Week 32 - 40:
        • Epithelial cords canalize and branch, forming lobuloalveolar structures
        • Depression in epidermis forms at the convergence of lactiferous ducts: mammary pit
      • Near term birth: nipple formed by evagination of mammary pit
      • Reference: Mills: Histology for Pathologists, 4th Edition, 2012
      Histology
      • Ductal - lobular system:
        • Consists of large lactiferous ducts that open to the surface of the nipple via lactiferous sinuses, which branch into ducts and terminal ductules which terminate in acini
        • Acini are grouped together in clusters to form lobules (terminal duct lobular unit)
        • Ductal - lobular system is arranged in series of segments (5 - 10)
        • Cellular lining of the ductal - lobular system is bilayered and consists of:
          • Inner (luminal) epithelial cells: cuboidal to columnar epithelium with pale eosinophilic cytoplasm
          • Outer (basal) myoepithelium: variably distinctive, varies in appearance from flattened cells with compressed nuclei to prominent epithelioid cells with abundant clear cytoplasm, can sometimes have myoid appearance
        • Both epithelial and myoepithelial cells thought to arise from a bipotent progenitor cell (Oncotarget 2017;8:10675)
        • Basement membrane surrounds mammary ducts, ductules and acini (Virchows Arch A Pathol Anat Histopathol 1992;421:323)
        • Basement membrane contains type IV collagen and laminin, demarcates ductal - lobular system from the surrounding stroma
        • Transgression of the myoepithelial layer and basement membrane by tumor cells defines invasion
      • Stroma:
        • Consists of varying amounts of fibrous tissue and adipose tissue and can be divided into:
          • Interlobular stroma:
            • Surrounds large ducts and terminal duct lobular units (TDLUs), denser, more collagenous than intralobular connective tissue
            • Multinucleated giant cells of unknown significance are sometimes present (Virchows Arch 2001;439:768)
          • Intralobular stroma: surrounds acini within TDLUs, loose, contains fibroblasts, scattered inflammatory cells and histiocytes
      • Nipple - areolar complex: skin variably pigmented, contains numerous sebaceous glands
        • Independent of hair follicles, except at periphery of areola; contains dense fibrous stroma with bundles of smooth muscle
        • Montgomery tubercles: sebaceous glands sharing ostium with a lactiferous duct, more prominent during pregnancy and lactation
        • Epidermis may contain occasional clear cells, either clear keratinocytes or Toker cells (epidermally located mammary duct epithelium)
        • Stratified squamous epithelium extends into duct lumens for 1 - 2 mm (Clin Radiol 2012;67:e42)
      Menstrual cycle related changes
      • Clinically manifested by changes in breast size and texture, varies between individuals
      • Proliferative phase (follicular) breast:
        • Small lobules with few terminal duct structures, smaller acinar lumina, rare mitotic figures, condensed intralobular stroma
        • Myoepithelial cells inconspicuous
        • Absence of inflammatory infiltrate in the intralobular stroma
      • Secretory phase (luteal) breast:
        • Larger lobules, more terminal duct structures, open acinar lumina, prominent apical snouting, stromal edema (breast fullness), increased mitotic figures
        • Myoepithelial cells with prominent cytoplasmic vacuolization
        • Mixed inflammatory infiltrate in the stroma
      • Perimenstrual breast:
        • Distended acinar lumina with secretions, luminal cells with scant cytoplasm, less apical snouting
        • Myoepithelial cells with extensive cytoplasmic vacuolization
        • Increased inflammatory infiltrate in the stroma
      • References: Mills: Histology for Pathologists, 4th Edition, 2012, Mod Pathol 2002;15:1348
      Pregnancy / lactation
      • Dramatic increase in the number of lobules, as well as acinar units
      • Increased epithelial cell proliferation and lobuloalveolar differentiation
      • Decrease in intralobular and interlobular stroma
      • Second and third trimester:
        • Increase in size of epithelial cells and cytoplasmic vacuolization, myoepithelial cells difficult to discern
      • Lactation:
        • Abundant secretory material and prominent cytoplasmic vacuoles, epithelial cells may have bulbous or hobnail appearance
        • Myoepithelial cells remain inconspicuous
        • Rarely areas of infarction (Exp Ther Med 2015;10:1888)
      Menopause
      • Reduction of estrogen and progesterone causes involution and atrophy of TDLUs
      • Reduction in size and complexity of acini, loss of specialized intralobular stroma
      • Hyalinized stroma with relative increase in fat
      • Reference: Mills: Histology for Pathologists, 4th Edition, 2012
      Neonatal breast
      • Identical in males and females at birth
      • Breast enlargement due to neonatal prolactin production, caused by falling maternal estrogen
      • Associated with production of colostrum
      • Extramedullary hematopoiesis may be seen (J Pediatr Hematol Oncol 2012;34:229)
      Male breast
      • Composed of ductal structures only within collagenized stroma, with no / rare acini
      Transsexual breast
      • Female to male conversion results from prolonged androgen administration, starting prior to mastectomy
      • Male to female conversion involves prolonged estrogen therapy and chemical castration with progestational agents
        • Changes include well developed lobular structures and pregnancy related changes
      Clinical features
      • Gross anatomy:
        • Lies on the anterior chest wall over the pectoralis major muscle, extends from second to sixth rib vertically and from sternal edge to midaxillary line horizontally
        • Also projects into the axilla as the tail of Spence
        • Microscopic foci of glandular tissue may extend through the pectoral fascia
        • Clinical relevance: total mastectomy does not remove all glandular breast tissue
      • Arterial supply, lymphatic flow:
        • Principal arterial supply by the internal mammary and lateral thoracic arteries
        • Lymphatic drainage: > 90% to axillary lymph nodes, remaining to internal mammary and posterior intercostal nodes
        • Relevance to staging, as metastasis to internal mammary node is higher nodal stage
      • Breast tissue is typically less nodular, late follicular phase is the optimal time for mammography (Radiol Phys Technol 2018;11:20)
      • Loss of myoepithelium is usually considered evidence of invasion, with the exception of microglandular adenosis, encapsulated papillary carcinoma and solid papillary carcinoma
      • Transsexual breast may develop any benign or malignant process seen in nontranssexual breast (Hoda: Rosen's Breast Pathology, 4th Edition, 2014)
      Gross description
      • Mastectomy attempts to remove all breast tissue
      • Different types of mastectomies: simple, modified radical, prophylactic, skin sparing, nipple sparing and radical
      • Axillary dissections performed for the staging of invasive carcinoma
        • Number of lymph nodes with metastases is the most important prognostic factor for breast carcinoma
      • Reference: Lester: Manual of Surgical Pathology, 3rd Edition, 2010
      Gross images

      Images hosted on other servers:
      Mastectomy specimen anterior view

      Mastectomy specimen, anterior view

      Microscopic (histologic) images

      Contributed by Indu Agarwal, M.D.
      Terminal duct lobular unit (TDLU) Terminal duct lobular unit (TDLU)

      Terminal duct lobular unit (TDLU)

      Extralobular and TDLU

      Extralobular and TDLU

      Myoepithelial cells

      Myoepithelial cells

      Myoid appearance Myoid appearance

      Myoid appearance


      Clear cell change Clear cell change

      Clear cell change

      Cross section of nipple Cross section of nipple

      Cross section of nipple

      Lactiferous duct

      Lactiferous duct


      Lactational changes Lactational changes

      Lactational changes

      Immunostain estrogen receptor (ER)

      Immunostain ER

      Immunostain p63

      Immunostain p63

      Positive stains
      Negative stains
      Electron microscopy images

      Images hosted on other servers:

      Acini

      Glandular and myoepithelial cells

      Videos

      Shotgun histology inactive breast

      Board review style question #1

      From the image above, what is the most likely diagnosis?

      1. Atypical apocrine adenosis
      2. Lactational changes
      3. Microglandular adenosis
      4. Secretory carcinoma
      Board review style answer #1
      B. Lactational changes. During lactation, epithelial cells are enlarged and have a bulbous or hobnail appearance with attenuated myoepithelial cells. These florid changes seen can be alarming and can be mistaken for atypia or malignancy.

      Comment Here

      Reference: Breast - Histology
      Board review style question #2
      Myoepithelial cells are always retained in which of the following?

      1. Atypical ductal hyperplasia
      2. Encapsulated papillary carcinoma
      3. Invasive tubular carcinoma
      4. Microglandular adenosis
      Board review style answer #2
      A. Atypical ductal hyperplasia. Myoepithelium is retained in atypical ductal hyperplasia and most benign and in situ lesions, with the exception of microglandular adenosis and a few in situ lesions, e.g. encapsulated papillary carcinoma.

      Comment Here

      Reference: Breast - Histology

      Hormonal therapy
      Definition / general
      • Endocrine / hormonal therapy may be utilized in the treatment of hormone receptor positive breast carcinoma in the adjuvant (postoperative) or neoadjuvant (preoperative) setting
      • Most commonly used endocrine therapies are the antiestrogen / inhibiting agent tamoxifen or aromatase inhibitors (e.g. letrozole, anastrozole)
      Essential features
      • Hormone receptor positive breast carcinomas may be treated with neoadjuvant endocrine therapy with lower toxicity and cost than the alternative of neoadjuvant chemotherapy
      • Neoadjuvant endocrine therapy with aromatase inhibitors shows response rates similar to those seen with neoadjuvant chemotherapy (JAMA Oncol 2016;2:1477)
      • Change in proliferative rate (as measured by Ki67), in the pre- vs posttreatment tumor, is related to long term prognosis and is a validated endpoint for neoadjuvant endocrine therapy (Eur J Surg Oncol 2016;42:333, Oncotarget 2017;8:26122)
      Prognostic factors
      Microscopic (histologic) description
      • Microscopic effects in invasive breast carcinomas (Eur J Cancer 2003;39:462)
        • Treatment with the aromatase inhibitor, letrozole, results in decreased mitoses and decreased progesterone receptor expression
        • Treatment with the antiestrogen / inhibiting agent, tamoxifen, results in increased tubule formation, decreased estrogen receptor expression and increased progesterone receptor
      • Microscopic effects in ductal carcinoma in situ (DCIS) (BMC Cancer 2009;9:285)
        • Treatment with antiestrogen therapy results in increased multinucleated histiocytes and degenerated cells, decrease duct extension and prominent periductal fibrosis
      Microscopic (histologic) images

      Images hosted on other servers:

      Various images

      Molecular / cytogenetics description
      • Invasive lobular and invasive ductal carcinomas demonstrate similar molecular changes in gene expression in response to endocrine therapy, such as downregulation of proliferation genes and upregulation of immune function and extracellular matrix remodeling genes (Cancer Res 2014;74:5371)
      Board review style question #1
      What tumor change, in response to preoperative endocrine therapy, would be considered predictive of a favorable outcome?

      1. Decreased histiocytes
      2. Decreased tubule formation
      3. Increased estrogen receptor expression
      4. Increased tumor cellularity
      5. Lower proliferative index
      Board review style answer #1
      E. The effects of preoperative endocrine therapy seen microscopically in the posttreatment surgical specimen may vary but most commonly include reduced tumor cellularity with decreased estrogen receptor expression in tumor cells. Response to therapy also typically includes increased histiocytic infiltration and tubule formation. A reduction in tumor proliferative index following endocrine therapy is associated with improved recurrence free survival.

      Comment Here

      Reference: Hormonal therapy

      Hormone receptors
      Definition / general
      • The most clinically important hormone receptors in breast cancer are estrogen, progesterone, and to a lesser extent, androgen receptors
      • Hormone receptor status is strongly associated with histology, prognosis and treatment response
      Essential features
      • Clinically used hormone receptor markers for breast cancers include estrogen, progesterone and androgen receptors
      • Expression of hormone markers is associated with favorable prognostic features and less aggressive histologic features
      • Estrogen and progesterone expression select breast cancer patients for hormone therapy
      Terminology
      • Estrogen receptor alpha
        • ERα is the routinely used marker in a clinical setting
        • Predictive value in prognostication and treatment response is better established in ERα compared to ERβ (Biomark Res 2020;8:39)
        • ER immunostain usually refers to ERα unless otherwise specified
      Pathophysiology
      Clinical features
      Interpretation
      • Estrogen, progesterone and androgen receptors are nuclear stains
      • Interpretation of ER and PR outlined by ASCO / CAP guideline on estrogen and progesterone receptor testing in breast cancer (J Clin Oncol 2020;38:1346)
        • Staining is considered positive regardless of intensity
        • Reported as percentage of tumor nuclei positive (0 - 100%)
        • 1 - 100% staining considered as positive, < 1% staining considered as negative
        • ER low positive category (1 - 10% staining) introduced in the latest ASCO / CAP Guideline
      • Other scoring methods (Ann Oncol 2013;24:47)
        • Allred score
          • Summation of proportion (0 - 5) and intensity (0 - 3) of ER positive tumor cells into a score of 0 to 8 out of 8
          • Proportion
            • 0: 0%
            • 1: < 1%
            • 2: 1 - 10%
            • 3: 11 - 33%
            • 4: 34 - 66%
            • 5: > 66%
          • Intensity
            • 0: negative
            • 1: weak
            • 2: moderate
            • 3: strong
          • Scores 0 and 2 interpreted as negative; scores 3 to 8 interpreted as positive
        • H score
          • Score of 0 to 300 from multiplying proportion as percentage (0 - 100) and intensity (0 - 3) of ER positive tumor cells
          • % of tumor cells with weak staining x 1 + % of tumor cells with moderate staining x 2 + % of tumor cells with strong staining x 3
      • Pitfalls in ER and PR assessment are not uncommon and should be avoided (Pathobiology 2022 Mar 4 [Epub ahead of print])
        • Positive staining in nonneoplastic or entrapped breast tissue, proliferative and in situ lesions should be excluded
        • Prolonged cold ischemia time can decrease ER and PR expression
        • ER and PR antigenicity are also sensitive to tissue processing, fixation protocol and decalcification
      Uses by pathologists
      • Routine assessment of breast cancer
        • For prognostication of breast cancers
        • Selecting ER positive breast cancers for hormonal therapy
        • ER and PR are components of the surrogate classification for breast cancer subtyping (Breast 2016;29:181)
          • ER / PR positivity indicates luminal-like breast cancers
      • Differentiating hyperplastic and neoplastic breast lesions
        • Homogenous uniform ER staining indicates clonality and favors a neoplastic process (Mod Pathol 2010;23:S1)
          • Homogenous uniform ER staining: columnar cell change, columnar cell hyperplasia, flat epithelial atypia, atypical ductal hyperplasia and carcinoma in situ
          • Heterogenous ER staining: normal breast epithelium, usual ductal hyperplasia and florid epithelial hyperplasia
        • AR can be used to demonstrate apocrine differentiation, usually with coexpression of GCDFP15 (Pathol Res Pract 1997;193:753, BMC Cancer 2014;14:546)
      • There are many other uses to ER and PR, including assessment of metastatic carcinomas and gynecological lesions
      Prognostic factors
      Microscopic (histologic) description
      • The most common type of ER positive (and PR, AR positive breast cancers which are correlated with ER expression) breast cancer is low grade invasive breast carcinoma of no special type (IBC, NST) (Breast Care (Basel) 2020;15:327)
        • Features include prominent tubule formation, mild nuclear pleomorphism and a low mitotic count
        • Other histological features include fewer tumor infiltrating lymphocytes, less lymphovascular invasion and more fibrotic foci (Br J Cancer 2020;123:1223, Ann Surg Oncol 2013;20:2842)
      • Special types of breast cancer which are commonly ER positive include lobular, micropapillary, mucinous, papillary and tubular carcinomas (Breast Care (Basel) 2020;15:327)
      Microscopic (histologic) images

      Contributed by Joshua J.X. Li, M.B.Ch.B. and Gary M. Tse, M.B.B.S.
      Low grade IBC, NST Low grade IBC, NST

      Low grade IBC, NST

      Invasive lobular carcinoma

      Invasive lobular carcinoma

      Papillary carcinoma

      Papillary carcinoma

      Estrogen receptor

      Estrogen receptor

      Progesterone receptor

      Progesterone receptor


      Androgen receptor

      Androgen receptor

      Apocrine carcinoma

      Apocrine carcinoma

      Androgen receptor

      Androgen receptor

      Atypical ductal hyperplasia

      Atypical ductal hyperplasia

      Estrogen receptor

      Estrogen receptor

      Columnar cell change

      Columnar cell change


      Estrogen receptor

      Estrogen receptor

      Florid epithelial hyperplasia

      Florid epithelial hyperplasia

      Estrogen receptor

      Estrogen receptor

      Estrogen receptor low positive Estrogen receptor low positive Estrogen receptor low positive

      Estrogen receptor low positive


      Estrogen receptor low positive

      Estrogen receptor low positive

      Virtual slides

      Images hosted on other servers:

      Invasive lobular carcinoma

      Low grade IBC, NST

      ER immunostaining

      Positive staining - normal
      Positive staining - disease
      Negative staining
      • Estrogen receptor:
      • Progesterone receptor and androgen receptor expression is closely related to estrogen receptor expression in most breast lesions, with the exception of apocrine lesions (see Positive staining - disease)
      Sample pathology report
      • Left breast, biopsy:
        • Invasive breast carcinoma of no special type (see comment)
        • Comment: Immunostaining shows that the invasive tumor cells are ER positive (strong [3+] intensity, 70%) and PR positive (strong [3+] intensity, 70%).
      Board review style question #1

      Which of the following is a histological feature associated with estrogen receptor positivity in breast cancers?

      1. Abundant intratumoral and stromal lymphocytes
      2. Frequent mitotic figures
      3. Highly pleomorphic nuclei
      4. Lymphovascular invasion
      5. Prominent tubule formation
      Board review style answer #1
      E. Prominent tubule formation. ER positivity is associated with low grade features (Breast Care (Basel) 2020;15:327). Lymphovascular invasion, medullary features and tumor infiltrating lymphocytes are associated with ER negativity (Br J Cancer 2020;123:1223, Cancer Biol Med 2020;17:293).

      Comment Here

      Reference: Hormone receptors

      Idiopathic granulomatous mastitis
      Definition / general
      • Lobulocentric granulomatous inflammation
      • Diagnosis of exclusion (excluding other causes of granulomatous inflammation)
      Essential features
      • Rare, benign inflammatory process affecting women of childbearing age
      • Clinical symptoms and radiologic findings concerning for breast cancer
      • Reported association with pregnancy, lactation, oral contraceptive pill use, hyperprolactinemia, autoimmunity
      • Lobulocentric granulomatous inflammation
      • Diagnosis of exclusion
      Terminology
      • Idiopathic granulomatous mastitis
      • Lobular granulomatous mastitis
      ICD coding
      • ICD-10: N61.2 - granulomatous mastitis
      • ICD-11: GB21 - inflammatory disorders of breast
      Epidemiology
      • Rare
      • Most common in parous women, childbearing age
      • Frequent in postpartum and breastfeeding women
      • Median age in the 30s
      Sites
      • Breast, most often unilateral, rarely bilateral
      Pathophysiology
      • Unknown
      Etiology
      • Unknown
      • Proposed causes or associations: pregnancy, lactation, oral contraceptive pill use, hyperprolactinemia, autoimmunity, infection
      Clinical features
      • Mimics breast cancer, including inflammatory breast cancer (Breast J 2019;25:1245)
      • Palpable breast mass
      • Breast pain
      • Abscess formation with fistula tract formation
      • Nipple retraction
      • Peau d'orange
      • Rare lymphadenopathy
      Diagnosis
      • Breast core biopsy or surgical excision with histologic examination
      Laboratory
      • Microbiology studies to exclude infection
      • Serologic markers to exclude autoimmune processes
      • Endocrine studies
      Radiology description
      Radiology images

      Images hosted on other servers:

      Mammogram; focal asymmetry

      Ultrasound; hypoechoic tubular extensions

      MRI; enhancing retroareolar mass

      Prognostic factors
      • Self limiting in approximately 50% of patients (Am J Surg 2015;210:258)
      • Can persist or recur, prompting treatment
      Case reports
      Treatment
      Clinical images

      Contributed by Nicolas Ajkay, M.D.
      Missing Image

      IGLM with multiple
      sinus tracts
      opening onto skin

      Missing Image

      IGLM with response
      to corticosteroids



      Images hosted on other servers:

      Appearance of second lump postbiopsy

      Firm mass with multiple sinuses

      Gross description
      • Gray-tan firm mass
      • Size variable; average 5 - 6 cm
      • Ill defined margins
      • Subtle nodularity
      Microscopic (histologic) description
      • Lobulocentric granulomatous inflammation (Pathology 2004;36:254)
      • Granulomas often contain neutrophils
      • Neutrophils can create microabscesses; overlapping features with cystic neutrophilic granulomatous mastitis
      • Multinucleated giant cells, lymphocytes, plasma cells and eosinophils within and around lobules
      • Extensive inflammation might obliterate lobulocentric distribution
      • Caseous necrosis not present
      • Schauman and asteroid bodies not common
      Microscopic (histologic) images

      Contributed by Jonathan Marotti, M.D. and Mary Ann Gimenez Sanders, M.D., Ph.D.
      Missing Image Missing Image

      Lobulocentric inflammation

      Missing Image

      Multinucleated giant cells

      Missing Image

      Nonnecrotizing granulomas


      Missing Image

      Lobulocentric inflammation

      Missing Image Missing Image

      Granulomas and multinucleated giant cells

      Cytology description
      Positive stains
      • CD68: positive in histiocytes
      Negative stains
      • Gram stain for gram positive bacilli (Corynebacterium)
      • Acid fast stain for mycobacteria
      • GMS or PAS stains for fungi
      • IgG4 in plasma cells for IgG4 related sclerosing mastitis
      Sample pathology report
      • Left breast, needle core biopsy:
        • Breast tissue with lobulocentric granulomatous inflammation (see comment)
        • Comment: Sections show a lobulocentric mastitis with histiocytes, granulomas, neutrophils and lymphocytes. The histologic findings are nonspecific but the main differential diagnosis includes idiopathic granulomatous mastitis, cystic neutrophilic granulomatous mastitis (typically caused by Corynebacterium), other infection, sarcoidosis and reaction to foreign material, among others. Given the lobulocentric nature of the granulomatous inflammation and lack of currently identifiable microorganisms, idiopathic granulomatous mastitis is favored. However, clinical and microbiologic correlation is required. Special stains for bacteria, acid fast bacilli and fungi are negative. Foreign material is not identified. There is no evidence of malignancy in this sample.
      Differential diagnosis
      Board review style question #1

      A 33 year old woman presents with a painful breast lump concerning for breast cancer. A core biopsy is performed. Which histologic finding is most characteristic of idiopathic granulomatous mastitis?

      1. Keratin debris with associated foreign body inflammatory response
      2. Large necrotizing granulomas
      3. Lobulocentric granulomatous inflammation
      4. Microcysts with gram positive bacilli
      5. Well formed granulomas within interlobular stroma
      Board review style answer #1
      C. Lobulocentric granulomatous inflammation. Although not specific, the most characteristic histologic finding of idiopathic granulomatous mastitis is granulomatous inflammation centered on lobules (lobulocentric inflammation). Idiopathic granulomatous mastitis can have overlapping histologic features with cystic neutrophilic granulomatous mastitis (CNGM); however, microcysts with gram positive bacilli (Corynebacterium) are identified in CNGM. Keratin debris with an associated foreign body inflammatory response is usually seen in squamous metaplasia of lactiferous ducts (SMOLD). Large necrotizing granulomas are more typical of an infectious processes, such as mycobacterial infection. The presence of well formed granulomas within the interlobular stroma suggests sarcoidosis.

      Comment here

      Reference: Idiopathic granulomatous mastitis

      Inflammatory
      Definition / general
      • Clinicopathologic entity characterized by rapid onset (≤ 6 months) and enlarged, erythematous and edematous (peau d’orange) breast due to dermal plugging of lymphatic vessels by tumor (Breast Cancer Res 2005;7:52)
      • Involvement of dermal lymphatics alone without the classic clinical presentation is not considered inflammatory carcinoma
      Essential features
      • Clinical presentation of breast cancer with dermal lymphatic involvement
        • Breast skin with erythema and edema occupying at least 33% of the breast is required to make the diagnosis
        • Must be of rapid onset; duration of symptoms of ≤ 6 months
      ICD coding
      • ICD-O: 8530/3 - inflammatory carcinoma
      • ICD-10: C50.9 - malignant neoplasm of breast of unspecified site
      • ICD-11: 2C62 - inflammatory carcinoma of the breast
      Epidemiology
      Sites
      • Breast, breast skin (dermal lymphatics)
      Pathophysiology
      • Tumor microemboli in the lymphatic spaces causes obstruction that leads to the clinical presentation of peau d’orange appearance
      • E-cadherin accumulation / overexpression within tumor emboli (Anticancer Res 2010;30:3903)
      Etiology
      • No association with inherited genetic mutation
      • No association with family history
      Clinical features
      • Rapid onset of erythema, edema, induration, warmth and tenderness of the breast skin (Ann Oncol 2011;22:515)
      • Patients may have a history of being diagnosed with mastitis that does not respond to at least 1 week of antibiotics (Ann Oncol 2011;22:515)
      • Erythema occupying at least 33% of the breast (Ann Oncol 2011;22:515)
      • Duration of history < 6 months (Ann Oncol 2011;22:515)
      • May or may not have a breast mass, typically not palpable due to rapid spread to lymphatics and distant sites but low volume of invasive carcinoma in breast
      Diagnosis
      • At least 2 skin punch biopsies between 2 - 8 mm of the most prominent area of skin discoloration (Ann Oncol 2011;22:515)
      Radiology description
      Radiology images

      Contributed by Kiran Manjee, M.D. and Julie M. Jorns, M.D.
      Skin thickening and mass

      Skin thickening and mass

      Prognostic factors
      • Previous studies have reported poor survival rates; however, this may be due to advanced stage at presentation and delayed diagnosis
      • 2010 - 2015 Surveillance, Epidemiology and End Results (SEER) database showed similar breast cancer specific survival in patients with inflammatory breast cancer and stage T4 noninflammatory breast cancer (BMC Cancer 2021;21:138)
      • 2010 - 2011 SEER data showed worse outcomes in hormone receptor positive / HER2 negative and triple negative breast cancer compared with triple positive and hormone receptor negative / HER2 positive subtypes (J Cancer Res Clin Oncol 2017;143:161)
      • Use of neoadjuvant therapy has improved survival rates
      • Histologic subtype does not impact survival (PLoS One 2016;11:e0145534)
      • 50% of patients with complete pathological response will subsequently recur
      • Overall 5 year survival rate is 41% (Cancer.Net: Breast Cancer - Inflammatory: Statistics [Accessed 27 December 2022])
        • 5 year survival with regional lymph node involvement is 56%
        • 5 year survival with distant metastasis is 19%
      • p53 status is not evaluated routinely but positive staining (≥ 10%) was associated with decreased response to chemotherapy (n = 59) (Clin Cancer Res 2004;10:6215)
      Case reports
      Treatment
      • Multidisciplinary team approach is critical
      • Neoadjuvant chemotherapy, mastectomy with lymph node dissection, radiation therapy and adjuvant endocrine therapy if indicated (Ann Oncol 2011;22:515)
      • If there is a good clinical response to neoadjuvant chemotherapy, some may be able to safely undergo breast conserving therapy instead of a mastectomy (Gland Surg 2018;7:520)
      Clinical images

      Contributed by Amanda L. Kong, M.D., M.S. and Mark R. Wick, M.D.
      Erythema and edema

      Erythema and edema

      Discoloration

      Discoloration

      Gross description
      • Skin findings and mass lesions are usually not present after neoadjuvant therapy
      Microscopic (histologic) description
      • Presence of tumor cells in dermal lymphovascular spaces
      • Tumor cells or lymphatic spaces may be scant and therefore assessment at multiple levels is recommended
      • It is not a specific histological subtype
        • Most associated invasive carcinoma is high grade invasive ductal carcinoma, no special type; however, other types of breast cancers can present as inflammatory carcinoma (PLoS One 2016;11:e0145534)
      • Ductal carcinoma in situ is often absent
      • Classified as T4d in AJCC cancer staging
      • T stage does not change following neoadjuvant therapy (ypT4d)
      Microscopic (histologic) images

      Contributed by Kiran Manjee, M.D., Julie M. Jorns, M.D., Weijie Li, M.D. and AFIP
      Dilated lymphatics Dilated lymphatics

      Dilated lymphatics

      Dermal lymphovascular invasion Dermal lymphovascular invasion

      Dermal lymphovascular invasion

      Dermal lymphovascular invasion Dermal lymphovascular invasion

      Dermal lymphovascular invasion


      Dermal lymphovascular invasion

      Dermal lymphovascular invasion

      HER2 / neu HER2 / neu

      HER2 / neu

      Locally advanced breast cancer Locally advanced breast cancer Locally advanced breast cancer

      Locally advanced breast cancer


      Skin punch biopsy

      Infiltrative nests

      Tumor emboli

      Tumor in dermal lymphatics

      Tumor in dermal lymphatics

      Virtual slides

      Images hosted on other servers:
      Breast, inflammatory carcinoma

      Inflammatory breast carcinoma

      Positive stains
      Negative stains
      Sample pathology report
      • Skin, left breast, punch biopsy:
        • Positive for high grade mammary (ductal) carcinoma within dermal lymphovascular spaces
        • Comment: Immunohistochemistry shows carcinoma to be:
          • GATA3 (subset, moderate) positive, supporting breast origin
          • Estrogen receptor (< 1%) negative
          • Progesterone receptor (< 1%) negative
          • HER2 / neu (0) negative for overexpression
      Differential diagnosis
      • Invasive carcinoma involving dermis:
        • CD31 / ERG negative
        • Lack of clear intralymphatic space involvement
      • Ductal carcinoma in situ (DCIS):
        • Tumor clusters within myoepithelial lined spaces
      • Infection (mastitis, cellulitis, abscess):
        • More common in lactating women or in those with history of recent trauma or surgery
        • Responds to antibiotic treatment
      • Radiation dermatitis:
        • Clinical history of radiation
        • Benign biopsies
      • Other causes of edema (lymphatic obstruction or surgical removal, congestive heart failure):
        • Clinical history of medical condition, surgery, travel (parasitic infections)
        • Benign biopsies
      • Locally advanced carcinoma with skin involvement:
        • Skin changes may be present due to dermal lymphatic involvement but clinical history is of longer duration (> 6 months), often with palpable mass
        • Histologically bulky invasive carcinoma is present alongside lymphovascular space involvement
      • Metastatic neoplasms from other sites:
        • Unusual histology for breast
        • Immunohistochemistry can help differentiate
      Board review style question #1

      A middle aged woman presents with rapid onset (< 6 months) of breast skin thickening and discoloration. A punch biopsy is done and the patient is diagnosed with inflammatory breast cancer. She undergoes neoadjuvant chemotherapy and subsequent mastectomy reveals focal ductal carcinoma in situ (DCIS) but no residual invasive or metastatic carcinoma. How would she be staged?

      1. ypTX
      2. ypT0
      3. ypTis
      4. ypT4d
      Board review style answer #1
      D. ypT4d. In inflammatory breast cancer, the T stage is classified as T4d and does not change after neoadjuvant chemotherapy.

      Comment Here

      Reference: Inflammatory
      Board review style question #2
      What feature best distinguishes inflammatory breast carcinoma from locally advanced breast cancer?

      1. Mammographic mass
      2. Peau d’orange
      3. Skin thickening and discoloration < 6 months
      4. Tumor emboli within lymphatic spaces
      Board review style answer #2
      C. Skin thickening and discoloration < 6 months. Inflammatory breast cancer is defined as breast cancer with rapid onset of clinical symptoms of skin thickening and discoloration / erythema involving at least 33% of the breast. There is frequent peau d’orange and tumor emboli within lymphatic spaces on skin punch biopsy; however, these findings are not necessary to make the diagnosis of inflammatory breast cancer nor are they unique to inflammatory breast cancer as they can also be seen in locally advanced breast cancer. A mass may or may not be seen on mammogram as in inflammatory breast cancer tumor quickly spreads to lymphatics (and beyond) and mammography has lower sensitivity in this setting.

      Comment Here

      Reference: Inflammatory

      Intraductal papilloma
      Definition / general
      • Benign intraductal proliferation of epithelial cells with fibrovascular cores and underlying myoepithelial cells
      Essential features
      • Common benign breast lesion
      • Distinction with malignant papillary lesions may be difficult on biopsy and immunohistochemistry may be necessary
      • No consensus regarding management of papilloma without atypia
      • Surgical excision of intraductal papilloma with atypical epithelial proliferation is recommended
      Terminology
      • Duct papilloma of the breast
      • Intracystic papilloma of the breast (old terminology)
      ICD coding
      • ICD-10: D24 - benign neoplasm of the breast
      Epidemiology
      Sites
      Pathophysiology
      • Poorly understood
      • Intraductal papilloma might originate from bipotent progenitor cells that differentiate as luminal and myoepithelial cells (Am J Pathol 2018;188:1106)
      Clinical features
      • Solitary central intraductal papilloma may present as a retroareolar mass with serous or serosanguineous nipple discharge
      • Peripheral intraductal papilloma is often discovered incidentally
      Diagnosis
      Radiology description
      Radiology images

      Contributed by Mark R. Wick, M.D.

      Mammogram



      Images hosted on other servers:

      Round or ovoid mass on mammogram and ultrasound

      Well circumscribed
      mass with
      calcifications

      Ductal dilatation on mammogram

      Prognostic factors
      Case reports
      Treatment
      • No consensus for intraductal papilloma without atypia
      • Complete surgical excision should be performed for intraductal papilloma with atypia
      Clinical images

      Images hosted on other servers:

      Male with nipple discharge

      Gross description
      Gross images

      Contributed by Mark R. Wick, M.D.
      Intracystic mass

      Intracystic mass



      Images hosted on other servers

      Intraductal polypoid mass

      Frozen section description
      • Frozen section not performed on lesions suspected to be intraductal papilloma
      Microscopic (histologic) description
      Microscopic (histologic) images

      Contributed by Xiaoxian (Bill) Li, M.D., Ph.D.
      Minute intraductal papillomas

      Minute intraductal papillomas

      Papillary cores

      Papillary cores

      Involved by usual ductal hyperplasia Involved by usual ductal hyperplasia Involved by usual ductal hyperplasia Involved by usual ductal hyperplasia

      Involved by usual ductal hyperplasia


      Involved by apocrine metaplasia

      Involved by apocrine metaplasia

      Involved by ductal carcinoma in situ Involved by ductal carcinoma in situ Involved by ductal carcinoma in situ Involved by ductal carcinoma in situ

      Involved by ductal carcinoma in situ



      Contributed by Jijgee Munkhdelger, M.D., Ph.D. and Andrey Bychkov, M.D., Ph.D.

      Intraductal papilloma with DCIS immunoprofile

      Intraductal papilloma with DCIS, CK5/6

      Intraductal papilloma with DCIS, p63

      Virtual slides

      Images hosted on other servers:

      Sclerosing intraductal papilloma

      Cytology description
      Positive stains
      Molecular / cytogenetics description
      • PIK3CA / AKT1 pathway mutations commonly involved (Mod Pathol 2010;23:27, Mod Pathol 2021;34:1044):
        • AKT1 mutations are commonly seen in papilloma without (54%) or with mild hyperplasia (15%)
        • PIK3CA mutations can be seen in 21 - 42% in papilloma with or without atypia
      Videos

      Intraductal papilloma

      Sample pathology report
      • Right breast, 9 o'clock, 1 cm from the nipple, ultrasound guided core biopsy:
        • Intraductal papilloma with apocrine metaplasia
        • Comment: Patchy positivity for CK5 and ER supports the above diagnosis.
      Differential diagnosis
      Board review style question #1

      Regarding this breast specimen, what is the correct diagnosis?

      1. Intraductal papillary carcinoma
      2. Intraductal papilloma with apocrine metaplasia
      3. Intraductal papilloma with DCIS
      4. Intraductal papilloma with LCIS
      5. Invasive papillary carcinoma
      Board review style answer #1
      B. This intraductal papilloma shows typical features of apocrine metaplasia.

      Comment Here

      Reference: Intraductal papilloma
      Board review style question #2
      What is the staining pattern of p63 for intraductal papilloma of the breast without atypia?

      1. At the periphery of involved duct only
      2. In the fibrovascular cores only
      3. Intraductal papillomas do not stain for p63
      4. At the periphery and in the fibrovascular cores
      Board review style answer #2
      D. p63 stain for myoepithelial cells is positive throughout the lesion, including in the fibrovascular cores and at the periphery of the involved duct.

      Comment Here

      Reference: Intraductal papilloma
      Board review style question #3
      What is the typical staining pattern of atypical epithelial proliferation involving an intraductal papilloma of the breast?

      1. Diffusely positive for ER and patchy positive for CK5/6, negative for p63
      2. Diffusely positive for ER, negative for p63 and CK5/6
      3. Diffusely positive for ER, p63 and CK5/6
      4. Negative for ER, p63 and CK5/6
      5. Patchy positive for ER, negative for p63 and CK5/6
      Board review style answer #3
      B. The atypical low grade epithelial proliferation is diffusely positive for ER, negative for CK5/6 and negative for p63.

      Comment Here

      Reference: Intraductal papilloma

      Invasive breast cancer of no special type (NST)
      Definition / general
      • Most common type of invasive breast carcinoma (75 - 80%)
      • Invasive carcinoma with evidence of mammary epithelial origin either by morphology or immunohistochemistry
      • Diagnosis of exclusion, lacks the histologic features to classify morphologically as a special subtype of breast cancer
      • Arises from epithelial (progenitor / stem) cells at the terminal duct lobular unit (as do lobular and other types of breast carcinoma)
      • Although the ductal and lobular nomenclature does not reflect the histogenesis of these tumor types, this terminology remains in common use
      • Patient age, tumor stage, lymph node status, histologic grade, clinical biomarker profile and gene expression signature are important prognostic factors
      • Treated with surgical excision, with or without radiotherapy, systemic chemotherapy or targeted therapies
      • ASCO / CAP guidelines require assessment by standard breast biomarkers (estrogen and progesterone receptors and HER2)
      Essential features
      • Large and heterogenous group of tumors that lack features diagnostic of special types
      • Range in morphology, grade, biomarker profile, molecular subtype and clinical behavior
      Terminology
      • Invasive breast carcinoma, not otherwise specified (NOS)
      • Invasive ductal carcinoma
      • Not recommended: scirrhous carcinoma
      ICD coding
      • ICD-10:
        • C50.812 - malignant neoplasm of overlapping sites of left female breast
        • C50.912 - malignant neoplasm of unspecified site of left female breast
      Epidemiology
      Sites
      • Breast
      • Can occur in axilla accessory breast tissue
      Etiology
      • Origin of breast cancer is multifactorial and complex but most studies point to hormones, reproductive factors, diet, environment and genetics as general factors
      • After menopause, about 40% of risk is modifiable (Am J Epidemiol 2008;168:404)
      • Hormone related risk factors:
      • Hormone factors associated with reduced risk of breast cancer:
        • Oophorectomy before age 35 or first child before age 18
        • Obesity prior to age 40 - due to anovulatory cycles and lower progesterone levels in late cycle (J Natl Cancer Inst 2011;103:250)
      • Genetic risk factors:
        • Breast cancer shows familial clustering
        • First degree relatives with breast cancer; having 1 first degree relative (mother, sister, daughter) creates a relative risk of 2 - 3x, higher if relative is affected before age 50 or had bilateral disease; relative risk with 2 first degree relatives is 4 - 6x (Int J Cancer 1997;71:800)
        • Li-Fraumeni syndrome (germline p53 mutations) - 25% of patients develop breast cancer
        • Mutations of BRCA1 and BRCA2 genes are associated with familial breast cancer at an early age, account for 20 - 60% of familial breast cancer but only 5% of all cases
        • Germline BRCA1 mutations are associated with risk for triple negative breast cancers, whereas germline BRCA2 mutations are associated with hormone receptor positive breast cancers (J Clin Oncol 2015;33:304)
        • Cowden disease (multiple hamartoma syndrome): autosomal dominant, due to 10q mutation: 30 - 50% risk of breast cancer (ductal carcinoma in situ or invasive ductal carcinoma) by age 50; also benign skin tumors (Hum Pathol 1998;29:47)
        • Heterozygous carriers for ataxia telangiectasia have an 11% risk of breast cancer by age 50
        • Black women (compared to white women) have more frequent breast cancers in women < age 40; present with higher stage tumors with higher nuclear grade that are more likely ER / PR negative, have higher mortality rate (J Natl Cancer Inst 200;100:1804, Cancer 1998;83:2509)
        • Women have 100x risk of breast cancer compared to men (J Clin Oncol 2011;29:4381)
      • Environmental risk factors:
      • Other risk factors:
      Clinical features
      • In patients > 65 years, 87% of patients with invasive breast carcinoma have NST (Crit Rev Oncol Hematol 2008;67:263)
      • Most (~90%) are unifocal and with higher frequency in the upper outer quadrant (Cancer Epidemiol 2016;44:186)
      • In nonscreened populations, NST carcinoma commonly presents as a palpable mass
      • Less commonly, it presents with skin retraction, nipple discharge, nipple inversion, a change in the size or shape of the breast or a change in the color or texture of the skin
      • In screened populations, it commonly presents as a spiculated mass with or without calcifications but may present as well circumscribed masses, architectural distortion or calcifications alone
      • Clinically, IBC NST is further divided into the following biomarker defined subtypes for treatment purposes:
        • ER positive, HER2 negative
        • ER positive, HER2 positive cancers
        • ER negative, HER2 positive cancers
        • ER negative, HER2 negative cancers
      • Frequency of these biomarker / clinical subtypes varies by population characteristics but the ER positive subtype is the most common
      • HER2 positive subtype comprises 10 - 15%, whereas ER negative / HER2 negative subtype comprises approximately 15 - 30% (Crit Rev Oncol Hematol 2010;76:44, JAMA 2006;295:2492)
      Diagnosis
      • Histologic examination of involved tissue
      Radiology description
      • Mammographically often presents as a spiculated mass with or without calcifications but may present as an architectural distortion or calcifications alone
      • Targeted ultrasonography often used to enhance visualization
      • MRI often shows enhancing mass lesion with variable washout kinetic patterns (Breast J 2010;16:394)
      • MRI is sensitive but nonspecific technique is reserved for screening patients at high risk (e.g., BRCA mutation carriers), patients with dense breast tissue, estimating extent of disease in patients with lobular carcinoma, evaluating response to neoadjuvant chemotherapy or evaluating indeterminate abnormalities on mammography / ultrasound (Breast 2013;22:S77)
      Radiology images

      Contributed by Mark R. Wick, M.D.

      Invasive

      Mammogram

      Spot film

      Spot film

      Prognostic factors
      Case reports
      Treatment
      • Surgical excision by breast conserving surgery (lumpectomy) or mastectomy, with or without axillary lymph node dissection
      • May be offered local radiotherapy to the breast after lumpectomy or to the chest wall after mastectomy to reduce local recurrence risk
      • May be offered systemic chemotherapy either neoadjuvantly or adjuvantly
      • May be offered targeted therapies (e.g., antiendocrine or anti-HER2 directed)
      • Selection of treatment (type of surgery, inclusion of radiotherapy and selection of systemic therapies) is based on multifactorial consideration of several factors including but not limited to tumor size, histologic grade, stage, biomarker status, results of genomic risk assessment (e.g., Oncotype DX Recurrence Score ®), anatomic location, patient age and comorbidities, heritable breast cancer risk (e.g., BRCA status), prior exposure to chemotherapy or radiation therapy, cosmetic outcome and patient's preference
      Clinical images

      Contributed by Mark R. Wick, M.D.

      Fungating, breast skin

      Gross description
      • Grossly evident mass, with an irregular, stellate outline or nodular configuration
      • Mass is usually poorly circumscribed and contracts from surrounding tissue
      • Firm or even hard on palpation and may have a gritty feel when cut with a knife, grating sound when scraped
      • May show streaks of chalky white elastotic stroma penetrating surrounding stroma (crab-like), calcification
      • Large tumors have hemorrhage, necrosis and cystic degeneration
      • May be fixed to chest wall and cause skin dimpling or nipple retraction
      • Some tumors, including neoadjuvant treated cancers, may be grossly inapparent and require careful correlation with the imaging at the time of gross examination and tissue sampling
      Gross images

      Contributed by Mark R. Wick, M.D. and @Andrew_Fltv on Twitter
      Irregular stellate mass Irregular stellate mass

      Irregular stellate mass

      Irregular stellate mass Irregular stellate mass

      Irregular stellate mass


      Invasive breast cancer of no special type (NST) Invasive breast cancer of no special type (NST) Invasive breast cancer of no special type (NST) Invasive breast cancer of no special type (NST)

      Invasive breast cancer of no special type (NST)

      Microscopic (histologic) description
      • Histologic grading is based on the Nottingham / modified Bloom & Richardson Score (Histopathology 1991;19:403):
        • Tubule formation (1 - 3 points):
          • > 75% (1 point)
          • 10 - 75% (2 points)
          • < 10% (3 points)
        • Nuclear pleomorphism (1 - 3 points):
          • Small, regular, uniform, similar to normal ductal epithelial cells, 2 - 3x RBC (1 point)
          • Moderate increase in size / variability (2 points)
          • Large nuclei, marked variation, often vesicular chromatin with prominent nucleoli (3 points)
        • Mitotic count (1 - 3 points), dependent on microscopic field area
        • Total score (add points for tubule formation, nuclear pleomorphism and mitotic count):
          • 3 - 5 points: grade 1
          • 6 - 7 points: grade 2
          • 8 - 9 points: grade 3
        • Histological features of IBC NST vary considerably from case to case and even within the same case
        • Margins vary from highly infiltrative, permeating the surrounding tissue, to continuous pushing margins
        • Architecture varies from sheets, nests, clusters, cords or individual cells (but lacks the cytomorphological characteristics of invasive lobular carcinoma)
        • Tubular formations are prominent in well differentiated tumors but absent in poorly differentiated tumors
        • Variable cytological features:
          • Cytoplasm may be abundant and eosinophilic but it can show other features in some tumors, including as clear, foamy or granular
          • Nuclei may be regular and uniform or highly pleomorphic with prominent or multiple nucleoli
        • Mitotic figures are variable from virtually absent to extensive
        • 2 distinct growth patterns exist:
          • Large and solid nests or syncytial infiltrative growth pattern with little associated stroma and an expansive growth that compresses the surrounding stroma (e.g., most basal-like breast cancers)
          • Tumors characterized by small cancer nests accompanied by marked fibrosis (desmoplastic / scirrhous); this type diffusely infiltrates the surrounding tissue as an irregular shaped spiculated mass
        • Calcification in 60% of cases, variable necrosis
        • Elastosis involves stroma, wall of vessels and ducts and causes grossly noted chalky streaks
        • Often ductal carcinoma in situ (DCIS) (up to 80%)
          • In some cases, DCIS is extensive
          • Associated DCIS is usually of same nuclear grade as the invasive carcinoma
        • Perineural invasion (28%)
        • Mast cells are associated with low grade tumors
        • Uncommon features: eosinophils intraluminal crystalloids (BMC Cancer 2007;7:165, Arch Pathol Lab Med 1997;121:593)
        • No myoepithelial cell lining (as seen in DCIS or benign lesions)
        • Angiolymphatic invasion in 35%; differs from tissue retraction because:
          • Occurs outside margin of carcinoma
          • Does not conform precisely to space it is in
          • Endothelial lining is present and is CD31+, ERG, D2-40+, CD34+ and factor VIII+
        • Mixed IBC NST and special subtype pattern:
          • IBC NST with < 10% special type should be classified as IBC NST with an option to describe the focal specialized pattern in the report comment
          • Invasive breast carcinoma with > 90% specialized type patterns should be classified as the special type
          • When IBC NST contains a component of specialized tumor type that comprises > 10% but < 90%, the tumor is classified as mixed tumor
          • It is recommended to report both the specific special type present and the overall percentage of the cancer with the special pattern (e.g., mixed IBC NST and mucinous carcinoma [40% mucinous])
          • Biomarker status of both areas should be reported since they can be distinct
          • Grade of the overall tumor should be considered
        • Special morphologic patterns of NST:
          • IBC NST shows a wide spectrum of differentiation with variable extent of these differentiation lineages
          • Some tumors comprises the end of the differentiation spectrum but the prognostic value and the morphological features are not sufficient to classify them as special types, even if such differentiation features comprise > 90%
          • Current opinion is to consider such tumors as special morphologic patterns of IBC NST to improve the diagnostic concordance of the classification system and to highlight the lack of prognostic and management implications
          • These patterns include:
            • Basal-like or medullary-like patterns:
              • These include breast carcinomas previous described as medullary-like carcinoma or carcinoma with medullary features, as well as basal-like carcinoma
              • These tumors typically show high histological grade, a prominent tumor associated lymphocytic (TIL) infiltrate, a triple negative phenotype and often have basal-like molecular profiles
            • Oncocytic, sebaceous, melanocytic, glycogen rich, lipid rich, carcinomas with osteoclast-like stromal giant cells, pleomorphic and choriocarcinomatous carcinoma patterns
            • Invasive carcinoma with neuroendocrine differentiation:
              • A proportion of IBC NST shows neuroendocrine differentiation of variable extent as determined by histological, histochemical and immunohistochemical analysis occurs but lack histologic features to classify as large cell or small cell neuroendocrine carcinoma or neuroendocrine tumor of the breast
              • These tumors do not carry prognostic or clinical significance and they can be considered as variant (morphological pattern) of IBC NST (Hum Pathol 2003;34:1001)
      Microscopic (histologic) images

      Contributed by Julie M. Jorns, M.D., Kristen E. Muller, D.O., Gary Tozbikian, M.D. and Emad Rakha, M.D.
      Invasive ductal carcinoma, grade 1 Invasive ductal carcinoma, grade 1 Invasive ductal carcinoma, grade 1

      Invasive ductal carcinoma, grade 1

      Invasive ductal carcinoma, grade 2 Invasive ductal carcinoma, grade 2 Invasive ductal carcinoma, grade 2

      Invasive ductal carcinoma, grade 2


      Invasive ductal carcinoma with necrosis, grade 3

      Invasive ductal carcinoma with necrosis, grade 3

      Invasive ductal carcinoma with scattered apoptotic cells

      Invasive ductal carcinoma with scattered apoptotic cells

      CK 5/6

      CK5/6

      Central necrosis

      Central necrosis

      Central fibrosis

      Central fibrosis

      Adjacent normal breast

      Adjacent normal breast


      CK7

      CK7

      GATA3

      GATA3

      Lobular and apocrine features, E-cadherin

      Lobular and apocrine features, E-cadherin

      Lobular and apocrine features

      Lobular and apocrine features

      Lobular features

      Lobular features

      Lobular features, E-cadherin

      Lobular features, E-cadherin


      GCDFP-15 positive invasive ductal carcinoma

      GCDFP-15 positive invasive ductal carcinoma

      GCDFP-15

      GCDFP-15

      Angiolymphatic invasion

      Angiolymphatic invasion

      Involvement of a fibroadenoma

      Involvement of a fibroadenoma

      “Involvement of a fibroadenoma, p63

      Involvement of a fibroadenoma, p63

      Microinvasion, p63

      Microinvasion, p63


      Microinvasion

      Microinvasion

      Actin - muscle specific

      Actin - muscle specific

      Pseudo DCIS pattern

      Pseudo DCIS pattern

      Calponin, pseudo DCIS pattern

      Calponin, pseudo DCIS pattern

      Mammaglobin positive invasive ductal carcinoma

      Mammaglobin positive invasive ductal carcinoma

      Mammaglobin

      Mammaglobin


      SOX10+ invasive ductal carcinoma SOX10 positive invasive ductal carcinoma

      SOX10+ invasive ductal carcinoma

      SOX10 SOX10

      SOX10

      p120+ invasive ductal carcinoma

      p120+ invasive ductal carcinoma

      p120

      p120


      Invasive ductal carcinoma, grade 3

      Invasive ductal carcinoma, grade 3

      Invasive ductal carcinoma, grade 2 Invasive ductal carcinoma, grade 2

      Invasive ductal carcinoma, grade 2

      Invasive ductal carcinoma, grade 1

      Invasive ductal carcinoma, grade 1

      Cytology description
      • Can use cellular pleomorphism, nuclear size, nuclear margin, nucleoli, lack of naked nuclei, cellular dyscohesion and mitoses in addition to necrosis to assess cytologic tumor grade, which correlates with histologic grade (Diagn Cytopathol 2003;29:185)
      Cytology images

      Contributed by Mark R. Wick, M.D. and Abdulaziz Mohamed, M.D.
      FNAB

      FNAB

      Diff quik

      Diff-Quik

      47 year old woman with 3 x 2 cm painless retroareolar mass and ipsilateral axillary lymphadenopathy 47 year old woman with 3 x 2 cm painless retroareolar mass and ipsilateral axillary lymphadenopathy 47 year old woman with 3 x 2 cm painless retroareolar mass and ipsilateral axillary lymphadenopathy 47 year old woman with 3 x 2 cm painless retroareolar mass and ipsilateral axillary lymphadenopathy

      47 year old woman with 3 x 2 cm painless retroareolar mass and ipsilateral axillary lymphadenopathy

      Positive stains
      Negative stains
      Electron microscopy description
      • Glandular differentiation (microvilli and terminal bars on luminal side)
      Molecular / cytogenetics description
      • ER+ pathway: characterized by gains of 1q, loss of 16q, sometimes amplification of 17q12
      • ER- pathway: characterized by loss of 13q, gain of 11q13, amplification of 17q12, p53 mutations common, increased expression of genes associated with cell proliferation
      • PI3KCA mutations associated with endocrine therapy resistance (Virchows Arch 2016;469:35, Int J Breast Cancer 2020;2020:3759179)
      • p53 mutations in many basal-like tumors (Nature 2012;490:61)
      • HER2 gene amplification in approximately 15% (Cancer Res 1993;53:4960, J Clin Oncol 1989;7:1120)
      • Gene expression profiling stratifies breast cancers into several intrinsic molecular subtypes (e.g., luminal A, luminal B, HER2 enriched, basal-like, etc.) (Nature 2012;490:61)
      • 5 - 10% of breast cancers are hereditary as result of mutations in various genes, such as BRCA1, BRCA2, TP53, STK11, CD1, PTEN, MDM2, RB, CHEK2 (Acta Oncol 2019;58:135)
      • ESR1 mutations associated with resistance to endocrine therapy (JAMA Oncol 2016;2:1310)
      Molecular / cytogenetics images

      Contributed by Mark R. Wick, M.D. and Semir Vranic, M.D.
      HER2 amp FISH HER2 amp FISH

      HER2 amp FISH

      TOP2A CEP17 FISH - no gene amplification

      TOP2A CEP17 FISH - no gene amplification

      Videos

      Ductal carcinoma - breast

      Breast ductal carcinoma (breast cancer)

      Breast carcinoma: invasive - pathology mini tutorial

      Sample pathology report
      • Left breast, mass, ultrasound guided core needle biopsy:
        • Invasive breast carcinoma, NST (invasive ductal carcinoma), grade 2, measuring 1.5 cm
        • Estrogen receptor: positive (95%, strong intensity)
        • Progesterone receptor: negative (0%)
        • HER2: negative (score 1+)
      Differential diagnosis
      • IBC NST should be differentiated from special types of breast carcinoma:
        • > 90% special tumor characteristics
        • IBC NST admixed with 10 - 90% of other special subtype(s) is designated as mixed IBC NST and special subtype with description of the percentage of special subtype components provided
      • Metastatic adenocarcinoma with overlapping features, including lung adenocarcinoma, endometrioid carcinoma, serous carcinoma and renal cell carcinoma:
        • Correlation with clinical and radiologic history
        • Appropriate use of immunohistochemical panel of stains (e.g., estrogen receptor, GATA3, TRPS1, etc.)
      • Rarely, examples of skin adnexal (sweat gland) carcinoma of the breast skin:
        • Morphology and immunohistochemical profile may be indistinguishable from breast cancer, superficial location centered in the skin versus within the breast (adjacent ducts / lobules)
      • Melanoma:
        • Can mimic high grade breast cancer
        • Melanin pigment, lack of cytokeratin expression, positive for melanoma melanocytic markers
      • Ductal carcinoma in situ involving sclerosing lesions (e.g., sclerosing adenosis):
        • IHC using myoepithelial markers shows retained myoepithelial cell layer
      Board review style question #1

      Which of the following immunohistochemical stains would be most helpful for distinguishing DCIS from an invasive beast carcinoma with a cribriform growth pattern?

      1. Calponin and p63
      2. CD31 and CK7
      3. CK5/6 and estrogen receptor
      4. Estrogen receptor and progesterone receptor
      5. HER2 and cytokeratin AE1 / AE3
      Board review style answer #1
      A. Calponin and p63. The use of myoepithelial markers is most useful for distinguishing in situ from invasive carcinoma. Utilization of 2 myoepithelial markers, including both a cytoplasmic marker (e.g., calponin or smooth muscle myosin) and a nuclear marker (p63 or p40) is recommended, as myoepithelial expression can be patchy or attenuated (p63 / p40) and show nonspecific expression in endothelial cells (SMMS); the combination of 2 markers provides increased sensitivity and specificity. Both DCIS and invasive breast carcinoma (NST) may show expression of hormone receptors, HER2 and cytokeratins.

      Comment Here

      Reference: Invasive breast cancer of no special type (NST)
      Board review style question #2

      Which of the following is true regarding the diagnosis of invasive breast carcinoma of no special type (NST)?

      1. Always shows extensive (> 90%) tubular / acinar formation
      2. Generally negative for E-cadherin and p120 (cytoplasmic)
      3. Low grade tumors are generally positive for HER2 overexpression by immunohistochemistry
      4. Majority of cases are hereditary due to mutations in either BRCA1 or BRCA2
      5. Requires assessment of standard biomarkers estrogen and progesterone receptors and HER2
      Board review style answer #2
      E. Per ASCO / CAP guideline recommendations, all new primary invasive breast cancers require assessment of standard breast biomarkers: estrogen receptor, progesterone receptor and HER2. IBC NST show positive membranous reactivity of E-cadherin and p120. Degree of tubular formation can range between tumors and within tumors. Generally, low grade tumors show greater tubule formation, are positive for hormone receptor expression and are negative for HER2. The majority of breast cancer is sporadic; only 5 - 10% of breast cancer is considered familial.

      Comment Here

      Reference: Invasive breast cancer of no special type (NST)
      Board review style question #3

      Which of the following findings would you expect from a low grade invasive breast carcinoma of no special type?

      1. Basal-like intrinsic molecular subtype
      2. Lack of desmoplastic stromal response, positive for S100, positive for laminin and collagen IV around glands, estrogen receptor negative
      3. Positive for HER2 overexpression
      4. Positive for hormone receptors (estrogen and progesterone receptors)
      5. Positive for p63 and calponin
      Board review style answer #3
      D. The majority of low grade invasive breast carcinomas of no special type belong to the luminal A intrinsic molecular subtype, are positive for hormone receptors (estrogen and progesterone receptors) and are negative for HER2. The invasive component will lack expression of myoepithelial markers (e.g., p63 and calponin). Lack of desmoplasia, expression of S100, negativity for estrogen receptor and expression of laminin and collagen IV around infiltrative glands would be consistent with microglandular adenosis.

      Comment Here

      Reference: Invasive breast cancer of no special type (NST)

      Juvenile papillomatosis
      Definition / general
      • Grossly distinct multinodular mass with clustering of cystic formations resembling swiss cheese, composed of epithelial proliferation and clustered cysts
      • First described by Rosen in 1980 (Am J Surg Pathol 1980;4:3)
      Terminology
      • Not part of WHO breast classification
      Epidemiology
      • Mean age 19 years
      • 2 / 3 are less than age 20 years
      Clinical features
      Case reports
      Clinical images

      Contributed by Mark R. Wick, M.D.

      Mammogram

      Gross description
      • Localized multinodular mass with clustering of cystic formations resembling swiss cheese
      Gross images

      Contributed by Mark R. Wick, M.D.

      Juvenile papillomatosis

      Microscopic (histologic) description
      • Florid epithelial hyperplasia and papillomatosis, cysts with foamy histiocytes and sclerosing adenosis
      • Variable apocrine metaplasia, atypia and necrosis
      Microscopic (histologic) images

      Contributed by Mark R. Wick, M.D.

      Juvenile papillomatosis



      AFIP images

      Multiple cystically dilated ducts

      Hyperplastic epithelium in dilated duct

      Intraductal epithelial proliferation with necrosis

      Apocrine metaplasia
      of hyperplastic
      intraductal
      epithelium


      Hyperplastic epithelium and foam cells

      With sclerosing adenosis
      and apocrine metaplasia
      adjacent to cystically
      dilated ducts

      Multiple small intraductal papillomas in 15 year old girl

          

      High power shows hyperplastic
      intraductal epithelium, myoepithelial
      cells are associated with the
      delicate fibrovascular stalks

      Cytology description
      • Difficult to diagnose
      • Cystic fluid, but mass persists after aspiration
      • Sheets of hyperplastic breast epithelium with areas resembling fibroadenoma, macrophages and apocrine cells (Diagn Cytopathol 1993;9:457)
      Cytology images

      Contributed by Areej M. Al Nemer, M.D.
      Suspicious nipple discharge

      Suspicious nipple discharge


      Ki67 breast
      Definition / general
      Essential features
      Terminology
      • MIB1 is not equivalent to Ki67; rather, it is a widely used monoclonal antibody against the Ki67 antigen (Breast Cancer 2020;27:1058)
      Pathophysiology
      • Anti-Ki67 antibody binds Ki67 protein in formalin fixed paraffin embedded tissue block and highlights nuclei of all cells in nonresting cell cycle phases (Pathology 2017;49:166)
      • Increased expression in tumor cells is a marker of cell cycle activation and proliferative activity
      • Ki67 often is heterogeneously expressed in breast carcinomas, with hot spots and inactive areas (J Natl Cancer Inst 2013;105:1897)
      Clinical features
      Interpretation
      Uses by pathologists
      • As an independent prognostic marker to predict disease free survival, overall survival and complete pathologic response to neoadjuvant chemotherapy
      • Subtyping ER positive breast carcinomas into 2 prognostically distinct subtypes:
        • Luminal A (low Ki67 index; better prognosis)
        • Luminal B (high Ki67 index; worse prognosis) (Mod Pathol 2014;27:554)
      • Ki67 IHC MIB1 pharmDx has been approved by the FDA as a companion diagnostic for detecting Ki67 expression in patients with high risk breast cancer who may be eligible for treatment with abemaciclib combined with endocrine therapy
        • FDA approval of abemaciclib was restricted to patients at high risk of recurrence and with a Ki67 score ≥ 20%, as determined by an FDA approved test; however, restricting the approval to patients with a high Ki67 and the 20% cutoff is debated (Ann Oncol 2022;33:234)
      Prognostic factors
      Microscopic (histologic) images

      Contributed by Anas Mohamed, M.D. and Joseph Geradts, M.D.
      Lobular carcinoma, low proliferation

      Lobular carcinoma, low proliferation

      Lobular carcinoma, low Ki67

      Lobular carcinoma, low Ki67

      Ductal carcinoma, low proliferation

      Ductal carcinoma, low proliferation

      Ductal carcinoma, low Ki67

      Ductal carcinoma, low Ki67


      Ductal carcinoma, intermediate proliferation

      Ductal carcinoma, intermediate proliferation

      Ductal carcinoma, intermediate Ki67

      Ductal carcinoma, intermediate Ki67

      Ductal carcinoma, high proliferation

      Ductal carcinoma, high proliferation

      Ductal carcinoma, high Ki67

      Ductal carcinoma, high Ki67

      Positive staining - normal
      Positive staining - disease
      • Low Ki67 index is observed in benign breast tumors, luminal A carcinomas, low grade ductal carcinomas, lobular carcinomas and tubular carcinomas
      • High Ki67 index is observed in triple negative breast carcinomas, including metaplastic and medullary carcinomas, HER2 positive carcinomas, luminal B carcinomas, high grade carcinomas and pleomorphic lobular carcinomas (BMC Res Notes 2019;12:605)
      Sample pathology report
      • Right breast, core needle biopsy:
        • Invasive ductal carcinoma, grade 2
        • Ki67 index: 5% (low)

      • Left breast, total mastectomy:
        • Invasive mammary carcinoma with medullary pattern
        • Ki67 index: 40% (high)
      Board review style question #1
      Which of the following breast neoplasms is most likely to be associated with an elevated Ki67 labeling index?

      1. Invasive ductal carcinoma, grade 1
      2. Invasive lobular carcinoma, classic type
      3. Metaplastic carcinoma
      4. Tubular carcinoma
      Board review style answer #1
      C. Metaplastic carcinoma. High Ki67 index is observed in triple negative breast carcinomas, including metaplastic and medullary carcinomas, as well as HER2 positive carcinomas, luminal B carcinomas, high grade carcinomas and pleomorphic lobular carcinomas (BMC Res Notes 2019;12:605).

      Comment Here

      Reference: Ki67 breast
      Board review style question #2

      Which of the following breast tumors is likely to demonstrate a Ki67 labeling index similar to the one shown in the picture?

      1. Invasive cribriform carcinoma
      2. Invasive ductal carcinoma, grade 3
      3. Medullary carcinoma
      4. Pleomorphic lobular carcinoma
      Board review style answer #2
      A. Invasive cribriform carcinoma. Low Ki67 index is observed in benign breast tumors, luminal A carcinomas, low grade ductal carcinomas, lobular carcinomas and tubular carcinomas and lower grade breast cancer subtypes, such as invasive cribriform carcinoma (BMC Res Notes 2019;12:605).

      Comment Here

      Reference: Ki67 breast

      Lactating adenoma
      Definition / general
      • Well circumscribed mass, arising during or shortly after pregnancy, composed of cuboidal cells with actively secreting, closely packed glands
      Essential features
      • Most prevalent breast lesion in pregnant women and during postpartum period
      • Must be distinguished from carcinoma, as breast cancer is the second most common malignancy during pregnancy
      • Age range (19 - 34 years), female
      • Benign, slow growing
      • Unknown capability of malignant transformation
      Terminology
      • Adenomatous lactational hyperplasia, nodular lactational hyperplasia (not recommended by WHO)
      ICD coding
      • ICD-0: 8204/0 - Lactating adenoma
      • ICD-10: D24 - Benign neoplasm of breast
      • ICD-11: 2F30.1 - Lactating adenoma of breast
      • ICD-11: XH0W31 - Lactating adenoma
      Epidemiology
      • 19 - 34 years, female, predominantly in third decade of life
      • Most prevalent breast lesion in pregnant women (mostly third trimester) and during postpartum period (puerperium)
      Sites
      Pathophysiology
      • Unknown if de novo neoplasm or hyperplastic condition
      • Some postulate may arise in a pre-existing adenoma (tubular adenoma, fibroadenoma) with superimposed lactational changes; however, lack of MED12 exon 2 mutations, frequently found in fibroadenomas, does not support this theory (Genes Chromosomes Cancer 2017;56:11)
      • Increase in serum estrogen, progesterone and prolactin
      Clinical features
      • Painless, soft, palpable, solid, mobile discrete mass
      • May be bilateral and multifocal
      • Infarction may lead to pain, tenderness and rapid enlargement
      Diagnosis
      Radiology description
      Radiology images

      Contributed by Kristen E. Muller, D.O.

      Well circumscribed hypoechoic mass

      Hypoechoic mass with hypervascularity

      Prognostic factors
      • Slow growing, spontaneous regression
      • No progression to cancer on follow up studies
      • Unknown if capable of carcinogenesis or risk factor
      • Coexistence of lactating adenoma with breast malignancy reported (Indian J Cancer 2015;52:585, J Clin Pathol 2005;58:87)
      Case reports
      Treatment
      • Observation for slow growing
      • Many spontaneously regress after termination of breastfeeding
      • Enucleation in cases that warrant excision (enlarging, worrisome clinical or histologic features)
      Clinical images

      Images hosted on other servers:
      Large mass

      Large mass

      Gross description
      Gross images

      Contributed by Dr. Mark R. Wick
      Lactating adenoma

      Lactating adenoma



      Images hosted on other servers:
      Well circumscribed mass

      Well circumscribed mass

      Microscopic (histologic) description
      • Well circumscribed proliferation of hyperplastic closely packed lobules with both epithelial and myoepithelial cell layers separated by thin, delicate connective tissue
      • Glands lined by actively secreting cuboidal or hobnail shaped cells with small round nuclei and granular to clear vacuolated cytoplasm
      • Variably prominent small, pinpoint nucleoli may be seen but cells lack cytologic atypia
      • Occasional mitoses may be seen
      • Resembles pregnancy-like (pseudolactational) changes
      • May resemble fibroadenoma or tubular adenoma with lactational change microscopically
      Microscopic (histologic) images

      Contributed by Kristen E. Muller, D.O.

      Lactating adenoma biopsy

      Vacuolated cytoplasm

      Hypersecretory features


      Delicate stroma

      Prominent hobnailing

      Psammomatous calcification

      Virtual slides

      Images hosted on other servers:

      Lactating adenoma

      Cytology description
      • Loose cohesive clusters of monomorphic cells or single cells
      • Cells contain foamy to finely vacuolated cytoplasm, round uniform nuclei with fine chromatin and small nucleoli (J Clin Diagn Res 2013;7:2417)
      • Background of foamy material
      Cytology images

      Contributed by Dr. Mark R. Wick

      FNAB



      Images hosted on other servers:

      Increased lobular size

      Cytoplasmic vacuoles and background foamy material

      Positive stains
      Molecular / cytogenetics description
      • 50 gene NGS panel showed no mutations in three lactating adenomas in one series, including MED12 exon 2 mutations (frequently seen in fibroadenomas) (Genes Chromosomes Cancer 2017;56:11)
      Sample pathology report
      • Right breast mass, needle biopsy:
        • Breast tissue with lactational change / lactating adenoma
        • Microcalcifications associated with lactational change / lactating adenoma
      Differential diagnosis
      • Lobular hyperplasia (normal physiologic event in pregnancy):
        • Not a well defined mass
      • Delayed involution of lactation:
        • Not a well defined mass
        • Hyperplastic and involuting lobules
        • Neutrophils, macrophages and lymphocytes in background
        • More frequently associated with calcifications
      • Fibroadenoma with secretory activity:
        • Fibroepithelial architecture
        • Prominent stroma
        • Focal, not diffuse, proliferation of cells with secretory activity
      Board review style question #1
      Which of the following is correct for the entity pictured from this breast biopsy from a 27 year old pregnant woman?



      1. Frequently shows a prominent stromal component similar to fibroepithelial lesions
      2. Histologically composed of closely packed glands lined by actively secreting cuboidal or hobnail shaped cells
      3. Never develops in ectopic breast tissue along milk line
      4. The glands lack a myoepithelial cell layer
      5. Third most prevalent breast lesion in pregnant women
      Board review style answer #1
      B. Histologically composed of closely packed glands lined by actively secreting cuboidal or hobnail shaped cells

      Comment Here

      Reference: Lactating adenoma
      Board review style question #2
      What is the most common breast lesion in pregnant and lactating women?

      1. Adenomyoepithelioma
      2. Atypical ductal hyperplasia
      3. Fibroadenoma
      4. Invasive ductal carcinoma
      5. Lactating adenoma
      Board review style answer #2
      E. Lactating adenoma

      Comment Here

      Reference: Lactating adenoma

      LCIS florid
      Definition / general
      • Nonclassical or variant form of lobular carcinoma in situ (LCIS) where acini and ducts are markedly distended with little to no intervening stroma, sometimes imparting a mass-like architecture and comprising cells with cytologic features of classical LCIS (type A or type B cells) with or without comedonecrosis and calcifications
      Essential features
      • Nonclassical or variant form of lobular carcinoma in situ (LCIS) defined by markedly distended acini / ducts of involved terminal duct lobular units (TDLUs) with scanty to no intervening stroma or an expanded acinus / duct filling at least 1 high power field (~40 - 50 cells in diameter)
      • Cells share cytologic features of classic LCIS (type A or type B cells) and lack significant cytologic atypia and pleomorphism
      • Commonly unifocal and shows a continuous distribution compared to classic LCIS
      • Shows features of a high risk precursor lesion of invasive carcinoma with a higher upgrade rate, increased association with invasive carcinoma and more genetic complexity compared to classic LCIS
      Terminology
      • Florid LCIS or LCIS, florid type (F-LCIS)
      • LCIS with necrosis / comedonecrosis
      • Lobular neoplasia (can include atypical lobular hyperplasia and LCIS)
      • LCIS, variant type (not recommended)
      • The term florid is not synonymous with extensive or diffuse
      ICD coding
      • ICD-10
        • D05 - carcinoma in situ of breast
        • D05.0 - lobular carcinoma in situ of breast
        • D05.00 - lobular carcinoma in situ of unspecified breast
        • D05.01 - lobular carcinoma in situ of right breast
        • D05.02 - lobular carcinoma in situ of left breast
      • ICD-11: 2E65.0 - lobular carcinoma in situ of breast
      Epidemiology
      Sites
      Pathophysiology
      • First described in 2006 as a form of LCIS with central necrosis (Am J Surg Pathol 2006;30:1445)
      • Early tumorigenesis of LCIS is marked by loss of heterozygosity of the wild type allele and inactivating mutations of E-cadherin (Br J Cancer 1997;76:1131)
      • E-cadherin, encoded by the CDH1 gene on chromosome 16q22.1, is a cell - cell adhesion molecule that helps maintains lobular architecture
      • Clonal relationship to classic LCIS: harbors characteristic signature of LCIS showing 16q loss and 1q gain (Hum Pathol 2013;44:1998)
      • Florid LCIS is proposed to progress from classic LCIS by acquiring additional genetic (i.e., frequent ERBB2 or ERBB3 alterations, CCND1 amplifications) and chromosomal alterations (Mod Pathol 2020;33:1287, Mod Pathol 2020;33:1078, Hum Pathol 2013;44:1998)
      • Significantly more genome alterations and amplifications compared to classic LCIS (Hum Pathol 2013;44:1998)
      • Similar genetic complexity as apocrine pleomorphic LCIS (Hum Pathol 2013;44:1998)
      Etiology
      Diagnosis
      • Diagnostic core or excisional biopsy, surgical excision
      Radiology description
      Radiology images

      Contributed by Kristen E. Muller, D.O. and Victoria M. Jones, M.D.
      Mammographic calcifications

      Mammographic calcifications

      Prognostic factors
      Treatment
      Microscopic (histologic) description
      • Diagnosis requires the presence of at least 1 of 2 architectural features (Mod Pathol 2020;33:1287, Am J Surg Pathol 2019;43:399, Ann Diagn Pathol 2020;45:151481)
        • Markedly distended acini / ducts of involved TDLUs with little to no intervening stroma or
        • Expanded acinus / duct filling at least 1 high power field (~40 - 50 cells in diameter)
      • Marked distention of acini and ducts with little intervening stroma may impart a confluent, mass-like architecture
      • Solid, dyshesive, monomorphic proliferation of classic LCIS type A cells (scant cytoplasm, small uniform nuclei, inconspicuous nucleoli) or type B cells (increased cytoplasm, larger nuclei, more prominent nucleoli) (Am J Surg Pathol 2019;43:399, Ann Diagn Pathol 2020;45:151481)
      • Absence of marked nuclear pleomorphism and atypia, distinguishes from pleomorphic LCIS
      • Comedo or single cell necrosis is common, up to 71% of cases (Mod Pathol 2020;33:1287, Mod Pathol 2021;34:1495)
      • Comedo type necrosis, mitosis and apoptosis are more frequent in florid LCIS than in pleomorphic LCIS (Hum Pathol 2018;78:163)
      • May observe apocrine features (rounded, enlarged nuclei with prominent nucleoli and abundant eosinophilic and granular cytoplasm), more commonly in ER negative cases (Mod Pathol 2021;34:1495)
      • Rarely composed entirely of signet ring cells, which has a high frequency of associated invasive lobular carcinoma (80% of 10 cases in small series) (Arch Med Res 2010;41:436)
      • Often coexists with classic or pleomorphic LCIS
      Microscopic (histologic) images

      Contributed by Kristen E. Muller, D.O. and Victoria M. Jones, M.D.
      Florid LCIS with mass-like architecture Florid LCIS with mass-like architecture

      Florid LCIS with mass-like architecture

      Single acinus

      Single acinus

      Little intervening stroma

      Little intervening stroma

      Necrosis

      Necrosis


      Calcifications

      Calcifications

      Type B cells Type B cells

      Type B cells

      Type A cells

      Type A cells

      Mitoses

      Mitoses


      Apocrine features Apocrine features

      Apocrine features

      Invasive lobular carcinoma

      Invasive lobular carcinoma

      E-cadherin

      E-cadherin

      Positive stains
      Negative stains
      Molecular / cytogenetics description
      • Florid LCIS shares the LCIS molecular hallmarks of 16q loss and 1q gain (Mod Pathol 2020;33:1287, Mod Pathol 2020;33:1078)
        • CDH1 gene encoding for E-cadherin on chromosome 16
      • Pathogenic alterations in CDH1, ERRB2, PIK3CA, ERRB3, RUNX1, FOXA1, CCND1 and CBFB (Mod Pathol 2020;33:1287, Mod Pathol 2020;33:1078)
      • Frequent ERBB2 / ERBB3 alterations (HER2 / HER3) are associated with more aggressive potential and invasive lobular carcinoma (Mod Pathol 2020;33:1078)
      • Additional, complex genomic alterations including (Mod Pathol 2020;33:1078)
        • Gain of interstitial or whole arm 17q, loss of distal or whole arm 8p, 11q and 17p, loss of distal or whole arm 18q or whole chromosome 18, loss of 13q or whole chromosome 13, gain of distal or whole arm 16p, gain of 8q or whole chromosome 8
      • Significantly more fraction genome alteration and loss, number of breakpoints and chromosomes with breakpoints and amplifications than classic LCIS (Hum Pathol 2013;44:1998)
      • Similar genetic complexity to apocrine pleomorphic LCIS (Hum Pathol 2013;44:1998)
      Videos

      Lobular carcinoma in situ and high risk breast lesions

      Sample pathology report
      • Breast, right, core needle biopsy:
        • Lobular carcinoma in situ (LCIS), florid type (see comment)
        • Comment: The biopsy shows ducts and lobules that are markedly distended by a proliferation of classic LCIS cells (small, round, discohesive, uniform cells) with comedo type necrosis and calcifications. An E-cadherin immunostain is negative in lesional cells supporting interpretation as florid LCIS.
      Differential diagnosis
      Board review style question #1

      The photomicrographs above are from a 63 year old woman who underwent core needle biopsy for screen detected calcifications. An E-cadherin immunostain is negative in lesional cells. What is the most appropriate diagnosis?

      1. Atypical lobular hyperplasia
      2. Ductal carcinoma in situ
      3. Lobular carcinoma in situ, classic type
      4. Lobular carcinoma in situ, florid type
      5. Lobular carcinoma in situ, pleomorphic type
      Board review style answer #1
      D. Lobular carcinoma in situ, florid type. The images show markedly expanded acini with central necrosis and calcification and little intervening stroma. The cells are uniform, small and round without marked pleomorphism. The differential usually includes ductal carcinoma in situ (DCIS) and lobular carcinoma in situ (LCIS). A negative E-cadherin immunostain supports interpretation as LCIS and given the marked distention with scanty stroma, florid LCIS is the best diagnosis. Answer B is incorrect because this lesion is negative for E-cadherin and DCIS would have it retained. Answer C is incorrect because this meets diagnostic criteria for florid LCIS given the marked acinar distention. Answer E is incorrect because the lesional cells lack cytologic features of pleomorphic LCIS. Answer A is incorrect because atypical lobular hyperplasia (ALH) shows similar cytologic features to florid LCIS; however, criteria for ALH include either type A or type B cells that distend 50% or more acini within a lobule but not uniformly present throughout the entire terminal duct lobular unit (TDLU) or involves all acini in a TDLU but does not distend the acini. In contrast, florid LCIS shows markedly distended acini in a TDLU.

      Comment Here

      Reference: LCIS florid
      Board review style question #2

      Which of the following is a true statement regarding florid lobular carcinoma in situ (LCIS)?

      1. Florid LCIS is always HER2 negative
      2. Florid LCIS is not typically detected by mammography
      3. Florid LCIS is rarely upgraded to invasive carcinoma; thus, surgical excision is not recommended
      4. Florid LCIS shares the classic LCIS molecular hallmarks of 16q loss and 1q gain
      Board review style answer #2
      D. Florid LCIS shares the classic LCIS molecular hallmarks of 16q loss and 1q gain. In addition, florid LCIS has significantly more genomic alterations and amplifications compared to classic LCIS and in some studies, shows a similar genetic complexity to apocrine pleomorphic LCIS. Answer B is incorrect because florid LCIS is frequently detected by imaging and presents with calcifications (most commonly) or a mass / density. Answer C is incorrect because the upgrade rate to invasive carcinoma or DCIS is between 17 and 39%. Although consensus guidelines are not yet available, most experts agree that surgical excision should be recommended. Answer A is incorrect because florid LCIS is reported to be HER2 positive 13 - 18% of the time and shows frequent ERBB2 / ERBB3 alterations in molecular studies.

      Comment Here

      Reference: LCIS florid

      LCIS pleomorphic
      Definition / general
      • A morphologic subtype of lobular carcinoma in situ (LCIS) composed of a noninvasive neoplastic proliferation of large dyscohesive cells with marked nuclear pleomorphism (nuclei are > 4 times the size of a lymphocyte, equivalent to the cells of high grade DCIS)
      • With or without apocrine features (Histopathology 2020;77:181)
      Essential features
      • Lobulocentric proliferation of cells which are significantly more pleomorphic and larger when compared to cells of classic LCIS
      • Lobular acini involved by pleomorphic LCIS and often substantially expanded
      • Cytologic features more closely resemble high grade DCIS rather than classic LCIS
        • Large nuclei (≥ 4x larger than a lymphocyte)
        • Pleomorphism: 2 - 3x variation in nuclear size and shape
        • May be associated with necrosis, including comedonecrosis
      • Other features overlap with classic LCIS
        • Loss / dysfunction of E-cadherin
        • Cellular dyscohesion
        • Lack of cell polarity
        • Intracytoplasmic lumina, may have signet ring cells
        • Pagetoid spread
      • Regarded as a genetically and biologically more advanced lesion than classic LCIS and is a direct precursor to invasive carcinoma
      Terminology
      • First described by Frost et al. in 1996 (AJSP Rev Rep 1996;1:27)
      • Pleomorphic LCIS (may be denoted as PLCIS, PL LCIS or P LCIS) is considered a morphologic variant of classic LCIS (CLCIS)
      • Although it falls under the category of lobular neoplasia, the terms nonclassic lobular carcinoma in situ and variant lobular carcinoma in situ are not preferred because they do not distinguish between PCLIS and another morphologic subgroup called florid LCIS (FLCIS)
      • In the past, some have referred to what is now PLCIS as pleomorphic ductal lobular carcinoma in situ (PL / DLCIS), which is no longer used
      • The terms lobular carcinoma in situ with pleomorphic morphology or lobular carcinoma in situ with pleomorphic features are not preferred as they are more vague than using pleomorphic lobular carcinoma in situ
      • While there is no established cutoff, some have suggested that lobular lesions be considered pleomorphic when at least 10% of the tumor cells show high grade nuclei (Histopathology 2014;64:981)
      • Lobular acini involved by pleomorphic lobular cells are often substantially expanded but they may be only mildly distended or show no distension; the latter still qualify as pleomorphic LCIS since there is no recognized category of pleomorphic atypical lobular hyperplasia (ALH) (analogous to not having a high grade atypical ductal hyperplasia)
      • Currently there is no term for a lesion with both the nuclear morphology of pleomorphic LCIS and architectural features of florid LCIS
      ICD coding
      • ICD-O:
        • 8520/2 - lobular carcinoma in situ, NOS
        • 8519/2 - lobular carcinoma in situ, pleomorphic
      • ICD-11:
        • 2E65.0 & XH6EH0 - lobular carcinoma in situ of breast & lobular carcinoma in situ, NOS
      Epidemiology
      • PLCIS is relatively rare, with an estimated incidence of < 5% of all LCIS (Histopathology 2014;64:981)
      • Prevalence may be underestimated given the similarities of PLCIS to DCIS, especially high grade DCIS and probable misdiagnoses as such in the past
      • Most patients with PLCIS are White females (Ann Surg Oncol 2015;22:4263)
      • Compared to CLCIS, patients with PLCIS are typically older and postmenopausal
      Sites
      • Occurs in the breast with no specific laterality predilection
      • PLCIS has been noted to be more unifocal and continuous in distribution than CLCIS, which is typically multicentric and bilateral (Am J Surg Pathol 2019;43:399)
      Pathophysiology
      • CDH1 inactivation, which leads to loss or impaired function of E-cadherin, is an early event and hallmark of lobular lesions, including PLCIS (Histopathology 2020;77:181)
        • E-cadherin plays a major role in intercellular adhesion and cell polarity
      • 1q gain and 16q loss combined with the lack of E-cadherin expression observed in PLCIS support a lobular lineage
      • PLCIS accumulates additional alterations and generally exhibits greater genomic instability than classic LCIS (J Pathol 2005;207:1, Mod Pathol 2020;33:1287)
      • Whether PLCIS arises from CLCIS and represents a genetically more advanced lesion or if PLCIS originates from a separate disease process is not entirely clear; however
        • PLCIS and pleomorphic invasive lobular carcinoma (ILC) are frequently associated with foci of classic ILC and areas showing a morphological continuum between PLCIS and CLCIS have also been described (J Pathol 2005;207:1)
        • Synchronous CLCIS and apocrine PLCIS share similar chromosomal changes with additional changes present in the apocrine PLCIS, suggesting evolution from the same precursor or through the same genetic pathway (Am J Surg Pathol 2009;33:1683)
      Etiology
      • No well known causative factors specific to PLCIS
      • Etiologic factors associated with the development of CLCIS currently have insufficient data for being considered etiologic factors for PLCIS
      • A study found that in their cohort of patients with nonclassic LCIS, which also included florid LCIS and LCIS with pleomorphic features, none had a personal history of breast cancer nor were known BRCA1 / 2 carriers but 41.7% (10/24) had a first degree family history of breast cancer (Surg Oncol 2019;28:190)
      Clinical features
      • Patients are usually asymptomatic
      • The majority of patients (up to 87%) present with abnormal screening detected mammographic findings, such as calcifications (Am J Surg Pathol 2009;33:1683)
      • Some cases of PLCIS lack specific radiologic features and may be an incidental microscopic finding in breast biopsies
      • Less commonly, patients can present with a palpable mass or nipple discharge (Histopathology 2014;64:981, Surg Oncol 2019;28:190)
      • Patients who present with a palpable mass usually have PLCIS involving other lesions (e.g., a fibroadenoma) or have associated invasive carcinoma (AJR Am J Roentgenol 2018;211:462)
      • Only 15 - 52.2% of patients have pure PLCIS alone on final surgical pathology on core biopsy or excision (Ann Surg Oncol 2015;22:4263, Am J Surg Pathol 2019;43:399)
      • In the majority of cases (up to 77%), there is an associated invasive carcinoma, usually consisting of pleomorphic ILC but classic ILC or other histologic types may occur (Am J Surg Pathol 2019;43:399)
      Diagnosis
      • Most patients have imaging abnormalities detected on mammogram or ultrasound, leading to image directed core needle biopsies
      • In a subset of patients, PLCIS may be detected at the time of excisional biopsy or operations for other lesions (Ann Surg Oncol 2018;25:3064)
      Radiology description
      • Most patients present with suspicious mammographic calcifications similar to those associated with DCIS (AJR Am J Roentgenol 2012;199:929, AJR Am J Roentgenol 2001;176:1255)
      • Rarely a patient may have a circumscribed mass identified on a diagnostic mammogram or ultrasound (AJR Am J Roentgenol 2018;211:462)
      • The following imaging findings (from most to least common) were described in one study: fine pleomorphic grouped calcifications on mammograms, mammographic distortion, amorphous calcifications or regional clumped nonmass enhancement at screening MRI (AJR Am J Roentgenol 2018;211:462)
      • Other studies using advanced breast imaging with MRI found nonmass-like enhancement (37.5 %), no MRI abnormality (31.3 %), mass / space occupying lesion (25.0%) or focal < 5 mm area of enhancement (6.3%) (Ann Surg Oncol 2015;22:4263)
      Radiology images

      Images hosted on other servers:

      50 year old woman screening mammogram

      54 year old woman screening mammogram

      Prognostic factors
      • Pure PLCIS is relatively rare and the natural history of PLCIS not associated with invasive carcinoma remains largely unknown
      • PLCIS is considered both a genetically advanced risk factor for and direct precursor to ILC / pleomorphic ILC
      • PLCIS is associated with a high incidence of associated invasive carcinoma either on core needle biopsy or excision
        • Reported numbers vary; one study cohort found 46 of 93 (49%) PLCIS patients had concurrent invasive disease on excision specimen analysis (World J Clin Oncol 2014;5:546)
      • Given the increased genomic instability of PLCIS, the risk of subsequent breast cancer is suspected to be higher than CLCIS but further studies with more patients and longer follow up are necessary
      • PLCIS is more likely to be ER or PR negative and HER2 positive when compared with CLCIS and FLCIS (Breast 2013;22:194)
      • Up to 18% of patients may be triple negative (Mod Pathol 2020;33:1287)
      • PLCIS also frequently has a high Ki67 labeling index and accumulation of p53 protein by immunohistochemistry, both features indicative of aggressive behavior (Breast 2013;22:194)
      • Overall, due to its unfavorable immunoprofile and a higher likelihood of being associated with invasive disease, it is regarded as having a worse prognosis than CLCIS
      Case reports
      • 44 year old woman with PLCIS of the breast composed almost entirely of signet ring cells (Pathol Int 2006;56:683)
      • 52 year old woman with metastatic invasive pleomorphic lobular carcinoma with PLCIS / high grade DCIS-like morphology in affected lymph nodes (Int J Surg Pathol 2020;28:436)
      • 68 year old woman with apocrine differentiation in invasive pleomorphic lobular carcinoma arising in a background of multiple in situ lesions, including apocrine PLCIS, classic LCIS and DCIS (Pathol Oncol Res 2002;8:151)
      • 79 year old woman with apocrine PLCIS associated with microglandular adenosis and metaplastic matrix producing carcinoma (Diagnostics (Basel) 2022;12:1458)
      Treatment
      • There are no well defined guidelines for the treatment of PLCIS
      • It is currently debated whether it is necessary to achieve negative margins and if there is any potential benefit of adjuvant radiation
      • WHO Classification of Tumors Editorial Board recommends excision for PLCIS diagnosed on core needle biopsy (Histopathology 2020;77:181)
      • In a resection specimen, the margin status of CLCIS is not reported but many recommend it should be reported for PLCIS to help inform subsequent management decisions (Arch Pathol Lab Med 2011;135:737)
      • For management guidelines, many organizations suggest considering excision of PLCIS with negative margins (World J Clin Oncol 2014;5:546)
      • Others recommend PLCIS should be treated similarly to DCIS; surgical excision with negative margins with or without other adjuvant treatments (World J Clin Oncol 2014;5:546)
      • As with high grade DCIS, some suggest that a sentinel lymph node biopsy may be considered at the time of PLCIS excision (World J Clin Oncol 2014;5:546)
      • Reported recurrence rates of PLCIS treated with conservative surgery with or without antiestrogen therapy vary widely from 0% to as high as 57% (Histopathology 2020;77:181)
      • It is unclear whether positive margin status affects the likelihood of recurrence
      • One study evaluating only PLCIS at or near an excision margin found an overall recurrence rate of 3.8% regardless of treatment (Arch Pathol Lab Med 2011;135:737)
      • Chemoprevention can be considered in ER positive PLCIS patients
      Gross description
      • Not associated with any specific grossly recognizable abnormalities
      • Cut surface of breast tissue harboring PLCIS may have a faintly granular appearance when viewed with tangential light because affected lobules may be sufficiently enlarged to be visible and microcalcifications are almost always present
      Microscopic (histologic) description
      • Pleomorphic LCIS has architectural features similar to CLCIS, characterized by solid proliferation of dyscohesive neoplastic cells within terminal duct lobular units (TDLUs)
      • Pagetoid spread to adjacent ducts may be seen, similar to CLCIS
      • Differs from classic LCIS with regard to the degree of nuclear atypia or lobular acinar expansion
      • Unlike CLCIS, PLCIS has more nuclear enlargement and variability
      • Lobular acini involved by pleomorphic lobular cells are often substantially expanded but they may be only mildly distended or show no distension
      • Individual cells are medium to large with high N:C ratios, PLCIS nuclei are larger (≥ 4x the size of a lymphocyte), often exhibit moderate to marked nuclear pleomorphism (> 2 - 3x variation in nuclear size) and have distinct small to prominent nucleoli (Mod Pathol 2002;15:1044)
      • Nuclei are often eccentrically placed and binucleated / multinucleated cells are frequently seen (Am J Clin Pathol 2015;144:722)
      • Chromatin is coarse and mitotic figures can be seen
      • Cytoplasm of PLCIS cells is moderate to abundant, dense eosinophilic or granular
      • Intracytoplasmic vacuoles are often present with or without mucin; may be large enough to produce signet ring cell forms
      • Apocrine differentiation is observed in a subset of pleomorphic LCIS, referred to as apocrine PLCIS
        • This subset is characterized by abundant eosinophilic cytoplasm, cytoplasmic granules and prominent nucleoli
      • Central, comedo type necrosis is often present but not required for diagnosis
      • Microcalcifications are a common associated finding
      • PLCIS coexists with classic LCIS in > 40% of cases (Mod Pathol 2002;15:1044)
      • Associated with invasive carcinoma, most commonly pleomorphic ILC but may be of any histologic type, careful assessment of surrounding tissue is essential

      Comparison of cytomorphologic features
      Type of carcinoma Nuclear sizea Nuclear pleomorphism Nucleoli Cytoplasm Dyshesion Central necrosis and calcifications
      PLCIS ≥ 4x Moderate / marked Small (occasionally prominent) Moderate / abundant Yes (often) Yes (common)
      Classic LCIS
      type A
      1.5x Absent Indistinct Scant Absent Absent
      Classic LCIS
      type B
      2x Mild / moderate Indistinct Moderate Yes (common) Absent
      FLCIS 1.5 - 2x Absent to moderate Indistinct Scant to moderate Occasional Yes (common)
      • Adapted and modified from Mod Pathol 2002;15:1044
      • aSize in comparison with a mature lymphocyte
      • Florid LCIS (FLCIS) cells show the cytological features of classic LCIS
      • FLCIS can be composed of type A or type B LCIS cells and is defined based on architectural not cytologic features (Mod Pathol 2002;15:1044)
      • Pleomorphic LCIS and florid LCIS often demonstrate comedonecrosis and calcifications
      Microscopic (histologic) images

      Contributed by Anna Biernacka, M.D., Ph.D.
      PLCIS with central necrosis and calcifications PLCIS with central necrosis and calcifications

      PLCIS with central necrosis and calcifications

      Marked nuclear pleomorphism

      Marked nuclear pleomorphism

      Cellular dyscohesion and pleomorphism

      Cellular dyscohesion and pleomorphism

      Apocrine PLCIS

      Apocrine PLCIS

      Solid proliferation of discohesive cells

      PLICS, solid proliferation of discohesive cells


      Absent E-cadherin

      PLICS, absent E-cadherin

      Attenuated E-cadherin Attenuated E-cadherin

      PLCIS, attenuated E-cadherin

      CLCIS and PLCIS

      CLCIS and PLCIS

      PLCIS with associated invasive carcinoma

      PLCIS with associated invasive carcinoma

      PLCIS within the lobule

      PLCIS within the lobule

      Virtual slides

      Images hosted on other servers:
      PLCIS distending lobules with necrosis and calcifications

      Distending lobules with necrosis and calcifications

      Cytology description
      • Samples are typically highly cellular
      • Cells are dyscohesive, predominately single but may form small, loosely cohesive aggregates
      • Cells have abundant dense cytoplasm and may have signet ring morphology
      • Nuclei are large, pleomorphic and frequently have nucleoli
      • Cells may be bi or multinucleated, lobulated, indented or eccentrically placed
      • Cells can resemble those of DCIS
      • Intracytoplasmic mucin or signet ring cells can be present, which can help to distinguish from DCIS
      • Microcalcifications or necrosis may be seen in the background
      • In ductal lavage (DL), the degree of cytologic atypia is lower than that seen in fine needle aspiration (FNA) (Acta Cytol 2008;52:207)
      • There are no reliable cytologic criteria for distinguishing between PLCIS and ILC
      • Reference: Cancer 2008;114:111
      Cytology images

      Images hosted on other servers:
      Cytoplasm

      Cytoplasm

      Nuclei

      Nuclei

      E-cadherin

      E-cadherin

      Positive stains
      • ER and PR are frequently positive
      • In PLCIS
      • Notably, PLCIS is more likely to be ER negative than CLCIS
      • CLCIS are typically diffusely and strongly positive for ER and PR (> 90% of cases) and very rarely show ERBB2 (HER2) overexpression or gene amplification (0 - 4%) (Histopathology 2020;77:181)
        • In a study comparing PLCIS and classic lobular with pleomorphic features (borderline CLCIS lesions with focal atypia), expression of ER was identified in 81% of PLCIS cases and 100% of CLCIS with pleomorphic features (Breast Cancer Res Treat 2017;165:411)
      • Apocrine PLCIS appears to be a biologically and immunohistochemically distinct subtype

      Comparison of biomarkers among lobular carcinoma in situ (LCIS) subtypes
      ER PR AR AR intensity Mean Ki67 HER2 GCDFP-15 CK5/6
      CLCIS 100% 100% 100% Moderate 4.20% 0% Not done 19%
      All PLCIS 66% 62% 100% Variable 11.5% 13% 74% 23%
      Nonapocrine PLCIS 100% 94% 100% Weak 9.9% 0% 50% 28%
      Apocrine PLCIS 23% 17% 100% Strong 13.9% 31% 100% 17%
      Negative stains
      • Loss of membranous expression of E-cadherin is the defining immunohistochemical feature of lobular differentiation including PLCIS
      • Inactivation of E-cadherin usually leads to loss of p120 and beta catenin expression on cell membranes and aberrant accumulation of p120 in the cytoplasm (Arch Pathol Lab Med 2017;141:1668)
      • Rarely attenuated E-cadherin expression (scattered cells showing dot-like discontinuous and weak membranous staining or patchy cytoplasmic staining) is seen but this does not preclude the diagnosis of PLCIS if typical morphology is present
      Molecular / cytogenetics description
      • The molecular hallmark of LCIS, including PLCIS, is the loss of expression or function of membranous E-cadherin, a transmembrane glycoprotein encoded by the CDH1 gene on the long arm of chromosome 16 (16q)
      • Overall, PLCIS and CLCIS share similar genomic alterations: both groups are characterized by 1q gain (75% in PLCIS versus 69% in CLCIS) and 16q loss (85% versus 76%, for PLCIS and CLCIS, respectively) (Am J Surg Pathol 2009;33:1683)
      • PLCIS is thought to evolve through a molecular genetic pathway similar to that of CLCIS but then undergoes additional genetic events
      • PLCIS exhibits greater genomic instability than CLCIS, with increased copy number changes, gene amplifications and additional mutations
      • Mutations (and amplification) of ERBB2 (HER2) are frequent molecular alterations in PLCIS
        • Amplification of HER2 is found in ~15 - 40% of cases
        • HER2 mutations in ~20% of cases (more frequent in apocrine PLCIS)
        • Mutations and amplifications are mutually exclusive
        • HER2 alterations are more common in ER- lesions
      • At least some PLCIS are nonobligate precursors of pleomorphic ILC, analogous to high grade DCIS and invasive ductal carcinoma
        • These lesions show additional molecular genetic changes, including gains of c-myc (8q24) and HER2 (17q12), gains on 8p, 8q and 13q and losses on 1p, 8p, 12p, 14q, 18q, 19p and 19q (J Pathol 2005;207:1)
      • When separating apocrine and nonapocrine PLCIS, one study found that the overall extent of genomic alterations was similar between CLCIS and nonapocrine PLCIS; additional genomic changes were observed only in the apocrine PLCIS (Am J Surg Pathol 2009;33:1683)
      Videos

      Lobular carcinoma in situ and invasive lobular carcinoma including variants

      Sample pathology report
      • Breast, left, seed localized partial mastectomy:
        • Pleomorphic lobular carcinoma in situ, largest focus measuring 1.5 cm in greatest dimension, surgical margins negative (see comment)
        • Comment: Immunohistochemical stains are performed and show the tumor cells to be negative for E-cadherin and beta catenin consistent with a lobular phenotype supporting the diagnosis above.
      Differential diagnosis
      • Florid LCIS:
        • Has nuclear features of CLCIS and lacks the nuclear pleomorphism of PLCIS
        • FLCIS shows marked distention of ducts and lobules with little residual stroma between the expanded spaces
        • It may or may not have necrosis or calcifications (Am J Surg Pathol 2019;43:399)
      • Classic LCIS with a predominance of type B cells:
        • Type B cells of CLCIS have slightly larger vesicular nuclei with mild variability in size and shape and with small nucleoli when compared to type A cells
        • They lack the marked nuclear pleomorphism of PLCIS
        • The distinction may be difficult in some cases
        • Many authors and the current WHO recommend LCIS lesions that are borderline between CLCIS composed of type B cells and PLCIS should be categorized as CLCIS composed of type B cells (Histopathology 2020;77:181, Ann Diagn Pathol 2020;45:151481)
      • High grade ductal carcinoma in situ:
        • No intracytoplasmic vacuoles, mucin or signet ring cells
        • E-cadherin positive
      • Apocrine intraductal carcinoma:
        • Predilection for lobular involvement
        • E-cadherin positive
      Board review style question #1

      The depicted image is of a breast lesion from a postmenopausal woman. The first image is stained with H&E. The second image is the same area as the first image, stained immunohistochemically with E-cadherin. Other areas (not depicted) had associated microcalcifications. What is the most likely diagnosis?

      1. Apocrine intraductal carcinoma
      2. Classic lobular carcinoma in situ
      3. High grade ductal carcinoma in situ
      4. Invasive ductal carcinoma
      5. Pleomorphic lobular carcinoma in situ
      Board review style answer #1
      E. Pleomorphic lobular carcinoma in situ, PLCIS. Answer B is incorrect because although the lesion has some similar characteristics to classic lobular carcinoma in situ (CLCIS), such as distension of an affected lobule and loss of membranous E-cadherin staining, the cells have enlarged markedly pleomorphic nuclei, which would not be seen in CLCIS. Also, the patient being postmenopausal and the lesion being associated with microcalcifications favors PLCIS rather than CLCIS. Answers A, C and D are incorrect because apocrine intraductal carcinoma, high grade ductal carcinoma in situ and invasive ductal carcinoma would not demonstrate loss of E-cadherin staining. Also, regarding answer D, PLCIS is an in situ lesion and not an invasive carcinoma.

      Comment here

      Reference: LCIS pleomorphic
      Board review style question #2
      Which of the following is true about pleomorphic lobular carcinoma in situ (PLCIS)?

      1. Displays positive beta catenin membranous staining by IHC
      2. Infrequently associated with central, comedo type necrosis
      3. Is a precursor of invasive carcinoma
      4. Most patients present with bloody nipple discharge
      5. The majority of patients are triple negative (estrogen receptor, progesterone receptor and HER2 negative)
      Board review style answer #2
      C. Is a precursor of invasive carcinoma. Although pleomorphic lobular carcinoma in situ (PLCIS) is an in situ lesion, it is considered a precursor of invasive carcinoma. Answer A is incorrect because beta catenin is part of the cadherin - catenin complex and PLCIS would lose membranous staining by immunohistochemistry. Answer B is incorrect because this lesion is frequently associated with central comedonecrosis. Answer D is incorrect because most patients present with a screening detected abnormality on imaging. Answer E is incorrect because ~10% of patients with PLCIS are triple negative.

      Comment here

      Reference: LCIS pleomorphic

      Leiomyoma
      Definition / general
      Essential features
      • Benign smooth muscle tumor
      • Spindle shaped cells with cigar shaped nuclei arranged in whorls and fascicles
      • SMA, desmin, caldesmon positive
      Epidemiology
      Sites
      Pathophysiology
      Etiology
      Clinical features
      Diagnosis
      Radiology description
      Radiology images

      Images hosted on other servers:
      Well defined, circumscribed, hypoechoic mass

      Well defined, circumscribed, hypoechoic mass

      Case reports
      Treatment
      Clinical images

      Images hosted on other servers:
      1 cm nodule (black arrow) on left areola

      1 cm nodule (black arrow) on left areola

      Left nipple (white arrow), nodule (black arrow)

      Left nipple (white arrow), nodule (black arrow)

      Swollen left nipple compared with the right nipple

      Swollen left nipple compared with the right nipple

      Gross description
      Gross images

      Images hosted on other servers:

      Well circumscribed skin covered mass

      Frozen section description
      Frozen section images

      Contributed by Reena Tomar, M.B.B.S., M.D.
      Scrapes

      Scrapes

      Scrapes (toluidine blue)

      Scrapes (toluidine blue)

      Microscopic (histologic) description
      Microscopic (histologic) images

      Contributed by Reena Tomar, M.B.B.S., M.D., Mark R. Wick, M.D. and Julie M. Jorns, M.D.
      Dermal circumscribed leiomyoma

      Dermal circumscribed leiomyoma

      Spindle shaped cells

      Spindle shaped cells

      Fascicles

      Fascicles

      Whorls and fascicles

      Whorls and fascicles

      SMA SMA

      SMA


      Caldesmon

      Caldesmon

      Desmin

      Desmin

      PR

      PR

      S100

      S100

      Fascicles

      Fascicles


      Leiomyoma - breast core biopsy Leiomyoma - breast core biopsy

      Breast core biopsy

      Leiomyoma - breast core biopsy Leiomyoma - breast core biopsy

      Breast core biopsy

      Negative stains
      Sample pathology report
      • Right breast (nipple), punch biopsy:
        • Nipple leiomyoma (see comment)
        • Comment: There is a circumscribed dermal proliferation of monotonous spindle cells. On immunohistochemistry, spindle cells are positive for SMA and desmin and negative for S100 and CK, supporting the diagnosis.
      Differential diagnosis
      Board review style question #1

      A 35 year old woman presents with a freely mobile lump in the nipple areolar area of the left breast. Microscopy shows features in image above with no mitosis or atypia. Which of the following immunohistochemical markers is best to confirm and highlight the diagnosis?

      1. CK72
      2. Ki67
      3. S100
      4. SMA
      Board review style answer #1
      D. SMA highlights smooth muscle origin. Answer A is incorrect because CK72 is an epithelial marker. Answer C is incorrect because S100 is a neural marker. Answer B is incorrect because Ki67 is a proliferation marker that will be low in benign leiomyoma but will not help in diagnosis.

      Comment Here

      Reference: Leiomyoma
      Board review style question #2

      Which of the following is typically true about breast leiomyoma?

      1. Has nipple discharge
      2. Has no hormonal association
      3. Is imaging occult
      4. May be painful
      Board review style answer #2
      D. May be painful. Nipple adenomas may be painful, which may be alleviated by calcium channel or alpha adrenergic blockers that cause smooth muscle relaxation. Answer C is incorrect because breast leiomyoma may be seen as a mass on breast imaging modalities such as mammogram and ultrasound. Answer B is incorrect because breast leiomyomas are associated with hormonal agents. Answer A is incorrect because breast leiomyoma has not been associated with nipple discharge.

      Comment Here

      Reference: Leiomyoma

      Lobular carcinoma in situ (LCIS) classic
      Definition / general
      • Noninvasive, neoplastic proliferation of small, uniform, dyscohesive cells, which originates in the terminal duct lobular unit (TDLU) and fills and distends most of the acini of the involved lobule (Histopathology 2020;77:181)
      Essential features
      • Lobulocentric proliferation of monomorphic cells, which expands lobular units; with or without pagetoid involvement of terminal ducts
      • Hallmark feature is loss of cellular cohesion due to dysfunctional E-cadherin catenin axis
        • Loss of E-cadherin protein is the most common etiology (> 95%) and can be caused by mutations, deletions or methylation
        • Aberrant E-cadherin expression can be observed in some cases
      • Characterized by uniform, loosely cohesive and evenly spaced cells (marbles in a bag); cells are slightly larger than normal, with indistinct cell borders and pale cytoplasm, small uniform nuclei, evenly distributed chromatin and inconspicuous nucleoli
      • Constitutes both a risk factor and nonobligate precursor of invasive breast cancer
      • No longer classified as a tumor in situ (Tis) because it is considered a risk factor, not a malignancy (Amin: AJCC Cancer Staging Manual, 8th Edition, 2016)
      Terminology
      • Lobular neoplasia (LN) of the breast includes atypical lobular neoplasia (ALH) and lobular carcinoma in situ (LCIS)
      • Distinction between ALH and LCIS is the percentage of the TDLU involved (> 50% in LCIS and < 50% in ALH) with acini that are filled and expanded to qualify as LCIS
      • Some authors use LN terminology to denote the histologic overlap and frequent coexistence of ALH and LCIS (Cancer 1978;42:737)
        • This approach is not recommended as it combines into one category lesions, which (although parts of a spectrum) differ in the magnitude of the associated breast cancer risk (Tokai J Exp Clin Med 1989;14:361)
      • Classic LCIS (CLCIS) consists of type A or B epithelial cells, which have different morphologic characteristics
      • Other morphologic subtypes of LCIS that are nonclassical include pleomorphic and florid
      ICD coding
      • ICD-10:
        • D05 - carcinoma in situ of breast
        • D05.0 - lobular carcinoma in situ of breast
        • D05.00 - lobular carcinoma in situ of unspecified breast
        • D05.01 - lobular carcinoma in situ of right breast
        • D05.02 - lobular carcinoma in situ of left breast
      Epidemiology
      Sites
      Pathophysiology
      • E-cadherin gene mutations and loss of the wild type allele by loss of heterozygosity (LOH) is an early genetic event that plays a critical role in pathogenesis (Br J Cancer 1997;76:1131)
      • E-cadherin mediates intracellular adhesion and cell polarity and plays a key role in the maintenance of lobular architecture; it can also inhibit the growth and invasion of breast cancer cells (Breast Cancer Res 2001;3:289)
      • Defective E-cadherin results in loss of cell to cell cohesion, increased cell proliferation and altered organization of the lobules, giving rise to the characteristic appearance of lobular neoplasia
      Etiology
      • Many risk factors for LCIS are similar to those established for DCIS and invasive breast carcinoma
      • Exposure to hormones likely plays a role in etiology; the majority of LCIS shows a strong diffuse expression of ER
      • Increased risk of LCIS is associated with a family history of breast cancer, a previous benign breast biopsy, nulliparity, older age at first full term birth, older age at menopause and high mammographic density (J Natl Cancer Inst 2001;93:1811, Cancer 2015;121:1369)
      • In postmenopausal women, the risk of LCIS increases with the use of unopposed estrogen replacement therapy (Am J Epidemiol 2017;186:1329)
      Diagrams / tables

      Images hosted on other servers:

      Classic LCIS

      LCIS IHC

      Clinical features
      • No specific reproducible clinical or imaging findings
      • Usually identified in biopsy or surgical excision specimen performed for other indications (Am J Pathol 1941;17:491)
      • Rare cases (< 2%) of classic LCIS can be targeted for biopsy due to associated imaging abnormalities; this includes grouped amorphous or granular calcifications on mammography, a shadowing, avascular, irregular, hypoechoic mass on the ultrasound or heterogeneous nonmass-like enhancement with persistent enhancement kinetics on MRI (Breast 2016;27:109, Acad Radiol 2013;20:463)
      • Uncommonly, after biopsy for a palpable mass or nipple discharge, patients may be found to have LCIS (J Clin Oncol 2015;33:3945)
      Diagnosis
      • Diagnosis of classic LCIS is usually an incidental finding in a breast biopsy performed for other indications, including screen detected calcifications or mass producing lesions (Breast Cancer Res 2003;5:258)
      • Columnar cell lesions (frequently associated with LCIS) often have calcifications; these are the targets on mammography leading to the discovery of LCIS (Am J Surg Pathol 2007;31:417)
      • Cases of LCIS or DCIS are distinguished based on histomorphology or by using E-cadherin and p120 catenin immunostains (Int J Clin Exp Pathol 2014;7:2551)
      Radiology description
      • There are no radiologic features specific to classic LCIS
      • In a retrospective radiologic review of patients that were diagnosed pathologically with only classic LCIS, the majority had no abnormalities on mammography; few patients had masses or calcifications (J Clin Ultrasound 2011;39:59)
      • Generally, the sensitivity of ultrasound is low; however, when present, associated sonographic findings include an avascular, irregularly shaped, ill defined, hypoechoic mass with posterior shadowing (Acad Radiol 2013;20:463)
      • When detected via MRI, the most common finding was heterogeneous nonmass-like enhancement with delayed persistent enhancement kinetics (Acad Radiol 2013;20:463)
      • Microcalcifications are not commonly seen in LCIS but can be present (AJR Am J Roentgenol 2001;176:1255)
        • In one study, small focal calcifications were associated with LCIS on biopsy in up to 20% of patients (Cancer 2013;119:1073)
        • Calcifications are more common in lesions that may coexist with LCIS, such as sclerosing adenosis, columnar cell change / hyperplasia, atrophic lobules and ducts and collagenous spherulosis
      Radiology images

      Images hosted on other servers:

      Classic LCIS ultrasound

      Classic LCIS mammogram

      LCIS MRI

      Prognostic factors
      • LCIS is both a risk factor for and a nonobligate precursor of invasive breast carcinoma
      • Compared with the general population, women with LCIS have a 7 - 10 fold increase in breast cancer risk (N Engl J Med 1985;312:146)
      • Absolute risk is ~1 - 2% per year for lifetime risk of 30 - 40%
      • Slightly greater risk to ipsilateral than contralateral breast
      • Time to cancer from LCIS diagnosis ranges 15 - 30 years
      • Subsequent invasive carcinomas are 3x more likely to be lobular (than in the general population) but most commonly are of no special type
      • Clinical and histologic features that may identify patients with LCIS that are likely to develop invasive breast cancer have not been consistently identified
      • Presence of LCIS at (or close to) the margin of resection is not associated with increased local recurrence (Ann Surg Oncol 2008;15:2263)
      • Concurrent lobular neoplasia increases the risk of ipsilateral breast cancer recurrence in patients with ductal carcinoma in situ treated with breast conserving therapy (Cancer 2009;115:1203)
      Case reports
      Treatment
      Gross description
      • Classic LCIS by itself does not cause any grossly visible / palpable alteration in breast tissue
      • May have abnormal findings secondary to the coexisting lesions / proliferative changes (e.g., cysts, firm / hard tissue or nodularity, which are not attributable to LCIS)
      Frozen section description
      • Due to the relatively bland cytological features of classic LCIS frozen section, diagnosis is generally not performed
      Microscopic (histologic) description
      • LCIS involves the terminal duct lobular unit (TDLU), filling and distending acini
      • > 50% of the acini in a TDLU must be filled and expanded to qualify as LCIS; otherwise, called atypical lobular neoplasia
      • Lobular distention is defined as the presence of ≥ 8 cells in the cross sectional diameter of an acinus
        • Involved lobules may be compared with uninvolved lobules to estimate the degree of distension
        • ALH only partially involves the lobule by filling up the acini without significant distention
      • LCIS most often involves lobules but may also grow along the basement membrane of ducts (i.e., pagetoid spread)
        • Pagetoid spread in ducts is the characteristic growth of cells between luminal and myoepithelial layers of a duct without destroying ductal epithelium or filling up ductal lumina
        • Often makes the ducts appear convoluted; this is called a cloverleaf pattern
      • LCIS may secondarily involve (or arise in) sclerosing adenosis, radial scar, fibroadenoma, collagenous spherulosis or papilloma
      • Rosen triad: the presence of tubular carcinoma, columnar cell lesions and LCIS / ALH
        • Initial morphologic observation of lesions that are genetically linked
      • Classic LCIS cells are monomorphic, evenly spaced, loosely cohesive and do not show polarization or gland formation
      • 2 types of cells are described:
        • Type A: nuclei are small to slightly enlarged (1 - 1.5x size of lymphocyte) with uniform round nuclei and inconspicuous nucleoli
        • Type B: nuclei larger (2x size of lymphocyte), more abundant cytoplasm and more prominent nucleoli
        • Type A and B cells can coexist in the same lesion
      • Cytoplasm of LCIS cells is typically pale to lightly eosinophilic with indistinct cell borders
      • In almost all cases of LCIS, at least some cells contain intracytoplasmic vacuoles or lumina, which may contain an eosinophilic globule; this feature is not specific to LCIS
        • Vacuoles may be subtle such that special histochemical stains for mucin are required in order to demonstrate; alternatively, they may be large enough to push the nucleus against the cell membrane and produce signet ring cell forms
      • Outer layer of myoepithelial cells is retained in the acini and ducts involved but it may be attenuated
        • In some cases, scattered myoepithelial cells can be admixed with the neoplastic epithelial cells within the involved spaces
        • Myoepithelial cells show weak positivity for E-cadherin due to the cross reactivity of P-cadherin
      • Cytologic or architectural features of LCIS may deviate from the classical appearances and exhibit distinct cell membranes, clear cell change or apocrine change, or have a myoid appearance with dark, eccentric nuclei and dense eosinophilic cytoplasm similar to rhabdomyoblasts
      • Classic LCIS does not show significant nuclear pleomorphism or mitosis
      • Classic LCIS may rarely display single cell apoptosis or minute foci of necrosis but typically does not show comedo type necrosis
      • Rarely, comedo necrosis can be present in classic LCIS that has typical cytological and architectural features; this (by itself) should not be diagnosed as pleomorphic LCIS (Am J Surg Pathol 2006;30:1445)
      • Pleomorphic LCIS is defined as having large cells with enlarged pleomorphic nuclei; additionally, central necrosis and apocrine features can be present
      • Florid LCIS, with or without central necrosis, refers to lesions with cytologic features of classic LCIS but with marked duct distension in a solid architectural pattern
      • LCIS lesions, which are borderline between classic LCIS composed of type B cells and pleomorphic LCIS, should be categorized as classic LCIS composed of type B cells
      • Lesions with cytological features of classic LCIS, which show distension of the acini borderline between classic LCIS and florid LCIS, should be designated as classic LCIS, even when LCIS is extensive
      Microscopic (histologic) images

      Contributed by Anna Biernacka, M.D., Ph.D.

      Classic LCIS involving terminal duct lobular units

      Classic LCIS, terminal duct lobular unit expansion

      Classic LCIS, cell morphology

      Lobular distension

      Pagetoid spread, cloverleaf pattern

      Pagetoid spread in ADH


      Preserved lobular architecture

      Type A cells

      Type B cells

      Type A and B cells

      LCIS, mixed cell population

      Signet ring cells, breast type


      Signet ring cells, gastric type

      Histiocytoid appearance

      Clear cell change

      LCIS involving an intraductal papilloma

      LCIS in intraductal papilloma

      Rosen triad


      LCIS in collagenous spherulosis

      LCIS in sclerosing adenosis

      LCIS, sclerosing adenosis, calcifications

      LCIS, E-cadherin

      LCIS and ILC,
      E-cadherin

      LCIS and ILC, p63


      LCIS, ER

      LCIS and DCIS, low grade

      LCIS versus DCIS, nucleoli

      Cytology description
      • Fine needle aspiration (FNA) of classic LCIS shows loosely cohesive groups of small, uniform and cytologically bland neoplastic cells (Am J Clin Pathol 1989;92:22, Diagn Cytopathol 1993;9:713, Diagn Cytopathol 2002;27:22)
      • Nuclei are mildly enlarged and eccentrically placed and may show small but prominent nucleoli (Diagn Cytopathol 2005;32:276)
      • Cytoplasm may have occasional intracytoplasmic lumina
      • Nonatypical spindled myoepithelial cells may be present in association with classic LCIS cells
      • Classic LCIS may be suspected in an FNA sample if signet ring cells are identified and associated with detached fragments of the lobular epithelium; however, definitive diagnosis in these sparely cellular samples is rarely possible
      • There are no reliable cytological criteria for distinguishing between classic LCIS and invasive lobular carcinoma (ILC) (Diagn Cytopathol 1993;9:713, Diagn Cytopathol 2002;27:22)
        • FNA samples of classic LCIS are usually cellular, whereas samples of ILC are often scant due to prominent desmoplasia that surrounds the infiltrative tumor cells (Diagn Cytopathol 2002;27:22)
        • ILC samples generally have a higher proportion of noncohesive cells with more nuclear atypia and pleomorphism (Am J Clin Pathol 1989;92:22)
      • > 50% of aspirates from classic LCIS are determined to be benign or nondiagnostic (Diagn Cytopathol 2002;27:22)
      Cytology images

      Images hosted on other servers:

      Cytology classic LCIS

      Positive stains
      Negative stains
      Molecular / cytogenetics description
      • Molecular data has shown that LCIS is a clonal neoplastic proliferation and an early lesion in the low grade neoplasia pathway
      • Studies comparing LCIS and synchronous ILC highlight the shared copy number aberrations and somatic genetic mutations, indicating that LCIS is a nonobligate precursor to ILC (Genes Chromosomes Cancer 2010;49:463, Cancer 2004;100:2562, J Pathol 2005;207:1)
        • LCIS associated with ILC has more genetic alterations than pure LCIS
      • Most frequent copy number changes include loss of 16q and gain of 1q (Breast Cancer Res Treat 2005;90:249, Hum Pathol 2001;32:292, Cancer Res 1998;58:4721, Mol Pathol 2000;53:118)
        • Loss of 16q and gain of 1q are also seen in other ER positive proliferations, including columnar cell lesions and well differentiated invasive ductal carcinoma, including tubular carcinoma and low grade DCIS
        • This has led to an understanding that LCIS belongs to a group of lesions in a low grade, ER positive pathway of breast carcinogenesis (Histopathology 2011;59:549, Arch Pathol Lab Med 2017;141:1668)
        • Although the target gene of 16q deletion in DCIS and IDC remains to be identified in lobular lesions, it involves the CDH1 tumor suppressor gene encoding for cell adhesion molecule E-cadherin (mapping at 16q22.1)
      • Loss of 16q is often accompanied by CDH1 inactivating mutations, CDH1 homozygous deletions or CDH1 gene promoter methylation, which results in biallelic silencing of the gene
      • Different combinations of alterations can result in different patterns of immunoreactivity
        • Completely absent, weak and beaded, or aberrantly present in the cytoplasm in a dot-like pattern
        • Rare cases (< 10%) may show membrane staining even though the protein is nonfunctional
      • Other tumor suppressor genes located on chromosome 16 have been shown to be lost in LCIS:
        • CCCTC binding factor (CTCF) gene, a transcriptional regulator of several genes linked to tumorigenesis
        • Dipeptidase 1 (DPEP1) gene, which is involved in the metabolism of important glutathione and may have a role in the degradation of the surrounding extracellular matrix
      • Somatic genetic mutations: CDH1 is the most frequently mutated gene in LCIS (in as many as 81% of cases), followed by PIK3CA (41%) and CBFB (12%); p53 and PTEN mutations are remarkably rare (Clin Cancer Res 2019;25:674, Mol Oncol 2016;10:360, Breast Cancer Res 2017;19:7)
      • Germline mutations in CDH1, which are highly penetrant, are associated with an increased risk of hereditary diffuse gastric cancer and an increased risk of lobular carcinoma, including bilateral LCIS, with or without ILC (J Med Genet 2018;55:431, Br J Cancer 2014;110:1053)
      • Variants of LCIS: pleomorphic and florid variants exhibit greater genomic instability than classic LCIS, with increased copy number alterations and gene amplification (Am J Surg Pathol 2009;33:1683)
      Videos

      Histopathology breast: lobular carcinoma in situ

      Lobular carcinoma in situ and invasive lobular carcinoma

      Sample pathology report
      • Right breast, core needle biopsy:
        • Lobular carcinoma in situ, classic type

      • No longer classified as Tis because it is considered a risk factor, not a malignancy (Amin: AJCC Cancer Staging Manual, 8th Edition, 2016)
      • Reporting on size or margin status is not necessary
      Differential diagnosis
      • Atypical lobular hyperplasia:
        • Identical to classic LCIS morphologically but defined by limited extent
        • < 50% of acini in a lobule are involved or all the acini are involved but are not distended
      • Artifactual dyshesion:
        • Poor tissue fixation may result in artifactual dyshesion of the epithelial cells lining normal ductal lobular structures or comprising benign lesions
        • Appearance can mimic LCIS; however, diagnostic cytologic features of LCIS are not present
      • Intraepithelial histiocytes:
        • May be mistaken for pagetoid, classic LCIS cells
        • Small nuclei, abundant foamy to granular cytoplasm and the lack of intracytoplasmic vacuoles distinguish these cells from classic LCIS
        • IHC specific for histiocytes, such as CD68, can be used in these cases
      • Myoepithelial hyperplasia:
        • Myoepithelial cells surrounding normal lobular acini or ducts may assume an epithelioid appearance, with abundant pale to clear cytoplasm and prominent round nuclei
        • Myoepithelial cell markers (calponin, smooth muscle myosin heavy chain and p63) are positive
      • Clear cell change:
      • Pseudolactational hyperplasia:
        • Vacuolated cytoplasm, apical apocrine tufts, hyperchromatic or atypical nuclei, psammoma-like calcifications, usually in postmenopausal women
      • Invasive carcinoma:
        • LCIS involving a pre-existing sclerosing lesion (e.g., sclerosing adenosis, radial scar or complex sclerosing lesion) may produce a pattern mimicking stromal invasion
        • Solid or alveolar patterns of invasive lobular carcinoma (ILC) can be difficult to distinguish from LCIS
        • Myoepithelial cell markers are retained in LCIS and absent in invasive carcinoma
      • Pleomorphic LCIS:
        • May be difficult to distinguish from LCIS composed of type B cells
        • Nuclei of type B LCIS are larger and more variable than in type A LCIS
        • Still, the nuclear size and shape variability is relatively subtle compared with pleomorphic LCIS
        • In cases of uncertainty, the lesion should be categorized as classic LCIS with type B cells rather than as pleomorphic LCIS
      • Ductal carcinoma in situ:
        • Solid low grade DCIS:
          • Cohesive cells, microacini with cells polarized around microlumen, no intracytoplasmic vacuoles, no pagetoid ductal involvement
          • E-cadherin positive
        • Cancerization of lobules by DCIS:
          • Cellular cohesion, moderate to high grade cytology, necrosis or calcifications
          • E-cadherin positive
        • Cribriform DCIS and collagenous spherulosis involved by LCIS:
          • Lumens of DCIS are formed by polarized epithelial cells and are empty or contain necrosis or calcifications
          • Myoepithelial cells in DCIS surround the periphery of the involved lobule
          • Lumens in collagenous spherulosis contain eosinophilic basement membrane material or myxoid material and are lined by myoepithelial cells
        • Pagetoid LCIS versus pagetoid DCIS:
          • Pagetoid ductal involvement is a common feature of LCIS, less commonly seen in DCIS
          • Cellular dyshesion and the presence of intracytoplasmic vacuoles favor LCIS, cellular cohesion and moderate to high grade nuclear atypia favor DCIS
        • LCIS with comedo necrosis versus DCIS with comedo necrosis:
          • Some examples of LCIS composed of classical type A or type B cells may show comedo necrosis in the involved spaces and may simulate DCIS
          • Recognition of characteristic cytologic features of LCIS (particularly dyshesion and intracytoplasmic vacuoles) and, in problematic cases, an immunostain for E-cadherin leads to the correct diagnosis
        • In situ carcinoma with mixed ductal and lobular phenotype:
          • Mixed architectural and cytological features; heterogeneous expression of E-cadherin
      Board review style question #1
      What is the Rosen triad?

      1. DCIS, LCIS, tubular carcinoma
      2. LCIS, flat epithelial atypia, columnar cell hyperplasia
      3. Tubular carcinoma, DCIS, apocrine metaplasia
      4. Tubular carcinoma, LCIS, columnar cell hyperplasia
      5. Tubular carcinoma, LCIS, flat epithelial atypia
      Board review style answer #1
      D. Tubular carcinoma, LCIS, columnar cell hyperplasia

      Comment here

      Reference: Lobular carcinoma in situ (LCIS) classic
      Board review style question #2
      Which of the following is true of classical lobular carcinoma in situ (classic LCIS)?

      1. Classic LCIS belongs to group lesions in a low grade, ER positive pathway of breast carcinogenesis
      2. Classic LCIS increases the patients' risk only for subsequent unilateral breast cancer
      3. Classic LCIS is staged as Tis using the AJCC 8th edition cancer staging
      4. If present at a surgical margin, it should be noted in the final pathology report
      5. Loss of nuclear expression of E-cadherin is the defining immunohistochemical feature of classic LCIS
      Board review style answer #2
      A. Classic LCIS belongs to group lesions in a low grade, ER positive pathway of breast carcinogenesis

      The presence of classic LCIS at or close to the margin of resection is not associated with increased local recurrence and does not need to be reported. Classic LCIS increases the patients' risk for breast cancer in either breast. E-cadherin typically displays a membranous staining pattern. Classic LCIS is no longer classified as Tis because it is considered a risk factor, not a malignancy.

      Comment here

      Reference: Lobular carcinoma in situ (LCIS) classic
      Board review style question #3

      Which of the following morphologic features would be most consistent with the entity depicted above?

      1. Comedo type necrosis
      2. Intracytoplasmic lumens / vacuoles
      3. Intranuclear inclusions
      4. Microcalcifications
      5. Single prominent nucleoli
      Board review style answer #3
      B. Intracytoplasmic lumens / vacuoles

      Intracytoplasmic lumens, vacuoles and even signet ring morphology are associated with the diagnosis of classic LCIS. Prominent nucleoli, microcalcifications and comedo type necrosis are morphologic features more commonly seen in DCIS or the pleomorphic variant of LCIS. Although type B LCIS cells may have nucleoli, they are typically multiple and scattered, with the lesional cells having uniform and smooth nuclear contours. Intranuclear inclusions are typically secondary to viral cytopathic effect and are not associated with breast lobular neoplasia.

      Comment here

      Reference: Lobular carcinoma in situ (LCIS) classic

      Low grade adenosquamous
      Definition / general
      • Rare variant of metaplastic carcinoma comprised of an intimately admixed combination of bland, well developed glands and solid squamous cell nests within a fibrotic or cellular spindle stroma
      • First reported in 1917 (Br J Surg 1917;5:417)
      • Formally recognized in 1987 (Am J Surg Pathol 1987;11:351)
      Essential features
      • Rare variant of metaplastic carcinoma with low grade cytomorphology
      • Largely clinically indolent but may be locally aggressive
      • Diagnosis is based primarily on morphologic features, which can be challenging in small samples
      • Immunostaining patterns can be variable and unreliable
      • May arise in association with other breast lesions
      Terminology
      • Syringomatous squamous tumor or infiltrative syringomatous adenoma
      ICD coding
      • ICD-10: C50.919 - malignant neoplasm of unspecified site of unspecified female breast
      Epidemiology
      • 0.2 to 1%
      • Median age: fifth decade; range: 20 - 85 years
      • Females; only one case report in male
      Sites
      • Breast parenchyma
      Pathophysiology
      • Unknown at this time
      • Association with other proliferative and sclerosing breast lesions has led some to suggest they represent precursors (Pathology 2014;46:402)
      Etiology
      • Multifactorial
      Clinical features
      Diagnosis
      • Most frequently diagnosed via surgical resection
      • Difficult to diagnose by fine needle aspiration, core biopsy or frozen section (Histopathology 2006;49:603)
      Radiology description
      Radiology images

      Images hosted on other servers:

      Mammogram: asymmetry

      Mammogram: calcifications

      Mammogram: mass

      Ultrasound: mass

      Prognostic factors
      Case reports
      Treatment
      • Surgical excision with negative margins (Am J Surg Pathol 1987;11:351)
      • Radiotherapy is standard for patients with breast conserving surgery
      • Role of chemotherapy is indeterminate
      Gross description
      Gross images

      Contributed by Paula S. Ginter, M.D.
      Excisional biopsy of mass

      Excisional biopsy of mass

      Microscopic (histologic) description
      Microscopic (histologic) images

      Contributed by Paula S. Ginter, M.D.
      Glands and squamous nests Glands and squamous nests Glands and squamous nests

      Glands and squamous nests

      Bland cytology Bland cytology

      Bland cytology


      Spindle stroma

      Spindle stroma

      Lymphoid aggregates

      Lymphoid aggregates

      Luminal keratin debris

      Luminal keratin debris

      Associated papilloma

      Associated papilloma

      Myosin: variable myoepithelial staining

      Myosin: variable myoepithelial staining


      p63: variable myoepithelial staining

      p63: variable myoepithelial staining

      CK7: core staining

      CK7: core staining

      ER negative

      ER negative

      Virtual slides

      Images hosted on other servers:
      Low grade adenosquamous carcinoma and DCIS

      Low grade adenosquamous carcinoma and DCIS

      p63: variable myoepithelial and luminal staining

      p63: variable myoepithelial and luminal staining

      ER negative (< 1%)

      ER negative
      (< 1%)

      Cytology description
      • Variable cellular yield; not reliable as a surrogate for considering malignancy
      • Irregularly clustered cells, angulated sheets and tubular structures with monomorphic cytology (Acta Cytol 2014;58:427, Diagn Cytopathol 2012;40:713, Diagn Cytopathol 1999;20:13)
      • Whorl-like arrangements of epithelial cells may hint of squamous differentiation
      • Individually dispersed, atypical cells may be seen
      • Background may show mildly atypical naked nuclei and spindle cells (representing the stromal component)
      • Combination of epithelial and spindle elements is distinctive and should raise the possibility of low grade adenosquamous carcinoma
      • Accurate cytological identification is challenging
      Positive stains
      Negative stains
      Electron microscopy description
      Molecular / cytogenetics description
      Sample pathology report
      • Left breast, 3 o'clock, lumpectomy:
        • Low grade adenosquamous carcinoma (see comment and synoptic report)
        • Ductal carcinoma in situ
        • Comment: Low grade adenosquamous carcinoma is grade 1 (2, 2, 1), measures 1.7 cm in greatest dimension, is associated with a sclerotic papilloma, is present less than 1 mm from the inked surgical margin and is ER negative (< 1%), PR negative (< 1%) and HER2 negative (0). No lymphovascular invasion identified. Ductal carcinoma in situ is solid type with intermediate nuclear grade and is present as a single 0.2 cm focus associated with the low grade adenosquamous carcinoma. Ductal carcinoma in situ is present > 2 mm from the inked surgical margin.
      Differential diagnosis
      • Tubular carcinoma:
        • Strong and diffuse ER positivity
        • Devoid of staining for myoepithelial markers (i.e. SMMHC, calponin, CD10 and SMA)
        • Lacks squamous differentiation
      • Radial scar with squamous metaplasia:
        • Heterogeneous, positive staining for ER
      • Syringomatous adenoma:
        • Morphologically similar to low grade adenosquamous carcinoma
        • Composed of tubules and solid nests with comma-like or tail-like extensions and squamous cysts embedded in desmoplastic stroma
        • More superficial, involving the nipple areolar complex
        • Lacks lymphoid aggregates
        • Multinucleated giant cell reactions to ruptured cysts may be seen
        • Some authors suggest these are identical entities (Histopathology 2014;65:9)
      • High grade adenosquamous carcinoma:
        • Shows both squamous and glandular components with moderate to high cellularity and nuclear atypia
        • Mitotic activity is moderate to high
        • Areas of necrosis may be present
      Board review style question #1


      Which of the following is true about low grade adenosquamous carcinoma of the breast?

      1. Associated with a poor prognosis
      2. Commonly estrogen receptor (ER) positive
      3. Consistent loss of myoepithelial cells by immunohistochemistry
      4. May be associated with other breast lesions
      Board review style answer #1
      D. May be associated with other breast lesions. Multiple reports have demonstrated an association of low grade adenosquamous carcinoma with other breast entities (i.e. papilloma, adenomyoepithelioma, radial scar / radial sclerosing lesions and fibroepithelial tumors). Low grade adenosquamous carcinoma is frequently ER negative. Low grade adenosquamous carcinoma demonstrates variable degrees of circumferential myoepithelial cell markers and immunohistochemistry is consistently inconsistent in this entity, limiting use. Low grade adenosquamous carcinoma is associated with a good prognosis.

      Comment Here

      Reference: Low grade adenosquamous

      Lymphocytic / diabetic mastitis
      Definition / general
      • Histologic triad of keloidal type fibrosis, lymphocytic inflammation and epithelioid myofibroblasts
      • Associated with type I (insulin dependent) diabetes or other autoimmune diseases
      Essential features
      • Young to middle aged women presenting with breast mass(es)
      • Histologic triad of keloidal type fibrosis, lymphocytic infiltrates and epithelioid stromal myofibroblasts
      • Associated with type I (insulin dependent) diabetes or other autoimmune diseases
      Terminology
      • Lymphocytic mastopathy / diabetic mastopathy
      • Sclerosing lymphocytic mastitis / lobulitis
      • Diabetic fibrous breast disease
      ICD coding
      • ICD-10: N61 - inflammatory disorders of breast
      • ICD-11: GB21 - inflammatory disorders of breast
      Epidemiology
      Sites
      • Breast
      Pathophysiology
      • Unknown
      • Theories include:
        • Hyperglycemia leads to stromal matrix expansion and accumulation of glycosylation end products and a B cell inflammatory response (Hum Pathol 1992;23:780)
        • Immunologic response to exogenous insulin (Hum Pathol 1994;25:819)
      Etiology
      • Unknown
      Clinical features
      Diagnosis
      • Needle core biopsy or surgical excision with histologic examination
      Laboratory
      • Correlate with serologic markers for diabetes and other autoimmune diseases
      Radiology description
      Radiology images

      Contributed by Kristen E. Muller, D.O.
      Acoustic shadowing

      Acoustic shadowing

      Prognostic factors
      Case reports
      Treatment
      Gross description
      • Firm, ill defined, homogenous, tan-white or white-gray cut surface
      • Size variable, up to 6.0 cm
      Gross images

      Images hosted on other servers:
      Dense white fibrous stroma

      Dense white fibrous stroma

      Firm white cut surface

      Firm white cut surface

      Microscopic (histologic) description
      • Triad of dense, keloidal type fibrosis, periductal, perilobular and perivascular lymphocytic infiltrates and epithelioid stromal myofibroblasts
      • Epithelioid characteristics of myofibroblasts include enlarged cells with round to polygonal nuclei, prominent nucleoli and abundant cytoplasm
      • Epithelioid features of myofibroblasts may be subtle or absent
      • Stromal mitoses absent
      • Mature lymphocytic infiltrates are nonclonal, predominantly B lymphocytes and lack germinal centers (Mod Pathol 2003;16:223)
      Microscopic (histologic) images

      Contributed by Kristen E. Muller, D.O.
      Dense fibrous stroma

      Dense fibrous stroma

      Perivascular lymphocytic inflammation Perivascular lymphocytic inflammation

      Perivascular lymphocytic inflammation

      Enlarged myofibroblasts

      Enlarged myofibroblasts

      Fibrous stroma, periductal and perivascular inflammation

      Fibrous stroma, periductal and perivascular inflammation

      Periductal lymphocytes

      Periductal lymphocytes


      Epithelioid myofibroblasts

      Epithelioid myofibroblasts

      CD20

      CD20

      CD3

      CD3

      Virtual slides

      Images hosted on other servers:
      Lymphocytic mastopathy

      Lymphocytic mastopathy

      Cytology description
      Positive stains
      Negative stains
      Molecular / cytogenetics description
      Sample pathology report
      • Left breast, needle core biopsy:
        • Breast tissue with dense stromal keloidal type fibrosis, epithelioid myofibroblasts and perilobular lymphocytic inflammation (see comment)
        • Comment: The histologic findings are compatible with lymphocytic / diabetic mastopathy in the appropriate clinical context.
      Differential diagnosis
      Board review style question #1

      For the breast stroma shown, which of the following is true?

      1. Lesion is associated with increased risk of developing lymphoma
      2. Lymphocytes are predominantly T cells and are clonal
      3. Patient may have insulin dependent diabetes or an autoimmune disorder
      4. Spindle cells in the stroma show beta catenin nuclear expression
      Board review style answer #1
      C. Patient may have insulin dependent diabetes or an autoimmune disorder

      Comment Here

      Reference: Lymphocytic / diabetic mastitis
      Board review style question #2
      Which of the following is true regarding diabetic / lymphocytic mastopathy?

      1. All patients have a history of diabetes
      2. Characteristic histologic triad includes dense keloid-like fibrosis, epithelioid myofibroblasts and periductal, perilobular and perivascular lymphocytic inflammation
      3. If surgically excised, these lesions never recur
      4. Typical mammographic finding is an infiltrative mass lesion
      Board review style answer #2
      B. Characteristic histologic triad includes dense keloid-like fibrosis, epithelioid myofibroblasts and periductal, perilobular and perivascular lymphocytic inflammation

      Comment Here

      Reference: Lymphocytic / diabetic mastitis

      Lymphoma
      Definition / general
      • Lymphomas of the breast may be primary or secondary
      • Primary breast lymphoma is generally defined as lymphoma limited to the breast, with or without axillary lymph node involvement, in a patient without evidence of distant disease at presentation and without prior history of lymphoma (Histopathology 1985;9:297, Cancer 1972;29:1705)
      • Some accept lymphoma as primary if the initial presentation and the dominant mass or symptoms occur in the breast, even if other involved sites are identified (Am J Surg Pathol 2008;32:1299, Ann Surg 1987;205:144, Cancer 1990;66:2602)
      • Most common types include diffuse large B cell lymphoma, extranodal marginal B cell lymphoma arising from mucosal associated lymphoid tissue (MALT lymphoma), follicular lymphoma and Burkitt lymphoma
      • Implant associated anaplastic large cell lymphoma is discussed in a separate section
      Essential features
      • Lymphoma seen in close proximity to breast parenchyma, not limited to lymph nodes
      • Primary breast lymphoma is uncommon and must be distinguished from secondary breast involvement from another primary site in the setting of widespread disease / relapse (more common)
      • Must distinguish from mammary carcinoma, as therapeutic / surgical management is different; use of immunohistochemical panel (cytokeratins and lymphoid lineage markers) will confirm the diagnosis
      Terminology
      • Diffuse large B cell lymphoma (DLBCL) / DLBCL not otherwise specified (NOS)
      • Extranodal marginal B cell lymphoma arising from mucosal associated lymphoid tissue (MALT lymphoma) / marginal zone lymphoma MALT type / MALT lymphoma
      • Follicular lymphoma / follicular center cell lymphoma
      • Burkitt lymphoma
      ICD coding
      • ICD-O:
        • 9680 / 3 - diffuse large B cell lymphoma (DLBCL)
        • 9699 / 3 - extranodal marginal B cell lymphoma arising from mucosal associated lymphoid tissue (MALT lymphoma)
        • 9690 / 3 - follicular lymphoma
        • 9687 / 3 - Burkitt lymphoma
      • ICD-10:
        • C83.3 - diffuse large B cell lymphoma
        • C82 - follicular lymphoma
        • C88.4 - MALT lymphoma
        • C83.7 - Burkitt lymphoma
      Epidemiology
      Etiology
      • Diffuse large B cell lymphoma (DLBCL):
        • Activated B cell type (IRF4+, FLIP+, BCL2+), germinal center type (BCL6+, CD10+, CD38+) or neither
        • Commonly arises de novo or can occur through progression / transformation of a less aggressive non-Hodgkin lymphoma
      • Extranodal marginal B cell lymphoma arising from mucosal associated lymphoid tissue (MALT lymphoma):
        • Less association with underlying autoimmune disease compared to MALT lymphomas of other sites
      • Follicular lymphoma:
        • Originates from germinal / follicular center B cells
      • Burkitt lymphoma:
        • 3 variants: endemic (> 95% Epstein-Barr virus [EBV]+), immunodeficiency associated (often HIV+) and sporadic (less frequently EBV+)
        • c-Myc translocation
      Clinical features
      • Middle age to elderly, median in sixth to seventh decade (Am J Surg Pathol 2008;32:1299, Leuk Lymphoma 2005;46:1321)
      • Reports of younger patients in pregnant or lactating women (Burkitt lymphoma)
      • Most present with painless palpable mass, with or without ipsilateral lymphadenopathy (11 - 50%) (Am J Surg Pathol 1993;17:574)
      • Bilateral in 10%
      • Treatment and prognosis are related to specific type of lymphoma:
        • Diffuse large B cell lymphoma (DLBCL):
          • Most common primary breast lymphoma (50 - 65%) (Cancer 2007;110:25, Am J Hematol 2009;84:133)
          • Wide age range, median in sixth decade
          • Median size 4 - 5 cm, may present as rapidly enlarging mass, few present with diffuse breast enlargement
          • Aggressive clinical behavior, most common sites of relapse include ipsilateral or contralateral breast and central nervous system
          • Must be distinguished from poorly differentiated carcinomas
        • Extranodal marginal B cell lymphoma arising from mucosal associated lymphoid tissue (MALT lymphoma):
          • Sixth to seventh decade
          • Typically unilateral
          • Indolent, > 90% 5 year survival (Ann Oncol 2009;20:1993)
          • Large cell transformation reported
        • Follicular lymphoma:
          • Middle aged and older women
          • Usually unilateral
          • Similar prognosis compared to nodal follicular lymphoma, some have reported worse prognosis (Ann Oncol 2009;20:1993)
        • Burkitt lymphoma:
          • Uncommon, younger to middle aged females, some pregnant or postpartum at presentation
          • Endemic variant is the most common presentation in the breast
          • May present with massive bilateral breast enlargement
          • Aggressive B cell lymphoma has more favorable prognosis with aggressive therapy
      Radiology description
      • On mammography, primary breast lymphomas have been reported to present as unilateral diffuse involvement (25%), bilateral diffuse involvement (8%), solitary masses (58%) or as multiple masses (8%) (Breast J 2002;8:294)
      • Masses may have either irregular, partially defined or well defined borders (Clin Imaging 2007;31:234)
      • Useful MRI features may include strong enhancement with penetrating vessels on early images of dynamic MR imaging, strong high signal intensity on diffusion weighted imaging, a cerebroid appearance and septal enhancement on delayed contrast enhanced images (Jpn J Radiol 2013;31:668)
      • Often mimic primary breast cancer
      Radiology images

      Contributed by Mark R. Wick, M.D. and Julia Braza, M.D.

      Mammogram, large cell lymphoma

      Mammogram, lymphoblastic lymphoma

      Mammogram, CLL / SLL



      Images hosted on other servers:

      Mammogram and ultrasound, DLBCL, 47 yo

      Mammogram and ultrasound, DLBCL, 56 yo

      T1w MRI, DLBCL, fat suppressed

      MRI, MALT lymphoma, 56 yo

      Prognostic factors
      • Prognosis is dependent on stage and type of lymphoma
      Case reports
      Treatment
      • Depends on diagnosis; is the same as lymphoma at any other site
      Clinical images

      Contributed by Mark R. Wick, M.D.

      Large cell lymphoma



      Images hosted on other servers:

      Burkitt lymphoma

      DLBCL

      Large cell lymphoma

      Gross description
      • Grossly circumscribed mass, tan-white, fleshy cut surface
      • Focal hemorrhage or necrosis may be present
      Gross images

      Contributed by Mark R. Wick, M.D. and AFIP images

      Large cell lymphoma

      Follicular lymphoma



      Images hosted on other servers:

      DLBCL

      Microscopic (histologic) description
      • Despite gross circumscription, microscopically these are diffusely infiltrative with invasion into surrounding breast parenchyma
      • Infiltrates around and occasionally obliterates mammary gland structures
      • Diffuse large B cell lymphoma (DLBCL):
        • Diffuse infiltration by large lymphoid cells, centroblastic > immunoblastic, activated B cell type > germinal center type
      • Extranodal marginal B cell lymphoma arising from mucosal associated lymphoid tissue (MALT lymphoma):
        • Median size 3 cm
        • Vaguely nodular growth pattern, neoplastic cells occupy the marginal zone surrounding the B cell follicle mantle zone but can have diffuse growth pattern if follicles infiltrated / replaced
        • Marginal zone B cells, variable plasma cells, reactive follicles (±), lymphoepithelial lesions not conspicuous
        • Monocytoid-like cells: medium sized lymphocytes with slightly irregular nuclei, dispersed chromatin, inconspicuous nucleoli, variable amounts of pale cytoplasm
        • Variable scattered large transformed immunoblasts can be present
        • Reactive follicles with infiltration by neoplastic marginal zones in some cases
        • Plasmacytic differentiation in up to 75% (Arch Pathol Lab Med 2007;131:1673)
        • Low mitotic rate
        • Lymphoepithelial lesions may be seen if glands / ducts present but are usually not prominent, less conspicuous than MALT lymphoma in other sites
        • Transformation into large B cell lymphoma described (Arch Pathol Lab Med 2007;131:1673)
      • Follicular lymphoma:
        • Median size 2 - 3 cm
        • Morphology similar to nodal follicular lymphomas; follicular and diffuse architectural patterns, grade 1 or 2 > grade 3
        • Monotonous population of small cleaved cells (centrocytes) and variable numbers of centroblasts (large noncleaved cells)
        • Associated sclerosis can cause single file infiltration pattern
      • Burkitt lymphoma:
        • Sheets of uniform medium sized lymphocytes with round nuclei, multiple basophilic nucleoli, course chromatin, scant to moderate basophilic cytoplasm, minimal intervening stroma
        • High mitotic rate
        • Tingible body macrophages containing apoptotic debris produce a "starry sky" appearance
      Microscopic (histologic) images

      Contributed by Gary Tozbikian, M.D. and Julia Braza, M.D.

      Marginal zone lymphoma

      Invasion of adipose

      Follicular colonization

      Monocytoid-like B cells

      DLBCL, activated B cell type

      DLBCL, germinal center type


      Sheet-like growth

      Lymphoid cells surround glands

      Marked atypia and frequent mitoses

      Large lymphoid cells


      CLL / SLL

      CLL / SLL, CD5

      CLL / SLL, CD20



      Images hosted on other servers:

      Follicular lymphoma

      Follicular lymphoma, BCL2

      Mantle cell lymphoma

      Cytology description
      • Primary cytomorphologic features include the presence of dissociated monomorphic cells with high nuclear cytoplasmic ratio, finely granular lymphoid chromatin and the presence of lymphoglandular bodies in the background of the cytologic smear (Diagn Cytopathol 2013;41:53)
      • Differential diagnosis includes lobular carcinoma, small cell carcinoma, lesions with prominent lymphoid population including medullary carcinoma, chronic mastitis and intramammary lymph node (Diagn Cytopathol 2016;44:235)
        • Lobular carcinoma: intracytoplasmic lumina, signet ring cells, intracytoplasmic mucin secretions
        • Small cell carcinoma: cell clustering, cell molding, lacks lymphoglandular bodies, positive keratin and neuroendocrine immunohistochemistry staining
        • High grade carcinomas with medullary features: admixture of pleomorphic epithelial cells as well as reactive lymphoid and plasma cells
        • Intramammary lymph nodes and mastitis can show reactive lymphoid infiltrate and pose a diagnostic problem (Diagn Cytopathol 2013;41:53)
      • Final categorization generally requires flow cytometric immunophenotyping / immunohistochemistry on cell blocks to allow morphologic and immunohistochemical evaluation (Diagn Cytopathol 2016;44:235)
      Cytology images

      Contributed by Mark R. Wick, M.D.

      Large cell lymphoma

      Positive stains
      Negative stains
      Molecular / cytogenetics description
      • Diffuse large B cell lymphoma (DLBCL):
      • Extranodal marginal B cell lymphoma arising from mucosal associated lymphoid tissue (MALT lymphoma):
      • Follicular lymphoma:
        • Presence of t(14;18)(q32;q21) translocation has not been systematically assessed in primary breast follicular lymphoma
      • Burkitt lymphoma:
        • t(8;14)(q24;q32) c-Myc and IgH translocation is most frequent
        • t(2;8)(p12;q24) IgΚ and c-Myc
        • t(8;22)(q24;q11) c-Myc and Igλ
      Differential diagnosis
      • Invasive ductal carcinoma:
        • Poorly differentiated carcinoma mimics high grade lymphoma
        • Presence of cohesive growth, identification of gland / tubule formation and presence of in situ component favor a diagnosis of ductal carcinoma
        • Cytokeratin+, CD45-, CD20-, CD3-
      • Invasive lobular carcinoma:
      • Chronic inflammatory infiltrates (e.g. diabetic mastopathy):
        • Mimics low grade lymphoma 
        • Immunohistochemistry and flow cytometry will not identify a clonal B cell population
        • Diabetic mastopathy shows perilobular / periductal / perivascular infiltrate with reactive lymphoid follicles, keloidal fibrosis and lobular atrophy
      • Nodal lymphoma:
        • Differentiation of lymphoma within a lymph node from a primary breast lymphoma may be difficult in small biopsy specimens if breast parenchyma is not identifiable
      Board review style question #1
      A 22 year old woman presents with bilateral breast swelling. Breast biopsy shows diffuse sheets of uniform medium sized cells with round nuclei, multiple nucleoli, coarse chromatin, a thin rim of basophilic cytoplasm and interspersed tingible body macrophages. The tumor cells are positive for CD45, CD20, CD10 and BCL6. The Ki67 proliferation index is nearly 100%. The tumor cells are negative for CD5, CD23 and BCL2.

      What molecular / cytogenetic abnormality may be associated with this lymphoma?



      1. Deletion of 13q14
      2. t(8;14)(q24;q32)
      3. t(11;14)(q13;q32)
      4. t(14;18)(q32;q21)
      5. Trisomy of 3, 12, 18
      Board review style answer #1
      B. Burkitt lymphoma; t(8;14)(q24;q32)

      Comment Here

      Reference: Lymphoma
      Board review style question #2
      What is the most common type of primary breast lymphoma?

      1. Burkitt lymphoma
      2. Diffuse large B cell lymphoma
      3. Follicular lymphoma
      4. Hodgkin lymphoma
      5. MALT lymphoma
      Board review style answer #2
      B. Diffuse large B cell lymphoma

      Comment Here

      Reference: Lymphoma

      Male invasive
      Definition / general
      • Male breast cancer is uncommon, accounting for < 1% of all male cancers and 0.1% of male cancer deaths
      • Tends to occur at an older age group (60 - 70s) when compared to women (50 - 60s) and has a larger tumor size (J Clin Oncol 2010;28:232)
      • Surveillance, Epidemiology and End Results (SEER) program shows a rise in the incidence across the U.S. (Cancer 2004;101:51)
      Essential features
      Terminology
      • Invasive ductal carcinoma, ductal carcinoma in situ
      ICD coding
      • ICD-10: C50 - malignant neoplasm of breast
      Epidemiology
      • Average age of onset of DCIS is sixth decade; for invasive carcinoma is seventh decade, although highest incidence occurs in the eighth decade of life (Breast Cancer Res Treat 2004;83:77)
      • Incidence is lower in Japanese and higher in African Americans from West Africa and United States when compared to Caucasians from United States
      • Death rate is higher among nonwhite men in the United States and lower among Japanese men in Japan (J Natl Cancer Inst 1978;60:1223, Am J Public Health Nations Health 1963;53:890)
      • Despite the differences in the hormone receptor and HER2 status expression among the different races and ethnicities, no difference in the overall survival (Cancer 2013;119:1611)
      • 2x increased risk among first degree relatives of male breast cancer patients
      Sites
      • Tend to be located centrally in retroareolar position and also in upper outer quadrants
      Etiology
      • Associated with increased serum estrogen: liver disease, obesity, exogenous therapy, antiandrogen therapy
      • Associated with decreased serum androgen: Klinefelter syndrome, testicular injury / atrophy, occupational exposure to high temperatures
      • Drugs, exogenous agents: digitalis, tricyclic antidepressants, marijuana, lavender oil, tea tree oil
      • Radiation
      • Pituitary gland dysfunction: hyperprolactinemia
      • Hereditary: BRCA2 mutation carriers, PTEN mutation (Cowden syndrome), CHEK2 mutation carriers
      Clinical features
      • Invasive carcinoma:
        • Average age of presentation is 60 to 70 years, although can present in younger men and children
        • ~75% present with a palpable mass
        • Can also present with nipple retraction, ulceration or nipple discharge (serous); may be nodular and cystic
        • Time lapse between symptoms and clinical consultation varies from 6 months to 1 year
        • Tends to present with larger size and at a higher stage in men than in women
        • Lymph node metastases are more common in men than women
        • Axillary nodal staging is done by sentinel lymph node biopsy
        • History of breast cancer does not significantly affect the age or stage at time of diagnosis or the prognosis (Cancer 1999;86:821)
        • Annual clinical screening exam recommended in men with BRCA mutations
      Diagnosis
      • Clinical presentation, imaging findings and ultimately image guided biopsies of the lesion provide tissue for an accurate pathologic diagnosis
      Radiology description
      • No guidelines recommending mammographic screening for men have been established, even with documented genetic predisposition
      • Mammographic findings in men with breast cancer reveal distinct lesions with invasive margins that contrast sharply with the surrounding fatty tissue
      • Microcalcifications found in 9 - 30%
      • Ultrasound: round mass with calcifications and papillary components like those seen in cystic or encapsulated papillary carcinoma
        • Can be used to differentiate gynecomastia from carcinoma
      • Inflammatory carcinoma of the male breast produces diffuse enlargement and thickening of the skin that can be detected by MRI
      Radiology images

      Images hosted on other servers:

      Ultrasound of lymph node metastasis

      Prognostic factors
      • Poor prognostic factors: lymphatic tumor emboli, tumor size over 2 cm, poor histologic differentiation, HER2 overexpression, p53 expression, amplification of CCND1 (11q13)
      • Men with a prior diagnosis of breast carcinoma have a 30x increased risk of invasive carcinoma in the contralateral breast, increasing to 110x if the original diagnosis was before age 50
      • Death rate is higher among nonwhite U.S. men and lower among Japanese men
      • Most investigators believe men and women with the same stage disease have similar prognosis, but some believe men have poorer prognosis
      • AR+ luminal type A male breast cancer (ER+ HER2-, tumor grade 1/2) has better overall survival at 5 years but not 10 years compared to female breast cancer (Breast Cancer Res Treat 2012;133:949)
      Case reports
      Treatment
      • Treatment recommendations for male breast cancer are based on guidelines established for female breast cancer
      • Most men are treated with mastectomy and axillary lymph node dissection
      • Radiation, hormonal therapy and chemotherapy are also used
      • Breast conservation therapy may be possible, especially in older patients
      • Tamoxifen has less mortality when compared to aromatase inhibitors in men with stage I - III ER+ carcinoma (Breast Cancer Res Treat 2013; 137:465)
      Clinical images

      Images hosted on other servers:

      Bilateral tumors

      Gross description
      • Male breast cancers are grossly identical to female breast cancers; cystic papillary carcinomas can be striking
      Gross images

      Images hosted on other servers:

      Intracystic papillary carcinoma

      Microscopic (histologic) description
      • In situ carcinoma with papillary architecture is the most common pattern; also cribriform, micropapillary and solid patterns
      • Most (~85%) male breast carcinoma is poorly differentiated infiltrating ductal carcinoma, followed by papillary carcinoma and other types similar to female breast cancer
      • Lobular carcinoma represents 2% of cases
      Microscopic (histologic) images

      Contributed by Carlos C. Diez Freire, M.D.

      Male invasive ductal carcinoma

      ER

      PR


      Male mucinous carcinoma

      Lymph node metastasis, mucinous

      Male mucinous carcinoma, ER

      Male mucinous carcinoma, PR



      Contributed by Semir Vranić, M.D., Ph.D.

      Man with adenoid cystic carcinoma

      c-kit

      CK14

      Cytology description
      • Reliable but underused
      • Dispersed epithelial cells with atypical features and high N/C ratio
      • Cell clusters can be present in papillary tumors
      • Gynecomastia present with sparsely cellular aspirate with loosely cohesive sheets of cells
      Positive stains
      • ER (85 - 90%), PR (85%), AR (90 - 95%), BCL2 (94%) (most luminal A or luminal B type)
      • Younger patients (under 45 years) tend to have HER2+ tumors (Cancer 2013;119:1611)
      • Older patients (over 65 years) are most likely to have ER+, PR+ and AR+ tumors
      Negative stains
      Molecular / cytogenetics description
      • p53 mutations in exon 6 found in over 90% of male cases compared to 33% of females with p53 mutations in exon 5 and 6 (Neoplasma 1996;43:305)
      • Gene expression performed in 66 male breast cancer tumors revealed 2 unique subgroups (luminal M1 and M2) that were different in biologic features and outcome when compared to female breast cancers (Breast Cancer Res 2012;14:R31)
      • EGFR and CCND1 genes are amplified more in male than female breast cancers
      • High methylation status of MSH6, WT1, PAX5, PAX6, GATA5 and CDH13 correlates with more aggressive phenotype and poor survival
        • Male and female breast cancers share many of these methylated genes but many genes are less frequently methylated in males
      • Methylation of RASSF1A (downregulation of ER alpha and micro RNAs) is more common in male than female breast carcinomas
      Differential diagnosis
      Board review style question #1
      What is the most common type of male breast carcinoma?

      1. Invasive lobular carcinoma
      2. Invasive papillary carcinoma
      3. Mucinous carcinoma
      4. Poorly differentiated invasive ductal carcinoma
      5. Well differentiated invasive ductal carcinoma
      Board review style answer #1
      D. Poorly differentiated invasive ductal carcinoma. Approximately 85% of male breast carcinomas are poorly differentiated infiltrating ductal carcinomas, followed by papillary carcinomas and other forms similar to those found in female breast cancer. Lobular carcinomas represent 2% of cases.

      Comment Here

      Reference: Male invasive breast carcinoma

      Medullary
      Definition / general
      • Medullary pattern is a histological pattern that can be applied to an invasive breast carcinoma of no special type that contains pushing borders, syncytial growth, high grade nuclei and prominent lymphoid infiltrate
      • Represents one end of the spectrum of tumor infiltrating lymphocyte (TIL) rich invasive breast carcinomas of no special type rather than a distinct morphological subtype
      Essential features
      • Medullary pattern of invasive breast carcinoma no special type has 4 characteristic histological features (ideally, all 4 should be present for the diagnosis):
        • Pushing border
        • Syncytial growth pattern
        • High grade nuclei
        • Prominent lymphoid infiltrate
      Terminology
      • Preferred term according to WHO: invasive breast carcinoma of no special type with medullary pattern
      • Historically described as medullary carcinoma, atypical medullary carcinoma, medullary features
      ICD coding
      • ICD-O: 8500/3 - infiltrating duct carcinoma, NOS
      • ICD-11: 2C61.0 & XH7KH3 - invasive carcinoma of breast, NOS & infiltrating duct carcinoma, NOS
      Epidemiology
      • Often present earlier in life, median age of 53 (45 - 62) years (JAMA Netw Open 2021;4:e214123)
      • Rare: less than 5% of all invasive breast cancers
      Sites
      • Breast
      Pathophysiology
      Clinical features
      • Younger age
      • Soft, palpable, circumscribed mass
      • May have lymphadenopathy (due to hyperplasia rather than metastasis, which is uncommon)
      Diagnosis
      • Diagnostic steps are identical to invasive breast carcinoma (screening mammogram, diagnostic mammogram, ultrasound, core biopsy, etc.)
      Radiology description
      • Round, oval or lobulated mass on mammography
      • Hypoechoic mass on ultrasound with thick echogenic halo
      Radiology images

      Contributed by Julie M. Jorns, M.D. and Mark R. Wick, M.D.

      Mammogram

      Mammogram

      Prognostic factors
      • Prognostic factors identical to invasive breast carcinoma of no special type: patient age, tumor histological grade, tumor stage, lymphovascular invasion, ER, PR and HER2 status, molecular subtype
      • Additional prognostic factors:
      Case reports
      Treatment
      • Identical to treatment options for invasive breast carcinoma of no special type with considerations for targeted therapy based on ER, PR, HER2 status
      • Tumor infiltrating lymphocytes do not alter therapy currently
      Gross description
      • Well circumscribed, 2 - 3 cm in size, soft and fleshy (may resemble fibroadenoma)
      • Homogenous with white to gray appearance
      Gross images

      Contributed by Mark R. Wick, M.D.

      White mass

      Microscopic (histologic) description
      • Well circumscribed pushing border
      • Cells in syncytial growth pattern with no glandular structures
      • High histologic grade, high grade nuclei with prominent nucleoli
      • Prominent tumor infiltrating lymphocyte infiltrate
      Microscopic (histologic) images

      Contributed by Shannon M. Welter, M.D., Julie M. Jorns, M.D. and @SeoparjooAzmel on Twitter

      Circumscribed tumor

      Prominent lymphoid infiltrate

      Syncytial growth pattern

      Stromal lymphocytes

      High grade nuclei


      High grade nuclei

      Numerous mitotic figures

      Biopsy site and circumscription

      Syncytial growth and lymphoplasmacytic infiltrate

      Lymphoplasmacytic
      infiltrate


      Invasive breast carcinoma of no special type with medullary pattern Invasive breast carcinoma of no special type with medullary pattern Invasive breast carcinoma of no special type with medullary pattern

      Invasive breast carcinoma of no special type with medullary pattern

      Virtual slides

      Images hosted on other servers:
      Missing Image

      Breast cancer with medullary pattern

      Cytology description
      • Hypercellular specimen
      • Numerous isolated cells and loose clusters
      • Markedly enlarged, vesicular nuclei
      • Prominent, often irregular macronucleolus
      • Many lymphocytes and some plasma cells
      Positive stains
      Negative stains
      Molecular / cytogenetics description
      • Genomic instability is common
      • Majority of these tumors fall within the basal-like molecular profile
      • Approximately 15% of tumors in BRCA1 mutation carriers are classified as invasive breast carcinoma with medullary pattern (J Natl Cancer Inst 1998;90:1138)
      Sample pathology report
      • Breast, right, segmental mastectomy:
        • Invasive breast carcinoma of no special type with medullary pattern (see synoptic report)
        • Carcinoma contains 90% stromal tumor infiltrating lymphocytes
      Differential diagnosis
      Board review style question #1

      The tumor in the above image is found in the breast of a 45 year old woman. The tumor was round and soft on palpation and had pushing borders on microscopy. What is the most likely diagnosis?

      1. Ductal carcinoma in situ
      2. Invasive breast carcinoma with medullary pattern
      3. Invasive lobular carcinoma
      4. Secretory carcinoma
      Board review style answer #1
      B. Invasive breast carcinoma with medullary pattern

      Comment Here

      Reference: Invasive breast carcinoma of no special type with medullary pattern
      Board review style question #2
      Which of the following is a feature of invasive breast carcinoma with medullary pattern?

      1. Extensive surrounding ductal carcinoma in situ
      2. Low histologic grade
      3. Present as calcifications on mammography
      4. Prominent lymphocytic infiltrate
      Board review style answer #2
      Board review style question #3

      This breast tumor most often falls into which molecular profile?

      1. Basal-like
      2. HER2 enriched
      3. Luminal A
      4. Luminal B
      Board review style answer #3

      Metaplastic
      Definition / general
      • Heterogeneous group of invasive breast carcinomas characterized by differentiation of the neoplastic epithelium towards squamous cells or mesenchymal looking elements, including but not restricted to spindle, chondroid and osseous cells
      Essential features
      • Invasive breast carcinoma with atypical squamous, spindle cell or mesenchymal / matrix producing differentiation
      • When lacking a ductal carcinoma in situ or conventual mammary carcinoma component, a panel of immunohistochemical stains (cytokeratins, including high molecular weight and p63) is needed to confirm the presence of epithelial differentiation
      Terminology
      • Acceptable: metaplastic carcinoma, NOS
      • Not recommended: carcinosarcoma, sarcomatoid carcinoma, carcinoma with pseudosarcomatous metaplasia, carcinoma with pseudosarcomatous stroma
      ICD coding
      • ICD-O: 8575/3 - metaplastic carcinoma, NOS
      • ICD-11: 2C6Y & XH0RD4 - metaplastic carcinoma of breast and metaplastic carcinoma, NOS
      Epidemiology
      • Uncommon (0.2 - 1% of breast carcinomas)
      • Average 55 years
      Sites
      • Any anatomical part of breast
      Pathophysiology
      Clinical features
      Diagnosis
      • By histopathological examination
      Radiology description
      Radiology images

      Images hosted on other servers:
      Mammography shows a spiculated mass with associated nipple retraction and skin thickening

      Spiculated mass

      Prognostic factors
      Case reports
      Treatment
      • Mastectomy or local excision, with or without radiation and chemotherapy
      • Lower response rates to conventional chemotherapy (Breast J 2019;25:418)
      Clinical images

      Images hosted on other servers:
      Right breast mass

      Right breast mass

      Gross description
      • Often firm, well circumscribed and solid
      • Pearly, white to grayish and glistening cut surface in areas of squamous and chondroid differentiation
      • Mean 3.9 cm, ranging from 2 cm to > 10 cm
      Gross images

      AFIP images

      Bisected squamous tumor

      Carcinoma with osteoclast-like giant cells



      Images hosted on other servers:
      Well circumscribed<br>mass with<br>central bleeding

      Well circumscribed
      mass with
      central bleeding

      Causing skin ulceration

      Causing skin ulceration

      Sharply circumscribed tumor

      Sharply circumscribed tumor

      Microscopic (histologic) description
      • Morphologically heterogenous
      • Can be epithelial only carcinomas, pure (monophasic) sarcomatoid carcinomas and biphasic epithelial and sarcomatoid carcinomas
      • Epithelial only carcinomas include low grade adenosquamous carcinoma, high grade adenosquamous carcinoma and squamous cell carcinoma
      • Pure (monophasic) sarcomatoid carcinomas include fibromatosis-like metaplastic carcinoma and spindle cell carcinoma
      • Heterologous mesenchymal components include chondroid, osseous, rhabdomyosarcomatous, angiosarcomatous, liposarcomatous and neuroglial differentiation or in combination
      • Mesenchymal component can show a wide spectrum of atypia, ranging from minimal atypia to frankly malignant
      • May need to look carefully, extensive sampling or performing immunohistochemistry (a combination of several stains) for identification of epithelial component
      Microscopic (histologic) images

      Contributed by Huina Zhang, M.D., Ph.D. and Julie M. Jorns, M.D.
      Spindle cell carcinoma Spindle cell carcinoma Spindle cell carcinoma Spindle cell carcinoma Spindle cell carcinoma Spindle cell carcinoma

      Spindle cell carcinoma


      Metaplastic carcinoma with squamous differentiation Metaplastic carcinoma with squamous differentiation

      Metaplastic carcinoma with squamous differentiation

      Matrix producing metaplastic carcinoma

      Matrix producing metaplastic carcinoma

      Squamous type Squamous type Squamous type

      Squamous type


      Spindle cell type Spindle cell type Spindle cell type

      Spindle cell type

      Osseous differentiation

      Osseous differentiation

      Spindle cell type Spindle cell type

      Spindle cell type


      Chondromyxoid matrix producing

      Chondromyxoid matrix producing



      Contributed by Indu Agarwal M.D. and Semir Vranić, M.D., Ph.D.
      Chondroid differentiation Chondroid differentiation

      Chondroid differentiation

      Squamous component

      EGFR expression

      Virtual slides

      Images hosted on other servers:
      Fibromatosis-like metaplastic carcinoma

      Fibromatosis-like metaplastic carcinoma

      Cytology description
      Positive stains
      Negative stains
      Molecular / cytogenetics description
      Videos

      Metaplastic carcinoma by
      Dr. Alexander Damron

      Sample pathology report
      • Breast, left, 3:00 (mass), ultrasound guided core needle biopsy:
        • Malignant spindle cell lesion, consistent with spindle cell carcinoma (see comment)
        • Comment: H&E sections show a high grade spindle cell lesion with pleomorphic nuclei, irregular nuclear borders and occasional mitotic figures, including atypical mitosis. There is no leaf-like growth pattern. Immunohistochemical staining shows that the neoplastic cells are positive for pancytokeratin, p63, CK5 and AE1 / AE3 and are negative for CD34. The morphology and immunoprofile of this malignant spindle cell lesion are consistent with a spindle cell (metaplastic) carcinoma.
      Differential diagnosis
      • Phyllodes tumor:
        • Leaf-like growth pattern
        • Benign epithelial elements
        • HMWCK, CK5/6 negative
      • Primary breast sarcoma:
      • Metastatic sarcoma:
      • Adenomyoepithelioma:
        • Bland epithelial and stromal component
        • Stromal only with myoepithelial differentiation
        • Lack of squamous differentiation
      • Myoepithelial carcinoma:
        • Entirely or almost entirely malignant spindle cells with myoepithelial differentiation
        • No squamous component or other mesenchymal differentiation
      • Pleomorphic adenoma:
        • Usually retroareolar region
        • Well organized biphasic pattern with inner layer of regular epithelial cells
        • Thin, continuous outer layer of myoepithelial cells
        • No cytologic atypia, atypical mitosis or necrosis
      • See others in variants sections
      Board review style question #1


      A 55 year old woman without significant medical history was found to have a 2.5 cm mass on mammogram and a biopsy was performed. Histology and immunohistochemical stain for p63 were shown (see images above). What is the likely diagnosis?

      1. Malignant phyllodes tumor
      2. Metaplastic carcinoma
      3. Metastatic sarcoma
      4. Primary breast sarcoma
      Board review style answer #1
      B. Metaplastic carcinoma

      H&E section shows a high grade spindle cell lesion with pleomorphic nuclei, irregular nuclear borders and occasional mitotic figures, including atypical mitosis. Based on the morphology, all the listed choices should be considered. Performing a panel of immunohistochemical stains including high molecular weight cytokeratin and p63 is very important and helpful. In this case, the positivity for p63 (as well as other cytokeratins, not shown here) supports the diagnosis of spindle cell carcinoma and makes other choices less likely. It needs to be noted that cytokeratins and p63 are often focal and patchy, therefore a panel of immunostains is usually needed.

      Comment Here

      Reference: Metaplastic carcinoma

      Metastases
      Definition / general
      • Metastasis to the breast from a malignancy arising outside the breast
      Essential features
      • Rare, < 1% of breast malignancies (Eur J Surg Oncol 2003;29:854, Cancer 2007;110:731, J Clin Pathol 2007;60:1333)
      • Can be initially misdiagnosed due to nonspecific / overlapping clinical, radiologic, morphologic and immunophenotypic features with primary breast carcinoma, especially if inadequate clinical history is provided
      • Accurate identification of extramammary metastases is critical to avoid unnecessary surgery and inappropriate treatment
      Terminology
      • Nonmammary metastases to the breast, extramammary metastases to the breast
      ICD coding
      • ICD-10: C80.0 - disseminated malignant neoplasm, unspecified
      Epidemiology
      Etiology
      Clinical features
      Diagnosis
      • Possibility of a nonmammary metastasis should be considered when encountering a breast lesion with unusual clinical, radiologic or histomorphologic features
      • Clinical history is critical to establish the correct diagnosis; there is often history of advanced stage, nonbreast malignancy
      • Biopsy is performed if clinical evaluation and imaging are inconclusive for primary versus metastasis
      • Identification of a definitive in situ component can be useful to rule out metastasis and confirm a breast primary
      • Absence of an in situ component does not conclusively exclude a nonmammary metastasis, as an in situ component is not present in all primary mammary carcinomas
      • Immunohistochemical work up is generally required for confirmation of the diagnosis
      Radiology description
      Radiology images

      Images hosted on other servers:
      Missing Image

      Gastric signet ring cell carcinoma

      Missing Image

      Malignant melanoma

      Missing Image

      Ovarian serous carcinoma

      Prognostic factors
      Case reports
      Treatment
      • Primary therapeutic approach is systematic treatment that is specific and appropriate to the primary lesion
      • Generally, surgical excision is not indicated for metastatic tumors to the breast, unless performed to palliate symptoms (e.g., bulky, ulcerated lesions)
      • It is critically important for the pathologist to recognize the metastatic nature of the lesion, as the treatment and prognosis differ significantly from primary breast cancer and misdiagnosis can result in unnecessary surgery and inappropriate treatment (Mod Pathol 2013;26:343)
      Gross description
      • Generally well circumscribed
      Gross images

      Images hosted on other servers:
      Missing Image

      Malignant melanoma

      Missing Image

      Pulmonary adenocarcinoma

      Microscopic (histologic) description
      • General
        • Often morphologic features are unusual for a primary mammary tumor (J Clin Pathol 2007;60:1333)
        • Lack of an in situ component (J Clin Pathol 2007;60:1333)
        • Metastatic tumors with certain morphologies (e.g., spindle cell, mucinous, squamous, clear cell, neuroendocrine) may show significant morphologic overlap with primary breast malignancies
      • Malignant melanoma
        • Wide range of histologic appearances, may be amelanotic
        • Generally high grade cytology
        • Pigment, spindle cell morphology, intranuclear inclusions can suggest melanoma (Histopathology 2000;36:387)
      • Pulmonary carcinoma
        • Wide range of histologic appearances, including cystic papillary and glandular architecture
        • Small cell features can suggest pulmonary site of origin
      • Ovarian carcinoma
        • Usually serous type, less commonly mucinous or clear cell types
        • Wide range of histologic appearances, including papillary, glandular and solid architecture
        • May show necrosis and high grade cytology
        • Papillary morphology, psammomatous calcifications can suggest ovarian serous carcinoma (Am J Surg Pathol 2004;28:1646)
        • Often morphologic overlap with breast primary
      • Gastrointestinal carcinoma
        • If mucinous morphology, can show histologic overlap with mucinous subtype of breast carcinoma
        • If signet ring cell morphology, can show histologic overlap with lobular subtype of breast carcinoma
        • If neuroendocrine / carcinoid, can show histologic overlap with neuroendocrine carcinoma of the breast
      • Renal cell carcinoma
        • Clear cell features can suggest renal site of origin
        • If spindle cell and sarcomatoid morphology, can show histologic overlap with metaplastic breast carcinoma
      • Prostatic adenocarcinoma
        • Can show morphologic overlap with breast primary
      Microscopic (histologic) images

      Contributed by Gary Tozbikian, M.D., Julie M. Jorns, M.D. and Case #504
      Missing Image Missing Image

      Metastatic ovarian serous carcinoma

      Missing Image Missing Image

      Metastatic leiomyosarcoma

      Missing Image Missing Image

      Metastatic malignant melanoma


      Metastatic melanoma Metastatic melanoma

      Metastatic melanoma

      Metastatic clear cell renal cell carcinoma Metastatic clear cell renal cell carcinoma

      Metastatic clear cell renal cell carcinoma


      Metastatic melanoma, SOX10

      Metastatic melanoma, SOX10

      Metastatic melanoma, AE1 / AE3

      Metastatic melanoma,
      AE1 / AE3

      Metastatic melanoma, HMB45

      Metastatic melanoma, HMB45

      Cytology description
      • Malignant melanoma
        • Dispersed polygonal and plasmacytoid cells with a moderate amount of granular and vacuolated cytoplasm, can show spindle cell morphology (Diagn Cytopathol 2017;45:446)
      • Pulmonary adenocarcinoma
      • Ovarian carcinoma
        • Medium sized cells arranged in papillary clusters with round nuclei with evident nucleoli and a moderate quantity of grayish cytoplasm, psammomatous microcalcifications (Ann Oncol 2008;19:682)
      Cytology images

      Images hosted on other servers:
      Missing Image

      Metastatic ovarian serous carcinoma

      Missing Image

      Metastatic lung adenocarcinoma

      Positive stains
      Negative stains
      Sample pathology report
      • Breast, left, 2:00 zone 2, ultrasound guided core biopsy:
        • Metastatic carcinoma, consistent with metastasis of Müllerian origin (see comment)
        • Comment: Per the electronic medical record, the patient's history of ovarian cancer is noted. Immunohistochemical stains show that the tumor cells are positive for CK7, estrogen receptor, PAX8 and WT1 and are negative for GATA3, BRST2 and mammaglobin. The findings are consistent with metastasis of Müllerian origin (ovarian serous carcinoma).
      Differential diagnosis
      Board review style question #1

      A 55 year old woman with history of cutaneous malignant melanoma presents with a solitary, palpable, rapidly enlarging breast mass. Ultrasound guided core biopsy shows a high grade malignant neoplasm. What panel of immunohistochemical stains would be most useful in distinguishing metastatic melanoma versus a primary breast malignancy?

      1. Cytokeratin and estrogen receptor
      2. Cytokeratin, HMB45 and MelanA
      3. Estrogen receptor and S100
      4. SOX10 and estrogen receptor
      5. SOX10 and S100
      Board review style answer #1
      B. Cytokeratin, HMB45 and MelanA. A panel of cytokeratins and melanoma markers (e.g., S100, HMB45, MelanA) would be most useful. 20% of breast cancers will be negative for estrogen receptor. A negative result for estrogen receptor would not rule out a breast primary. SOX10 can be expressed in both melanoma as well as triple negative breast cancer.

      Comment Here

      Reference: Metastases
      Board review style question #2

      A 45 year old BRCA+ woman with a history of ER+ primary ductal carcinoma of the breast 5 years ago now presents with ascites and omental caking. Peritoneal biopsy shows a high grade malignant neoplasm with glandular and papillary features. Given the history, an initial panel of breast biomarkers is performed. The tumor is strongly positive for ER and PR and negative for HER2. What panel of additional immunohistochemical stains would be most useful in the diagnosis?

      1. CK20 and CDX2
      2. GATA3 and androgen receptor
      3. MelanA and HMB45
      4. PAX8 and WT1
      5. TTF1 and Napsin A
      Board review style answer #2
      D. PAX8 and WT1. Given the clinical presentation, hormone receptor positivity and history of BRCA mutation, a metastasis from a gynecological primary (e.g., ovarian carcinoma) is the main concern. The most appropriate immunohistochemical stain panel would include PAX8 and WT1.

      Comment Here

      Reference: Metastases

      Microcalcifications
      Definition / general
      • Small deposits of calcium in breast tissue that measure < 0.5 mm and are visible on mammographic imaging (Mol Clin Oncol 2022;16:81)
      Essential features
      • Common finding on mammograms and can represent both benign and malignant conditions
      • Microcalcifications are the most common mammographic presentation (~42%) of nonpalpable cancer (AJR Am J Roentgenol 1986;146:661)
      • 2 main types based on biochemical composition
        • Type I: calcium oxalate
        • Type II: calcium phosphate (hydroxyapatite)
      Terminology
      • Mammary microcalcifications
      • 2 main types based on biochemical composition
        • Type I: calcium oxalate (CO)
        • Type II: calcium phosphate / hydroxyapatite (HA)
      ICD coding
      • Considered a radiologic finding rather than a diagnosis; related ICD codes include
        • ICD-O
          • 8010/2 - carcinoma in situ, NOS
          • 8500/3 - infiltrating duct carcinoma, NOS
        • ICD-10
          • D24 - benign neoplasm of breast
          • C50 - malignant neoplasm of breast
          • N64.89 - other specified disorders of breast
          • N63 - unspecified lump in breast
        • ICD-11
          • 2F30 - benign neoplasm of breast
          • 2C6Z - malignant neoplasms of breast, unspecified
          • GB20.Y - other specified benign breast disease
          • MF30 - breast lump or mass female
      Epidemiology
      • Can occur at any age but is more common in older women
      • Women with a history of breast cancer may have a higher likelihood of developing microcalcifications (Breast Cancer Res 2022;24:96)
      • Women with microcalcifications have a higher risk of developing breast cancer (Breast Cancer Res 2022;24:96)
      • Strong association between the presence of calcifications and HER2 overexpression (Mol Oncol 2015;9:967)
      Sites
      • Found throughout the breast tissue, most commonly within the mammary ducts and the surrounding fibroglandular tissue (stroma, vessels, skin)
      Pathophysiology
      • Thought to arise from mineral deposition which results from various physiologic processes
        • Secretory
        • Inflammatory
        • Senescence
        • Traumatic injury
        • Necrosis
      Etiology
      • Dystrophic calcification in breast tissue is traditionally perceived as a passive phenomenon but type II microcalcifications (MCs) are often associated with abnormal epithelial cells; suggesting an active process
      • Bone hydroxyapatite (HA) deposition is recognized as an active and physiological process; however, in breast tissue, HA deposition is considered pathological
      • Breast MCs may undergo regulation similar to that of physiologic bone mineralization, despite being considered a pathologic process
      • Formation of mammary MCs likely involves active secretion and may include epithelial to mesenchymal transition (EMT) of breast cells
      • Upregulation of osteogenic proteins such as osteopontin and BMP2 in breast cancer suggests a connection between breast cancer and osteogenic characteristics (Mol Clin Oncol 2022;16:81)
      Diagrams / tables
      Not relevant
      Clinical features
      • Asymptomatic and too small to be palpated
      • Identified during a mammogram and classified based on number, size, morphology and distribution using the BI-RADS (breast imaging reporting and data system) scoring system
      • Presence of microcalcifications on mammography has led to the detection of breast tumors as small as 1 - 2 mm (Mol Clin Oncol 2022;16:81)
      • Microcalcifications are present in 50% of carcinomas, compared to 20% in benign breast disease (Mol Clin Oncol 2022;16:81)
      • Only 20% of suspicious microcalcifications are actually part of a malignant process (Br J Cancer 2021;125:759)
      Diagnosis
      • Breast microcalcifications are typically first identified during mammography (Mol Clin Oncol 2022;16:81)
      • Radiographical findings and correlation with clinical history is necessary
      • Core biopsy is frequently performed if there are microcalcifications (Korean J Radiol 2015;16:996)
      • Pathological diagnosis needs to correlate with imaging findings
      Laboratory
      Not relevant
      Radiology description
      • Microcalcifications are detected during mammography
      • Based on their number, size, morphology and distribution, a BI-RADS category indicative of the likelihood of malignancy is assigned (see Table 1 and 2)
      • Suspicious microcalcifications are irregular and fine; nonsuspicious are coarse and chunky
      • Pathologists must detect microcalcifications in glass slides that correspond to those observed in radiographs; if microcalcifications are not initially identified, they should consider submitting additional tissue, obtaining additional levels or employing polarized microscopy to identify calcium oxalate (Pathologica 2007;99:5)
      • Note: microcalcifications may be absent from biopsy specimen due to retrieval failure (Radiology 2006;239:61)
      • Note: it is recommended to histologically examine all vacuum assisted breast biopsy specimens, regardless of the presence or absence of microcalcifications (Eur Radiol 2008;18:925)
      • Detection of calcium phosphate microcalcifications is diminished with glyoxal fixative (Eur Radiol 2008;18:925)


      Table 1: BI-RADS assessment categories and likelihood of cancer (adapted from Biochim Biophys Acta Rev Cancer 2018;1869:310)
      Category Assessment Management Likelihood of cancer
      0 Incomplete; needs additional imaging evaluation or prior mammograms for comparison Additional imaging or comparison with prior examination(s) N/A
      1 Negative Routine mammography screening 0%
      2 Benign Routine mammography screening 0%
      3 Probably benign Short interval (6 month) follow up or continued surveillance mammography > 0% but ≤ 2%
      4 Suspicious for malignancy
      • 4A: Low suspicion
      • 4B: Moderate suspicion
      • 4C: High suspicion
      Tissue diagnosis > 2% but < 95%
      • > 2% to ≤ 10%
      • > 10% to ≤ 50%
      • > 50% to < 95%
      5 Highly suggestive of malignancy Tissue diagnosis ≥ 95%
      6 Known biopsy proven malignancy Surgical excision when clinically appropriate N/A


      Table 2: Mammographic characteristic features of benign versus suspicious breast calcifications (Biochim Biophys Acta Rev Cancer 2018;1869:310, Mol Clin Oncol 2022;16:81, YouTube: Breast Imaging Calcifications - module 2 | Health4TheWorld Academy [Accessed 8 April 2024], Nov Approaches Cancer Study 2021;6:582)
      Features Typically Benign Suspicious for Malignancy
      Quantity Few Many
      Size (diameter) Large and chunky, 1.0 - 4.0 mm < 0.5 mm
      Shape Coarse, round, oval, uniform Fine, linear
      Borders Well defined, regular Irregular, less defined
      Groups 5 calcifications within 1 cm > 5 calcification within 2 cm
      Distribution Diffuse / scattered, regional Linear, segmental
      Morphology Eggshell like, pop-corn-like, rod-like Amorphous, coarse heterogenous, fine pleomorphic, fine linear branching
      Other associations Skin, vascular, dystrophic, milk of calcium, suture, breast infarcts
      Radiology images

      Contributed by Agnes Ikpoto Udoh, M.D., M.B.A.
      Microcalcifications in grouped distribution

      Microcalcifications in grouped distribution

      Round microcalcifications (BI-RADS 4A)

      Round microcalcifications (BI-RADS 4A)

      Linear microcalcifications (BI-RADS 4B)

      Linear microcalcifications (BI-RADS 4B)

      Linear microcalcifications (BI-RADS 4C)

      Linear microcalcifications (BI-RADS 4C)



      Images hosted on other servers:
      Involuting Fibroadenoma with popcorn microcalcifications

      Involuting fibroadenoma with popcorn microcalcifications

      Breast calcification BI-RADS 2

      Breast calcification BI-RADS 2

      Linear microcalcifications in DCIS

      Linear microcalcifications in DCIS

      Prognostic factors
      • Microcalcifications are associated with both favorable prognostic factors (i.e., small size and hormone receptor positivity) and unfavorable factors (i.e., high grade), depending on the associated diagnosis (Cancer 2017;123:219)
      • Casting (duct centric) calcification subtype is an independent unfavorable prognostic factor in breast cancer patients, affecting ~10 - 12% of cases; it is associated with specific genetic and molecular characteristics and is linked to decreased overall survival rates (Semin Cancer Biol 2021;72:165)
      Case reports
      • 27 year old woman with pleomorphic calcifications and segmental distribution on mammography, highly suggestive of breast cancer; however, the pathological findings were fibrocystic disease (Radiol Case Rep 2023;18:3828)
      • 42 year old breastfeeding woman with multiple groups of microcalcifications in bilateral breasts (Breast J 2004;10:247)
      • 54 year old woman with a history of gastric cancer and pleomorphic microcalcifications in the breast (J Breast Cancer 2012;15:356)
      • 73 year old woman diagnosed with primary breast amyloidosis and persistent microcalcifications (Int J Surg Pathol 2013;21:177)
      Treatment
      Clinical images
      Not available
      Gross description
      • Typically, specimens come as core biopsies
      Gross images
      Not relevant to this topic
      Frozen section description
      Not relevant to this topic
      Frozen section images
      Not relevant to this topic
      Microscopic (histologic) description
      • There are 2 primary types of microcalcifications based on biochemical composition: calcium oxalate and calcium phosphate
      • Calcium oxalate (type I)
        • Less common
        • These crystals do not bind with the hematoxylin stain and appear as clear refractile structures; they are best visualized under polarized light, with the condenser flipped down for optimal observation (Am J Surg Pathol 1990;14:961, Mod Pathol 1992;5:146)
        • Predominantly found within benign cysts or terminal ductules, often demonstrating histological features of apocrine cells
        • Can be associated with lobular carcinoma in situ and are rarely seen in association with invasive carcinoma (Am J Surg Pathol 1991;15:586)
      • Calcium phosphate (type II)
        • More common
        • Manifest as purple / blue psammoma-like aggregates, either solitary or clustered, when stained with H&E
        • May be observed in both benign and malignant lesions; they are commonly associated with premalignant and malignant lesions (Mol Clin Oncol 2022;16:81)
      • If a mammogram reveals the presence of microcalcifications, diligent efforts should be undertaken to locate them; furthermore, the presence of microcalcifications must be documented in the histology report, along with any associated lesions
      • Location of the microcalcifications may suggest the following possible differentials
        • Lobular calcifications
          • Cystic breast disease
          • Milk of calcium
          • Sclerosing adenosis
          • Atypical lobular hyperplasia (ALH)
          • Lobular carcinoma in situ (LCIS)
          • Invasive lobular carcinoma (ILC)
        • Ductal calcifications
          • Columnar cell change
          • Flat epithelial atypia (FEA)
          • Atypical ductal hyperplasia (ADH)
          • Ductal carcinoma in situ (DCIS)
          • Invasive ductal carcinoma (IDC)
      • The following steps are recommended to locate the microcalcifications
        • Step 1: examine the H&E stained slides meticulously for microcalcifications
        • Step 2: if not apparent on H&E, utilize polarized light microscopy, especially for detecting calcium oxalate crystals
        • Step 3: consider cutting deeper levels of the H&E slides to further evaluate for microcalcifications
        • Step 4: utilize Xray imaging of the tissue block for enhanced detection
        • Step 5: consult with the radiologist to confirm that the lesion has been biopsied
      Microscopic (histologic) images

      Contributed by Jing He, M.D.
      DCIS with microcalcifications DCIS with microcalcifications

      DCIS

      Collagenous spherulosis with microcalcifications

      Collagenous spherulosis

      Fibroadenoma with microcalcifications

      Fibroadenoma

      Fibroadenoma with microcalcifications

      Columnar cell lesion

      Apocrine metaplasia with microcalcifications

      Apocrine metaplasia


      Radial scar with microcalcifications

      Radial scar

      Papillary lesion with microcalcifications

      Papillary lesion

      Fibrocystic changes with microcalcifications

      Fibrocystic changes

      Invasive tubular carcinoma with microcalcifications

      Invasive tubular carcinoma

      IDC after neoadjuvant chemotherapy with microcalcifications

      IDC after neoadjuvant chemotherapy

      Sclerosing adenosis with microcalcifications

      Sclerosing adenosis


      LCIS with microcalcifications

      LCIS

      Fat necrosis with microcalcifications

      Fat necrosis

      Flat epithelial atypia with microcalcifications

      Flat epithelial atypia

      Usual ductal hyperplasia with microcalcifications

      Usual ductal hyperplasia

      Fibrocystic changes with calcium oxalate calcifications

      Fibrocystic changes with calcium oxalate calcifications

      Calcium oxalate calcifications with polarized light

      Calcium oxalate calcifications with polarized light

      Virtual slides
      Not available.
      Cytology description
      Not relevant to this topic
      Cytology images
      Not relevant to this topic
      Immunofluorescence description
      Not relevant to this topic
      Immunofluorescence images
      Not relevant to this topic
      Positive stains
      Not relevant to this topic
      Negative stains
      Not relevant to this topic
      Electron microscopy description
      Not relevant to this topic
      Electron microscopy images
      Not relevant to this topic
      Molecular / cytogenetics description
      Not relevant to this topic
      Molecular / cytogenetics images
      Not relevant to this topic
      Videos

      Breast imaging: calcifications (basic radiology)

      Breast imaging calcifications (module 1)

      Breast imaging calcifications (module 2)

      Sample pathology report
      • Breast, right, 3 o'clock, 5 cm from nipple, microcalcifications, stereotactic core biopsy:
        • Sclerosing adenosis with associated microcalcifications
      Differential diagnosis
      • Dystrophic calcification:
        • These can develop in areas of old scarring and may warrant biopsy during follow up after conservation surgery for malignant disease
        • Typically, they appear coarse and may exhibit an eggshell-like appearance
        • They can pose a diagnostic challenge if they are arranged in clusters
      • Vascular calcification:
        • Typically conspicuous but can resemble casting calcifications
        • They are arranged in parallel streaks and distributed along a vessel
      Additional references
      Not available
      Board review style question #1

      Which of the following characteristics is most suggestive of benign microcalcifications on mammography?

      1. Fine pleomorphic calcifications
      2. Linear or branching distribution
      3. Round or oval shape with smooth borders
      4. Segmental distribution
      Board review style answer #1
      C. Round or oval shape with smooth borders. Benign microcalcifications commonly exhibit a round or oval shape with smooth margins, which typically suggest benign conditions like fibrocystic changes or benign breast calcifications. Answer B is incorrect because linear or branching distribution is more commonly associated with malignant lesions, particularly ductal carcinoma in situ (DCIS). Answer A is incorrect because fine pleomorphic calcifications are irregular in shape and size and have a higher likelihood of being associated with malignant lesions than the classifications that are round with smooth borders. Answer D is incorrect because segmental distribution is usually suggestive of malignancy; however, there are few reports of its association with fibrocystic change (Radiol Case Rep 2023;18:3828).

      Comment Here

      Reference: Microcalcifications
      Board review style question #2
      What is the recommended next step if microcalcifications are not initially identified in histological examination?

      1. Consult with a radiologist for further imaging
      2. Submit additional tissue for analysis
      3. Disregard the findings as insignificant
      4. Perform electron microscopy for better resolution
      Board review style answer #2
      B. Submit additional tissue for analysis. Of the provided options, the best option would be to submit additional tissue for analysis. It would be most appropriate to first obtain additional levels before doing so, after which you should employ polarized microscopy to identify calcium oxalate crystals if still not seen on H&E. Answer A is incorrect because while consulting with a radiologist for further imaging might be a further step taken in the diagnostic process, it is not the recommended next step if microcalcifications are not initially identified in histological examination. Answer C is incorrect because disregarding the findings as insignificant without further investigation would be inappropriate and potentially harmful. Microcalcifications can be indicative of various benign and malignant conditions, so their presence or absence could have implications for diagnosis and treatment planning. Dismissing them without proper evaluation could lead to missed diagnoses or incorrect management decisions. Answer D is incorrect because electron microscopy is a highly specialized technique used for examining ultrastructural details of tissues at a very high resolution. While electron microscopy can provide detailed information, it is not routinely used for identifying microcalcifications in breast tissue.

      Comment Here

      Reference: Microcalcifications
      Board review style question #3

      A 49 year old woman is seen for bilateral breast tenderness. She is not pregnant or on hormone therapy. Mammography of the left breast revealed grouped punctate and indistinct calcifications measuring 6 mm at 12 o'clock position and 5 cm from the nipple, BIRADS 4A. No lesions are seen in the right breast. A follow up biopsy is shown above. IHC showed heterogenous ER staining and positive CK5/6. What is the most likely diagnosis?

      1. DCIS with microcalcifications
      2. Fat necrosis with microcalcificatons
      3. Invasive ductal carcinoma with microcalcificatons
      4. Usual ductal hyperplasia with microcalcifications
      Board review style answer #3
      D. Usual ductal hyperplasia with microcalcifications. The histology shows benign appearing hyperplastic ductal cells admixed with myoepithelial cells with associated microcalcifications. Answer A is incorrect because of the presence of myoepithelial cells and heterogenous ER staining. In DCIS, ER is diffuse strong positive. Answer C is incorrect because the cells appear benign with myoepithelial cells admixed with the ductal cells. Myoepithelial cells are absent in invasive breast cancer. Also, radiology had a low suspicion for malignancy. Answer D is incorrect because fat necrosis usually follows trauma and consists of cystic spaces surrounded by lipid laden (foamy) macrophages. Though histiocytes can be seen in this picture the more appropriate diagnosis is usual ductal hyperplasia.

      Comment Here

      Reference: Microcalcifications

      Microcalcifications
      Definition / general
      • Deposits of calcium in breast tissue visible on mammographic imaging
      Terminology
      Epidemiology
      • Can occur at any age but more common after menopause
      Etiology
      • Can be associated with both benign and malignant lesions
      Clinical features
      • Presence of microcalcifications on mammography has led to detection of breast tumors as small as 1 - 2 mm
      • Microcalcifications are present in 50% of carcinomas versus 20% of benign breast disease but only 20% of "suspicious" microcalcifications are actually part of a malignant process
      Radiology description
      • Suspicious microcalcifications are irregular and fine; nonsuspicious are coarse and chunky
      • Pathologists must detect microcalcifications in glass slides that correspond to those in radiographs - if not present, submit additional tissue, obtain additional levels or use polarized microscopy to look for calcium oxalate (Pathologica 2007;99:5)
      • Exhaustive searching for microcalcifications yields a small increase in specific diagnostic information but with a high technical cost (Mod Pathol 2001;14:350)
      • Note: microcalcifications may be missing from biopsy due to retrieval failure (Radiology 2006;239:61)
      • Note: recommended to examine all vacuum assisted breast biopsy specimens histologically, even those without microcalcifications (Eur Radiol 2008;18:925)
      • Detection of calcium phosphate microcalcifications is reduced with glyoxal fixative (Hum Pathol 2004;35:1058)

      Radiologic BI-RADS (Breast Imaging Reporting and Data System of American College of Radiology) classification:
      • Category 0 - need additional imaging evaluation
      • Category 1 - negative
      • Category 2 - benign finding
      • Category 3 - probably benign finding - short term interval follow up suggested
      • Category 4 - suspicious abnormality - biopsy should be considered
      • Category 5 - highly suggestive of malignancy - appropriate action should be taken
      • References: American College of Radiology: ACR BI-RADS® Atlas, Fifth Edition, 2013

      • Suggested that radiologists subcategorize BI-RADS 4 as 4A (low suspicion for malignancy), 4B (intermediate suspicion of malignancy) and 4C (moderate concern but not classic for malignancy, Breast J 2010;16:28)

      LeGal classification of microcalcifications
      • Type 1 - annular
      • Type 2 - regularly punctiform
      • Type 3 - too fine for precizing the shape
      • Type 4 - irregularly punctiform
      • Type 5 - vermicular (Bull Cancer 1984;71:57)
      Radiology images

      Images hosted on other servers:

      DCIS

      Case reports
      Microscopic (histologic) description
      • Calcium phosphate microcalcifications are associated with benign and malignant disease; are blue / purple psammoma like chunks
      • Calcium oxalate crystals are typically within benign cysts or terminal ductules that are histologically apocrine or GCDFP-15 positive; are associated with LCIS but only rarely with invasive carcinoma (Am J Surg Pathol 1991;15:586)
      • Calcium oxalate crystals may be present in centrifuged fixative (Am J Surg Pathol 1997;21:255)
      Microscopic (histologic) images

      AFIP images

      Calcium oxalate calcifications

      With polarized light

      Without polarized light



      Benign lesions with microcalcifications

      ADH

      Sclerosing adenosis



      Images hosted on other servers:

      Benign lesions with microcalcifications:

      Mucocele-like lesion

      Positive stains
      Differential diagnosis

      Microcysts
      Definition / general
      • Round to ovoid fluid filled structures of variable size lined by bland epithelium, part of nonproliferative fibrocystic changes
      Essential features
      • Very common nonproliferative fibrocystic change
      • Round to ovoid fluid filled cysts lined by flat, cuboidal or columnar cells that may be attenuated and denuded
      • Cysts alone do not increase risk of breast cancer
      Terminology
      • Blue dome cysts: based on gross appearance
      • Type 1 cysts:
        • Na/K ratio of 3 or less: increased breast cancer risk; associated with higher levels of estrogen, melatonin, epidermal growth factor and DHEA-S and lower levels of TGF-B2 than type 2 cysts (Breast Cancer Res Treat 2007;103:331)
      • Type 2 cysts:
        • Na/K above 3: reduced breast cancer risk, although patients may have both types of cysts and type 2 cysts can be associated with cancer (Breast 2005;14:37)
      • Microcysts: seen during imaging or microscopic exam; not palpable
      • Macro or gross cysts: large enough to be palpated

      Note:
      • Type 1 and type 2 terminology is not commonly used
      ICD coding
      • ICD-10:
        • N60.0 - solitary cyst of breast
        • N60.19 - diffuse cystic mastopathy of unspecified breast
      • ICD-11: GB20.0 - fibrocystic change
      Epidemiology
      • Most common nonproliferative change in breast along with metaplastic changes
      • Any age
      • 20 - 25% of palpable breast abnormalities that underwent fine needle aspiration were simple cysts (Cancer 2001;93:263)
      • Prevalence estimated between 50 - 90% (Breast Cancer Res Treat 2006;97:115)
      Sites
      • Usually multifocal and bilateral
      Pathophysiology
      • Arises in the terminal ductal lobular unit (TDLU) → dilation and coalescence of lobular acini
      • Gross cysts defined by Haagensen are palpable (Cancer 1989;63:2156)
      Etiology
      • Unknown
      Clinical features
      • Large cysts may present as a palpable mass
      Diagnosis
      • Fine needle aspiration:
        • Mass disappears after aspiration and the fluid is nonbloody, yellow or green and serous, no further workup necessary (Can Fam Physician 2012;58:1240)
        • Further workup starting with imaging required after aspiration if no fluid, bloody or tenacious fluid or mass persists (Am Fam Physician 2003;68:1983)
        • Bloody or cloudy or turbid fluid should be sent for cytopathology review
      Radiology description
      Radiology images

      Contributed by Kristen E. Muller, D.O.
      Hypoechoic mass, ultrasound

      Hypoechoic mass, ultrasound

      Circumscribed mass, mammogram

      Circumscribed mass, mammogram

      Prognostic factors
      Case reports
      Treatment
      • Aspiration / decompression if symptomatic or for diagnostic purposes (See Diagnosis section)
      Gross description
      • Variable size, usually visible grossly, contains clear, straw colored or brown fluid
      • Larger intact cysts may appear blue (blue domed cyst) (Breast Cancer Res Treat 2006;97:115)
      Gross images

      Contributed by Kristen E. Muller, D.O. and AFIP images
      Numerous grossly visible cysts

      Numerous grossly visible cysts

      Large hemorrhagic cysts

      Large hemorrhagic cysts



      Images hosted on other servers:

      Unopened cyst

      Extensive cystic disease

      Microscopic (histologic) description
      Microscopic (histologic) images

      Contributed by Kristen E. Muller, D.O.
      Simple epithelial lining

      Simple epithelial lining

      Flattened epithelial lining

      Flattened epithelial lining

      Denuded epithelial lining

      Denuded epithelial lining

      Cysts, columnar cell change Cysts, columnar cell change Cysts, columnar cell change

      Cysts, columnar cell change


      Surrounding fibrosis and inflammation Surrounding fibrosis and inflammation

      Surrounding fibrosis and inflammation

      Ruptured cyst Ruptured cyst

      Ruptured cyst

      Cluster of microcysts

      Cluster of microcysts

      Cytology description
      • Hypocellular aspirate, macrophages with other inflammatory cells, with or without few clusters of benign ductal epithelial and myoepithelial cells (Patholog Res Int 2011;2011:547580)
      • Prior rupture of the cyst may result in turbid or milky fluid, degenerated cells and debris in an abundant background of inflammatory cells (Patholog Res Int 2011;2011:547580)
      Cytology images

      Contributed by Kristen E. Muller, D.O.
      Predominantly debris, cell block

      Predominantly debris, cell block

      Hypocellular specimen, LBP

      Hypocellular specimen, LBP

      Sample pathology report
      • Left breast, 2 o’clock, 1 cm from nipple, core needle biopsy:
        • Benign breast tissue with fragments of cyst wall with surrounding fibrosis and chronic inflammation
      Differential diagnosis
      Board review style question #1

      The findings pictured are from a 38 year old woman with a cystic breast mass. What is the diagnosis?

      1. Atypical ductal hyperplasia
      2. Cysts
      3. Duct ectasia
      4. Flat epithelial atypia
      5. Lymphocytic mastitis
      Board review style answer #1
      B. Cysts

      Comment Here

      Reference: Microcysts
      Board review style question #2

      The findings pictured are from a 67 year old woman with a palpable breast mass. Ultrasound showed a complex cystic and solid mass. What is the diagnosis?

      1. Duct ectasia
      2. Ductal carcinoma in situ
      3. Granulomatous mastitis
      4. Ruptured cyst wall
      Board review style answer #2
      D. Ruptured cyst wall

      Comment Here

      Reference: Microcysts

      Microglandular adenosis
      Definition / general
      • Rare benign lesion composed of haphazard, irregularly distributed, small, uniform, rounded, open glands with eosinophilic secretions
      • Mimics invasive carcinoma due to infiltrative stromal growth and single epithelial layer devoid of myoepithelium
      Essential features
      • Disordered proliferation of small open glands comprised of bland epithelial cells
      • Retains basement membrane but lacks myoepithelium
      • Infiltrates adipose and fibrous stroma without a stromal response
      • S100+, ER-, PR-
      • Potential precursor to triple negative invasive breast cancer
      Terminology
      • Microglandular adenosis (MGA)
      • Less frequently referred to as:
        • Microglandular hyperplasia
        • Microglandular adenoma
      ICD coding
      • ICD-10:
        • N60.8 - other benign mammary dysplasias
        • N60.9 - unspecified benign mammary dysplasia
      • ICD-11:
        • GB20.Y - other specified benign breast disease
        • GB20.Z - benign breast disease, unspecified
      Epidemiology
      • Rare, represented in case reports and series and small studies
      • No specific known epidemiologic associations (reported in women over a wide age range)
      Sites
      • No specific location in breast
      Pathophysiology
      • Gain of somatic mutations including TP53, PIK3CA pathway related and tyrosine kinase receptor signaling elated genes have been identified in MGA and MGA associated triple negative breast cancers, supporting a role as a nonobligate cancer precursor (J Pathol 2016;238:677)
      Etiology
      • Unknown
      Clinical features
      • Can present as a thickening, palpable mass or no clinical abnormality (seen on imaging only or incidental finding)
      Diagnosis
      • Histologic examination of tissue often with immunostains performed
      Radiology description
      • Mammographic density or calcifications
      Radiology images

      Contributed by Julie M. Jorns, M.D.
      Missing Image Missing Image

      Mammographic density: MGA / atypical MGA

      Prognostic factors
      Case reports
      Treatment
      Gross description
      • Indiscernible from surrounding tissue
      • Mass / nodule: more frequently identified with coexisting carcinoma
      • Ill defined margins
      Microscopic (histologic) description
      • Haphazardly infiltrating collection of small, uniform, rounded, open glands with eosinophilic secretions, irregularly distributed in fibrous or adipose tissue
      • Glands lined by single layer of cuboidal / flat cells with vacuolated / granular cytoplasm and bland nuclei
      • No apocrine snouts, no nucleoli, no myoepithelial layer but thick basement membrane
      • Atypical microglandular adenosis:
      Microscopic (histologic) images

      Contributed by Julie M. Jorns, M.D.
      Missing Image

      Scattered round glands

      Missing Image

      Eosinophilic secretions

      Missing Image

      PAS

      Missing Image

      S100

      Missing Image

      p63


      Missing Image

      With atypia, cribriforming

      Missing Image

      With atypia, complex architecture

      Missing Image

      With atypia, S100

      Missing Image

      With atypia, collagen IV

      Cytology description
      • Sparse cellularity, monotonous population of medium sized cells with vacuolated clear cytoplasm, round and uniform nuclei, small nucleoli
      • Also clear cells that are solitary or clustered with spindly fibroblasts (Diagn Cytopathol 1993;9:72)
      Positive stains
      Electron microscopy description
      Molecular / cytogenetics description
      • Clonal in some cases, array CGH and sequencing studies demonstrate molecular progression to matched invasive carcinoma, supports the role of MGA as a nonobligate precursor to triple negative breast cancer (Hum Pathol 2019;85:65, Histopathology 2012;60:E115)
      • Mutations in PI3K pathway (PIK3CA, PTEN, INPP4B and BRCA1) in MGA and atypical MGA (Mod Pathol 2017;30:69)
      Sample pathology report
      • Right breast, core biopsy:
        • Microglandular adenosis (MGA) (see comment)
        • Comment: Immunohistochemistry shows the lesion to be S100 positive (diffusely, strongly), estrogen receptor negative and to lack myopithelium via p63 and calponin, supporting the diagnosis. Controls are appropriate.
      Differential diagnosis
      Board review style question #1

        The pictured lesion shows 3 regions in a breast core biopsy. What is the expected biomarker profile of the invasive carcinoma?

      1. ER-, PR-, HER2-
      2. ER+, PR-, HER2-
      3. ER+, PR+, HER2-
      4. ER+, PR+, HER2+
      Board review style answer #1
      A. ER-, PR-, HER2-. The pictured lesion is microglandular adenosis (MGA) with associated invasive carcinoma, which is supported by S100 positivity in the MGA. Both MGA and associated carcinomas are most frequently ER-, PR- and HER2- (triple negative).

      Comment Here

      Reference: Microglandular adenosis of breast
      Board review style question #2
        What is a feature that is characteristic of atypical microglandular adenosis (versus typical microglandular adenosis)?

      1. Bland cytology
      2. Fused / cribriform glands
      3. Intraluminal secretions
      4. S100 positivity
      Board review style answer #2
      B. Fused / cribriform glands. Typical microglandular adenosis (MGA) is characterized by haphazardly infiltrative tubules with characteristic luminal secretions and lined by a single layer of bland epithelial cells. Atypical forms have atypical cytology and architecture, including multilayered epithelium, gland fusion and cribriform architecture. Atypical MGA may not have as prominent secretions as in more typical forms. Both typical and atypical MGA are S100 (diffusely, strongly) positive.

      Comment Here

      Reference: Microglandular adenosis of breast

      Microinvasive
      Definition / general
      • Dominant lesion is not invasive, but there are 1+ separate small, microscopic foci of infiltration, each 1 mm or less in size
      Terminology
      • “Minimal breast carcinoma” includes microinvasive carcinoma and DCIS
      • Can confirm using myoepithelial stains (myoepithelial layer is not intact) and keratin (to observe infiltrative growth)
      • Diagnosis requires certainty of invasion; if doubt remains after recuts and immunostains, call DCIS or suspicious
      Epidemiology
      • Mean age 61 years
      Clinical features
      • Less than 1% of all breast cancers
      • Usually detected by mammography due to abnormal calcifications in associated DCIS
      • 72% associated with comedo DCIS, 89% with high nuclear grade and 89% with necrosis
      • Sentinel lymph node dissection may be appropriate, although axillary nodal metastases occur in less than 10% (Breast J 2008;14:335, Breast 2008;17:395); controversial whether to perform complete axillary dissection if positive sentinel node (yes - Breast 2007;16:146, no - Am J Surg 2007;194:845)
      • Commonly misdiagnosed, as true diagnosis is usually DCIS or T1a carcinoma (Cancer 2000;88:1403)
      • In breast core needle biopsies, invasive carcinomas 1 mm or less are rare, are associated with DCIS and ADH, and often with large invasive foci at excision (Arch Pathol Lab Med 2004;128:996)
      • Report number of foci of invasion, size of largest focus
      Treatment
      Microscopic (histologic) description
      • Usually ductal, rarely tubular or lobular morphology
      • Nodules of invading neoplastic cells in periductal or perilobular stroma, none exceeding 1.0 mm
      • Usually arises in background of high grade DCIS; stromal microinvasion typically associated with fibroblast proliferation, collagenization and focal inflammation
      • False positives: lobular cancerization, radial scar, sclerosing adenosis (Arch Pathol Lab Med 2001;125:1259)
      • False negatives: masking of invasion by inflammatory cells or histiocytes; use cytokeratin to highlight tumor cells
      Microscopic (histologic) images

      Images hosted on other servers:

      Invasive carcinoma (arrow) < 1 mm, surrounded by DCIS

      Figures 1, 2 and 3

      Figure 1

      Negative stains
      Additional references

      Micropapillary
      Definition / general
      Essential features
      • Pseudopapillae, morulae or tubules with inside out growth pattern characterized by reverse polarity of cells
      Terminology
      ICD coding
      • ICD-11:
        • XH9C56 - Invasive micropapillary carcinoma of breast
        • 2C60 - Carcinoma of breast, specialized type
        • 2C6Y - Other specified malignant neoplasms of breast
      • ICD-10:
        • C50 - Malignant neoplasm of breast
      Epidemiology
      • < 2 - 8% of all breast cancers (Mod Pathol 2018;31:956)
      • Pure micropapillary carcinoma is rare, 0.9 - 2% of all breast cancers; often admixed with other histologic subtypes
      • Reported median age: 50 - 62 years
      • Mean age: 59 years; range: 25 - 92 years
      • Also described in male breast (Breast Care 2018;13:192)
      Sites
      • Breast parenchyma
      Pathophysiology
      Etiology
      • Unknown
      Clinical features
      • Almost 90% present as a palpable mass
      • Radiologic findings, locations and gross features similar to conventional invasive breast carcinoma (BMC Cancer 2017;17:839)
      • Aggressive clinical presentation with large tumor size (often T2), frequent positive lymph nodes at initial presentation and advanced stage when compared with invasive ductal carcinoma (Cancer Med 2017;6:2775)
      • Mean size: 1.5 - 3.9 cm
      • Frequent and higher number of axillary nodal metastasis (44 - 85%) at initial diagnosis
      Diagnosis
      • Tissue biopsy or surgical resection
      Radiology description
      • Multiple lesions, accompanying nonmass enhancement and axillary lymph node enlargement
      • Irregular spiculated mass on mammogram
      • Irregular hypoechoic mass with spiculated margins and posterior acoustic shadowing on ultrasound
      • Irregular mass with washout kinetics and diffuse heterogeneous mass-like enhancement in MRI
      • Radiologists to consider axillary sonography if this entity is found in a core needle biopsy specimen (AJR Am J Roentgenol 2013;200:689)
      Prognostic factors
      • Traditionally, presence of micropapillary carcinoma component has been considered a poor prognostic factor; however, the data has been controversial
      • Meta analysis of 14 published studies showed that presence of micropapillary carcinoma histology is an unfavorable prognostic factor for recurrence free survival and locoregional recurrence free survival (BMC Cancer 2017;17:839)
      Case reports
      Treatment
      • Tumors are typically ER positive and are eligible for antiestrogen treatment
      • Treatment varies depending on clinical stage (tumor size and lymph node metastasis)
      Gross description
      • No specific gross features; tumors with > 50% of micropapillary growth tend to be larger (mean: 6 cm) than those with lesser amount of this pattern (mean: 3.5 cm)
      • Mean tumor size: 2 cm; range: 0.1 - 10 cm
      • Lobulated outline
      Microscopic (histologic) description
      • Tufts of cells arranged in pseudopapillae, hollow tubules and morula that are surrounded by empty clear spaces formed by fibrocollagenous stroma (Arch Pathol Lab Med 2016;140:799)
      • Cells show characteristic reverse polarity, also called inside out pattern, with apical surface abutting the epithelial stromal interface
      • Densely cellular, with minimal stroma
      • Abundant eosinophilic cytoplasm
      • Nuclei around the periphery bulge with knobby / serrated appearance
      • Extensive true angiolymphatic invasion (78%)
      • Foci of necrosis may be present
      • May have intracytoplasmic mucin
      • Uncommon variant feature: psammoma bodies, microcystic dilation of lumens with cell clusters, apocrine differentiation, multinucleated giant cells (e.g. osteoclast)
      • Variable histologic grade, usually grade 2 or 3
      • Mucinous carcinoma can contain micropapillary component, also called mucinous micropapillary carcinoma
      Microscopic (histologic) images

      Contributed by Kamaljeet Singh, M.D.
      Micropapillary carcinoma Micropapillary carcinoma Micropapillary carcinoma

      Micropapillary carcinoma

      Lymphovascular invasion

      Lymphovascular invasion

      EMA

      EMA

      E-cadherin

      E-cadherin


      p120

      p120

      HER2 2+

      HER2 2+

      HER2 3+

      HER2 3+

      Mucinous micropapillary carcinoma Mucinous micropapillary carcinoma Mucinous micropapillary carcinoma

      Mucinous micropapillary carcinoma


      Mucinous hypercellular tumor

      Mucinous hypercellular tumor

      HER2+ mucinous micropapillary carcinoma

      HER2+ mucinous micropapillary carcinoma

      Cytology description
      • Increased cellularity, angulated clusters, abortive papillae and isolated malignant cells
      • Staghorn epithelial structures in 35%, giving serrated border (Pathology 2007;39:401)
      • May have few malignant appearing multinucleated giant cells or focal mucin (Acta Cytol 2009;53:463)
      • No fibrovascular cores (Cancer 2002;96:280)
      Positive stains
      Electron microscopy description
      • Microvilli on their cell membranes lining the outer surfaces of the cell clusters
      • Secretory activity in stroma facing surface of tumor cells
      Molecular / cytogenetics description
      Sample pathology report
      • Breast, right, 12:00 mass, core needle biopsy:
        • Invasive breast carcinoma with micropapillary component, preliminary Nottingham (or modified Bloom-Richardson) grade 2
        • Lymphovascular invasion is present
        • Comment: The tumor is ER positive (90%; 3+), PR positive (50%; 3+) and HER2 (score 2+) equivocal for overexpression.
        • Microscopic (optional): The sample shows a moderately differentiated carcinoma comprised of invasive ductal carcinoma and micropapillary carcinoma components.
          • ±% of subtype components: This may be of particular importance if micropapillary pattern makes up the majority (> 90%) of the biopsy sample and the diagnosis of pure micropapillary carcinoma is of consideration, pending full histologic evaluation at resection.
      Differential diagnosis
      Board review style question #1

      Which of the following is true regarding the pictured special subtype of breast carcinoma?

      1. Low stage disease is typical presentation
      2. Pure micropapillary carcinoma is common
      3. Tumor cells show circumferential HER2 staining
      4. Tumor cells show inside out pattern with EMA staining
      5. Tumor is frequently ER negative
      Board review style answer #1
      D. Tumor cells show inside out pattern with EMA immunostaining. The pictured lesion shows characteristic features of the micropapillary subtype of invasive breast carcinoma, which is frequently ER positive, with cup shaped basolateral HER2 immunostaining. Tumors often present at high stage with lymph node involvement. Tumor cells show reverse polarity, also known as inside out pattern with apical surface placed towards the stroma. Inside out pattern can be demonstrated by EMA staining of the apical surface.

      Comment Here

      Reference: Micropapillary carcinoma
      Board review style question #2
      The HER2 immunohistochemical staining pattern of tumor cells in micropapillary carcinoma of the breast is

      1. Apical
      2. Basal
      3. Basolateral
      4. Circumferential
      5. Cytoplasmic
      Board review style answer #2
      C. Basolateral. The basal and lateral surfaces of tumor cells in the micropapillary variant of invasive breast carcinoma frequently show membranous staining (cup shaped pattern). No staining is noted on the apical surface that stains with EMA.

      Comment Here

      Reference: Micropapillary carcinoma

      Mixed NST
      Definition / general
      • Mixture of ductal carcinoma NOS and lobular carcinoma
      Terminology
      • Not part of WHO breast classification
      Clinical features
      • Age of onset similar to infiltrating ductal carcinoma
      • Survival similar to pure ductal or lobular types
      • Compared to pure ductal carcinomas, have higher rate of second primary breast cancers (World J Surg Oncol 2010;8:51, Breast Cancer Res Treat 2009;114:243)
      • Compared to pure lobular carcinomas, have lower rate of synchronous contralateral breast cancer
      • 4% of breast carcinomas
      • 89% have DCIS, 31% have LCIS
      • 41% have positive lymph nodes at diagnosis
      • May have higher plasma levels of Soluble human leukocyte antigen s(HLA)-G (Anticancer Res 2012;32:1021)
      Microscopic (histologic) description
      • Definite features of invasive ductal carcinoma and invasive lobular carcinoma in same tumor
      Microscopic (histologic) images

      Images hosted on other servers:

      Mixed ductal / lobular tumor

      E-cadherin immunostaining of ductal components of tumor

      Positive stains
      Differential diagnosis
      • Tubulolobular carcinoma: typical areas of invasive lobular carcinoma with cords of single file cells, which merge with small round to angulated tubules with minute or undetectable lumina
      • Collision of two separate neoplasms

      Molecular subtypes
      Definition / general
      • Gene expression profiling and hierarchical cluster analyses have categorized breast cancer into 4 major intrinsic subtypes with distinct molecular characteristics, risk factors, treatment effectiveness and prognosis (Nature 2000;406:747, Proc Natl Acad Sci USA 2001;98:10869, J Pathol 2010;220:263, Cancer Res 2018;78:6011)
      • Traditional immunohistochemical biomarkers (ER, PR, HER2, Ki67 proliferative index) and pathologic features such as histological type and tumor grade serve as surrogates to classify tumors into molecular subtypes in clinical practice
      Essential features
      • Luminal subtype (subdivided into luminal A and luminal B)
        • Luminal A (ER positive, PR positive, HER2 negative, Ki67 proliferative index low)
        • Luminal B
          • HER2 negative (ER positive, PR negative / low, HER2 negative, Ki67 proliferative index high)
          • HER2 positive (ER positive, PR positive or negative, HER2 positive, Ki67 proliferative index varies)
      • Triple negative subtype (ER negative, PR negative, HER2 negative)
        • Umbrella term that includes basal-like (1 and 2), luminal androgen receptor positive, mesenchymal, mesenchymal stem-like and immunomodulatory
      • HER2 enriched (HER2 positive; ERBB2 positive)
        • ER negative, PR negative, HER2 / neu amplified or overexpressed
      • Other rare subtypes: claudin low, normal breast-like
      Terminology
      • Intrinsic subtypes, molecular classification of breast cancer
      Epidemiology
      • Frequency and lifetime risk by molecular subtype and age (Cancers (Basel) 2015;7:908, Surg Oncol Clin N Am 2018;27:95)
        • Lifetime risk
          • Luminal A and B: 6.79%
          • HER2 positive: 1.78%
          • Triple negative: 1.2%
        • Overall frequency
          • Luminal A: 30 - 40%
          • Luminal B: 20 - 30%
          • HER2 positive: 12 - 20%
          • Triple negative: 15 - 20%
        • Molecular subtype by age
          • < 40: triple negative > luminal B > HER2 > luminal A
          • 40 - 59: triple negative > luminal B > HER2 > luminal A
          • 60 - 69: luminal A > HER2 > luminal B > triple negative
          • > 70: luminal A > luminal B and HER2 > triple negative
      • Triple negative
      Pathophysiology
      • Luminal
      • Triple negative (Annu Rev Pathol 2022;17:181, Cell Stem Cell 2010;7:403)
        • Originally thought to arise from basal stem cells
        • Most likely cell of origin for nonmetaplastic triple negative breast cancer (TNBC) is an ER negative luminal progenitor cell
        • TNBC with metaplastic features likely arises from the myoepithelial progenitor cells
        • Defects in homologous recombination (i.e., double stranded DNA repair)
          • ~15% of all TNBCs harbor deleterious germline mutations in a homologous recombination deficiency (HRD) related gene (BRCA1, BRCA2, ATM, CHK2, RAD51, PALB2, BRIP1)
          • Nonobligate precursors
            • Low grade: microglandular adenosis (MGA) and atypical microglandular adenosis, which contain nearly identical copy number variants and TP53 somatic variants to those seen in high grade TNBC
            • High grade: ER negative ductal carcinoma in situ with basal-like phenotypes (~7% of all ductal carcinoma in situ)
      • HER2 enriched (Oncologist 2002;7:2)
        • HER2 is part of the epidermal growth factor (EGF) family, along with 3 other receptors: epidermal growth factor receptor HER1, HER2, HER3 and HER4
        • HER2 gene is located on the long arm of chromosome 17 and encodes a 185 kDa transmembrane protein and an activator of several downstream signaling cascades, including the MAPK and PI3K pathways
      Etiology
      • Risk factors by subtype (Cancer Res 2018;78:6011, Int J Epidemiol 2017;46:373, Breast Cancer Res 2019;21:40)
        • Luminal A and B
          • Nulliparity, alcohol consumption, early menarche, use of menopausal hormones, increased BMI / weight, older age at first birth, late menopause
            • For parous women, greater number of years between menarche and first birth
        • Triple negative
          • Ever parous (23% higher risk than nulliparous women), high BMI
          • Breastfeeding might ameliorate the higher risk of triple negative breast cancer associated with parity (Ann Oncol 2015;26:2398)
          • Disproportionally affects African American women and germline BRCA1 and PALB2
        • HER2 enriched
          • Risk factors are less well defined than luminal and triple negative subtypes
          • HER2 positive / enriched tumors overexpress HER2 and genes associated with the HER2 pathway or the HER2 amplicon on 17q12
        • All subtypes: family history of breast cancer, personal history of benign breast disease, smoking (women who initiate smoking before first childbirth)
      Diagnosis
      • Luminal (subdivided into luminal A and B)
        • Luminal A (ER positive, PR positive, HER2 negative, Ki67 proliferative index low)
        • Luminal B, HER2 negative (ER positive, PR negative / low, HER2 negative, Ki67 proliferative index high)
        • Luminal B, HER2 positive (ER positive, PR positive or negative, HER2 positive, Ki67 proliferative index varies)
      • Triple negative (ER negative, PR negative, HER2 negative)
        • Umbrella term that includes basal-like (1 and 2), luminal androgen receptor positive, mesenchymal, mesenchymal stem-like and immunomodulatory
      • HER2 enriched (ERBB2 positive)
        • ER negative, PR negative, HER2 amplified or overexpressed
          • Invasive breast tumors that are HER2 positive (amplification via FISH testing or overexpression via IHC)
      • Reference: J Pathol 2010;220:263
      Radiology description
      • Imaging findings do not distinguish between molecular subtypes; however
        • Luminal subtype is associated with architectural distortion
        • HER2 positive subtype is more likely to be associated with mammographic calcifications
        • Triple negative subtype is most likely to be associated with a mammographic mass (Ann Surg Oncol 2013;20:3247, Radiology 2008;246:367)
      Prognostic factors
      • Survival rate at 4 years from diagnosis (Cancer Epidemiol Biomarkers Prev 2018;27:619)
        • Luminal (92.5%) > luminal HER2 positive (90.3%) > HER2 enriched (82.7%) > triple negative (77.0%)
      • Local regional recurrence and distant metastasis rates by molecular subtype (Cureus 2016;8:e924)
        • Local regional
          • Luminal A: 2%
          • Luminal B: 4%
          • HER2 positive: 8%
          • Triple negative: 12%
        • Distant
          • Luminal A: 6.4%
          • Luminal B: 12.1%
          • HER2 positive: 19.2%
          • Triple negative: 27.4%
      • Luminal A
        • Generally low grade, indolent and slow growing, favorable prognosis
        • Less frequent and less extensive lymph node involvement
        • Risk of late recurrence and mortality after 10+ years from diagnosis
          • ~30% of ER positive patients will eventually develop anti-estrogen resistant metastatic or locally advanced breast cancer
      • Luminal B
        • More aggressive clinical behavior and worse prognosis compared to luminal A
        • Higher histologic grade than luminal A
        • Loss of PR expression adverse prognostic factor (J Clin Oncol 2013;31:203)
      • Triple negative
        • Aggressive phenotype with shorter time to relapse and metastasis compared to stage matched HR positive / HER2 positive tumors, despite more aggressive therapy (Breast J 2009;15:454)
        • African American women are twice as likely to die from triple negative breast cancer even when controlling for treatment, stage and socioeconomic factors (Cancer J 2021;27:8)
        • Of note, a subset of TNBC with distinct histomorphology is associated with significantly better prognosis (Am J Pathol 2017;187:2133)
          • Adenoid cystic carcinoma, secretory carcinoma, medullary carcinoma, low grade metaplastic carcinomas (adenosquamous carcinoma, fibromatosis-like metaplastic carcinoma)
      • HER2 enriched
      Treatment
      • Luminal
        • Standard of care surgery and adjuvant endocrine therapy with or without irradiation (depending on surgical, pathological and clinical factors)
        • Endocrine therapy for 5 - 10 years (tamoxifen, aromatase inhibitor) with or without ovarian suppression
        • Duration and choice of endocrine therapy is dependent upon menopausal status at time of diagnosis
        • Luminal tumors are less sensitive to chemotherapy compared to triple negative and HER2 enriched (pathologic complete response rates for neoadjuvant chemotherapy is 10 - 20% compared to 50 - 60%, respectively) (Lancet 2014;384:164)
        • Neoadjuvant endocrine therapy is not widely used (Ann Surg Oncol 2017;24:418)
          • Trials such as neoadjuvant aromatase inhibitor (NAOMI) therapy for ER positive breast cancer evaluate the effects of neoadjuvant aromatase inhibitor (letrozole) in postmenopausal women with stage I - III ER positive, HER2 negative breast cancer
        • The NCCN Guidelines allow for the use of breast conserving surgery (pathologically negative margin required) with 5 years of tamoxifen or an aromatase inhibitor, without breast irradiation, for patients over 70 years of age with clinically negative lymph nodes and ER positive T1 breast cancers
      • Triple negative
        • Surgery and chemotherapy with or without irradiation (depending on surgical, pathological and clinical factors)
        • Endocrine therapy and anti-HER2 therapies are ineffective due to absent expression of ER, PR and HER2
        • While surgery and postoperative radiation remains standard of care, chemotherapy, either in the neoadjuvant or adjuvant setting, is also recommended
          • Neoadjuvant therapy is preferred in individuals with ≥ cT2 or ≥ cN1 TNBC and can be considered in those with cT1c, cN0 TNBC as well
          • Pathologic complete response (pCR) is associated with extremely favorable disease free and overall survival (J Clin Oncol 2012;30:1796, Lancet 2014;384:164)
        • Preferred therapeutics include anthracyclines, taxanes, antimetabolites, microtubule inhibitors, platinum based therapy, cyclophosphamide
        • If recurrent / unresectable or metastatic, additional testing for
          • PDL1 expression (first line) variable
          • Germline BRCA1 / BRCA2 testing (second line) variable
          • Directed biomarkers (i.e., MSI high, TRK, mTOR inhibitors, RET, etc.) are warranted (Oncol Lett 2021;22:512, Int J Mol Sci 2022;23:1665)
          • If any are positive, the relevant targeted agents can be utilized in concert with the standard chemotherapy regimen
            • In PDL1 positive TNBC, pembrolizumab and chemotherapy significantly increased event free survival in high risk early stage TNBC patients (N Engl J Med 2020;382:810)
      • HER2 enriched
      Microscopic (histologic) description
      • Luminal
        • Invasive ductal carcinoma, no special type (IDC, NST)
        • Also tubular, mucinous, invasive lobular carcinoma (classic and variants), cribriform, invasive carcinoma with neuroendocrine differentiation, micropapillary, some apocrine / oncocytic carcinomas
          • Luminal A: IDC, NST, grade 1 or 2
            • ER positive, PR positive, HER2 negative, Ki67 low
          • Luminal B: IDC, NST, grade 2 or 3
            • ER positive, HER2 positive (Ki67 and PR can show any degree of expression)
            • ER positive, PR negative or low, HER2 negative, Ki67 high
      • Triple negative (J Pathol 2008;216:141, Breast Care (Basel) 2020;15:327, Semin Cancer Biol 2021;72:136)
        • ER negative, PR negative, HER2 negative, includes high and low grade tumors
          • High grade TNBC: IDC, NST grade 3, spindle cell metaplastic breast carcinoma, medullary carcinoma, apocrine carcinoma
          • Low grade TNBC: salivary gland-like tumors (secretory carcinoma, adenoid cystic carcinoma, acinic cell carcinoma, mucoepidermoid carcinoma), tall cell carcinoma with reverse polarity, low grade metaplastic carcinomas (adenosquamous carcinoma, low grade fibromatosis-like metaplastic carcinoma)
        • Broken down into basal-like type 1, basal-like type 2, mesenchymal and luminal androgen receptor
          • Debated distinct subtypes include immunomodulatory (IM) (high rates of tumor infiltrating lymphocytes [TILs]) and mesenchymal stem-like (MSL) (high levels of tumor associated mesenchymal tissue)
          • Immunomodulatory and mesenchymal stem-like characteristics that may be present in different degrees of all TNBC subtypes
        • Basal-like: high grade IDC, NST, solid tumor growth with pushing borders, geographic necrosis, nuclear pleomorphism, brisk inflammatory infiltrate, high mitotic index, high Ki67
          • Most express basal cytokeratins (keratin 5/6, 14, 17) and EGFR
        • Medullary (immunomodulatory), metaplastic (mesenchymal and mesenchymal stem-like), adenoid cystic, acinic cell, secretory carcinoma
        • Molecular apocrine: invasive lobular carcinoma (ILC) (pleomorphic), apocrine
          • Breast carcinomas with apocrine differentiation may cluster into several intrinsic subtypes, including luminal (ER positive), HER2 enriched and molecular apocrine (also known as luminal AR)
      • HER2 enriched
        • Predominantly IDC, NST of high histological grade
        • Tumor cells frequently have pleomorphic nuclei and more abundant eosinophilic cytoplasm than triple negative cancers
        • Ki67 proliferative rate is high (20 - 60%)
        • Frequently associated with high grade comedo ductal carcinoma in situ
        • Some pleomorphic invasive lobular carcinoma are HER2 positive
      Microscopic (histologic) images

      Contributed by Kristen E. Muller, D.O. and Evan R.J. Goyette, M.D.
      Luminal A1

      Luminal A

      Luminal A1

      ER positive

      High grade IDC NST High grade IDC NST

      Luminal B

      High grade tumor HER2 positive invasive ductal carcinoma

      HER2 positive


      TNBC basal-like TNBC basal-like

      TNBC basal-like

      TNBC basal-like TNBC basal-like

      TNBC basal-like

      CK5/6 expression

      CK5

      EGFR expression

      EGFR

      Positive stains
      Negative stains
      Molecular / cytogenetics description
      • Luminal A (Nature 2012;490:61)
        • PIK3CA (45%) > GATA3 (14%) > TP53 (12%)
        • Gains 1q and 16p, loss 16q
      • Luminal B (Nature 2012;490:61)
        • PIK3CA (29%) and TP53 (29%) > GATA3 (15%)
        • Gains 1q, 8q, 17q, 20q; losses 1p, 3q, 8p, 13q, 16q, 17p, 22q
      • Triple negative (J Clin Invest 2011;121:2750, Nature 2012;490:61, PLoS One 2016;11:e0157368, Semin Cancer Biol 2021;72:136, Annu Rev Pathol 2022;17:181)
        • TP53 (80%) > PIK3CA (9%) > GATA3 (2%)
        • Many gains and losses across the entire genome
        • Distinct gene profiles are associated with each TNBC subtype
          • Basal-like 1
            • Cell cycle and cell division pathways, BRCA1 / BRCA2 in minority
          • Basal-like 2
            • Growth factor signaling (EGFR, NGF, MET, Wnt / beta catenin)
          • Luminal androgen receptor positive
            • Steroid synthesis, androgen metabolism
          • Mesenchymal-like
            • Cell motility, cell differentiation
          • Mesenchymal stem-like
            • Cell motility, cell differentiation, genes associated with stem cells
            • Growth factor signaling pathways (EGFR / PDGF)
            • Low levels of claudin expression
            • Epithelial mesenchymal transition
          • Immunomodulatory
            • Immune cell processes (TH1 / TH2, NK cell, B, T cell, antigen processing and presenting cytokine signaling)
      • HER2 enriched (Nature 2012;490:61)
        • TP53 (72%) > PIK3CA (39%) > GATA3 (2%)
        • Many gains and losses across entire genome
        • HER2 / neu gene is a member of a family of genes encoding transmembrane receptors for growth factors, including epidermal growth factor receptor (EGFR), HER2, HER3 and HER4 (N Engl J Med 2005;353:1652)
        • Amplification of the HER2 gene (ERBB2 gene) at the DNA level results in protein overexpression (Science 1989;244:707)
      Board review style question #1

      The breast tumor pictured above shows strong expression of estrogen receptor (ER) and progesterone receptor (PR) (both > 90%) and is negative for HER2 / neu amplification. What is the most likely molecular subtype?

      1. Basal-like
      2. HER2 enriched
      3. Luminal A
      4. Luminal B
      Board review style answer #1
      C. Luminal A. Invasive breast tumors classified as luminal A by gene expression profiling are typically low grade invasive ductal carcinomas. By definition, the luminal A intrinsic subtype shows strong expression of hormone receptors (ER and PR) and is negative for HER2 / neu overexpression and amplification. Special histologic subtypes that cluster with the luminal A subtype include tubular, mucinous and cribriform carcinomas. Answers A, B and D are incorrect because basal-like, HER2 enriched and luminal B tumors are characteristically high grade.

      Comment Here

      Reference: Molecular subtypes
      Board review style question #2

      A 67 year old woman underwent a partial mastectomy for a breast tumor shown in the image. Microscopic sections revealed a high grade invasive ductal carcinoma growing as a solid mass with pushing borders, areas of necrosis, brisk mitoses and abundant tumor infiltrating lymphocytes. The tumor was negative for ER, PR and HER2 and was classified as basal-like by gene expression profiling. What other immunostain(s) might be positive?

      1. Androgen receptor (AR)
      2. EGFR
      3. KIT
      4. Synaptophysin
      Board review style answer #2
      B. EGFR. Basal-like breast cancers are typically high grade invasive ductal carcinoma of no special type that show solid tumor growth with pushing borders, geographic necrosis, nuclear pleomorphism, a brisk inflammatory infiltrate, a high mitotic index and a high proliferative index with Ki67. Most basal-like breast cancers express basal cytokeratins (keratin 5/6, 14, 17) and EGFR. Answer A is incorrect because while a minor proportion of triple negative breast cancers can express androgen receptor, the prototypical AR positive breast tumor has apocrine features (apocrine carcinoma). Answer C is incorrect because KIT is not characteristically expressed in breast carcinomas. Answer D is incorrect because synaptophysin may be expressed in breast tumors with neuroendocrine differentiation, which typically cluster within the luminal category.

      Comment Here

      Reference: Molecular subtypes

      Mucinous
      Definition / general
      • Breast neoplasm with mucinous component that comprises > 90% of tumor, usually with favorable prognosis
      Essential features
      • Rare tumor occurring in older women
      • Epithelial clusters that lie within secreted mucin (type A) or large sheets of tumor cells with neuroendocrine features and mucin production (type B)
      • Mucinous component must be > 90% of tumor to be pure and have favorable prognosis
      • ER and PR hormone receptor positive and HER2 negative (luminal A subtype)
      Terminology
      • Colloid carcinoma
      • Mucinoid carcinoma
      • Gelatinous carcinoma
      • Mucoid carcinoma
      • Mucinous adenocarcinoma
      ICD coding
      • ICD-O: 8480/3 - mucinous adenocarcinoma
      • ICD-11: 2C60 - carcinoma of breast, specialized type
      • ICD-11: XH1S75 - mucinous adenocarcinoma
      Epidemiology
      • < 5% of breast carcinomas have mucinous component; of these, only 2% are pure mucinous carcinoma
      • F > M, sixth through eighth decades
      Sites
      • Most are unifocal and can occur in any quadrant
      Pathophysiology
      • Occurs through ER positive pathway, though lack gains of 1q, loss of 16q and PIK3CA mutations, which are typical of low grade neoplasms (Breast 2020;49:87)
      • Lesions express hormone receptors while lacking HER2 and basal markers
      Etiology
      • Multifactorial, with factors including hormones (early menarche, nulliparity or exogenous hormone use), genetics and diet
      Clinical features
      • Imaging can vary from a well circumscribed mass (more likely to be pure) to an irregular, spiculated mass (more likely to have mixed mucinous component) (Breast 2020;49:87)
      • Due to potential circumscription seen on imaging, may be mistaken for benign process
      Diagnosis
      Radiology images

      Contributed by Carissa LaBoy, M.D.
      Mammogram of right breast

      Mammogram of right breast

      Ultrasound of left breast

      Ultrasound of left breast

      Mammogram of breast mass

      Mammogram of breast mass

      Mammogram of left breast

      Mammogram of left breast

      Prognostic factors
      • Prognosis dependent on extent of mucinous involvement; pure mucinous carcinoma has favorable prognosis with 10 year survival rate of 90.4% (Breast 2020;49:87)
      • No prognostic significance between type A and type B
      • Patients tend to have localized disease with low recurrence rate and rare involvement of axillary lymph nodes (Breast 2020;49:87)
      • Component of micropapillary pattern is associated with worse prognosis, with increased rate of lymph node metastasis (Breast J 2018;24:339)
      • Acellular mucin seen in postneoadjuvant chemotherapy specimens does not correlate with residual disease
      • Only when residual neoplastic epithelium is identified is the patient classified as partial response and staged with residual disease by measuring the extent of extracellular mucin (Histopathology 2018;72:965)
      Case reports
      Treatment
      • Surgical excision and adjuvant hormone therapy
      • Cases associated with worse prognosis, such as micropapillary variant, may warrant neoadjuvant chemotherapy (Breast J 2018;24:339, Histopathology 2018;72:965)
      • HER2 positivity is associated with tamoxifen resistance and should be treated with chemotherapy, anti-HER2 therapy and endocrine therapy via an aromatase inhibitor (Medicine (Baltimore) 2020;99:e20996)
      Gross description
      • Well circumscribed mass of variable size (from < 1 cm to > 20 cm) with gelatinous cut surface
      Gross images

      Contributed by Mark R. Wick, M.D.
      Pure Pure

      Pure mucinous carcinoma



      Images hosted on other servers:
      Gross mass in the breast Gross mass in the breast

      Gross mass in the breast

      Microscopic (histologic) description
      • Clusters / nests of tumor cells with low or intermediate nuclear grade floating in pools of extracellular mucin
      • Mucin pools separated by fibrous septa with capillaries
      • Considered pure when mucinous component comprises > 90% of the tumor, which is associated with favorable prognosis
      • Otherwise, tumor is mixed (mucinous component comprises 50 - 90% of tumor) or has mucinous features (mucinous component comprises < 50% of tumor), both of which have less favorable prognosis
      • Capella type A: abundant extracellular mucin production with scattered small epithelial clusters, strips or cribriform structures floating in pools of mucin
      • Capella type B: large sheets of tumor cells with mucin production and neuroendocrine features
      • Micropapillary pattern
        • Clusters of tumor cells with intermediate to high grade nuclei, occasional hobnailing and reverse polarity; more likely to be HER2 positive, which in turn has a worse prognosis (more likely to have lymph vascular space invasion and metastasis to lymph nodes) (Breast J 2018;24:339)
        • PIK3CA and TP53 mutations more frequent with recurrent gains of 8q (Histopathology 2019;75:139)
      Microscopic (histologic) images

      Contributed by Carissa LaBoy, M.D., Mark R. Wick, M.D. and AFIP images
      Type A mucinous carcinoma

      Type A mucinous carcinoma

      Type A ER positive

      Type A ER positive

      Type A PR positive

      Type A PR positive

      Type A HER2 negative

      Type A HER2 negative

      Clusters in mucins pools

      Clusters in mucin pools

      Bland epithelial cell clusters

      Bland epithelial cell clusters


      Strips of epithelium

      Strips of epithelium

      Type B mucinous carcinoma

      Type B mucinous carcinoma

      Neuroendocrine features

      Neuroendocrine
      features

      Grimelius stain

      Grimelius staining denotes neuroendocrine differentiation Grimelius staining denotes neuroendocrine differentiation

      Grimelius stain

      Virtual slides

      Images hosted on other servers:

      Type A, tumor clusters

      Type B, neuroendocrine features

      Cytology description
      • Small to intermediate sized cohesive groups of epithelial cells with low grade nuclei floating within pools of extracellular mucin
      • Mucinous carcinoma type B displays neuroendocrine differentiation in discohesive clusters with plasmacytoid cells with low grade nuclei (Cytopathology 2016;27:193)
      • Low cellularity coupled with bland nuclear features and potentially scant background mucin can make the FNA diagnostically challenging (Arch Pathol Lab Med 2011;135:1533)
      Cytology images

      Images hosted on other servers:

      FNA of mucinous carcinoma

      Cytology smear of mucinous carcinoma

      Cytospin preparation

      Colloid carcinoma

      Negative stains
      Electron microscopy images

      AFIP images

      Cytoplasmic mucigen granules

      Molecular / cytogenetics description
      Videos

      Review of mucinous carcinoma

      Sample pathology report
      • Left breast, needle localized lumpectomy:
        • Invasive mucinous carcinoma, grade 1 (1+2+1), measuring 1.4 cm in greatest dimension (see synoptic report)
        • Margins negative with invasive carcinoma > 0.5 cm from all margins
      Differential diagnosis
      • Mucocele-like lesion:
        • Ruptured cyst with detached strips of bland epithelium lined by myoepithelial cells floating within extravasated mucin
      • Mucinous cystadenocarcinoma:
      • Invasive micropapillary carcinoma:
        • Nests of tumor cells floating in spaces, mucin production absent
      • Metastasis:
        • Correlate with clinical and radiologic history, such as colorectal, lung and gynecologic
        • Expression profile dependent on origin
      • Solid papillary carcinoma:
        • Nests of cells with neuroendocrine features, fibrovascular cores and possible mucin production; may potentially be precursor to mucinous carcinoma
      • Invasive ductal carcinoma, no special type, with mucinous features:
        • Histologic features are those of pure mucinous carcinoma but mucinous component does not fulfill > 90% criteria
        • May be initial diagnosis on core needle biopsy for pure mucinous carcinoma
        • Due to limited material sampled on core needle biopsy, amount of mucinous component is more reliably assessed on excision specimen
      Board review style question #1

      What is the most likely hormone receptor profile of the above breast lesion?

      1. ER+, PR+, HER2-
      2. ER-, PR-, HER2-
      3. ER+, PR+, HER2+
      4. ER-, PR+, HER2+
      Board review style answer #1
      A. ER+, PR+, HER2-

      Mucinous carcinomas are ER+, PR+ and largely HER2-, making them a luminal A lesion, which is associated with a favorable prognosis.

      Comment Here

      Reference: Breast - Mucinous
      Board review style question #2

      What percentage of a breast carcinoma must have a mucinous component to be considered pure mucinous carcinoma?

      1. 75%
      2. 50 - 90%
      3. > 90%
      4. 30%
      Board review style answer #2
      C. > 90%

      To be considered a pure mucinous carcinoma with a favorable prognosis, the mucinous component must comprise > 90% of a breast carcinoma. If it comprises 50 - 90% of the tumor, it is termed mixed. If the mucinous component is < 50%, the tumor is termed invasive ductal carcinoma with mucinous features. The latter two diagnoses do not have as favorable a diagnosis.

      Comment Here

      Reference: Mucinous carcinoma

      Mucinous cystadenocarcinoma
      Definition / general
      • Very rare as primary breast tumor (< 50 cases reported)
      • Carcinoma composed of tall columnar cells with basal, bland nuclei and abundant intracytoplasmic and extracytoplasmic mucin (as opposed to mucinous / colloid carcinoma with only extracellular mucin, and signet ring cell carcinoma with only intracellular mucin)
      • Either grossly cystic (mucinous cystadenocarcinoma) or solid (columnar cell mucinous carcinoma)
      • Resembles similar tumors of ovary, pancreas and appendix (Am J Surg Pathol 1998;22:698)
      Epidemiology
      • Mean age 58 - 68 years, range 49 - 96 years
      Case reports
      Gross description
      • Multicystic, up to 19 cm
      • Resembles ovarian and pancreatic cystadenocarcinoma, cystic hypersecretory carcinoma of breast
      Gross images

      Images hosted on other servers:

      Ovary - multicystic tumor

      Microscopic (histologic) description
      • Cystic spaces lined by bland, tall columnar cells with abundant intracytoplasmic mucin
      • Tumor cells exhibit stratification, tufting and papillary formation
      • Variable cytologic atypia
      • Gradual loss of intracytoplasmic mucin and transformation to eosinophilic squamoid cells that are focally invasive
      • Columnar cell variant has round and convoluted glands in loose aggregates, lined by tall columnar mucinous epithelium with basal bland nuclei
      • Demonstrates both intracellular and extracellular mucin
      Microscopic (histologic) images

      Images hosted on other servers:

      Mucinous cystadenocarcinoma of breast with estrogen receptor expression

      Retroperitoneal tumor

      Positive stains
      Negative stains
      Differential diagnosis

      Mucocele-like lesion
      Definition / general
      • Rare lesion characterized by dilated epithelium lined ducts filled with mucin; associated with extravasation of acellular mucin into the stroma
      • Cysts lined by flat or low cuboidal epithelium
      Essential features
      • Characterized by extravasated acellular mucin in periductal stroma
      • Although there is variable associated hyperplasia of the cyst lining in mucocele-like lesions, there are no epithelial cells floating within the luminal or extravasated mucin, which is a critical finding in distinguishing mucocele-like lesions from mucinous carcinoma of the breast
      • Although originally described as a benign lesion, associations with atypical ductal hyperplasia (ADH), ductal carcinoma in situ (DCIS) and invasive carcinoma have been reported in several studies
      • Searching for atypia when a mucocele-like lesion is present is important to exclude possibility of ADH, DCIS or invasive carcinoma
      • Pure mucocele-like lesions without atypia typically have benign behavior
      Terminology
      • Mucocele-like lesion (MLL), mucocele-like tumor
      Epidemiology
      Sites
      • Anywhere in the breast
      Etiology
      • Pathogenesis is unclear but excessive mucinous secretions or ductal obstruction may be responsible
      Clinical features
      Diagnosis
      • Diagnosis can be made on core biopsy or surgical specimen but careful evaluation of excised tissue, multiple H&E levels and clinicopathologic correlation may be helpful to rule out in situ lesions and invasive mucinous carcinoma
      Radiology description
      Radiology images

      Images hosted on other servers:
      Microcalcifications

      Microcalcifications

      Ultrasound findings of mucocele-like lesion with focal atypical proliferation

      Mucocele-like lesion with focal atypical proliferation

      Pleomorphic calcifications

      Pleomorphic calcifications

      Sonogram of palpable mass showing tubular cystic structures

      Palpable mass with tubular cystic structures

      Prognostic factors
      Case reports
      Treatment
      Microscopic (histologic) description
      • Cysts and dilated ducts filled with mucin
      • Mucin containing cysts that often rupture with extravasation of mucin into surrounding stroma
      • Myoepithelial cells adhere to strips of cells floating in lakes of mucin
      • Calcifications are often present
      • Epithelium lining the cysts may show typical and atypical proliferative changes including benign / flat, hyperplasia, ADH, DCIS or mucinous carcinoma
      • Microscopic examination of the entire specimen is important to rule out any atypia / malignancy
      Microscopic (histologic) images

      Contributed by Cansu Karakas, M.D.
      Mucin filled cyst

      Mucin filled cyst

      Multiple cysts

      Multiple cysts

      Coarse calcifications

      Coarse calcifications

      Extravasation of mucin

      Extravasation of mucin

      Cytology description
      • Poorly cellular with cohesive clusters of bland cells in 2 dimensional sheets with abundant mucoid background, no / rare intact single tumor cells, no atypia if mucocele only (Am J Surg Pathol 1999;23:552, Am J Clin Pathol 1991; 95:875)
      • Excisional biopsy is necessary to confirm diagnosis (Breast Cancer 2009;16:77)
      • Most important features favoring a benign mucocele-like lesion over mucinous carcinoma on FNA are:
        • Younger patient
        • Cells arranged in cohesive monolayers
        • No significant nuclear atypia
        • Scant cellularity
        • No or rare single, intact tumor cells
      • Mucinous carcinomas are usually more cellular with more single tumor cells, 3 dimensional clusters, mild / moderate nuclear atypia and a solid mass by imaging (Cytopathology 2004;15:104, Acta Cytologica 2000;44:765)
      Positive stains
      Sample pathology report
      • Left breast, upper outer quadrant, core needle biopsy:
        • Mucocele-like lesion
      Differential diagnosis
      • Cystic mastopathy:
        • Associated with prominent apocrine differentiation
      • Florid duct ectasia with luminal mucin:
        • Generally contains lipid rich material within ducts with prominent foamy histiocytes
      • Mucinous carcinoma:
        • Prominent luminal cell proliferation and variable number of tumor cells floating within the mucin
        • Luminal cells in mucin are not associated with myoepithelial cells
      • Nodular mucinosis:
        • Rare lesion with accumulation of stromal mucin, typically located under or adjacent to nipple
        • Can be distinguished from mucocele-like lesions by location and staining feature of mucous substance
      Board review style question #1

      Which of the following statements is true about mucocele-like lesions of the breast?

      1. Frequently found in a retroareolar location
      2. Mammographically they usually form a palpable mass
      3. Most of the patients are elderly
      4. Usually benign, although they can be associated with atypia or malignancy
      Board review style answer #1
      D. Mucocele-like lesions are typically benign, although they can be associated with atypia or malignancy. Mucocele-lesions without atypia or carcinoma are benign lesions. The other statements are false. They can arise anywhere in the breast. They commonly present with calcifications with variable circumscribed mass on mammography. There is a wide range at presentation with mean age of 40.

      Comment Here

      Reference: Mucocele-like lesion (MLL) of breast
      Board review style question #2

      A 35 year old woman had a mammogram showing polymorphous, grouped microcalcifications. The histologic details of the core needle biopsy are shown in the image above. Which of the following statements are true about this entity?

      1. CK5/6 and p63 are negative
      2. Likelihood of carcinoma is very high
      3. Mucicarmine staining is negative
      4. When there is a rupture to stroma, the possibility of mucinous carcinoma must always be considered
      Board review style answer #2
      D. When there is a rupture to stroma, the possibility of mucinous carcinoma must always be considered. A mucocele-like lesion may show CK5/6 and p63 expression and highlight the myoepithelial cells surrounding the benign cystic lining of mucocele-like lesion. Mucin in the cyst and stroma usually show strong and diffuse mucicarmine staining in mucocele-like lesion. Mucocele-like lesion without associated carcinoma is a benign lesion.

      Comment Here

      Reference: Mucocele-like lesion (MLL) of breast

      Mucoepidermoid
      Terminology
      Clinical features
      Case reports
      Microscopic (histologic) description
      • Resembles salivary gland counterpart (Virchows Arch 2004;444:13)
      • Mixture of neoplastic mucus secreting, squamous and intermediate cells
      Microscopic (histologic) images

      Images hosted on other servers:

      In breast affected by burn scars

      Salivary gland tumors: H&E and Alcian blue - PAS

      Cytology description
      High grade tumors:
      • Clusters of epithelial ductal cells with mixed glandular, squamous and intermediate cells
      • May be scant intra- and extracellular mucin (Acta Cytol 2006;50:344)
      Positive stains
      Negative stains
      Electron microscopy description
      Differential diagnosis

      Multigene products
      Definition / general
      Essential features
      • New AJCC (8th edition) incorporates multigene panel results into staging, placing T1 and T2 hormone receptor positive, HER2- and lymph node negative tumors with low risk scores into the T1a-T1bN0M0 prognostic category
      • Multigene panels are not currently incorporated into staging for triple negative or HER2+ tumors
      • Use of these assays is not required for staging
      • Oncotype DX, with Level 1 evidence, is the only assay incorporated into the Pathological Prognostic Stage Table
      • No one multigene assay is preferred by the AJCC over another assay for treatment decisions
      Oncotype Dx®
      • ER+, HER2 - tumors, can have up to 3 involved lymph nodes
      • Most widely accepted and used multigene expression assay; offered by Genomic Health
      • First available in January 2004
      • Measures changes in 21 genes on paraffin fixed tissue to predict likelihood of disease recurrence and which patients are most likely to respond to chemotherapy (Breast Cancer Res 2006;8:R25)
      • Results in recurrence score (RS) of 0 - 100, categorized into low (< 11), intermediate (11 - 25) or high (> 25) recurrence risk
      • TAILORx trial found that endocrine therapy was "noninferior" (comparable) to chemotherapy plus endocrine therapy for disease free survival in up to 85% of women with hormone receptor positive, early invasive breast cancer
      • Particularly relevant for women > 50 years old with a recurrence score ≤ 25 as they may be spared adjuvant chemotherapy (N Engl J Med 2018;379:111)
      Breast Cancer Index
      • 11 gene assay including 4 normalization genes; offered by BioTheranostics
      • Results are predictive for benefit of extended endocrine therapy and prognostic for the risk of late (> 5 years) and overall (0 - 10 years) distant recurrence following 5 years of endocrine therapy
      • ER+ tumors can have up to 3 involved lymph nodes
      • Breast Cancer Index prognostic score reported as continuous percentage risk of late distant recurrence (> 5 years) and Breast Cancer Index predictive score reported as categorical (low versus high) likelihood of benefit from extended endocrine therapy
      • Breast Cancer Index scores resulted in a 26% net decrease in the recommendation of extended endocrine therapy (Breast Cancer Res Treat 2015;154:533)
      EndoPredict®
      • 12 gene expression assay, including 3 proliferation related genes, 5 hormone related, 3 normalization genes and 1 control gene for DNA contamination (Br J Cancer 2015;112:1405)
      • ER+, HER2- tumors, can have up to 3 involved lymph nodes
      • EPclin Risk Score integrates the 12 gene molecular score with tumor size and nodal status to categorize patients into low or high risk of distant recurrence within 10 years if treated with 5 years of endocrine therapy alone
      MammaPrint®
      • 70 gene assay, originally developed in 2002 to determine risk of distant metastases (Nature 2002;415:530); offered by Agendia
      • Originally FDA approved in 2007, requiring fresh tissue but later FDA approved in 2015 for formalin fixed paraffin embedded tissue
      • No estrogen or HER2 status restriction
      • Results in binary stratification into low risk or high risk of cancer recurrence within 10 years without any adjuvant treatment (either hormonal therapy or chemotherapy)
      • MINDACT trial found that 46% of women with early stage breast cancer at high clinical risk (defined as 10 year probability of breast cancer specific survival without systemic therapy of more than 88% for ER+ and more than 92% for ER- tumors as calculated by Adjuvant! Online version 8.0 [website currently under construction]) and low genomic risk for recurrence (as determined by MammaPrint low versus high risk result) may be spared adjuvant chemotherapy (N Engl J Med 2016;375:717)
      PAM 50 (Prosigna)®
      • Developed by NanoString, the assay measures expression profiles of 50 genes by PCR and classifies them into intrinsic subtypes (luminal A, luminal B, HER2 enriched, basal-like)
      • Results in risk of recurrence score (ROR) of 0 - 100 that correlates with probability of distant recurrence within 10 years (BMC Med Genomics 2012;5:44, J Clin Oncol 2013;31:2783)
      • Can be used on stage 1 or stage 2 and lymph node negative for stage 2 with 1 to 3 positive nodes
      • Node negative cancers are classified as low (0 - 40), intermediate (41 - 60) or high (61 - 100) risk
      • Node positive cancers are classified as low (0 - 40) or high (41 - 100) risk
      • Is incorporated into the Prosigna Breast Cancer Prognostic Gene Signature Assay (BMC Med Genomics 2015;8:54)
      Mammostrat
      • Developed by Clarient Diagnostic Services
      • IHC evaluation of the expression of five genes (p53, NDRG1, CEACAM5, SLC7A5 and HTF9C) in early stage, hormone receptor positive breast cancers to predict outcomes (J Clin Oncol 2006;24:3039)
      • Subdivides patients into low, moderate and high risk, age specific groups
      IHC4
      Prognostic factors
      • T1 and T2, hormone receptor-positive, HER2- and lymph node negative tumors with any of the following multigene panel results:
        • Oncotype Dx® recurrence score less than 11 (Level 1 Evidence)
        • Breast Cancer Index in the low risk range (Level 2 Evidence)
        • EndoPredict® low risk score (Level 2 Evidence)
        • Mammaprint® low risk score (Level 2 Evidence)
        • PAM 50® risk of recurrence score in the low range (Level 2 Evidence)
      • Are placed into the same prognostic category as T1a-T1bN0M0 tumors
      Board review style question #1
        Which multigene test has been integrated into the 8th edition of AJCC staging system for breast cancer with Level 1 evidence?

      1. Oncotype Dx
      2. Breast Cancer Index
      3. Endopredict
      4. Mammaprint
      5. PAM 50
      Board review style answer #1
      A. The 21 - gene recurrence score from Oncotype Dx is incorporated into the 8th edition of AJCC staging for breast cancer based on Level 1 Evidence (large scale prospective clinical trial published data) indicating that for patients with ER positive, lymph node negative, tumors with a recurrence score < 11, prognosis is excellent.

      Comment Here

      Reference: Multigene products

      Myoepithelioma and myoepithelial carcinoma
      Definition / general
      • Myoepithelioma: bland spindle cell lesion with myoepithelial phenotype
      • Myoepithelial carcinoma: very rare malignant lesion, mostly arising in association with adenomyoepitheliomas, composed of purely malignant myoepithelial cells, primarily spindle shaped to polygonal
      Essential features
      • Myoepithelioma: benign lesion comprised of cells of myoepithelial derivation
      • Myoepithelial carcinoma: rare, malignant tumor of the breast comprised of cells of myoepithelial derivation
      Terminology
      ICD coding
      • ICD-O: 8982/3 - malignant myoepithelioma
      • ICD-11:
          • Myoepithelioma:
            • 2F30.Y - other specified benign neoplasm of breast
            • 2F30.Z - benign neoplasm of breast, unspecified
          • Myoepithelial carcinoma:
            • 2C6Z - malignant neoplasms of breast, unspecified
            • 2C6Y & XH43E6 - other specified malignant neoplasms of breast & myoepithelial carcinoma
            • XH43E6 - myoepithelial carcinoma
      Epidemiology
      • Myoepithelioma: premenopausal and postmenopausal women; wide age range
      • Myoepithelial carcinoma:
        • Incidence is 15 per 100,000 population
        • Mean age for myoepithelial carcinoma in breast: 50 years; range 45 - 86 years (Transl Cancer Res 2017;6:441)
      Sites
      • Breast, without specific site predilection
      Pathophysiology
      • Tumors that arise from the peripheral myoepithelial cell layer
      Etiology
      • Unknown
      Clinical features
      • Myoepithelioma is benign but may recur locally
      • Myoepithelial carcinoma may recur or metastasize to regional lymph nodes or distant sites (Eur J Surg Oncol 2004;30:357)
      Diagnosis
      • Histologic evaluation of tissue, typically with assistance of immunohistochemistry
      Radiology description
      • Mammography: circumscribed or irregular, noncalcified mass
      • Ultrasound: hypoechoic, irregular or oval mass with microlobulated or indistinct margins
      • MRI: enhancing mass with an irregular or spiculated margin and delayed washout or plateau kinetics
      • References: Breast J 2007;13:203, Taehan Yongsang Uihakhoe Chi 2020;81:207
      Prognostic factors
      • Myoepithelioma: can recur locally
      • Myoepithelial carcinoma (Eur J Surg Oncol 2004;30:357):
        • Associated with aggressive course, including recurrence and metastasis
        • 2 and 5 year survival is 88% and 55%
        • Increased tumor size is associated with worse outcome
      Case reports
      Treatment
      • Myoepithelioma: complete surgical excision (J Cytol 2013;30:62)
      • Myoepithelial carcinoma:
        • Wide local excision, segmental mastectomy or total mastectomy with regional lymph node excision and radiotherapy (Mol Clin Oncol 2016;4:723)
        • Role of chemotherapy is not well defined
        • Platinum analogs and paclitaxel combination has shown some efficacy in few cases reported (Oncologist 2016;21:1492)
      Gross description
      Microscopic (histologic) description
      Microscopic (histologic) images

      Contributed by Julie M. Jorns, M.D. and Koteeswaran Govindaswamy, M.D. (Case #407)
      Myoepithelioma with adjacent benign glands

      Myoepithelioma with adjacent benign glands

      Myoepithelioma

      Myoepithelioma

      Myoepithelial carcinoma Myoepithelial carcinoma Myoepithelial carcinoma

      Myoepithelial carcinoma


      Myoepithelioma - p63

      Myoepithelioma - p63

      Tumor cells positive for p63 IHC

      Myoepithelial carcinoma - p63

      Tumor cells positive for CK 5/6 IHC

      Myoepithelial carcinoma - CK 5/6

      Tumor cells positive for pan CK IHC

      Myoepithelial carcinoma - pan CK

      Tumor cells positive for Ki67 IHC with high Ki67 index (approx. 15-20/hpf)

      Myoepithelial carcinoma - Ki67 IHC with high Ki67 index


      Tumor cells are negative for ER, positive for PR and negative for Her2Neu

      Myoepithelial carcinoma - tumor cells are negative for ER, positive for PR and negative for Her2Neu

      Cytology description
      Cytology images

      Images hosted on other servers:

      Myoepithelioma FNA

      Positive stains
      Negative stains
      Electron microscopy description
      Molecular / cytogenetics description
      Molecular / cytogenetics images

      Images hosted on other servers:

      Summary karyogram 10 cases

      Sample pathology report
      • Left breast, core biopsy:
        • Spindle cell neoplasm (see comment)
        • Comment: Histologic sections show a spindle cell neoplasm arranged in storiform pattern. The tumor cells have vesicular nuclei, prominent nucleoli, variable nuclear atypia and eosinophilic cytoplasm. Mitoses are readily identifiable (with up to 4 mitoses/10 high power fields). Focal areas of cystic degeneration and necrosis are seen. Immunohistochemistry shows tumor cells to be positive for p63 and cytokeratins 5/6 and 903 and negative for cytokeratin 7, desmin, ER, PR and HER2 / neu. Overall, findings are suggestive of metaplastic carcinoma, spindle cell type or myoepithelial carcinoma.
      Differential diagnosis
      • Metaplastic carcinoma:
        • Variable group of breast carcinomas with mesenchymal differentiation
        • Myoepithelial carcinoma is considered a subtype by WHO
      • Spindle cell sarcoma:
        • Negative for myoepithelial markers
      • Leiomyosarcoma:
        • Blunt ended nuclei with abundant cytoplasm
        • Positive for desmin
      • Fibromatosis:
        • Infiltrative fibroblastic and myofibroblastic proliferation
        • Minimal pleomorphism with no mitosis
        • Positive for actin, desmin and S100
        • Negative for cytokeratins
      • Adenomyoepithelioma:
        • Biphasic, circumscribed tumor composed of variable number of myoepithelial cells surrounding epithelial lined spaces
      • Myofibroblastoma:
        • Cellular, collagenized, epithelioid, palisaded, lipomatous and hemangiopericytoma-like, with infiltrative features
        • Positive for vimentin, CD34 and desmin
      Board review style question #1

      Biopsy of a breast nodule showed the histology in the image above. All tumor cells are strongly and diffusely positive for p63. What is the most likely diagnosis?

      1. Adenoid cystic carcinoma
      2. Adenomyoepithelioma
      3. Myoepithelial carcinoma
      4. Myoepithelioma
      Board review style answer #1
      C. Myoepithelial carcinoma. The biopsy shows an atypical proliferation with plump spindle cells and readily identifiable mitoses. p63 positivity supports myoepithelial differentiation. The morphology is malignant, arguing against adenomyoepithelioma and myoepithelioma. Adenomyoepithelioma and adenoid cystic carcinoma additionally have a biphasic composition, making these answers incorrect. In this case, tumor cells are of uniform myoepithelial differentiation, making the best diagnosis myoepithelial carcinoma, which is considered a metaplastic carcinoma of no special type by the WHO.

      Comment Here

      Reference: Myoepithelioma and myoepithelial carcinoma

      Myofibroblastoma
      Definition / general
      • Benign spindle cell tumor of mammary stroma composed of fibroblasts and myofibroblasts with recurrent monoallelic loss of 13q14 region including RB1
      • First recognized in 1981 and described by Wagotz in 1987 (Am J Surg Pathol 1987;11:493)
      • When present in mesenchymal soft tissue, termed mammary type myofibroblastoma
      • Rarely involves the lower genital tract, predominantly vagina (less commonly vulva and cervix)
      Essential features
      • Well circumscribed spindle cell tumor comprised of short fascicles of bland spindled cells and dense hyalinized collagen bundles
      • Variants include collagenized / fibrous, cellular, infiltrative, myxoid, deciduoid, epithelioid, lipomatous and atypical
      • Part of 13q / Rb family of tumors, as majority show deletion or rearrangement of 13q (13q14), which may result in loss of Rb expression by immunohistochemistry
      • Benign and adequately treated by excision alone
      Terminology
      • Mammary type myofibroblastoma
      • Benign stromal spindle cell tumor with predominant myofibroblastic differentiation
      ICD coding
      • ICD-O: 8825/0 - myofibroblastoma
      • ICD-10: D24.9 - benign neoplasm of unspecified breast
      • ICD-11: 2F30 & XH3NQ0 - benign neoplasm of breast & myofibroblastoma
      Epidemiology
      Sites
      Pathophysiology
      Etiology
      Clinical features
      • Painless, nontender, slow growing, mobile mass
      • Clinically mistaken for fibroadenoma
      Diagnosis
      • Imaging: ultrasound, mammogram, MRI
      • Invasive procedure: biopsy, fine needle aspiration (FNA)
      Radiology description
      Radiology images

      Contributed by Kristen E. Muller, D.O.

      Ultrasound, well circumscribed mass

      Ultrasound, multilobulated mass

      Mammogram

      Mammogram, multiple masses

      Prognostic factors
      • Clinically benign behavior, local excision is curative
      • No tendency for recurrence or distant metastasis (Virchows Arch 2002;440:249)
      Case reports
      Treatment
      • Surgical excision alone
      Clinical images

      Images hosted on other servers:

      Myofibroblastoma in gynecomastia

      15 cm tumor

      Gross description
      Gross images

      Images hosted on other servers:

      13 cm, lobulated
      white cut surface

      Microscopic (histologic) description
      • Well circumscribed, lacks true capsule, rarely infiltrative
      • Bland, uniform, short to elongated spindle cells arranged as short haphazard intersecting fascicles admixed with bands of hyalinized, brightly eosinophilic collagen and variable amounts of fat
      • No more than mild nuclear atypia
      • Mitoses usually absent, atypical mitoses and necrosis absent
      • Mast cells common, perivascular lymphocytic infiltrates on occasion
      • Focal myxoid stromal changes common
      • May show smooth muscle leiomyomatous differentiation (elongated spindle cells with cigar shaped nuclei and pink cytoplasm)
      • Rarely cartilaginous or osseous components
      • Variants:
        • Collagenized / fibrous: collagenous stroma predominates, may have hypocellular myofibroblastic spindle cell component
        • Cellular: dense proliferation of myofibroblasts, ratio of spindle cells to collagen increased compared with classical variant, may have infiltrative borders, storiform or herringbone arrangement
        • Infiltrative: irregular margins, grows into surrounding mammary parenchyma entrapping glandular tissue
        • Myxoid: stellate and spindle cells in abundant myxoid stroma (Histol Histopathol 2016;31:1)
        • Deciduoid: large round, polygonal cells with abundant eosinophilic glassy cytoplasm, single or multiple prominent nucleoli which may be eccentrically placed, binucleation, sharp cell borders, eosinophilic intracytoplasmic inclusions (Histopathology 2008;52:652)
        • Lipomatous: abundant adipocytic component
        • Epithelioid: oval to polygonal cells arranged in clusters, cords, alveolar groups, linear strands, mono, bi or multinucleated, may have eccentrically placed nuclei with small nucleoli, well defined cell borders, single file arrangement may mimic invasive lobular carcinoma (Am J Surg Pathol 2009;33:1085)
        • Atypical: single or scattered atypical mono or multinucleated cells with mild to severe nuclear pleomorphism, regarded as degenerative (Arch Pathol Lab Med 2008;132:1813)
      Microscopic (histologic) images

      Contributed by Kristen E. Muller, D.O. and Victoria Jones, M.D., M.S. (Case #536)
      Well circumscribed mass

      Well circumscribed mass

      Classical type

      Classical type

      Hypocellular, collagenized / fibrous variant

      Hypocellular, collagenized / fibrous variant

      Collagenized / fibrous variant

      Collagenized / fibrous variant

      Admixed adipocytes

      Admixed adipocytes

      Lipomatous variant

      Lipomatous variant


      Myxoid stroma

      Myxoid stroma

      Myxoid stroma and fat

      Myxoid stroma and fat

      Mast cells

      Mast cells

      Cellular variant

      Cellular variant

      Epithelioid variant

      Epithelioid variant

      Epithelioid / deciduoid variant

      Epithelioid / deciduoid variant


      Epithelioid with single file growth

      Epithelioid with single file growth

      Smooth muscle differentiation

      Smooth muscle differentiation

      Chondroid metaplasia Chondroid metaplasia

      Chondroid metaplasia

      Focal cytological atypia

      Focal cytological atypia

      Lipomatous variant

      Lipomatous variant


      Lipomatous variant

      Lipomatous variant

      Keratin

      Keratin

      CD34

      CD34


      CD34

      CD34

      Desmin

      Desmin

      ER

      ER

      PR

      PR

      Loss Rb1

      Loss Rb1

      Virtual slides

      Images hosted on other servers:

      Breast, myofibroblastoma

      Cytology description
      Cytology images

      Images hosted on other servers:

      Loosely cohesive myoepithelial cell groups

      Positive stains
      Negative stains
      Electron microscopy description
      Molecular / cytogenetics description
      Sample pathology report
      • Left breast, mass, needle core biopsy:
        • Bland spindle cell lesion consistent with myofibroblastoma (see comment)
        • Comment: Sections show bland spindle cells arranged in short fascicles admixed with hyalinized collagen bundles and adipose tissue. The lesional cells express CD34, desmin, smooth muscle actin, BCL2 and PR, are negative for cytokeratins and show a loss of Rb1 by immunohistochemistry. Overall, the morphologic and immunohistochemical findings are compatible with mammary type myofibroblastoma.
      Differential diagnosis
      Board review style question #1

      Which of the following immunohistochemical stains is positive in the lesion depicted above?

      1. AE1 / AE3
      2. Calponin
      3. CD34
      4. p63
      5. S100
      Board review style answer #1
      C. CD34. The image shows a myofibroblastoma.

      Comment Here

      Reference: Myofibroblastoma
      Board review style question #2
      Which of the following is true regarding mammary type myofibroblastomas?

      1. The cellular and epithelioid variants are positive for cytokeratins
      2. They are always CD34 positive
      3. They are genetically related to spindle cell lipoma and cellular angiofibroma and cytogenetically show monoallelic loss of 13q14 region
      4. They are related to solitary fibrous tumors and are STAT6 positive by immunohistochemistry
      5. They may have aggressive behavior if cytologic atypia is present
      Board review style answer #2
      C. They are genetically related to spindle cell lipoma and cellular angiofibroma and cytogenetically show monoallelic loss of 13q14 region

      Comment Here

      Reference: Myofibroblastoma
      Board review style question #3
      Which immunoprofile is most suggestive of mammary type myofibroblastoma?

      1. CD34 negative, desmin positive, smooth muscle actin positive, Rb1 positive
      2. CD34 negative, desmin negative, cytokeratin positive, Rb1 negative
      3. CD34 positive, desmin positive, CD10 negative, Rb1 positive
      4. CD34 positive, desmin positive, STAT6 negative, Rb1 negative
      5. CD34 positive, STAT6 positive, Rb1 positive
      Board review style answer #3
      D. CD34 positive, desmin positive, STAT6 negative, Rb1 negative

      Comment Here

      Reference: Myofibroblastoma

      Neoadjuvant chemotherapy
      Definition / general
      • Chemotherapy preceding surgical resection to downstage disease
      • Associated with characteristic histological features on the resection specimen, with prognostic implications
      Essential features
      • The treatment effects associated with neoadjuvant chemotherapy can be seen in the primary tumor (tumor bed), axillary nodal metastases and normal breast tissue
      • Neoadjuvant chemotherapy improves survival and functional outcome by allowing resection in initially nonresectable disease or by reducing the extent of surgical resection
      • Degree of response (extent of residual disease) is predictive of outcome, in particular, a complete pathologic response without residual tumor detected is indicative of good prognosis (Mod Pathol 2015;28:1185)
      Clinical features
      • Indications for neoadjuvant chemotherapy include
        • Locally advanced disease, to allow resection of unresectable disease
        • Disease where breast conservation is not possible but potentially feasible if downstaging is achieved
        • Temporary / treatable conditions that contraindicate surgery and thus require bridging therapy (Curr Oncol 2021;28:1338)
        • Clinically, node positive disease with response to neoadjuvant chemotherapy and negative sentinel lymph nodes may be spared from axillary dissection (Br J Surg 2021;108:748)
        • Grade, stage and biomarker status are additional considerations for neoadjuvant chemotherapy (J Clin Oncol 2021;39:1485)
      • Clinical response to neoadjuvant chemotherapy is classified as
        • Clinical complete response: complete disappearance of tumor
        • Clinical partial response: ≥ 50% reduction greatest dimension of tumor
        • Clinically positive nodes may also respond (reduce in size) after neoadjuvant chemotherapy
      • Clinical assessment may overestimate tumor size due to posttreatment fibrosis (Radiology 2017;285:358)
      • Clinical response is predictive of outcome and correlates with pathological response (J Surg Oncol 2002;80:4, Zhonghua Bing Li Xue Za Zhi 2010;39:734)
      Radiology description
      • Conventional imaging modalities for assessment of radiological response includes mammography, ultrasound, MRI
      • Functional imaging is also used for assessment of radiological response
        • Response is defined by normalization of tracer uptake (complete response) or by change in tracer uptake (Radiology 2017;285:358)
        • MRI is more accurate in predicting complete pathological response than PET / CT (Oncologist 2016;21:931)
      Radiology images

      Contributed by Joshua J.X. Li, M.B.Ch.B. and Gary M. Tse, M.B.B.S.
      Pretreatment MRI

      Pretreatment MRI

      Posttreatment MRI

      Posttreatment MRI

      Prognostic factors
      • Complete pathologic response demonstrates good prognostic prediction in certain types of breast cancers but not all (J Clin Oncol 2012;30:1796)
        • Strong positive correlation with survival in HER2 positive, triple negative and luminal B / HER2 negative (ER / PR positive, HER2 negative, grade 3) breast cancers
        • Poor correlation with survival in luminal A (ER / PR positive, HER2 negative) and luminal B / HER2 positive (ER / PR positive, HER2 positive) breast cancers
      • Multiple systems for reporting and evaluating response to neoadjuvant chemotherapy have been described (Drug Des Devel Ther 2020;14:2423)
      Gross description
      • Gross appearance of tumor may indicate degree of pathological response
        • Tumors with poor response demonstrate a firm, gritty and spiculated appearance (i.e., no difference from typical untreated tumors)
        • Response can be indicated by fibrosis or softening of the tumor bed (Mod Pathol 2015;28:1185)
        • Tumors with partial response can be nodular, multifocal with and less well defined borders (Breast 2022;62:S25)
        • In cases of complete response, the tumor bed may be soft and indistinguishable from normal breast tissue
      • Tumor mapping with radiographs, photographs or drawings of the sectioned gross specimen is recommended (Mod Pathol 2015;28:1185)
      • Considerations in tumor sampling
        • Sampling should include all grossly residual disease, visible tumor bed, marker clips and adjacent tissue
        • Recommendations on sampling extent are based on pretreatment size and whether residual tumor is grossly identifiable
          • Residual carcinoma grossly present
          • No invasive carcinoma grossly identified
            • Minimum of 1 block per cm of the pretreatment tumor size or at least 10 blocks in total, whichever is greater (Food and Drug Administration) (Surg Pathol Clin 2018;11:213)
            • 5 blocks of a cross section per 1 - 2 cm, up to 25 blocks for very large tumors (Breast International Group and the North American Breast Cancer Group)
        • In cases where the tumor bed cannot be identified, correlation with preoperative clinical and radiological findings is recommended for determining the location of sampling
      Gross images

      Contributed by Emily S. Reisenbichler, M.D.

      Complete response to neoadjuvant chemotherapy

      No significant response to neoadjuvant chemotherapy

      Microscopic (histologic) description
      • Breast
        • Multiple reporting systems are described but generally response is classified into
          • Pathological complete response (pCR)
            • Most widely agreed upon definition of pCR is no residual invasive carcinoma in both the breast and axillary lymph nodes, regardless of the presence of residual DCIS ypT0 ypN0 or ypTis ypN0 (Lancet 2014;384:164, Mod Pathol 2021;34:1271)
            • Residual tumor within lymphovascular spaces (lymphovascular invasion) is not allowed for pCR (Mod Pathol 2015;28:1185)
            • Currently, presence of in situ cancer after treatment in the absence of residual invasive cancer in the breast and lymph nodes constitutes pCR
            • However, there is debate on whether ductal carcinoma in situ should be considered as residual disease due to its association with shorter disease free survival but not overall survival (Mod Pathol 2015;28:1185, J Clin Oncol 2012;30:1796)
          • Incomplete response / residual disease: evidence of tumor response but residual tumor cells are present
            • Cutoffs for categories vary for different systems
          • No response: treatment associated changes are not seen histologically
        • Histologic evidence of tumor response includes fibrosis, hyalinization, lymphohistiocytic infiltration, hemosiderin laden macrophages, giant cell formation and vascular proliferation
          • These histologic features are indicative of the tumor bed
        • Patterns observed in partial response includes concentric tumor shrinkage, reduction in tumor cellularity and scattered multifocal tumor deposits (Mod Pathol 2015;28:1185)
        • Tumor cells show degenerative changes including nuclear and cytoplasmic vacuolation, karyorrhexis, karyolysis and pyknosis (Int J Appl Basic Med Res 2012;2:111)
          • There may be a change in histologic grading after neoadjuvant therapy
        • Normal breast tissue shows ductal and lobular atrophy, atypia and hyalinization of vessel wall
      • Lymph node
        • Possible scenarios include
          • Presence of tumor cells, with no evidence of histologic treatment response
          • Presence of tumor cells, histologic evidence of treatment response seen
          • Absence of tumor cells, histological evidence of treatment response seen
          • Absence of tumor cells and histologic evidence of treatment (true negative nodes)
        • Histological evidence of tumor response in the lymph node includes fibrosis, mucin pools, histiocytic infiltrates and lymphoid depletion
        • Presence of tumor response indicates possible previous nodal involvement and is associated with prognosis intermediate between node negative and node positive patients (Ann Surg Oncol 2003;10:734)
        • Residual nodal disease is associated with shorter disease free and overall survival (J Clin Oncol 2012;30:1796)
      Microscopic (histologic) images

      Contributed by Joshua J.X. Li, M.B.Ch.B., Gary M. Tse, M.B.B.S. and Emily S. Reisenbichler, M.D.
      Tumor with treatment effect Tumor with treatment effect Tumor with treatment effect

      Tumor with treatment effect

      Fibrotic tumor bed

      Fibrotic tumor bed

      Minimal residual disease

      Minimal residual disease


      Complete pathologic response

      Complete pathologic response

      Posttreatment lymph node

      Posttreatment lymph node

      No residual invasive or in situ tumor

      Small clusters of residual invasive tumor cells

      Virtual slides

      Images hosted on other servers:

      Tumor with incomplete response

      Near complete response with isolated residual tumor focus

      Positive stains
      • Cytokeratins are also used in assessment of sentinel lymph nodes in a postneoadjuvant chemotherapy setting (Breast 2022;62:S25)
        • Entrapped normal breast epithelium in the tumor bed is positive to cytokeratins and should not be confused for residual tumor cells
          • Myoepithelial markers may help in distinguishing normal breast epithelium and residual in situ carcinoma from residual invasive carcinoma (Arch Pathol Lab Med 2011;135:422)
          • Histiocytic markers (CD68, CD163) may help distinguish residual invasive carcinoma from infiltrating histiocytes in the tumor bed (Histopathology 2015;67:279)
          • Endothelial markers (CD31, ERG) may help distinguish residual invasive carcinoma from lymphovascular invasion
      • Biomarker status may change after neoadjuvant chemotherapy (Pathobiology 2022;89:297)
      Sample pathology report
      • Left breast, local excision:
        • No residual tumor (complete pathologic response) (see comment)
        • Comment: Sections of the tumor bed shows fibrosis and atrophic changes, with lymphohistiocytic infiltrates and hemosiderin deposition. No viable tumor cells are seen, indicating a pathologic complete response to neoadjuvant chemotherapy. The remaining breast tissue is unremarkable.

      • Left breast, local excision:
        • Residual invasive breast carcinoma, NST type, with therapy effect status postneoadjuvant chemotherapy, indicating partial / incomplete response (see comment)
        • Comment: Sections show residual invasive carcinoma of ______ (histotype) and _______ (grade). Fibrosis, lymphohistiocytic infiltrates and hemosiderin deposition in keeping with treatment effects are / are not noted. The residual tumor cellularity is ______%. In situ carcinoma is / is not noted and comprises ______% of the residual tumor cellularity. Lymphovascular invasion is / is not identified. The tumor measures ______ mm and is clear from (specify if < 1 cm ____) / involves the resection margins.

      • Left axillary dissection:
        • __ out of ___ lymph nodes involved by metastatic carcinoma (see comment)
        • Comment: Sections show a total of ____ lymph nodes. ____ lymph nodes show metastatic carcinoma with ____ showing fibrosis, mucin pools and histiocytic infiltrates, suggestive of treatment response. Extracapsular spread is / is not seen. Similar features are seen / not seen in ___ of the remaining benign lymph nodes.
      Board review style question #1

      Which of the following cases is considered a complete pathologic response to neoadjuvant chemotherapy?

      1. Absence of invasive carcinoma in the tumor bed but isolated tumor cells present in an axillary lymph node
      2. Absence of invasive carcinoma in the tumor bed but with viable tumor cells present in lymphovascular spaces of quadrant samplings
      3. Fibrous tumor bed with adjacent lobules showing atypical epithelial lining cells
      4. Significant tumor necrosis in the tumor bed with rare viable invasive carcinoma
      5. Small clusters of invasive carcinoma constituting < 5% of cellularity among a fibrous tumor bed
      Board review style answer #1
      C. Fibrous tumor bed with adjacent lobules showing atypical epithelial lining cells. Complete pathologic response is defined as the absence of residual tumor cells. Presence of any viable tumor cells, regardless of cellularity, in the tumor bed, lymph nodes or lymphovascular spaces should not be considered as complete pathologic response. Chemotherapy can result in atypia in normal breast epithelium and should not be confused with residual disease.

      Comment Here

      Reference: Neoadjuvant chemotherapy
      Board review style question #2
      In which of the following scenarios is neoadjuvant chemotherapy generally considered?

      1. Early stage, low grade, hormone positive / HER2 negative invasive breast cancer
      2. Patient with ductal carcinoma in situ of the breast
      3. Patient with local invasive breast cancer and active hepatitis B infection
      4. Patient with radiological evidence of bone metastasis with biopsy proven stage IV breast cancer
      5. Patient with suspected muscle invasive breast cancer
      Board review style answer #2
      E. Patient with suspected muscle invasive breast cancer. General contraindications to chemotherapy apply to neoadjuvant chemotherapy, including active hepatitis. Patients with unresectable disease or bulky nodal disease are also not indicated for neoadjuvant chemotherapy. The current role of adjuvant therapy in phyllodes tumor is unclear and is generally not offered (Histopathology 2016;68:5). T4 disease (muscle invasion) is not a contraindication to neoadjuvant therapy.

      Comment Here

      Reference: Neoadjuvant chemotherapy

      Neuroendocrine carcinoma (NEC)
      Definition / general
      • Invasive carcinoma with characteristic high grade neuroendocrine features and supported by immunoreactivity to neuroendocrine markers
      Essential features
      • Neuroendocrine markers helpful for diagnosis
      • Not to be confused with distinct breast neoplasms that exhibit neuroendocrine differentiation (e.g., solid papillary carcinoma, mucinous carcinoma of the breast)
      • Morphologically indistinguishable from neuroendocrine carcinomas (small cell carcinoma and large cell carcinoma) of other organ primaries
      Terminology
      • Small cell neuroendocrine carcinoma
      • Large cell neuroendocrine carcinoma
      • Oat cell carcinoma (for small cell carcinoma, not recommended by the WHO)
      ICD coding
      • ICD-O:
        • 8246/3 - neuroendocrine carcinoma, NOS
        • 8041/3 - small cell neuroendocrine carcinoma
        • 8013/3 - large cell neuroendocrine carcinoma
      • ICD-10: C50.9 - malignant neoplasm of breast of unspecified site
      • ICD-11:
        • 2C6Y - other specified malignant neoplasms of breast
        • XH0U20 - neuroendocrine carcinoma, NOS
        • XH9SY0 - small cell neuroendocrine carcinoma
        • XH0NL5 - large cell neuroendocrine carcinoma
      Epidemiology
      Pathophysiology
      • Unconfirmed at this time but proposed theories include:
        • Neoplastic transformation from native neuroendocrine cells in the breast (J Clin Pathol 2012;65:699)
        • Divergent neuroendocrine differentiation of neoplastic stem cells (Arch Pathol Lab Med 2017;141:1577)
        • Invasive and in situ ductal carcinomas occasionally identified in small cell carcinomas of the breast (Am J Surg Pathol 2000;24:1231)
          • May suggest acquisition of neuroendocrine features and transformation from invasive or in situ breast carcinomas
      Etiology
      • Unknown at this time
      Clinical features
      Diagnosis
      • Largely dependent on histologic evidence of neuroendocrine differentiation and small cell / large cell features (see Microscopic description)
        • Supplemented by positivity to neuroendocrine markers
      • Clinical exclusion of metastasis to breast from a nonmammary primary site is essential
      • Identification of in situ component is the most helpful feature for confirmation of the breast as the primary site of origin (e.g. gastrointestinal tract, pulmonary) (Am J Case Rep 2021;22:e932274)
      Laboratory
      • Serum chromogranin A not demonstrated to be useful in detection and disease monitoring in a large case control cohort (Int J Biol Markers 2001;16:268)
      Radiology description
      Radiology images

      Contributed by Mark R. Wick, M.D.

      Large cell carcinoma

      Left lateral MRI



      Images hosted on other servers:

      Small cell carcinoma

      Hyperdense mass on mammography

      Hypoechoic mass on ultrasound

      Prognostic factors
      Case reports
      Treatment
      Clinical images

      Images hosted on other servers:

      Before and after chemotherapy

      Gross description
      Gross images

      Contributed by Mark R. Wick, M.D.

      Small cell carcinoma

      Microscopic (histologic) description
      • Small cell carcinoma:
        • Infiltrative growth pattern
          • Crush artifact frequent
          • Necrosis and lymphovascular invasion common
        • Tumor cells densely packed with ill defined cell borders
          • High N/C ratio with scanty cytoplasm
          • Small dark hyperchromatic nuclei and inconspicuous nucleoli
          • Mitotic count high
        • Associated with proliferative changes, in situ carcinomas and invasive carcinomas (Am J Surg Pathol 2000;24:1231)
          • In situ small cell carcinoma present in 5 out of 9 cases in a case series
          • In situ / invasive ductal, lobular and squamous neoplastic components occasionally identified
      • Large cell carcinoma:
        • High grade features (frequent necrosis, lymphovascular invasion and mitosis) similar to small cell carcinoma
        • Tumor cells have a larger amount of cytoplasm
        • Chromatin pattern coarse with occasional distinct nucleoli
      • Neuroendocrine carcinomas of the breast histologically indistinguishable from their counterparts of other primaries
      Microscopic (histologic) images

      Contributed by Joshua J.X. Li, M.B.Ch.B., Gary M. Tse, M.B.B.S. and Kristen E. Muller D.O.

      Small cell carcinoma

      Tumor cell in cords

      Fine chromatin pattern

      Uniform tumor cells


      Cords and necrosis

      Crush artifact

      Abundant mitoses

      Synaptophysin



      Large cell carcinoma

      Infiltrative tumor edge

      Frequent mitotic figures

      Coarse chromatin pattern

      Marked nuclear pleomorphism

      CD56

      Chromogranin

      Virtual slides

      Images hosted on other servers:

      Small cell carcinoma

      Large cell carcinoma

      Cytology description
      • Neuroendocrine features, including nuclear molding and fine salt and pepper chromatin, may be identified in small cell carcinoma
      • Necrosis can be present in the background (J Cytol 2011;28:91)
      • Smear preparation can accentuate crush artifact
      Cytology images

      Contributed by Joshua J.X. Li, M.B.Ch.B. and Gary M. Tse, M.B.B.S.

      Crush artifact

      Neuroendocrine nuclear features

      Metastatic small cell carcinoma

      Molecular / cytogenetics description
      • TP53 (75%) and PIK3CA (33%) mutations common in small cell carcinoma of the breast (Breast Cancer Res Treat 2016;158:195):
        • PIK3CA mutation not present in small cell carcinoma of lung
        • RB1 mutation present in small cell carcinoma of lung but not of breast primary
      • Mutations in AKT1, ARID1A, CDH1, FOXA1, GATA3 and TBX3 also detected (J Pathol 2017;241:405)
        • Compared to luminal breast cancers, PIK3CA mutations are less frequent, whereas FOXA1 and TBX3 mutations are more common
      Sample pathology report
      • Left breast, local excision:
        • Small cell carcinoma (see comment)
        • Comment: Sections show breast tissue infiltrated by cords, nests and sheets of tumor cells. The tumor cells possess small dark hyperchromatic nuclei and inconspicuous nucleoli with scanty cytoplasm. The mitotic count is at __ per 10 high power fields. The tumor measures __ in maximal dimension and is clear from (specify if < 1 cm __) / involves the resection margins.
        • The tumor cells are diffusely positive for synaptophysin and focally positive for chromogranin.

      • Right breast, local excision:
        • Large cell carcinoma (see comment)
        • Comment: Sections show breast tissue infiltrated by nests and sheets of tumor cells. The tumor cells possess enlarged nuclei with coarse chromatin pattern, distinct nucleoli and a moderate amount of cytoplasm. The mitotic count is at __ per 10 high power fields. The tumor measures __ in maximal dimension and is clear from (specify if < 1 cm __) / involves the resection margins.
        • The tumor cells are diffusely positive for synaptophysin and focally positive for chromogranin.
      Differential diagnosis
      • Neuroendocrine carcinomas of other, nonmammary origin:
      • Neuroendocrine tumors:
        • Majority of neuroendocrine tumors are Nottingham grade 1 or 2 (low nuclear grade and infrequent mitosis)
        • Necrosis and lymphovascular invasion not features of neuroendocrine tumors
        • Ki67 index is lower in neuroendocrine tumors than neuroendocrine carcinomas
      • Invasive carcinoma with neuroendocrine differentiation:
        • Invasive carcinomas with neuroendocrine differentiation show neuroendocrine features in terms of nuclear morphology or positivity to neuroendocrine markers in 10 - 90% of the tumor
        • Complete small cell or large cell nuclear features usually absent
        • Negativity to neuroendocrine markers, in particular synaptophysin, may help identify nonneuroendocrine areas
      • Adenoid cystic carcinoma (solid):
        • Solid pattern of adenoid cystic carcinoma can resemble small cell carcinoma
        • Focal cribriform pattern, tubular pattern and luminal secretion, if identified, aids in diagnosing adenoid cystic carcinoma
        • Markedly hyperchromatic nuclei with crush artifact not a feature of adenoid cystic tumor
        • Positivity to KIT, MYB and SOX10 with negativity to hormonal markers confirms a diagnosis of adenoid cystic carcinoma
      • Lymphoma:
        • Markedly hyperchromatic nuclei with crush artifact in small cell carcinoma can resemble lymphoma
        • Lymphoma does not show cellular cohesion
        • Cytokeratins, hormonal, neuroendocrine and lymphoid markers differentiate lymphomas from carcinomas
      • Melanoma:
        • Degree of nuclear atypia and nuclear features of melanoma can overlap with large cell carcinoma
        • Melanin pigment, when identified, indicates melanoma
        • Definitive confirmation may require melanocytic markers
      Board review style question #1

      Which of the following features distinguishes large cell carcinoma from small cell carcinoma of breast primary?

      1. Coarse chromatin pattern and distinct nucleoli
      2. Positive axillary lymph nodes
      3. Positivity to estrogen and progesterone receptors
      4. Positivity to synaptophysin but not chromogranin
      5. Presence of extensive tumor necrosis
      Board review style answer #1
      A. Coarse chromatin pattern and distinct nucleoli. Large cell carcinoma is distinguished from small cell carcinoma based on cellular morphologic features, including a coarse chromatin pattern, distinct nucleoli and a greater amount of cytoplasm. The immunoprofile of large cell and small cell carcinomas are similar (Breast Care (Basel) 2015;10:281). Other histological and clinical features are also not useful in distinguishing large cell carcinoma from small cell carcinoma.

      Comment Here

      Reference: Neuroendocrine carcinoma (NEC)
      Board review style question #2
      Which of the following findings most favors small cell carcinoma of breast primary over metastatic small cell carcinoma from the lung?

      1. Positivity to estrogen and progesterone receptors
      2. Positivity to TTF1 stain
      3. Presence of adjacent ductal carcinoma in situ
      4. Presence of RB1 mutation
      5. Presence of TP53 mutation
      Board review style answer #2
      C. Presence of adjacent ductal carcinoma in situ. The majority of small cell carcinomas of the breast are associated with proliferative changes, in situ and invasive carcinomas (Am J Surg Pathol 2000;24:1231). TP53 mutation is detected in small cell carcinomas of both lung and breast primaries, whereas RB1 mutation was only present in small cell carcinomas of lung primary from a cohort described by McCullar et al. (Breast Cancer Res Treat 2016;158:195). TTF1, estrogen receptor and progesterone receptor expression do not accurately distinguish small cell carcinomas of breast and lung (Front Endocrinol (Lausanne) 2020;11:228, Am J Surg Pathol 2000;24:1231).

      Comment Here

      Reference: Neuroendocrine carcinoma (NEC)

      Neuroendocrine tumor
      Definition / general
      • Primary, low to intermediate grade neoplasm of the breast with histological features of neuroendocrine differentiation and supported by immunoreactivity to neuroendocrine markers
      Essential features
      • Uncommon, primary neuroendocrine neoplasm of the breast of low to intermediate grade
      • Morphology is the primary feature for classification, requires > 90% neuroendocrine tumor (NET) pattern
      • Neuroendocrine markers helpful for diagnosis
      • Not to be confused with distinct breast neoplasms that exhibit neuroendocrine differentiation (e.g., solid papillary carcinoma, mucinous carcinoma of the breast, invasive ductal carcinoma, no special / specific type [IDC-NST] with neuroendocrine differentiation)
      Terminology
      • Carcinoid tumor, atypical carcinoid, not recommended
      • Prior to WHO 5th edition, often referred to as the category of tumors with neuroendocrine differentiation; the current WHO 5th terminology is neuroendocrine neoplasms, which includes neuroendocrine tumors (NET), neuroendocrine carcinomas (small cell and large cell NEC)
      ICD coding
      • ICD-O:
        • 8240/3 - neuroendocrine tumor, NOS
        • 8240/3 - neuroendocrine tumor, grade 1
        • 8249/3 - neuroendocrine tumor, grade 2
      • ICD-10:
        • C50.9 - malignant neoplasm of breast of unspecified site
      • ICD-11:
        • 2C6Y - other specified malignant neoplasms of breast
        • XH8DS0 - neuroendocrine tumor, NOS
        • XH9LV8 - neuroendocrine tumor, grade 1
        • XH51K1 - neuroendocrine tumor, grade 2
      Epidemiology
      Pathophysiology
      Etiology
      • Unknown at this time
      Clinical features
      Diagnosis
      • Requires histomorphologic evidence of neuroendocrine differentiation
      • Supplemented by positivity to neuroendocrine immunohistochemical markers
      • Radiology and serology not sensitive nor specific in diagnosis
      Laboratory
      Radiology description
      Radiology images

      Contributed by Joshua J.X. Li, M.B.Ch.B. and Gary M. Tse, M.B.B.S.

      Mammogram

      Doppler ultrasound

      Prognostic factors
      • Tumor stage (Histopathology 2014;64:647)
      • Histological grade
        • Most neuroendocrine tumors of the breast are Nottingham grades 1 and 2
      • Ki67 proliferation index
        • High Ki67 proliferative index may indicate tumor aggressiveness
        • No formal grading system defined
      Case reports
      Treatment
      • Mainstay of treatment is surgical resection, with or without lymphadenopathy (Eur J Surg Oncol 1995;21:609)
      • Report of good response in metastatic disease with combination of traditional chemotherapy and somatostatin analog (Cureus 2021;13:e16860)
      Gross description
      Microscopic (histologic) description
      • Requires morphologic threshold of > 90% of NET pattern, consistent with the criteria defining other special subtypes of the breast
      • Densely cellular, insular, organoid nests and trabeculae with delicate intervening fibrovascular stroma (Arch Pathol Lab Med 2017;141:1577, Korean J Radiol 2013;14:395)
        • Papillary and insular patterns and alveolar-like structures may be seen
      • Tumor cells display classical neuroendocrine morphology:
        • Spindle, plasmacytoid or polygonal cells
        • Eosinophilic granular cytoplasm
        • Eccentric nuclear placement
        • Salt and pepper stippled chromatin may be present, often not obvious
        • Intranuclear inclusions
      • Mammary NET often lacks classic carcinoid-like features (ribbons, cords and rosettes) (Histopathology 2015;66:761)
      • Majority Nottingham grade 1 or 2
      Microscopic (histologic) images

      Contributed by Joshua J.X. Li, M.B.Ch.B. and Gary M. Tse, M.B.B.S.

      Nests of tumor cells

      Neuroendocrine morphology

      Neuroendocrine nuclear features


      Synaptophysin

      Ki67

      Chromogranin

      CD56

      Virtual slides

      Images hosted on other servers:

      Neuroendocrine tumor, grade 1

      Positive stains
      Electron microscopy description
      • Dense core vesicles and presynaptic vesicles
      Electron microscopy images

      AFIP images

      Dense core granules

      Molecular / cytogenetics description
      Sample pathology report
      • Left breast, local excision:
        • Neuroendocrine tumor, grade __, measuring ___ cm (see comment and synoptic report)
        • Comment: Sections show breast tissue infiltrated by groups and rounded nests of tumor cells. The tumor cells possess low grade nuclei that are rounded to ovoid, scanty to moderate amount of cytoplasm and some of them show plasmacytoid features with stippled chromatin. The mitotic count is at __ per 10 high power fields. The features are those of neuroendocrine tumor, grade __. The tumor measures __ in maximal dimension and is clear from (specify if < 1 cm __) / involves the resection margins.
        • The tumor cells are diffusely positive for synaptophysin and focally positive for chromogranin. Ki67 proliferative index is approximated at __%.
      Differential diagnosis
      • Metastatic neuroendocrine tumors of other primaries:
        • Cannot be reliably distinguished by morphology
        • Dependent on history of neuroendocrine tumors in other sites, correlation with clinical history and imaging is required
        • Positivity to hormonal markers and breast markers; negativity to markers associated with other primaries (e.g., TTF1, glypican 3) may be helpful
        • Identification of an in situ component can help distinguish primary breast from metastatic nonmammary tumors
      • Breast neoplasms with neuroendocrine differentiation:
        • Neuroendocrine carcinomas:
          • Small cell carcinoma:
            • Small hyperchromatic nuclei, nuclear molding, often crushed
            • Very high nuclear / cytoplasmic ratio
            • Mitotic count and Ki67 proliferative index high
            • Necrosis
          • Large cell carcinoma:
            • Grade 3, large pleomorphic nuclei with coarse chromatin
            • Mitotic count and Ki67 proliferative index high
            • Often necrosis
        • Invasive carcinoma with neuroendocrine differentiation:
          • If an NEN pattern is present in 10 - 90% of a malignancy, the terminology for mixed invasive (NST or other special type) and NET may be used, with an estimation of the percent of each component reported
          • Negativity to neuroendocrine markers, in particular synaptophysin, may help identify nonneuroendocrine areas
          • IBC-NSTs can be immunoreactive for neuroendocrine markers, expression of neuroendocrine markers alone is not sufficient for a diagnosis of mammary NET
        • Solid papillary carcinoma:
          • Nuclear and cytologic features can be similar
          • Fibrovascular cores with palisading, when identified, are helpful for diagnosing solid papillary carcinoma
          • Majority of solid papillary carcinomas are positive for neuroendocrine markers (Am J Surg Pathol 2016;40:1334)
        • Mucinous carcinoma:
          • Nuclear and cytologic features can be similar
          • > 90% mucinous morphology (e.g. extracellular mucin pools)
          • Mucinous carcinomas can be positive to neuroendocrine markers (Mod Pathol 2004;17:568)
      • Apocrine carcinoma:
        • Abundant eosinophilic cytoplasm may resemble neuroendocrine differentiation
        • Prominent nucleoli and vesicular chromatin distinguish apocrine from neuroendocrine differentiation
        • Apocrine carcinomas are negative to neuroendocrine markers
      • Lobular neoplasia:
        • Monomorphic low grade nuclei in lobular neoplasia can be confused with neuroendocrine differentiation
        • In situ lesions may appear similar to the nested pattern in neuroendocrine tumors
        • Genuine neuroendocrine nuclear features are not observed in lobular neoplasia
        • Lobular neoplasia is noninvasive with preserved myoepithelial cells
        • Loss of E-cadherin staining confirmatory of lobular neoplasia
      Board review style question #1

      Which of the following clinicopathological features best supports the diagnosis of a neuroendocrine tumor of breast primary (as compared to neuroendocrine tumors metastasizing to the breast)?

      1. Absence of carcinoid syndrome
      2. Negative imaging and endoscopy findings
      3. Negativity to TTF1
      4. Positivity to CK7
      5. Positivity to ER and PR
      Board review style answer #1
      B. Negative imaging and endoscopy findings. CK7 is also positive in neuroendocrine tumors of the lung. ER and PR positivity are not uncommon in neuroendocrine tumors of other primaries such as the lung and pancreas. TTF1 negative tumors are less likely of lung origin but do not exclude other primaries. Carcinoid symptoms are not reported in neuroendocrine tumors of the breast but do not exclude metastasis. Negative imaging and endoscopy findings best support the diagnosis of a primary neuroendocrine tumor of the breast by excluding metastasis from other sites (Hum Pathol 2001;32:1087, Arch Pathol Lab Med 2008;132:1889, Neuroendocrinology 2011;93:249, Am J Surg 2006;191:799).

      Comment Here

      Reference: Neuroendocrine tumor of breast
      Board review style question #2
      In which of the following breast tumors are neuroendocrine markers most likely to be negative?

      1. Grade 1 neuroendocrine tumor
      2. Grade 2 neuroendocrine tumor
      3. Mucinous carcinoma, type A
      4. Mucinous carcinoma, type B
      5. Small cell carcinoma
      Board review style answer #2
      C. Mucinous carcinoma, type A. Neuroendocrine tumors and small cell carcinomas are neuroendocrine neoplasms of the breast. Solid papillary carcinomas and mucinous carcinomas are specific breast cancer subtypes with frequent neuroendocrine differentiation and positivity to neuroendocrine markers. Positivity to neuroendocrine markers is more common in type B than type A mucinous carcinoma (Front Oncol 2021;10:558760).

      Comment Here

      Reference: Neuroendocrine tumor of breast

      Nipple adenoma
      Definition / general
      • Nipple adenoma is a benign epithelial proliferation with a variety of morphological patterns that arise near the collecting ducts of the nipple and may be continuous with the overlying epidermis
      Essential features
      • Benign, often florid, epithelial proliferation with retained myoepithelial cell layer involving superficial ducts of the nipple; the proliferation may extend and replace the overlying squamous epithelium resulting in erosion, hemorrhage and inflammation
      • Histologic patterns include a mix of sclerosing adenosis, papillary hyperplasia and usual ductal hyperplasia
      Terminology
      • Synonym: nipple duct adenoma
      • Not recommended: florid papillomatosis of the nipple, erosive adenomatosis of the nipple, papillomatosis of the nipple, superficial papillary adenomatosis, papillary adenoma of the nipple
      ICD coding
      • ICD-O: 8506/0 - adenoma of nipple
      • ICD-11:
        • 2F30.Y - other specified benign neoplasm of breast
        • XH7GN3 - adenoma of nipple
      Epidemiology
      • Rare; occurs in males and females
      • Most common in women in the fifth decade of life but has a wide age range (5 months to 89 years)
      Sites
      • Breast; superficial duct orifices of the nipple, the associated stroma and often the adjacent overlying epidermis
      Etiology
      • Unknown
      Clinical features
      • Detected when small due to skin changes
      • Clinically, epidermal involvement mimics the appearance of Paget disease
      • On physical examination, small, palpable dermal nodule
        • Nipple may appear eroded and erythematous as glandular epithelium replaces squamous epithelium
        • Bleeding and exudative crust may be interpreted as nipple discharge
      • If left untreated for years, may develop into large pedunculated mass (World J Surg Oncol 2014;12:91)
      Diagnosis
      • Diagnosis can be made on biopsy (often of the skin due to clinical presentation) or cytology specimen and confirmed on excision
      • Imaging findings are generally nonspecific (see Radiology description)
      Radiology description
      Radiology images

      Images hosted on other servers:
      Mammography, ultrasound and MRI of palpable left nipple mass with clear nipple discharge Mammography, ultrasound and MRI of palpable left nipple mass with clear nipple discharge Mammography, ultrasound and MRI of palpable left nipple mass with clear nipple discharge

      Mammography, ultrasound and MRI of palpable left nipple mass with clear nipple discharge

      Mammography, sonography and MRI images of nipple adenoma

      Mammography, sonography and MRI images of nipple adenoma

      Prognostic factors
      • Can recur if not completely excised
      • Risk of developing a subsequent breast cancer is comparable to other proliferative lesions of the breast (Am J Surg Pathol 1986;10:87)
      Case reports
      Treatment
      Clinical images

      Images hosted on other servers:
      Nipple erythema and induration

      Nipple erythema and induration

      Eroded crusted nipple adenoma

      Eroded crusted nipple adenoma

      Nipple nodule with erosion

      Nipple nodule with erosion

      Pedunculated papillomatous nipple adenoma

      Pedunculated papillomatous nipple adenoma


      Accessory breast nipple adenoma

      Accessory breast nipple adenoma

      Erosion and bloody discharge

      Erosion and bloody discharge

      Nipple adenoma, preoperative and postconservative excision

      Nipple adenoma, preoperative and postconservative excision

      Gross description
      • Rubbery; white to gray; 0.5 cm to > 4 cm
      Gross images

      Images hosted on other servers:
      Complete nipple resection

      Complete nipple resection

      Frozen section description
      • Glandular nests of various sizes with diffuse growth within ducts lined by myoepithelial cells; complex and irregular structure may falsely mimic invasion (Chin Med J (Engl) 2007;120:630)
      Microscopic (histologic) description
      • Nodular florid epithelial proliferation of the lactiferous ducts of the nipple with papillary hyperplasia / adenosis / sclerosis / mixed pattern
      • Epithelium is cytologically bland
      • Myoepithelial cell layer is preserved uniformly (Mod Pathol 1990;3:288)
      • 4 major morphologic patterns:
        • Sclerosing papillary hyperplasia pattern:
          • Intraductal papillary growth with prominent stromal proliferation (collagenous bands, myxoid change or elastosis)
          • Focal central necrosis may be present
          • Squamous cysts can be present in duct orifices
        • Papillary hyperplasia pattern:
          • Papillary growth in large ducts with less prominent stromal proliferation and focal necrosis
          • Replacement of epidermis with glandular epithelium
        • Adenosis pattern:
          • Small duct proliferation with a pattern resembling sclerosing adenosis and may be pseudoinfiltrative
          • Necrosis is uncommon
        • Mixed pattern:
      • May be in continuity with the skin surface, which may appear eroded or ulcerated
      • Fairly well circumscribed but unencapsulated (Br Med J 1963;1:563)
      • Other features often seen:
        • Squamous metaplasia and superficial keratin cysts
        • Acanthosis
        • Toker cell hyperplasia in the epidermis
        • Multinucleated giant cells
        • Apocrine metaplasia (Oncol Lett 2014;7:1839)
      • In rare cases, ductal carcinoma in situ involving an adenoma or a contiguous invasive carcinoma have been reported (Mod Pathol 1995;8:633)
      Microscopic (histologic) images

      Contributed by Hal Berman, M.D., Ph.D., Miralem Mrkonjic, M.D., Ph.D. and Mary Ann Gimenez Sanders, M.D., Ph.D.
      Epidermal involvement Epidermal involvement

      Epidermal involvement

      Focal ulceration Focal ulceration

      Focal ulceration

      Shave biopsy of nipple

      Shave biopsy of nipple

      Nipple duct involved by benign papillary proliferation

      Nipple duct involved by benign papillary proliferation


      Well circumscribed epithelial proliferation

      Well circumscribed epithelial proliferation

      Epithelial hyperplasia and epidermal ulceration

      Epithelial hyperplasia and epidermal ulceration

      Usual ductal hyperplasia with focal squamous metaplasia

      Usual ductal hyperplasia with focal squamous metaplasia

      Prominent squamous hyperplasia

      Prominent squamous hyperplasia

      Ductal carcinoma in situ Ductal carcinoma in situ

      Ductal carcinoma in situ


      Prominent epithelial proliferation with papillary architecture and tufting

      Prominent epithelial
      proliferation with
      papillary architecture
      and tufting

      Nipple section Nipple section

      Nipple section

      Missing Image

      Mixed pattern on mastectomy

      Sclerosing adenosis

      Sclerosing adenosis

      Papillary growth within ducts

      Papillary growth within ducts


      Mixed pattern on punch biopsy

      Mixed pattern on punch biopsy

      Florid usual ductal hyperplasia

      Florid usual ductal hyperplasia

      Small tubules

      Small tubules

      Prominent epithelial proliferation Prominent epithelial proliferation Prominent epithelial proliferation

      Prominent epithelial proliferation

      Virtual slides

      Images hosted on other servers:
      Nipple adenoma sclerosing

      Nipple adenoma sclerosing

      Nipple adenoma adenosis pattern

      Nipple adenoma adenosis pattern

      Nipple adenoma papillomatous pattern

      Nipple adenoma papillomatous pattern

      Nipple adenoma florid usual ductal hyperplasia

      Nipple adenoma florid usual ductal hyperplasia

      Cytology description
      • Obtained by fine needle aspiration (FNA), brush cytology or tumor imprint cytology
      • Reported features vary; benign when myoepithelial cells are detected
      • FNA:
        • Large cell clusters with a papillary or sheet structure, as well as small clusters and solitary epithelial cells in a necrotic background
        • Duct epithelial and myoepithelial 2 cell pattern in the clusters
        • Round to oval or spindle nuclei, fine and bland chromatin
      • Brush cytology (Diagn Cytopathol 2015;43:664):
        • Small papillary clusters with attached myoepithelial cells and solitary epithelial cells in a hemorrhagic or neutrophilic background
        • Round to oval nuclei, with smooth contour, bland and granular chromatin
        • Squamous metaplasia-like cells present
      • Cytology often classified as atypical or suspicious, especially in the presence of coagulative necrosis (ANZ J Surg 2015;85:444)
      Positive stains
      Negative stains
      Molecular / cytogenetics description
      • Activating PIK3CA mutations reported in 12/24 cases
      • KRAS (1/24 cases) and BRAF mutations (2/24 cases) (Histopathology 2017;70:195)
      Videos

      Dr. Jerad Gardner presents normal nipple histology and 2 cases of nipple adenoma

      3 minute overview of the presentation and histology of nipple adenomas

      Sample pathology report
      • Right breast (nipple), punch biopsy:
        • Nipple adenoma (see comment)
        • Comment: Sections show ductal proliferation involving a large nipple duct with areas of papillary growth and usual ductal hyperplasia in continuity with the epidermis. Ductal epithelial cells are cuboidal to columnar with eosinophilic cytoplasm and bland nuclei. Focal areas of stromal sclerosis are identified. Immunohistochemistry for smooth muscle myosin and p63 highlights the myoepithelial cell layer while ER and CK5/6 show heterogeneous staining patterns of epithelial cells. Taken together, the histomorphology and immunohistochemistry are consistent with a nipple adenoma.
      Differential diagnosis
      Board review style question #1

      A punch biopsy of the nipple was performed to sample a nodular lesion associated with skin changes in a 48 year old woman. What is the diagnosis?

      1. Ductal adenoma
      2. Intraductal papilloma
      3. Nipple adenoma
      4. Paget disease of the breast
      5. Tubular adenoma
      Board review style answer #1
      C. Nipple adenoma. Nipple adenoma is a benign epithelial proliferation in the breast, often in continuity with the epidermis of the nipple. It forms a nodular mass that can have a number of different morphologic patterns including sclerosing adenosis, papillary hyperplasia and epithelial hyperplasia. Myoepithelial cells are retained in nipple adenoma. Focal necrosis can be seen and squamous cysts can be seen superficially.

      Comment Here

      Reference: Nipple adenoma
      Board review style question #2
      A fairly well circumscribed lesion associated with the nipple of a 55 year old woman and consisting of papillary growth within ducts with areas of focal central necrosis in areas of florid epithelial hyperplasia is suspected to be a nipple adenoma on needle core biopsy. What stain can be used to confirm this diagnosis?

      1. CK7
      2. ER
      3. HER2
      4. p53
      5. p63
      Board review style answer #2
      E. p63. Nipple adenoma is a benign epithelial proliferation of the breast that retains a myoepithelial cell layer. p63, a marker of myoepithelial cells, can be used to rule out invasive carcinoma and support the diagnosis of nipple adenoma.

      Comment Here

      Reference: Nipple adenoma
      Board review style question #3
      A 54 year old woman presents with erythema, erosion and crusting of the nipple. A punch biopsy is performed showing nipple adenoma. Which of the following statements is true regarding nipple adenomas?

      1. HER2 positivity is an essential characteristic of nipple adenomas
      2. Myoepithelial cells are absent in nipple adenoma
      3. Sclerosing adenosis, papillary hyperplasia and epithelial hyperplasia can be seen in mixed patterns in a nipple adenoma
      4. The presence of necrosis excludes nipple adenoma in the differential diagnosis
      5. The skin changes that can be seen in nipple adenoma are clinically distinct from Paget disease
      Board review style answer #3
      C. Sclerosing adenosis, papillary hyperplasia and epithelial hyperplasia can be seen in mixed patterns in a nipple adenoma. Nipple adenoma is a benign breast lesion and can consist of sclerosing adenosis, papillary hyperplasia and epithelial hyperplasia, which can be seen in mixed patterns. Necrosis can sometimes be seen in areas of florid epithelial hyperplasia. The myoepithelial cell layer is retained and HER2 IHC is negative. Nipple adenoma can present clinically with skin changes that are very similar to Paget disease.

      Comment Here

      Reference: Nipple adenoma

      Nonproliferative fibrocystic changes
      Definition / general
      • Nonspecific, general term to describe a range of common and benign breast conditions, which may occur together or in isolation
      • Cause clinical, radiographic (e.g. calcifications, mass or architectural distortion) or histologic changes of the breast tissue that may result in concern for malignancy
      • Nonproliferative and nonatypical fibrocystic changes are not associated with increased risk of subsequent breast carcinoma
      • Some proliferative fibrocystic changes are associated with a slightly increased risk of subsequent breast carcinoma
      Essential features
      • Histologically benign structural alterations in epithelial and stromal elements
      Terminology
      ICD coding
      • ICD-10: N60.19 - diffuse cystic mastopathy of unspecified breast
      Epidemiology
      Sites
      • Breast
      • Can occur in axilla from the accessory breast tissue
      Pathophysiology
      • Excess estrogen leads to proliferation of epithelium in terminal duct lobular units and induces stromal fibrosis
      • Fibrosis and epithelial proliferation may lead to obstruction of ducts and acini, leading to involution or cyst formation
      • Some cysts may rupture, inducing adjacent fibroinflammatory stromal reactions
      • Reference: J Adv Sci Res 2020;11:30
      Etiology
      Diagrams / tables

      Images hosted on other servers:

      Breast tissue with cysts

      Clinical features
      • Breast pain, tenderness, lumpiness, cysts or mass
      • Manifestations may be cyclic, reflecting menstrual cycle
      • Associated with polycystic ovary syndrome (Arch Gynecol Obstet 2009;280:249)
      • Usually bilateral, although one breast may be affected more than the other
      • Symptoms tend to abate after menopause
        • Nonproliferative lesions are the most common finding in breast cancer screening biopsies, accounting for about 70% of all cases (N Engl J Med 2005;353:275)
      Diagnosis
      • Needle biopsy, needle aspiration or excisional biopsy
      Radiology description
      • Ultrasound: may be a solid mass, cyst, heterogeneous echogenic tissue or no visible abnormality (Invest Radiol 2005;40:436)
      • MRI: mass or a nonmass-like regional enhancing lesion with benign enhancement kinetics (Invest Radiol 2005;40:436)
      • Can also present as densities with associated calcifications, areolar skin thickening alone or normal dense fibroglandular tissue with no abnormality on mammogram (Invest Radiol 2005;40:436)
      Radiology images

      Images hosted on other servers:

      Increased fibroglandular density

      Prognostic factors
      • Risk of breast cancer (Am J Surg Pathol 2003;27:836)
        • Nonproliferative lesions: no increased risk
        • Proliferative lesions without atypia: 1.5 - 2 fold risk
      Case reports
      Treatment
      Gross description
      • Breast tissue is heterogeneously fibrous and indurated
      • Cysts ranging from 1 - 20 mm, clear or blue domed, may be seen
      Gross images

      Images hosted on other servers:

      Cysts surrounded by fibrous tissue

      Irregular fibrosis and small cysts

      Fibrosis and dilated ducts

      Microscopic (histologic) description
      • Nonproliferative fibrocystic changes; characterized by 3 features: adenosis, fibrosis and cyst formation
        • Adenosis:
          • Increased number of acini per lobule
          • Acini are lined by columnar cells, which may be benign or have atypia
          • Adenosis is frequently seen in pregnancy and may be focal in nonpregnant women
        • Fibrosis:
          • Stromal fibrosis
          • Cyst rupture may result in chronic inflammation, prominent histiocytic reaction and fibrosis
          • Causes increased density observed on imaging and nodularity observed on palpation
        • Cyst formation:
          • Cystically dilated ducts or lobules
          • May contain eosinophilic secretions, foamy macrophages and calcifications
          • Lined by cuboidal epithelium or may have apocrine metaplasia
          • Outer myoepithelial layer present but may be attenuated
        • Apocrine metaplasia: single or multilayered ductal epithelium with abundant granular eosinophilic cytoplasm, apical snouts, enlarged nuclei and prominent nucleoli
        • Columnar cell changes / columnar cell hyperplasia: enlargement of terminal duct lobule with variably dilated lumens and irregular contours, lined by cuboidal to columnar cells, lacking cytologic atypia, frequently with apical snouts
      Microscopic (histologic) images

      Contributed by Apeksha N. Agarwal, M.B.B.S., M.D.
      Fibrocystic change

      Fibrocystic change

      Apocrine metaplasia

      Apocrine metaplasia

      Fibrosis, cysts and microcalcifications

      Fibrosis, cysts and microcalcifications

      Cysts

      Cysts

      Cytology description
      • Nonproliferative breast lesions
        • Apocrine cells, foam cells and small, uniform, evenly spaced ductal epithelial cells
      • Proliferative breast lesions
        • Sheets and tight clusters of cells without significant nuclear overlap
      • Reference: Clin Lab Med 2005;25:713
      Cytology images

      Contributed by Areej M. Al Nemer, M.D.
      Fibrocystic change

      Fibrocystic change

      Fibrocystic change

      Apocrine metaplasia

      Sample pathology report
      • Breast, 12:00, microcalcifications, stereotactic core needle biopsy:
        • Benign breast parenchyma with fibrocystic changes, including florid usual type ductal hyperplasia and apocrine metaplasia
        • Microcalcifications associated with apocrine metaplasia
      Differential diagnosis
      • Proliferative lesions without atypia:
        • Mild usual ductal hyperplasia:
          • Proliferation of ductal epithelium no more than 3 or 4 cell layers in thickness
        • Moderate to severe (florid):
          • Proliferation of ductal epithelium more than 4 cell layers in thickness that may fill and extend the luminal space
          • Arrangement of nuclei is haphazard, with cellular heterogeneity, oval nuclei with grooves and small indistinct nucleoli
        • Sclerosing adenosis:
          • Lobulocentric proliferation of acini within dense hyaline sclerotic stroma
          • Intraluminal calcifications may be present
      • Cystic fibroadenoma:
      • Pseudoangiomatous stromal hyperplasia:
        • Stroma with prominent slit-like spaces lined by bland spindle cells; the slit-like spaces are empty and have an anastomosing appearance
      Board review style question #1
      Which of the following is true about proliferative fibrocystic changes in the breast?

      1. Apocrine metaplasia is considered a proliferative fibrocystic change
      2. Duct ectasia is considered a proliferative fibrocystic change
      3. Proliferative fibrocystic change has no associated increased lifetime risk of breast cancer
      4. Proliferative fibrocystic change without atypia has an associated 1.5 - 2 fold increased lifetime risk of breast cancer
      Board review style answer #1
      D. Proliferative fibrocystic change without atypia has an associated 1.5 - 2 fold increased lifetime risk of breast cancer

      Comment Here

      Reference: Fibrocystic changes
      Board review style question #2

      A 40 year old woman presents with heterogeneous echogenic tissue in the breast on ultrasound. Which of the following is shown in the biopsy above?

      1. Atypical ductal hyperplasia
      2. Columnar cell change
      3. Duct ectasia
      4. Flat epithelial atypia
      Board review style answer #2
      B. Columnar cell change. The biopsy shows classic findings of fibrosis, cystic changes and microcalcifications along with the lining epithelium in a couple of cysts lined by columnar epithelium.

      Comment Here

      Reference: Fibrocystic changes

      NST, rare variants
      Choriocarcinomatous / pleomorphic patterns (pending)
      Clear cell (pending)
      Glycogen rich
      Definition / general
      • Breast carcinoma in which at least 90% of the neoplastic cells have abundant clear cytoplasm due to glycogen
      • First described in 1981 (Cancer 1981;48:2003)
      • Rare, 1% - 3% of breast carcinomas
      • May be a variant of apocrine carcinoma

      Radiology images

      Images hosted on other servers:

      Mammogram shows circumscribed mass



      Prognostic factors

      Case reports

      Gross images

      Images hosted on other servers:

      Solid papillary pattern within cystically dilated duct



      Microscopic (histologic) description
      • Solid or solid / papillary patterns of large clear cells with distinct cell borders containing glycogen in 90% or more cells
      • Often associated with intraductal component of varied type
      • Often has apocrine features
      • Cells have clear to granular cytoplasm
      • May have scant intracellular mucin (Histopathology 1987;11:857)
      • No cytoplasmic vacuoles

      Microscopic (histologic) images

      AFIP images

      PAS+ dark granules

      Clear cells

      Sheets of polygonal cells



      Images hosted on other servers:

      Ample clear cytoplasm

      PAS positive

      Abundant calcifications

      Resembles renal clear cell carcinoma


      Intraductal lipid rich carcinoma with component of glycogen rich carcinoma

      CK8 / 18+

      Polygonal cells



      Cytology description
      • Hypercellular with tumor cells in loosely cohesive syncytial groups and some single cells
      • Most tumor cells have abundant, finely granular eosinophilic cytoplasm or foamy to clear cytoplasm with well defined cytoplasmic membranes and moderate / marked nuclear pleomorphism with central round/oval nuclei containing prominent nucleoli (Acta Cytol 2008;52:65)
      • PAS staining may be helpful (J Med Invest 2002;49:193)

      Positive stains

      Negative stains

      Electron microscopy description
      • Non membrane bound glycogen and empty glycogen lakes
      • Tight junctions between tumor cells, immature desmosomes, occasional short microvilli (Am J Surg Pathol 1986;10:553)

      Differential diagnosis

      Additional references
      Lipid rich
      Definition / general
      • 90%+ cells have prominent intracytoplasmic neutral lipid

      Clinical features
      • 1% - 2% of breast carcinomas
      • Axillary metastases may resemble histiocytes

      Prognostic factors
      • Poor prognosis due to frequent (70%) nodal metastases at presentation

      Case reports

      Gross description
      • Lobulated, variable circumscription, firm
      • 1 to 15 cm

      Microscopic (histologic) description
      • Nests, cords and sheets of large polygonal cells with foamy or vacuolated cytoplasm containing lipid
      • May resemble clear cells or lipoblasts
      • Irregular nuclei with coarse chromatin, moderate atypia, prominent nucleoli
      • Other patterns are large pleomorphic cells in alveolar pattern with hobnail appearance, oncocytic or apocrine type change

      Microscopic (histologic) images

      AFIP images

      Polygonal tumor cells



      Positive stains

      Negative stains

      Electron microscopy description

      Electron microscopy images

      AFIP images

      Luminal microvilli, lipid droplets and mitochondria



      Differential diagnosis
      Oncocytic (pending)
      Osteoclast
      Definition / general
      • Part of WHO classification
      • Osteoclastic giant cells are present in stroma
      • Presence of giant cells does not alter prognosis

      Case reports

      Gross description
      • Often brown due to vascular stroma with hemosiderin

      Microscopic (histologic) description
      • Giant cells are associated with vascular stroma with extravasated red blood cells and hemosiderin, also chronic inflammatory cells and fibroblasts
      • Giant cells have variable size and variable numbers of nuclei
      • Similar histologic features in nodal metastases and recurrences (Arch Pathol Lab Med 1986;110:636)
      • Carcinoma component may be any type

      Microscopic (histologic) images

      Images hosted on other servers:

      Invasive ductal carcinoma of the breast with osteoclast-like giant cells

      Various images



      Differentiated well cribriform carcinoma

      Osteoclast-like giant cells in close contact to tumor cells



      Cytology description

      Positive stains

      Negative stains

      Electron microscopy description
      • Osteoclasts are histiocytes

      Additional references
      Sebaceous
      Definition / general
      • Very rare primary breast carcinoma resembling skin adnexal tumor with sebaceous differentiation, but no evidence of cutaneous derivation (see J Med Case Rep 2008;2:276 for cutaneous tumor of breast)

      Clinical features

      Case reports

      Microscopic (histologic) description
      • Well defined solid sheets or lobules of atypical epithelial cells, including large, pale or clear cells with coarsely vacuolated cytoplasm, containing Oil red O staining lipid and often scalloped nuclei
      • Often focal squamous morules

      Microscopic (histologic) images

      Images hosted on other servers:

      Skin



      Positive stains

      Negative stains

      Electron microscopy description
      • Empty appearing, non membrane bound vacuoles

      Differential diagnosis
      Signet ring cell
      Definition / general

      Case reports

      Microscopic (histologic) description
      • Mucin fills cytoplasm and displaces nucleus
      • Usually coexists with invasive ductal NOS
      • High nuclear grade; DCIS may be present, but no lobular features (by definition)

      Microscopic (histologic) images

      Images hosted on other servers:

      Breast metastasis to stomach



      Cytology description
      • Look for signet ring cells if plasmacytoid cells are identified
      • Favor ductal carcinoma if hypercellular, single signet ring cells, high nuclear grade and tubule formation
      • Favor lobular carcinoma if hypocellular, single signet ring cells and mild / moderate nuclear grade (Cytopathology 2009;20:321)

      Differential diagnosis

      Other sarcomas
      Leiomyosarcoma
      Definition / general
      • Malignant smooth muscle neoplasm of the breast

      Essential features
      • Rare, < 1% of all breast malignancies
      • < 50 primary leiomyosarcomas of the breast reported in the literature
      • Should be distinguished from metastatic tumor

      Etiology
      • Primary tumors likely arise from smooth muscle of the nipple or vascular wall
      • Have been reported to occur secondary to radiation (Surg Case Rep 2015;1:76)

      Clinical features

      Radiology description
      • Mammographically presents as a mass or architectural distortion
      • By ultrasound, lesions are homogenously hypoechoic, lobular or oval with microlobulated or indistinct margins (Acta Radiol 2011;52:597)

      Prognostic factors
      • Type of surgery does not affect prognosis as long as the tumor is excised with an adequate (2 - 3 cm) margin (Breast 2011;20:389)
      • Due to the rare nature of this tumor, no specific prognostic features are established in primary leiomyosarcoma arising in the breast
      • If the tumor is cutaneous and limited to the dermis of the breast skin (i.e., atypical intradermal smooth muscle neoplasm), rather than arising in the mammary parenchyma, prognosis is excellent without metastases

      Case reports

      Treatment
      • As no axillary lymph node metastases have been reported with primary breast leiomyosarcoma, sentinel lymph node excision is not indicated (Breast 2011;20:389)
      • Wide excision with at least 2 cm margin but 3 cm margin is recommended (Breast 2011;20:389)
      • In tumors > 3 cm or with inadequate excision, radiotherapy may be indicated
      • Adjuvant chemotherapy may result in greater clinical response in radiation induced sarcomas (Surg Case Rep 2015;1:76)

      Gross images

      Images hosted on other servers:

      Lobulated elastic tumor



      Microscopic (histologic) description
      • Intersecting fascicle of spindled cells with abundant eosinophilic cytoplasm and cigar shaped nuclei demonstrating variable pleomorphism, mitotic activity and necrosis
      • Tumor borders are typically infiltrative

      Microscopic (histologic) images

      Contributed by Emily S. Reisenbichler, M.D., Mark R. Wick, M.D. and AFIP

      Interesecting fascicles, pleomorphic nuclei and mitotic activity

      Nuclear pleomorphism and mitotic activity

      Dense cellularity and intersecting fascicles


      Dermis involvement

      Parenchyma

      Desmin



      Cytology description
      • Large, dissociated round to spindle cells, medium to large, with abundant vacuolated cytoplasm with occasional intranuclear cytoplasmic invaginations
      • Mitotic activity, including abnormal forms, nuclear atypia and a necrotic background favor a malignant process
      • Immunohistochemistry is typically required to distinguish leiomyosarcoma from other malignant spindle cell tumors of the breast (Diagn Pathol 2006;1:13)

      Cytology images

      Images hosted on other servers:

      Spindle cell lipoma



      Positive stains

      Negative stains

      Molecular / cytogenetics description

      Differential diagnosis
      Low grade myofibroblastic sarcoma
      Definition / general
      • Spindle cell sarcoma with myofibroblastic differentiation
      • Also called myofibrosarcoma

      Essential features
      • Uncommon tumor overall, occurring predominantly in the head and neck or extremities with only rare reports within the breast
      • At least moderate cytologic atypia must be seen focally
      • Not part of WHO breast classification

      Prognostic factors
      • Frequently recurs, particularly when incompletely excised
      • Low rates of metastasis but lung is most common site when it occurs (Am J Surg Pathol 2001;25:219)

      Case reports

      Treatment

      Gross description
      • Well circumscribed tumor with firm, pale cut surface, despite infiltrative microscopic growth

      Microscopic (histologic) description
      • Spindle cells in a fascicular or storiform growth pattern
      • Variable stromal collagen
      • Cells demonstrate pale eosinophilic cytoplasm and the nuclei must show at least moderate atypia focally and hyperchromasia
      • Diffusely infiltrative borders

      Microscopic (histologic) images

      Contributed by Mark R. Wick, M.D.

      Low grade myofibroblastic sarcoma



      Positive stains

      Negative stains

      Electron microscopy description
      • Prominent rough endoplasmic reticulum, peripheral myofilaments and the fibronectin fibrils of the fibronexus junctions (J Clin Pathol 2009;62:236)

      Molecular / cytogenetics description
      • Gains at 1p11 → p36.3 (66%), 12p12.2 → p13.2 (45%), 5p13.2 → p15.3 (31%), +22 (28%), loss at 15q25 → q26.2 (24%) (Am J Clin Pathol 2009;131:701)
      • Complex DNA copy number changes
      • Higher average number of chromosomal aberrations than seen in nodular fasciitis

      Differential diagnosis
      Osteosarcoma
      Definition / general
      • Malignant osteoid producing neoplasm

      Essential features
      • Exceedingly rare tumor in the breast with < 150 reported cases in the literature
      • Comprised of cytologically malignant cells with osteoid production, lacking morphologic or immunohistochemical evidence of an epithelial / carcinomatous component
      • Must extensively sample tumors to exclude diagnoses of malignant phyllodes with osteosarcomatous differentiation and metaplastic carcinoma with osteogenic matrix production
      • Some experts believe that most, if not all, reported osteosarcomas of the breast actually represent matrix producing carcinomas (Breast 2013;22:13)

      Etiology

      Clinical features
      • Typically presents in the fifth or sixth decade of life
      • Large, palpable mass

      Radiology description
      • Irregular hyperdense mass by mammography
      • Hypoechoic mass by ultrasound

      Prognostic factors

      Case reports

      Treatment
      • Complete resection with negative margins
      • Routine axillary lymph node dissection not necessary due to tendency to metastasize to lungs rather than regional lymph nodes
      • Utility of adjuvant radiation or chemotherapy is unclear due to rarity of this tumor type in this location

      Gross description
      • Well circumscribed large tumor with firm to gritty cut surface
      • Difficulty in sectioning possible if abundant osteoid production present

      Gross images

      Contributed by Nitin Marwaha, M.D.
      Missing Image

      Well circumscribed, bony hard mass



      Microscopic (histologic) description
      • Spindle to epithelioid cells with nuclear atypia, hyperchromasia and mitotic activity, including atypical forms
      • Lace-like osteoid production
      • Absence of an in situ carcinoma component

      Microscopic (histologic) images

      Contributed by Emily S. Reisenbichler, M.D. and Nitin Marwaha, M.D.

      Osteosarcoma, 40x magnification

      Osteosarcoma, 60x magnification

      Missing Image Missing Image Missing Image Missing Image

      45 year old woman with right breast mass



      Positive stains

      Negative stains
      • Cytokeratins (typically high molecular weight keratins such as CK5 / 6) and myoepithelial markers (such as p63): to rule out a metaplastic carcinoma

      Differential diagnosis
      Rhabdomyosarcoma
      Definition / general
      • Malignant neoplasm with skeletal muscle differentiation

      Essential features
      • Comprises < 3% of all adult primary soft tissue sarcomas and exceedingly rare as a primary breast tumor
      • Primary rhabdomyosarcoma of the breast is most commonly alveolar subtype and is seen more frequently in the pediatric / adolescent population (Med Pediatr Oncol 1997;29:181)
      • Typically composed of small, round to spindled cells, with eosinophilic cytoplasm, some of which show cytoplasmic cross striations (embryonal and alveolar subtypes); larger cells with greater degree of atypia seen in pleomorphic subtype
      • Must exclude other tumor types with heterologous rhabdomyosarcomatous differentiation (such as malignant phyllodes)

      Clinical features
      • Primary rhabdomyosarcoma of the breast is seen most frequently in the pediatric / adolescent population and therefore is not typically identified by routine screening mammography
      • Presents as a palpable, circumscribed, rapidly growing mass
      • Presentation as a metastasis from a primary rhabdomyosarcoma elsewhere in the body occurs in 3 - 6% cases and portends a very poor prognosis (J Pediatr Hematol Oncol 2017;39:62)

      Radiology description
      • Due to the breast density of the pediatric and adolescent population, ultrasound is the imaging modality of choice for radiographic characterization of this tumor type (Eur J Radiol 2003;48:282)
      • Hypoechoic, round or lobulated nodule / mass by ultrasound

      Prognostic factors
      • Patients with primary breast alveolar rhabdomyosarcoma have a poor prognosis, frequently dying of disease with disseminated metastases (J Pediatr Hematol Oncol 2017;39:62)
      • Embryonal subtype portends much better prognosis than alveolar or pleomorphic subtypes

      Case reports

      Treatment
      • As with all primary breast sarcomas, wide excision without axillary dissection is the mainstay of treatment
      • Adjuvant or neoadjuvant radiation often used in large or incompletely resected tumors
      • Utility of chemotherapy is unclear due to the rarity of this tumor type

      Clinical images

      Contributed by Mark R. Wick, M.D.

      Alveolar



      Gross description
      • Circumscribed, nonencapusulated mass

      Gross images

      Images hosted on other servers:

      Large fungating mass involving right breast

      Large firm lobulated tumor



      Microscopic (histologic) description
      • 3 microscopic subtypes:
        • Alveolar: sheets of small, round cells clustered with variable amounts of fibrous septa; may contain scattered giant cells
        • Embryonal: cytologically round to spindle cells with scant cytoplasm in a myxoid background; elongated cells with more abundant eosinophic cytoplasm referred to as strap cells or tadpole cells
        • Pleomorphic: sheets of large cells demonstrating marked nuclear atypia or "anaplasia" with eosinophilic cytoplasm

      Microscopic (histologic) images

      Contributed by Mark R. Wick, M.D.

      Alveolar

      Alveolar with differentiation

      Embryonal



      Cytology images

      Contributed by Mark R. Wick, M.D.

      Embryonal, FNAB



      Positive stains

      Negative stains

      Electron microscopy description
      • Skeletal differentiation, as evidenced by ultrastructural specific myofilament arrays, focal density, Z band material and pinocytotic vesicle, is suggestive of rhabdomyosarcomatous differentiation but is not specific for the diagnosis of rhabdomyosarcoma (J Pathol Transl Med 2015;49:93)
      • Features may not be evident in solid variants of the alveolar subtype

      Molecular / cytogenetics description
      • Alveolar: t(2;13)(q35;q4), PAX3::FOXO1 fusion more frequent than t(1;13)(p36;q14), PAX7::POXO1 fusion
      • Embryonal: chromosome loss, deletion or uniparental disomy resulting in loss of chromosome 11 loci
      • Pleomorphic: complex karyotype

      Differential diagnosis
      Stromal sarcoma
      Definition / general
      • The 4th WHO edition (2012) reclassified this periductal stromal tumor as the preferred neutral term in the family of fibroepithelial tumor
      • Stromal proliferation around open, benign tubules

      Essential features
      • Considered a type of fibroepithelial tumor lesion with stromal proliferation around benign tubules
      • Lacks the defined tumor borders and compressed tubules of fibroadenoma and leaf-like architecture of phyllodes tumor
      • Must demonstrate stromal atypia and mitotic activity to be considered a sarcoma

      Clinical features
      • Occurs in peri and postmenopausal women (median 55.3 years), ~10 years older than the median phyllodes presentation (Am J Surg Pathol 2003;27:343)
      • Variable clinical presentation

      Prognostic factors

      Case reports

      Treatment
      • Given the likely relationship to fibroepithelial lesions, treatment is similar to that of phyllodes tumor
      • Wide excision with negative margins
      • As with primary breast sarcoma, assessment of axillary sentinel lymph nodes is not necessary
      • Adjuvant chemotherapy and radiation are not typically warranted unless presenting with a higher grade sarcomatous component

      Clinical images

      Contributed by Mark R. Wick, M.D.

      Mammogram



      Gross description
      • Ill defined nodular mass (mean 3 cm)

      Microscopic (histologic) description
      • Increased stromal cellularity of spindled stroma surrounding or cuffing benign ducts in a pericanalicular pattern
      • Involved tubules and ducts are not malignant
      • Stromal cells demonstrate at least moderate cytologic atypia and mitotic activity
      • Infiltrative and often multinodular microscopic border
      • May be seen with areas of angiosarcomatous or liposarcomatous components (Am J Surg Pathol 2003;27:343, Int J Surg Pathol 2015;23:221)

      Microscopic (histologic) images

      Contributed by Emily S. Reisenbichler, M.D. and Mark R. Wick, M.D.

      Cytologically atypical spindled cells cuffing benign ducts

      Numerous mitoses present in
      cytologically atypical stromal
      cells adjacent to ducts

      Various images



      Positive stains
      • CD34 (mostcases), CD117 (approximately ¼ of cases), HHF35 (< 10% of cases)

      Negative stains

      Electron microscopy description
      • Undifferentiated mesenchymal cell features such as free ribosomes and irregular profiles of rough endoplasmic reticulum
      • Fibroblastic differentiation such as well organized dilated rough endoplasmic reticulum, Golgi complexes, cytoplasmic filaments and pericellular collagen (Cancer 1979;43:209)

      Differential diagnosis
      • Periductal stromal hyperplasia:
        • Nodular growth, lacks the cytologic atypia and mitoses typical of a stromal sarcoma (0 - 2 mitoses per 10 high power fields)
      • Phyllodes tumor:
        • Has a more defined tumor border with compression and distortion of ductal elements causing a leaf-like architecture
        • Abundant stromal overgrowth of intralobular stroma, not limited only to periductal areas
      Undifferentiated pleomorphic sarcoma
      Definition / general
      • See also Soft tissue topic
      • A malignant tumor showing no line of differentiation
      • Not part of the WHO classification of breast tumors
      • Previously known as malignant fibrous histiocytoma (MFH)

      Essential features
      • High grade sarcoma with no distinctive microscopic, immunohistochemical or molecular features fitting any specific line of differentiation
      • Diagnosis of exclusion
      • Undifferentiated sarcomas account for ~20% of soft tissue sarcomas in other parts of the body but are exceedingly rare in the breast

      Etiology
      • May occur as a primary / de novo tumor or in the postradiation setting

      Clinical features
      • Most occur in fifth to sixth decade of life
      • Majority present as rapidly enlarging palpable mass but can be incidental finding at screening

      Diagnosis
      • A limited core biopsy may underestimate tumor grade as grading systems such as the 3 tier French (FNCLCC) system are dependent on percentage necrosis and mitotic rate

      Radiology description

      Prognostic factors
      • Close / involved margins, large tumor size (> 5 cm) and high grade histology typically indicate a worse prognosis

      Case reports

      Treatment
      • Surgical resection with a negative margin (R0) is standard treatment
      • Sentinel lymph node biopsy is not warranted
      • Adjuvant radiation is often utilized in large tumors or following incomplete (R1 / R2) surgical resections
      • Response to adjuvant chemotherapy is typically low but may be used in patients with large, unresectable or metastatic tumors

      Clinical images

      Images hosted on other servers:

      Cystic tumor with skin tethering



      Gross images

      Images hosted on other servers:

      Tumor invades skin

      Specimen includes ribs posteriorly



      Microscopic (histologic) description
      • Cell morphology may be spindle, round / epithelioid or pleomorphic, demonstrating marked cytologic atypia
      • Often high mitotic activity
      • May be associated with multinucleated giant cells or an inflammatory background

      Microscopic (histologic) images

      Contributed by Emily S. Reisenbichler, M.D. and Mark R. Wick M.D.
      Missing Image

      Spindle and epithelioid cells

      Pleomorphic cells

      Various images



      Cytology description
      • Single or clustered spindled cells with nuclear pleomorphism
      • Absence of an epithelial component

      Positive stains

      Negative stains
      • Typically negative for most markers

      Molecular / cytogenetics description
      • Complex karyotypes without specific recurrent aberrations

      Differential diagnosis

      Paget disease
      Definition / general
      • Mammary Paget disease (MPD) is a proliferation of malignant glandular epithelial cells (in situ carcinoma) in the nipple areolar epidermis (Histopathology 2020;77:181)
      • Disease is named after the pathologist and surgeon Sir James Paget, who published his findings of a relationship between a nipple rash and mammary gland tumors in 1874 (JAMA Dermatol 2018;154:335)
      Essential features
      • Uncommon clinical presentation of breast cancer
      • Most often presents as an eczematous / erythematous change of the nipple areolar skin
      • Cutaneous manifestation is due to tumor cells involving the epidermis and disrupting intercellular junctions
      • Underlying high grade ductal carcinoma in situ (DCIS) or invasive carcinoma is present in > 95% of patients
      • Majority of Paget cells and associated underlying carcinoma are HER2+
      Terminology
      • Mammary Paget disease
      • Paget disease of the breast
      • Paget disease of the nipple
      • DCIS involving nipple skin
      • Paget disease with dermal invasion occurs when Paget cells cross the basement membrane of the epidermis; the invasive component is usually small in size (Arch Pathol Lab Med 2013;137:72)
      • Secondary Paget disease occurs when invasive carcinoma arising deep in the breast invades the skin; the invasive carcinoma is usually large in size
      ICD coding
      • ICD-O: 8540/3 - Paget disease of breast
      • ICD-10: C50.019 - malignant neoplasm of nipple and areola, unspecified female breast
      • ICD-11: 2E65.5 & XH3E21 - Paget disease of nipple and Paget disease, mammary
      Epidemiology
      • Relatively uncommon; occurs in 1 - 4% of women and 1 - 2% of men with breast cancer (Breast Cancer Res Treat 2008;111:313)
      • Wide age range of 20 - 90 years, with a peak incidence between sixth and seventh decade (mean: 64 years) (Breast Dis 2020;39:119)
      • Between 20 - 30% of patients are premenopausal
      • Histologic (subclinical) evidence of MPD may be more frequent
      Sites
      Pathophysiology
      • 2 theories on the pathogenesis:
        • Epidermotropic theory (most widely accepted): Paget cells are DCIS cells that migrate along the basement membrane of the nipple, supported by the presence of DCIS deeper in the breast identical to Paget cells in almost all cases; migration may be mediated through a motility factor heregulin α, produced by keratinocytes and exerting its effect via the HER2 receptor (J Natl Cancer Inst 2000;92:622)
        • Transformation theory: Paget cells originate from malignant transformation of keratinocytes or Toker cells; it would explain rare cases (< 5%) in which cancer is not present in underlying breast (Intractable Rare Dis Res 2019;8:203)
      • When no parenchymal cancer is identified, there is a possibility that the underlying tumor has gone undetected / unsampled
      Etiology
      • Specific risk factors beyond those recognized for breast cancer have not been identified
      Diagrams / tables

      Images hosted on other servers:
      Development of Paget disease

      Development of Paget disease

      Clinical features
      • Majority of patients present clinically with slow onset nipple changes (Breast Cancer Res Treat 2008;111:313)
      • Most common: nipple erythema, scaling crust, ulceration, eczematoid type rash, pruritus or bleeding
      • Less common: bloody nipple discharge, pain, retraction or no clinically identifiable changes of the nipple
      • May present as a pigmented macule and can be mistaken for melanoma (Am J Dermatopathol 2009;31:223, Br J Dermatol 2018;178:e143)
      • Changes begin at the nipple / areola and may spontaneously regress or continue to progress with extension into the skin of the breast
      • Approximately 50% of patients present with an associated palpable mass, usually indicating an underlying invasive carcinoma
      • Affected men have similar clinical presentations as women (Am J Dermatopathol 2020;42:981)
      • Clinical presentation may be mistaken for skin inflammatory disease or infection, leading to delayed diagnosis
      Diagnosis
      • In symptomatic cases, the diagnosis is primarily clinical and confirmed by histologic examination
      • Most commonly used techniques include skin punch or shave biopsy and less often, core needle biopsy; samples are usually small due to concern about the cosmetic appearance of the nipple (Breast J 2019;25:1328, Radiol Med 2021;126:936)
      • Nipple scrape cytology can detect tumor cells on the surface of the epidermis or in the scale crust but it may be painful for the patients and is rarely used (Diagn Cytopathol 2019;47:249)
      • In addition to nipple biopsy, any underlying masses or mammographic abnormalities must be biopsied, usually by core needle; in patients with a palpable lump, alternatively by fine needle aspiration cytology
      • If these methodologies are not definitive for a preoperative diagnosis, an open surgical biopsy / wedge excision may be needed
      • In asymptomatic / unsuspected cases, MPD may be an incidental finding in a mastectomy specimen; detecting Paget cells depends on the extent of sampling of the nipple and the use of IHC (Am J Surg 1979;138:135)
      Radiology description
      • Imaging is mainly used to detect and evaluate the extent of an underlying malignancy and enable appropriate treatment planning (Eur J Radiol 2006;60:256, Radiographics 2011;31:1973)
      • Mammography is most useful in detecting masses, distortions or calcifications; however, approximately half of the patients with MPD have normal mammograms
      • Ultrasound may show ductal ectasia or changes of the nipple areolar complex, such as flattening, asymmetry and thickening
      • Magnetic resonance imaging (MRI) is sensitive, particularly for mammographically occult underlying disease; it may show abnormal nipple enhancement, thickened nipple areolar complex, associated enhancing DCIS or invasive tumor or a combination of these findings (J Am Coll Surg 2008;206:316)
      Radiology images

      Images hosted on other servers:
      Abnormal mammogram

      Abnormal mammogram

      Normal mammogram

      Normal mammogram

      Ultrasound

      Ultrasound

      MRI

      MRI

      Prognostic factors
      • Prognosis largely depends on the presence, extent and characteristics of the underlying tumor
      • Patients with MPD and DCIS only have excellent outcomes (> 95% survival at 20 years) (Cancer 2006;107:1448)
      • Patients with isolated MPD (no parenchymal tumor detected) have even better outcomes; 5 and 10 year survival rates of 100%, compared with those with underlying DCIS of 94.7% and 75.8%, in 1 study (Am Surg 2013;79:1009)
      • If invasive carcinoma is present in the underlying breast, prognosis depends on tumor size and lymph node status, with the 5 year relative survival rate declining with increasing tumor stage (stage I, 95.8%; II, 77.7%; III, 46.3%; IV, 14.3%) (Breast Dis 2020;39:119)
      • In general, women presenting with MPD and a palpable mass (indicating more advanced disease) have worse survival than women without a palpable mass
      • Dermal invasion arising directly from MPD in the epidermis and localized to the nipple (invasive Paget disease) is typically detected early and has a favorable outcome (Arch Pathol Lab Med 2013;137:72, Hum Pathol 2014;45:2480)
      • MPD must be distinguished from inflammatory carcinoma and locally advanced invasive carcinoma ulcerating skin, as they have poorer prognoses
      Case reports
      Treatment
      • Extent of surgery is determined by the extent of underlying breast carcinoma (Am J Surg 2010;200:241)
      • Patients with limited disease are candidates for breast conserving therapy with central lumpectomy and whole breast irradiation (J Surg Res 2019;241:178)
      • Those with larger extent of disease (multicentric cancer, diffuse calcifications) undergo simple mastectomy
      • Entire nipple areolar complex is removed in all cases; a variety of techniques can be used to reconstruct the nipple
      • Evaluation and treatment of the axilla in MPD are the same as for any breast cancer (Am J Surg 2006;192:481)
      • Recommendations regarding systemic therapy in neoadjuvant or adjuvant settings are based solely upon any associated invasive carcinoma or DCIS characteristics (Breast Cancer Res Treat 2008;112:513)
      Clinical images

      Contributed by Jennifer Tseng, M.D. and Mark R. Wick, M.D.
      Nipple, crusted lesion

      Nipple, crusted lesion

      Nipple erosion and erythema

      Nipple erosion and erythema



      Images hosted on other servers:
      Well demarcated, eczematous scaly lesion

      Well demarcated, eczematous scaly lesion

      Gross description
      • Nipple areolar complex with red-pink crusting lesion, discoloration, thickening, ulceration, exudate, nipple retraction, etc.
      • Typically well demarcated; epidermal involvement usually does not extend beyond the grossly evident lesion (Cancer Treat Rev 2001;27:9)
      • Scaling crust may be removed by skin preparation prior to surgery
      Gross images

      Contributed by Alexis Synder, M.H.S. and Emily S. Reisenbichler, M.D.
      Nipple ulceration

      Nipple ulceration

      Nipple ulceration, enlarged

      Nipple ulceration, enlarged

      MPD, cut section with underlying mass

      MPD, cut section with underlying mass

      Frozen section description
      • Frozen section for intraoperative management of Paget disease has been studied more extensively in extramammary sites but not in MPD
      Microscopic (histologic) description
      • Single cells or clusters of cells spread throughout the epidermis
      • Cells have abundant pale cytoplasm, large irregular nuclei with prominent nucleoli
      • Underlying dermis may have chronic inflammation
      • Epidermis with hyperkeratosis and possibly ulceration
      • Florid cases can show gland formation
      • Paget cells may phagocytose melanin, mimicking melanocytes
      • Underlying carcinoma is most commonly high grade invasive carcinoma of no special type (NST; 53 - 64%) or DCIS (24 - 43%) (Histopathology 2020;77:181)
      Microscopic (histologic) images

      Contributed by Anna Biernacka, M.D., Ph.D. and Mary Ann Gimenez Sanders, M.D, Ph.D.
      Paget disease with dermal reaction

      Paget disease with dermal reaction

      Paget cells in basal location

      Paget cells in basal location

      Superficial extension and inflammation

      Superficial extension and inflammation

      Cytologic features

      Cytologic features

      Spread to skin adnexa

      Spread to skin adnexa

      Paget cells, shrinkage artifact

      Paget cells, shrinkage artifact


      Paget cells, melanin uptake

      Paget cells, melanin uptake

      Melanocyte hyperplasia

      Melanocyte hyperplasia

      Full thickness with erosion Full thickness with erosion

      Full thickness with erosion

      Invasive Paget disease Invasive Paget disease

      Invasive Paget disease


      Mammary Paget disease CK7

      Mammary Paget disease CK7

      Mammary Paget disease CK5/6

      Mammary Paget disease CK5/6

      Mammary Paget disease HER2

      Mammary Paget disease HER2

      Secondary Paget disease

      Secondary Paget disease

      Florid Paget disease

      Florid Paget disease

      Virtual slides

      Images hosted on other servers:
      Paget cells involving nipple ducts

      Paget cells involving nipple ducts

      Cytology description
      • Tumor cells are often present on the surface of the epidermis and in the scale crust
      • Nipple scrape preparation can be performed by using a scalpel to scrape the nipple, smearing it on 2 slides and performing either a Papanicolaou or May-Grunwald-Giemsa (MGG) stain
      • 1 study, by comparing scrape smears and histological specimens, found that nipple scraping correctly detected MPD in 91.4% of cases (Diagn Cytopathol 2019;47:249)
      • Cytomorphologic features include isolated and loosely clustered malignant glandular cells with enlarged nuclei, prominent nucleoli and pale cytoplasm, detected among squamous cells
      • Cytologic differential diagnoses include nipple adenoma and squamous cell carcinoma detected
      • In cases of clinical doubt, a punch biopsy may be indicated in parallel to scrape smears
      Cytology images

      Images hosted on other servers:
      Dyscohesive cell groups

      Dyscohesive cell groups

      Positive stains
      Electron microscopy description
      • Desmosomal attachments have been identified between Paget cells and adjacent epidermal keratinocytes (Am J Pathol 1969;57:49)
      • Same authors described a pre-Paget cell with an electron microscopy appearance intermediate between that of a keratinocyte and a Paget cell, suggesting that epidermal cells can acquire the characteristics of ductal cells as they undergo malignant transformation (transformation theory)
      Electron microscopy images

      Images hosted on other servers:
      Large, pale intraepidermal Paget cells

      Large, pale intraepidermal Paget cells

      Single Paget cell

      Single Paget cell

      Molecular / cytogenetics description
      • No association with a specific gene mutation has been reported
      • Paget cells are genetically similar to the underlying carcinoma cells in 80% of cases (Histopathology 2020;77:181)
      Videos

      Brief overview of Paget disease with pathologic images by Pathology mini tutorials

      Sample pathology report
      • Right nipple areolar complex, biopsy:
        • Ductal carcinoma in situ involving the nipple epidermis (Paget disease) (see comment)
        • Comment: The tumor cells are positive for CK7 and HER2 (3+) by immunohistochemistry, supporting the above diagnosis of Paget disease. ER and PR show no staining of in situ nuclei.

      • For synoptic reporting, the appropriate AJCC T category is Tis (Paget) if no underlying DCIS is present or Tis (DCIS) if underlying DCIS is present
      • If there is associated invasive carcinoma in the underlying breast parenchyma, the T category is based on the size of the invasive tumor
      • If there is invasion from MPD into the dermis, only the size of the invasion should be used as size for T classification
      Differential diagnosis
      Board review style question #1
      Which immunohistochemical stain is most likely to be positive in Paget disease?

      1. CK5/6
      2. CK20
      3. ER
      4. HER2
      5. PR
      Board review style answer #1
      D. HER2. The majority of Paget disease cases are HER2 positive and therefore HER2 can be used to confirm the diagnosis. In the absence of invasive carcinoma, HER2 positivity does not influence treatment, since Paget disease is an in situ carcinoma. Paget disease is negative for CK5/6 and CK20. ER and PR show variable expression.

      Comment Here

      Reference: Paget disease
      Board review style question #2

      Which of the following is true regarding the lesion identified in the nipple in the image shown above?

      1. Invasion into the dermis is associated with a poor prognosis
      2. Lesional cells can contain melanin pigment
      3. Lesional cells have a specific recurring genetic alteration
      4. Most disease is an extension of lobular carcinoma in situ (LCIS) from the underlying ducts
      5. Nipple always has eczematoid change
      Board review style answer #2
      B. Paget cells may contain melanin, which is a diagnostic pitfall. Invasion into the dermis is not associated with a poor prognosis. The nipple in Paget disease may appear grossly normal. Most mammary Paget disease (MPD) is an extension of ductal carcinoma in situ (DCIS) from the underlying ducts, not LCIS. Paget cells have no known specific recurring genetic alterations and are genetically similar to the underlying breast carcinoma cells in most cases.

      Comment Here

      Reference: Paget disease

      Papillary
      Definition / general
      Essential features
      • A rare subtype of invasive ductal carcinoma with infiltrative papillary growth
      • Imperative to distinguish it from other invasive and noninvasive mammary lesions, as well as extramammary tumors metastasizing to the breast
      Terminology
      • Invasive papillary carcinoma (IPC)
      ICD coding
      • ICD-O: 8503/3 - invasive papillary carcinoma
      • ICD-11:
        • XH8KR8 - papillary carcinoma of breast
        • 2C60 - carcinoma of breast, specialized type
      Epidemiology
      Sites
      • No specific location in the breast
      Etiology
      Clinical features
      • Due to the rarity of this tumor, specific clinical features have not been described
      • Typically presents with bloody nipple discharge, an abnormal mass or radiographic abnormalities
        • However, it is not clear if these features refer to true IPC or other types of papillary carcinomas misdiagnosed as IPC (J Surg Res 2021;261:105)
      Diagnosis
      • Mammogram
      • Breast ultrasound (US)
      • Breast MRI
      Radiology description
      Prognostic factors
      Case reports
      Treatment
      Gross description
      Microscopic (histologic) description
      Microscopic (histologic) images

      Contributed by Marilin Rosa, M.D.
      Invasive mixed papillary and micropapillary carcinoma

      Invasive mixed papillary and micropapillary carcinoma

      No surrounding fibrous capsule

      No surrounding fibrous capsule

      Desmoplasia

      Desmoplasia

      Invasive papillary carcinoma

      Invasive papillary carcinoma

      No surrounding fibrous capsule

      No surrounding fibrous capsule


      Invasive papillary fronds

      Invasive papillary fronds

      Invasive papillary carcinoma

      Invasive papillary carcinoma

      Papillae lack myoepithelial cells

      Papillae lack myoepithelial cells

      Necrosis and calcifications

      Necrosis and calcifications

      Mitotic figures

      Mitotic figures

      Sample pathology report
      • Right / left breast, mastectomy:
        • Invasive papillary carcinoma, Nottingham histologic grade (X), measuring (X) cm (please see detailed synoptic report)
        • Associated ductal carcinoma in situ (DCIS), (X) nuclear grade, (X) patterns, comprising X% of tumor (if present).
        • Lymphovascular invasion (is / is not) identified.
        • Biopsy site changes are identified.
        • Margins are (involved / uninvolved).
      Differential diagnosis
      Board review style question #1

      Which of the following is a relevant feature of invasive papillary carcinoma of the breast?

      1. Commonly encountered breast tumor
      2. Graded according to nuclear size
      3. May mimic metastases from other organs
      4. Myoepithelial cells at the periphery
      5. Positive for Napsin A and PAX8
      Board review style answer #1
      C. It may mimic metastases from other organs (metastases to the breast from other organs such as ovary, lung and thyroid must be excluded)

      Comment Here

      Reference: Invasive papillary carcinoma

      Pertuzumab
      Definition / general
      • Humanized IgG1 kappa monoclonal antibody targets extracellular domain of HER2 tyrosine kinase receptor
      • Discovered and developed by Genentech, Inc., South San Francisco, CA
      • Synonym: 2C4 (monoclonal antibody)
        • rhuMAb-2C4
      Trade name
      • Perjeta®
      Pathophysiology
      • HER2, a member of HER family, does not have a receptor specific ligand binding site
      • HER2/neu signaling in cancer cells (N Engl J Med 2007;357:39)
        • HER2 heterodimerizes with HER1, HER3 or HER4, phosphorylates and activates intracellular tyrosine kinase domain
        • The activated tyrosine kinase on HER2 activates PI3K-Akt pathway and induces cellular survival
        • The activated tyrosine kinase activates SOS, induces a cascade of activation of RAS-RAF-MAPK-MEK and MAPK, eventually promotes cellular proliferation
        • Cleavage of HER2 extracellular domain produces phosphorylated P95 that could activate downstream signal transduction
      • Mechanism of action of pertuzumab (Oncology (Williston Park) 2014;28:186)
        • Upon binding the extracellular dimerization subdomain II of HER2, pertuzumab reduces activation of HER2 by disrupting dimerization of HER2 with HER1, HER3 or HER4
      • See Diagrams / tables, figure 7
      Diagrams / tables

      Images hosted on other servers:

      Mechanisms of action of trastuzumab and pertuzumab

      Trastuzumab and pertuzumab comparisons

      Clinical information
      Uses by pathologists
      • Identify HER2 positive breast adenocarcinomas
        • HER2 testing needed on all newly diagnosed breast adenocarcinomas
        • Metastatic diseases (test performed in a metastatic site)
      • HER2 immunohistochemistry in breast adenocarcinoma (see Diagrams / tables, figure 1)
        • ASCO-CAP HER2 Test 2013 and 2018 Guideline Recommendation (Arch Pathol Lab Med 2014;138:241, Arch Pathol Lab Med 2018;142:1364)
          • Negative (score 0):
            • No staining
            • Incomplete faint membrane staining ≤ 10% invasive tumor cells
          • Negative (score 1+):
            • Incomplete faint membrane staining > 10% invasive tumor cells
          • Equivocal (score 2+): perform HER2 ISH (see Diagrams / tables, figure 2)
            • Incomplete, weak / moderate membrane staining > 10% invasive tumor cells
            • Complete, intense membrane staining ≤ 10% invasive tumor cells
          • Positive (score 3+):
            • Complete, intense circumferential membrane staining > 10% invasive tumor cells
      • HER2 single probe in situ hybridization (ISH) in breast adenocarcinoma (see Diagrams / tables, figure 2)
        • ISH negative (not amplified)
          • Single probe HER2 copy number < 4.0
        • ISH positive (amplified)
          • Single probe HER2 copy number ≥ 6.0 signals/cell
        • ISH equivocal: perform HER2/CEP17 dual probe ISH (see Diagrams / tables, figure 3)
          • Single probe HER2 copy number ≥ 4.0 and < 6.0 signals/cell
      • HER2 dual probe ISH in breast adenocarcinoma (see Diagrams / tables, figure 3)
        • Negative:
          • Dual probe HER2/CEP17 < 2.0, HER2 copy number < 4.0 signals/cell
        • Positive:
          • Dual probe HER2/CEP17 ≥ 2.0, HER2 copy number ≥ 4.0 signals/cell
        • Equivocal (see Diagrams / tables, figures 4, 5 and 6):
          • HER2/CEP17 ≥ 2.0, with HER2 copy number < 4.0 signals/cell (see Diagrams / tables, figure 4)
          • HER2/CEP17 < 2.0, HER2 copy number ≥ 6.0 signals/cell (see Diagrams / tables, figure 5)
          • HER2/CEP17 < 2.0, HER2 copy number ≥ 4.0 and < 6.0 signals/cell (see Diagrams / tables, figure 6)
      Side effects
      Board review style question #1
        Which of the following drugs target HER2 / neu on cancer cells?

      1. Dabrafenib
      2. Larotrectinib
      3. Pertuzumab
      4. Trametinib
      Board review style answer #1
      C. Pertuzumab

      Comment Here

      Reference: Pertuzumab
      Board review style question #2
        Which of the following results is interpreted as positive staining (score 3+) for HER2 in breast adenocarcinoma?

      1. Complete, intense basolateral membranous staining > 10% of tumor cells
      2. Complete, intense circumferential membranous staining > 10% of tumor cells
      3. Complete, intense luminal membranous staining > 10% of tumor cells
      4. Complete, intense nuclear and cytoplasmic staining > 10% of tumor cells
      Board review style answer #2
      B. Complete, intense circumferential membranous staining > 10% of tumor cells

      Comment Here

      Reference: Pertuzumab

      Phyllodes tumor
      Definition / general
      • Biphasic fibroepithelial neoplasm with leaf-like epithelial (phyllodal) pattern and stromal proliferation
      Essential features
      • Biphasic fibroepithelial lesion characterized by leaf-like phyllodal epithelial pattern
      • Graded and prognosticated by histologic changes of the stromal proliferation
      • Epithelial component is benign in phyllodes tumor
      Terminology
      • Phyllodes tumor
      • Cystosarcoma phyllodes (the use of this term is discouraged)
      ICD coding
      • Phyllodes tumor
        • ICD-O: 9020/1 - Phyllodes tumor, NOS
      • Benign phyllodes tumor
        • ICD-O: 9020/0 - Phyllodes tumor, benign
        • ICD-10: D24 - Benign neoplasm of breast
        • ICD-11: 2F30.3 - Benign phyllodes tumor of breast
        • ICD-11: XH50P7 - Phyllodes tumor, benign
      • Borderline phyllodes tumor
        • ICD-O: 9020/1 - Phyllodes tumor, borderline
        • ICD-11: 2F75 - Neoplasms of uncertain behavior of breast
        • ICD-11: XH5NK4 - Phyllodes tumor, borderline
      • Malignant phyllodes tumor
        • ICD-O: 9020/3 - Phyllodes tumor, malignant
        • ICD-10: D48.6 - Cystosarcoma phyllodes (the use of this term is discouraged)
        • ICD-11: 2C63 - Malignant phyllodes tumor of breast
        • ICD-11: XH8HJ7 - Phyllodes tumor, malignant
      Epidemiology
      Sites
      Pathophysiology
      Etiology
      Clinical features
      Diagnosis
      Radiology description
      • Oval and lobulated lesions
      • Hypoechoic on ultrasound
      • Microcalcifications rare
      Radiology images

      Contributed by Mark R. Wick, M.D.
      Mammogram

      Mammogram, benign

      Prognostic factors
      Case reports
      Treatment
      Clinical images

      Images hosted on other servers:
      Large tumor

      Large tumor

      Malignant tumor

      Malignant tumor

      Swollen breast

      Swollen breast

      Gross description
      • Rounded, shelled out borders
        • Infiltrative in malignant and less commonly borderline phyllodes tumor
      • Whorled, bosselated cut surface in a leaf-like pattern
      • Skin ulceration, hemorrhage and cystic changes in large lesions (Int J Surg Case Rep 2020;67:114)
        • Ulceration and hemorrhage do not indicate malignant behavior per se
        • Infarcted or very large benign phyllodes tumor can also show necrosis
      Gross images

      Contributed by Mark R. Wick, M.D.

      Benign phyllodes tumor



      Images hosted on other servers:

      Large, well circumscribed tumor, benign

      Malignant

      Microscopic (histologic) description
      • Leaf-like (phyllodal) epithelial pattern formed by an exaggerated intracanalicular pattern
      • Subepithelial condensation with increased stromal cellularity adjacent to epithelium
        • Some regard tumors with predominant periductal stromal expansion (i.e., periductal stromal tumor) as a distinct subtype of phyllodes tumor
      • Graded into benign, borderline and malignant histologic grades


        Benign Borderline Malignant
        Stromal atypia Mild Moderate Marked
        Stromal cellularity Mildly increased, can be focal Moderately increased, can be focal Markedly and diffusely increased
        Stromal overgrowth* Absent Absent or very focal Present
        Mitotic count < 5/10 HPF or < 2.5/mm² 5 - 9/10 HPF or 2.5 - < 5/mm² ≥ 10/10 HPF or ≥ 5/mm²
        Tumor border Well defined Well defined or focally permeative Diffusely permeative
        Malignant heterologous elements Absent Absent Presence directly upgrades to malignant category**
          *Defined as absence of epithelial elements containing stroma only in 1 low power field
          **Includes chondrosarcoma, liposarcoma (except well differentiated liposarcoma), osteosarcoma, rhabdomyosarcoma, angiosarcoma and leiomyosarcoma

      • Cystic degeneration, hemorrhage, stromal hyalinization and myxoid change reported
      • Multinucleated stromal giant cells occasionally seen (Pathology 2001;33:153)
        • Can be found in phyllodes tumor of all histologic grades
      • Epithelial elements usually bland
      • Myoepithelial layer is preserved but can be attenuated
      Microscopic (histologic) images

      Contributed by Joshua J.X. Li, M.B.Ch.B. and Gary M. Tse, M.B.B.S.

      Benign phyllodes tumor
      Leaf-like epithelial pattern

      Leaf-like epithelial pattern

      Low stroma to epithelium ratio

      Low stroma to epithelium ratio

      Rounded borders

      Rounded borders

      Shelled out tumor

      Shelled out tumor

      Benign stromal cells

      Benign stromal cells



      Borderline phyllodes tumor
      Phyllodal pattern

      Phyllodal pattern

      Stromal overgrowth

      Stromal overgrowth

      Distorted epithelium

      Distorted epithelium

      Stromal mitosis

      Stromal mitosis

      Stromal atypia

      Stromal atypia



      Malignant phyllodes tumor
      Infiltrative borders

      Infiltrative borders

      Marked stromal overgrowth

      Marked stromal overgrowth

      High stromal cellularity

      High stromal cellularity

      Stromal atypia

      Stromal atypia

      Stromal giant cells

      Stromal giant cells

      Atypical mitosis

      Atypical mitosis

      Virtual slides

      Images hosted on other servers:

      Benign phyllodes tumor
      Benign phyllodes tumor

      Benign phyllodes tumor

      Intraductal growth pattern

      Intraductal growth pattern


      Borderline phyllodes tumor
      Borderline phyllodes tumor

      Borderline phyllodes tumor



      Malignant phyllodes tumor
      Malignant lipomatous element

      Malignant lipomatous element

      Malignant osteosarcomatous element

      Malignant osteosarcomatous element

      Extensive stromal overgrowth

      Extensive stromal overgrowth

      Cytology description
      • Fibromyxoid stromal clumps
        • Reduced epithelial stromal ratio compared with fibroadenomas
        • Higher nuclear atypia and cellularity in phyllodes tumor of higher grades
      • Large wavy and folded epithelial clusters
        • Usually exhibits benign cytomorphology
        • Occasionally, hyperplastic changes with enlarged and vesicular nuclei and small visible nucleoli may be seen
      • Fibroblastic pavements
      • Increased atypia in dispersed cells in phyllodes tumor of higher grades (Cancer Cytopathol 2010;118:33)
      • Multinucleated tumor cells and marked stromal anaplasia reported in malignant phyllodes tumor (Cancer Cytopathol 2010;118:33)
      Cytology images

      Contributed by Joshua J.X. Li, M.B.Ch.B. and Gary M. Tse, M.B.B.S.

      Benign phyllodes tumor
      Biphasic pattern

      Biphasic pattern

      Fibromyxoid stromal clumps

      Fibromyxoid stromal clumps

      Benign stromal cells

      Benign stromal cells


      Borderline phyllodes tumor
      Increased stroma to epithelium ratio

      Increased stroma to epithelium ratio

      Stromal cellularity

      Stromal cellularity

      Stromal atypia

      Stromal atypia



      Malignant phyllodes tumor
      Atypical stromal cells

      Atypical stromal cells

      Positive stains
      Negative stains
      Molecular / cytogenetics description
      • Recurrent MED12 and RARA mutations in phyllodes tumors of all grades and fibroadenoma (Nat Genet 2015;47:1341)
        • More frequent in benign phyllodes tumor
      • Mutations in cancer driver genes (NF1, RB1, PIK3CA, EGFR, TP53 and ERBB4) exclusively seen in borderline and malignant phyllodes tumors (Nat Genet 2015;47:1341)
      • TERT promoter mutation more common in borderline phyllodes tumor (Sci Rep 2018;8:3881)
      • Chromosome abnormalities increase with grade (Mod Pathol 2007;20:435)
        • Notably 1q gain and 13q loss
      Sample pathology report
      • Left breast, local excision:
        • Benign phyllodes tumor
        • Microscopic (optional): Sections show a stromal proliferation in a leaf-like pattern capped by benign ductal epithelium and intact intervening myoepithelial cell layer. The stromal proliferation is of low cellularity and the stromal cells show mild atypia. Stromal mitosis and stromal overgrowth are not prominent. The tumor is circumscribed. The tumor measures ____ in maximal dimension and is clear from (specify if < 1 cm ____) / involves the resection margins.
      • Left breast, wide local excision:
        • Borderline phyllodes tumor
        • Microscopic (optional): Sections show a stromal proliferation featuring areas of leaf-like pattern and stromal expansion with moderate cellularity and stromal atypia. Scattered stromal mitotic figures are noted. The tumor is largely circumscribed with focally infiltrative borders. The tumor measures ____ in maximal dimension and is clear from (specify if < 1 cm ____) / involves the resection margins.
      • Left breast, local excision:
        • Malignant phyllodes tumor
        • Microscopic (optional): Sections show a malignant spindle cell proliferation with diffusely increased cellularity, stromal overgrowth and marked stromal atypia. Stromal mitotic figures are frequent. The tumor shows infiltrative borders. The tumor measures ____ in maximal dimension and is clear from (specify if < 1 cm ____) / involves the resection margins.
      Differential diagnosis
      • Benign phyllodes tumor
        • Fibroadenoma:
          • Shows a combination of pericanalicular and intracanalicular epithelial patterns
          • No phyllodal epithelial pattern
          • Relatively uniform stromal cellularity, no subepithelial condensation
          • Overlap in stromal cellularity and mitosis in cellular and juvenile fibroadenomas
          • No significant stromal expansion
        • Fibromatosis:
      • Borderline / malignant phyllodes tumor
      Board review style question #1

      A 36 year old woman presents with a self palpated breast mass. Imaging shows a rounded nodule; excision with narrow negative margins was performed. Stromal atypia and mitosis are absent. What is the most suitable follow up management for the patient?

      1. Adjuvant chemotherapy
      2. Hormonal therapy
      3. Local radiotherapy
      4. Lymph node dissection
      5. Observation
      Board review style answer #1
      E. Observation. Benign phyllodes tumor is not associated with mortality or metastasis; additional therapy is not indicated.

      Comment Here

      Reference: Phyllodes tumor
      Board review style question #2
      Which of the following features is diagnostic of malignant phyllodes tumor?

      1. Focus of infiltrative tumor border
      2. Involvement by low grade ductal carcinoma in situ
      3. Multinucleated stromal giant cells
      4. Small focus of osteosarcomatous differentiation
      5. Stromal overgrowth
      Board review style answer #2
      D. Small focus of osteosarcomatous differentiation. Borderline phyllodes tumor allows for, albeit a lesser degree compared to malignant phyllodes tumor, stromal overgrowth and an infiltrative tumor border. Multinucleated stromal giant cells can be seen in all grades of phyllodes tumor and does not impact histologic grading. The epithelial component of phyllodes tumor can show metaplasia, hyperplasia, in situ or invasive malignant changes, which may impact prognosis but does not affect histologic grading. Presence of malignant heterologous differentiation directly upgrades a phyllodes tumor into the malignant category.

      Comment Here

      Reference: Phyllodes tumor
      Board review style question #3
      Which of the following features can be seen in malignant phyllodes tumors but not metaplastic carcinomas?

      1. Bizarre anaplastic tumor cells
      2. Cytokeratin expression
      3. MED12 mutation
      4. p53 expression
      5. Rhabdomyosarcomatous differentiation
      Board review style answer #3
      C. MED12 mutation. MED12 and RARA are frequent mutations in fibroepithelial lesions of the breast and are believed to be involved in the tumorigenesis of fibroepithelial lesions. MED12 and RARA mutations are absent in metaplastic carcinomas (Clin Cancer Res 2017;23:3859). p53 expression and TP53 mutation are detected in both malignant phyllodes tumor and metaplastic carcinomas. Aberrant cytokeratin expression, albeit patchy, can be seen in malignant phyllodes tumor. Bizarre tumor cells and malignant heterologous elements can be seen in both entities.

      Comment Here

      Reference: Phyllodes tumor

      Pleomorphic adenoma
      Definition / general
      • Biphasic benign breast tumor with varying epithelial and stromal components
      Essential features
      • Similar to pleomorphic adenoma of the salivary gland in morphology, immunohistochemistry, prognosis and treatment, except rarer in breast
      • Hypothesized to be a variant of intraductal papilloma
      • Biphasic benign tumor easily misdiagnosed as mucinous or matrix producing metaplastic carcinoma
      Terminology
      ICD coding
      • ICD-11: 2F30.Y - Other specified benign neoplasm of breast
      • ICD-11: XH2KC1 - Pleomorphic adenoma
      Epidemiology
      Sites
      • Periareolar, upper outer quadrant
      Pathophysiology
      • Breast and salivary glands are embryologically similar exocrine glands that originate from the same ectodermal layer and can differentiate towards dual epithelial myoepithelial cell differentiation
      Etiology
      Clinical features
      Diagnosis
      • Diagnosis may be made on core biopsy or FNA with radiologic correlation but definitive with excision
      Radiology description
      Radiology images

      Contributed by Shabnam Jaffer, M.D., Mark R. Wick, M.D.

      Periareolar nodule

      Mammogram

      Prognostic factors
      Case reports
      Treatment
      Gross description
      • Circumscribed yellow white gritty solid nodules > polypoid, multinodular or satellite lesions
      • Size: typically 0.8 - 4.5 cm (mean = 2.0 cm) (South Med J 1975;68:97)
      • Intraductal nodule with cystification
      Gross images

      Contributed by Shabnam Jaffer, M.D.

      Circumscribed tumor

      Microscopic (histologic) description
      • Admixture of epithelial and myoepithelial cells embedded in a chondromyxoid stroma, may show chondroid or osseous metaplasia
      • Architecture includes tubules, islands, cords, trabeculae, sheets or pseudoglandular structures lined by 2 cell types
        • Inner epithelial cells: cuboidal or columnar epithelial cells, with or without apocrine or squamous metaplasia
        • Outer myoepithelial cells: polygonal, plasmacytoid, fusiform or stellate with clear to eosinophilic cytoplasm
      • No atypia, mitoses, necrosis or infiltrative growth
      • Rare malignant transformation
      Microscopic (histologic) images

      Contributed by Shabnam Jaffer, M.D.

      Encapsulated tumor

      Biphasic benign tumor

      LMWCK

      SMA

      Virtual slides

      Images hosted on other servers:

      Myxoid stromal breast tumor

      Cytology description
      • Cellular smears consisting of epithelial and myoepithelial cell clusters and sheets arranged in branched glandular structures
        • Epithelial cells: round cells with abundant cytoplasm
        • Myoepithelial cells: stellate, plasmacytoid
      • Stroma: myxochondroid, light green on Pap, metachromatic (red-purple) on Giemsa
      • No atypia, mitoses or necrosis
      • Pitfall for misdiagnosis as mucinous or matrix producing metaplastic carcinoma (Breast Dis 2017;37:105)
      Positive stains
      Negative stains
      Electron microscopy description
      • Biphasic tumor (Am J Clin Pathol 1990;93:795)
        • Epithelial: microvilli and lumina
        • Myoepithelial: spindled stromal cells with intermediate filaments, dense bodies and intercellular junctions
      Molecular / cytogenetics description
      Sample pathology report
      • Left breast, mass, lumpectomy:
        • Pleomorphic adenoma, 3.2 cm (see comment)
        • Surgical margins, negative for tumor
        • Comment: The lesion is a circumscribed benign biphasic tumor by morphology and immunohistochemistry. AE1/3 and p63 are expressed. There is associated myxoid stroma. These findings are consistent with pleomorphic adenoma.
      Differential diagnosis

      Pleomorphic lobular carcinoma
      Definition / general
      • Cytologic variant of invasive lobular carcinoma (ILC)
      Essential features
      • Aggressive variant which accounts for approximately 10% of ILC
      • May show focal areas of classical ILC
      Terminology
      • Predominant cytologic features determine the histologic type of ILC
      Epidemiology
      • ILC are more common in older women (mean age 57, range 24 - 92) and very rare in men
      • When they do occur in men, it is often in BRCA2 carriers
      Etiology
      • CDH1 germline mutation is seen in many ILC tumors, resulting in the loss of E-cadherin gene expression in approximately 85% of ILC
      Clinical features
      Diagnosis
      Radiology description
      • Tumors are most commonly detected as spiculated mass lesions on mammography
      • Use of magnetic resonance imaging can aid in the detection of multicentric and contralateral lesions
      Prognostic factors
      • Factors which result in worse prognosis include high histologic grade (particularly high mitotic scores) and negative hormone receptor status
      • Although the prognosis is typically worse for this variant versus the classic type ILC, it is thought to be more related to tumor grade versus variant type
      Case reports
      Treatment
      • Similar to that of invasive ductal carcinoma and depends on the tumor pathologic stage and hormone receptor status
      Gross description
      • Spiculated mass with ill defined margins; often no mass because of diffuse growth pattern
      Gross images

      Images hosted on other servers:

      Circumscribed tumor

      Microscopic (histologic) description
      • Moderately to poorly differentiated carcinomas
      • Multifocal nodular aggregates of pleomorphic, high grade tumor cells with grade 2 - 3 nuclei which may contain prominent nucleoli
      • Typical single file pattern or targetoid growth in a background of dense fibrotic breast parenchyma
      • Distinctive cytologic cellular features with abundant eosinophilic cytoplasm (may look apocrine), signet ring and plasmacytoid cells
      • Commonly positive for apocrine marker GCDFP-15
      • Prognosis is related to the high tumor grade
      Microscopic (histologic) images

      Contributed by Mirna B. Podoll, M.D.
      Missing Image

      Desmoplastic stroma

      Missing Image Missing Image Missing Image Missing Image

      Pleomorphic invasive lobular carcinoma


      Missing Image

      E-cadherin

      Missing Image

      ER

      Missing Image

      PR



      Contributed by Mark R. Wick, M.D.

      Pagetoid spread

      Cadherin neg



      Images hosted on other servers:

      Pleomorphic tumor cells

      E-cadherin negative

      p53

      Cytology description
      • More cellular than classic lobular, large tumor cells with single filing, tumor cells have cytoplasmic vacuoles and pleomorphic nuclei (Cancer 1997;81:29)
      • May have apocrine features and resemble atypical mesothelial cells (Diagn Cytopathol 2008;36:657)
      • Ductal lavage: similar features, although less striking, including epithelial cells in small clusters, single-file or isolated; also nuclear atypia, cytoplasmic vacuoles and signet ring features (Acta Cytol 2008;52:207)
      Cytology images

      Images hosted on other servers:

      Various images

      Positive stains
      • GCDFP-15: 71%, due to apocrine nature
      • HER2: overexpression in approximately 20%
      • p53: 20 - 60%
      • Variable ER and PR (approximately 5 - 10% are hormone receptor negative)
      Negative stains
      Molecular / cytogenetics description
      Molecular / cytogenetics images

      Contributed by Mark R. Wick, M.D.

      HER2 amp

      Differential diagnosis
      • Invasive ductal carcinoma single cell pattern of infiltration in ILC; can be difficult to recognize particularly at metastatic sites
      • Lymphoma: may involve breast as primary site but is not seen associated with LCIS Immunohistochemical studies for cytokeratin and lymphoid markers can aid in the diagnosis
      • Metastatic melanoma: one of the most common metastasis to the breast; immunohistochemical studies for cytokeratin and melanoma markers can aid in the diagnosis
      • Chemotherapy or radiation treatment effect
      Board review style question #1
        Which of the following is true of pleomorphic variant of invasive lobular carcinoma?

      1. Are usually hormone receptor negative
      2. Do not show HER2 overexpression
      3. Frequency of p53 expression is similar to that of classic ILC
      4. The aggressive nature of these tumors is reflected by the pleomorphic variant designation
      5. The high histologic grade contributes to the poor patient outcomes
      Board review style answer #1
      E. The aggressive nature of the pleomorphic variant of invasive lobular carcinoma (ILC) and poorer outcomes are related to the high histologic grade of this variant, rather than the pleomorphic designation itself. While these tumors may be ER and PR negative in approximately 5 - 10% of cases, the majority are hormone receptor positive. HER2 expression can be seen in approximately 20% of cases. In addition, the pleomorphic variant of ILC more commonly shows p53 expression when compared to the classic type ILC.

      Comment Here

      Reference: Pleomorphic lobular carcinoma

      Polymorphous
      Definition / general
      • Primary breast tumor with morphology similar to that of polymorphous low grade adenocarcinoma of the salivary glands
      • "Low grade" terminology not used in these tumors due to reported case with widespread metastases
      Essential features
      • Very rare tumor; only 3 reported cases, first described in 2006 (Virchows Arch 2006;448:29)
      • Similar to tumors of the salivary glands; they are composed of monotonous cells arranged in nests and cords
      • Difficult to know prognosis of such a rare tumor but shows slow, aggressive growth with metastatic potential
      Clinical features
      • The rare reported cases all presented as:
        • Palpable masses
        • 1.5 - 4 cm in greatest dimension (mean 3 cm) with infiltrative borders
        • Patient mean age 55 (range 37 - 74)
        • No axillary metastases reported but one patient developed distant metastases within 3 years
      Treatment
      • All reported cases were treated with surgical resection (plus radiotherapy if breast conserving surgery) and axillary dissection
      Microscopic (histologic) description
      • Monotonous proliferation of neoplastic cells arranged in varying architectural patterns including tubules, trabeculae, alveolar, cribriform and solid nests
      • Nuclei are round to ovoid with vesicular chromatin, moderate mitotic activity without necrosis
      • Central and peripheral areas of the tumor may demonstrate differing architectural and immunohistochemical features
      Microscopic (histologic) images

      Contributed by Emily S. Reisenbichler, M.D.
      Missing Image

      Salivary gland equivalent polymorphous low grade adenocarcinoma

      Positive stains
      Negative stains
      Differential diagnosis
      • Adenoid cystic carcinoma: both have cribriform and alveolar growth with possible basal lamina-like material and are triple negative (ER / PR / HER2)
        • Adenoid cystic carcinoma has two cell types with a population of actin positive cells that are absent in polymorphous adenocarcinoma
      • Invasive lobular carcinoma: both can have single file pattern of growth but polymorphous adenocarcinoma has membranous E-cadherin and is negative for ER / PR and AR
      • Salivary tumor metastatic to the breast: clinical history of a salivary neoplasm
      Board review style question #1
        What receptor profile is expected in a primary polymorphous adenocarcinoma of the breast?

      1. ER+, PR+, HER2- (Luminal A)
      2. ER-, PR-, HER2- (Basal-like)
      3. ER-, PR-, HER2+ (HER2 positive)
      4. ER+, PR+, HER2+
      Board review style answer #1
      B. Like many of the salivary gland-like tumors of the breast, polymorphous adenocarcinoma is a triple negative, basal-like tumor. Unlike the poor prognosis seen in many triple negative breast tumors, salivary gland-like tumors, including polymorphous adenocarcinoma, may have a more favorable prognosis. (Appl Immunohistochem Mol Morphol 2013;21:283)

      Comment Here

      Reference: Polymorphous carcinoma

      Pseudoangiomatous stromal hyperplasia
      Definition / general
      • Benign stromal myofibroblastic proliferation forming clefts, simulating a vascular lesion
      • First described in 1986 (Hum Pathol 1986;17:185)
      Essential features
      • Benign stromal myofibroblastic proliferation
      • Usually an incidental finding but may produce palpable or mammographic mass
      • Complex interanastomosing spaces in dense collagenous, keloid-like stroma resembling blood vessels
      • Some of these spaces have spindle shaped myofibroblasts at their margins, simulating endothelial cells
      • Spindle cells usually positive for ER, PR and CD34 but negative for other vascular markers (e.g., CD31) (Breast J 2012;18:242)
      • Pseudoangiomatous stromal hyperplasia (PASH) is an incidental microscopic finding in up to 23% of breast surgical resections (Int J Surg 2011;9:20)
      Terminology
      • Also called pseudoangiomatous hyperplasia of mammary stroma
      ICD coding
      • ICD-11: GB20.Y - other specified benign breast disease
      Epidemiology
      Sites
      • Breast stroma
      Etiology
      Clinical features
      • Incidental finding, can occur in both females and males
      • Can be associated with other benign and malignant lesions
      Diagnosis
      • Histologic examination of resected or biopsied tissue
      Radiology description
      • Incidental but may present as a mass, well defined or irregular (SA J Radiol 2018;22:1366)
      • Nonspecific findings on magnetic resonance imaging (MRI), ranging from an enhancing mass to clumped, nonmass-like enhancement (Clin Radiol 2010;65:145)
      Radiology images

      Images hosted on other servers:
      Missing Image

      Round mass

      Prognostic factors
      • Nonneoplastic but mass forming lesion may rarely recur, especially in younger patients
      Case reports
      Treatment
      • Local excision needed only in symptomatic mass forming lesions
      • If diagnosed on core needle biopsy, no surgical excision is required, provided the diagnosis is concordant with radiologic findings
      Gross description
      Gross images

      Images hosted on other servers:
      Missing Image

      Well demarcated lesion

      Microscopic (histologic) description
      • Complex interanastomosing spaces in dense collagenous, keloid-like stroma
      • Some of these spaces have spindle shaped myofibroblasts at their margins that simulate endothelial cells
      • Spaces are usually empty but may contain rare erythrocytes
      • Cellular areas or plump spindle cells may obscure pseudoangiomatous structure
      • Often gynecomastia-like changes (Mod Pathol 2008;21:201)
      • Rarely multinucleated giant cells (Breast J 2007;13:568)
      • No mitotic figures, no necrosis, no atypia
      • Fascicular PASH: cellular variant in which myofibroblasts aggregate into fascicles with reduced or absent clefting, resembles myofibroblastoma (Am J Surg Pathol 1995;19:270)
      Microscopic (histologic) images

      Contributed by Indu Agarwal, M.D.
      Missing Image

      Interlobular PASH

      Missing Image

      Slit-like spaces

      Missing Image

      Spindle cells lining the spaces

      PASH within areas of fibroadenoma

      PASH within areas of fibroadenoma

      Missing Image

      CD31

      Missing Image

      CD34

      Negative stains
      Sample pathology report
      • Left breast, at 5 o'clock and 4 cm from the nipple, ultrasound core needle biopsy:
        • Breast tissue with pseudoangiomatous stromal hyperplasia (PASH)
      Differential diagnosis
      • Low grade angiosarcoma:
        • Hemorrhagic, soft, interanastomosing vascular channels containing red blood cells with invasion into breast parenchyma
        • Papillary endothelial growth and hyperchromatic endothelial cells
        • CD31+, factor VIII+ (Breast J 2012;18:242)
      • Myofibroblastoma:
        • Absence of typical breast epithelial elements within myofibroblastoma
        • Neoplastic clonal tumors with characteristic genetic change (del 13q14) (this can be demonstrated by loss of Rb protein immunohistochemistry in myofibroblastoma)
      • Other spindle cell tumors (e.g., fibromatosis):
        • Tumor cells arranged in long fascicles without significant clefting, nuclear beta catenin staining and CD34-
      Board review style question #1

      The finding shown in the image above was obtained on the core needle biopsy of the breast in a 35 year old woman. What is the most likely mammographic finding?

      1. Architectural distortion
      2. Calcifications
      3. Irregular mass
      4. Mass with smooth borders
      5. No abnormality; incidental finding
      Board review style answer #1
      E. No abnormality; incidental finding. Pseudoangiomatous hyperplasia is usually an incidental finding but may less commonly produce palpable or mammographic mass. Answers A, B, C and D are incorrect because the most common presentation is an incidental finding only.

      Comment Here

      Reference: Pseudoangiomatous stromal hyperplasia
      Board review style question #2
      Which of the following sets of immunohistochemical stains would be most suitable for the diagnosis of pseudoangiomatous hyperplasia (PASH)?

      1. CD34-, CD31-, ER-, nuclear beta catenin+
      2. CD34-, CD31-, nuclear beta catenin+, AE1 / AE3+
      3. CD34+, CD31-, ER+, factor VIII-
      4. CD34+, CD31+, ER-, factor VIII+
      Board review style answer #2
      C. CD34+, CD31-, ER+, factor VIII-. This immunoprofile is indicative of PASH. Answer D is incorrect because the immunoprofile is most consistent with angiosarcoma or other vascular lesions, with positive vascular markers. Answer A is incorrect because it is the immunoprofile for fibromatosis with positive nuclear beta catenin. It is important to note that fibromatosis is negative for CD34 as it does not arise from myofibroblasts. Answer B is incorrect because it is the classic staining pattern for a spindle cell carcinoma with positive AE1 / AE3.

      Comment Here

      Reference: Pseudoangiomatous stromal hyperplasia

      Pubertal macromastia
      Definition / general
      • Rare form of breast hypertrophy that is characterized by an atypical, alarmingly rapid and continual breast growth during puberty or in the peripubertal age group (J Breast Health 2014;10:122)
      Essential features
      • Rare form of breast hypertrophy characterized by rapid and continual breast enlargement
      • Pathogenesis is not clear
      • Surgery with or without medical therapy is the treatment of choice
      • Breast hypertrophy is defined as a medical condition of the breast connective tissues in which the breasts become excessively large
        • There is no universal classification or definition
        • Based on the severity, it can be of 2 types (Hoda: Rosen's Breast Pathology, 5th Edition, 2020)
          • Gigantomastia: refers to cases with extreme breast enlargement, where excessive tissue is > 2.5 kg
          • Macromastia: refers to sustained, steady breast enlargement that develops throughout puberty, where excessive tissue is < 2.5 kg
      Terminology
      ICD coding
      • ICD-10: N62 - hypertrophy of breast
        • Applicable to
          • Hypertrophy of breast, NOS
          • Gynecomastia
          • Massive pubertal hypertrophy of breast
      Epidemiology
      Sites
      Pathophysiology
      Etiology
      Clinical features
      Diagnosis
      • Physical examination typically reveals normal sexual development
      • Breasts are generally enlarged, pedunculated, ptotic with widened areolas
      • Often firm on palpation and diffusely tender
      • Discrete nodules may be present
      • Breast growth, velocity and stability should be evaluated along with pubertal development and onset of thelarche (Semin Plast Surg 2013;27:49)
      • Family history regarding breast disorders particularly related to breast hypertrophy should be noted (Semin Plast Surg 2013;27:49)
      Laboratory
      • Endocrinology profile, specifically estradiol, progesterone, luteinizing hormone (LH), follicle stimulating hormone (FSH) and prolactin, does not reveal any abnormalities and is not routinely indicated (Semin Plast Surg 2013;27:62)
      Radiology description
      • Breast imaging is of limited value owing to the dense breast tissue
      • May be required if mass lesion is present
      Radiology images

      Images hosted on other servers:
      Breast mammogram

      Breast mammogram

      Case reports
      Treatment
      Clinical images

      Contributed by Pooja Ramakant, M.S., M.Ch.
      Bilateral pubertal macromastia

      Bilateral pubertal macromastia



      Images hosted on other servers:

      Adolescent macromastia

      Virginal (juvenile) breast hypertrophy

      Autoimmune related cases

      Gross description
      • No specific gross findings; only fibrofatty tissue seen
      Frozen section description
      • No role of frozen in these cases
      • May be used whenever there is preoperative suspicion of cancer
      Microscopic (histologic) description
      • Clinical diagnosis
      • Histomorphological findings resemble gynecomastia
        • Increased stromal collagenization and fibrosis
        • Poorly developed or reduced lobules
        • Sometimes usual ductal hyperplasia is also seen
      • Reference: Case Rep Surg 2017;2017:3491012
      Microscopic (histologic) images

      Contributed by Chanchal Rana, M.D. and AFIP
      Increased stromal collagenization

      Increased stromal collagenization

      Increase in interlobular stroma

      Increase in interlobular stroma

      Juvenile / pubertal breast hypertrophy Juvenile / pubertal breast hypertrophy Juvenile / pubertal breast hypertrophy

      Juvenile / pubertal breast hypertrophy

      15 year old girl

      Marked stroma collagenization

      Cytology description
      • It is a clinical diagnosis, which does not require use of fine needle aspiration cytology
      • Findings are nonspecific
      • Smears may be paucicellular
      • Resembles benign breast disease characterized by sheets of benign ductal and myoepithelial cells with bare nuclei in the background
      Sample pathology report
      • Bilateral breast, reduction mammoplasty:
        • Consistent with clinical diagnosis of bilateral breast hypertrophy
        • No evidence of malignancy

      • Bilateral breast, subcutaneous mastectomy:
        • Consistent with clinical diagnosis of bilateral breast hypertrophy
        • No evidence of malignancy
      Differential diagnosis
      • Clinical diagnosis
      • Need to exclude other causes of breast hypertrophy
      • Breast hypertrophy associated with endocrine disorder:
        • Histology is similar but also associated with laboratory abnormalities like aromatase excess, hyperprolactinemia and excessive production of parathyroid hormone
      • Drugs:
        • Penicillamine, ciclosporin, prednisolone, indinavir, etc.
        • There is a history of drug intake followed by development of hypertrophy
      • Obesity
      Board review style question #1

      Which of the following statements relating to pubertal macromastia is true?

      1. Characterized by increased sensitivity to normal levels of hormone
      2. Characterized by presence of increased levels of hormone
      3. Does not recur after reduction mammoplasty
      4. Medical management is the treatment of choice
      Board review style answer #1
      A. Characterized by increased sensitivity to normal levels of hormone. There is heightened end organ sensitivity with normal hormonal as well as growth factor levels. Answer B is incorrect because cases of pubertal macromastia have normal hormonal levels, only the sensitivity to their levels is increased. Answer D is incorrect because pubertal / virginal / juvenile macromastia is generally treated surgically with or without medical therapy. Answer C is incorrect because recurrence rates are high with reduction mammoplasty.

      Comment Here

      Reference: Pubertal macromastia
      Board review style question #2
      Which of the following statements about breast hypertrophy is true?

      1. Drugs such as bromocriptine and tamoxifen are used as a part of medical management
      2. Medical therapy is never indicated
      3. Obesity is not one of the major causes
      4. Subcutaneous mastectomy is never done
      Board review style answer #2
      A. Drugs such as bromocriptine and tamoxifen are used as a part of medical management. Bromocriptine and tamoxifen are used as medical therapy to reduce breast proliferation. Answer C is incorrect because pubertal / virginal / juvenile macromastia can be associated with obesity. Answer D is incorrect because subcutaneous mastectomy is also one of the surgical management options. Answer B is incorrect because medical management may be given before or after the surgical procedures for breast reduction to prevent further growth.

      Comment Here

      Reference: Pubertal macromastia

      Radial scar / complex sclerosing lesion
      Definition / general
      • Benign breast lesion comprised of a central fibroelastotic stromal core with radiating ducts and lobules with varying degrees of proliferative and cystic changes arranged in a stellate configuration
      Essential features
      • Benign breast lesion comprised of a central fibroelastotic stromal core with radiating ducts and lobules with varying degrees of proliferative and cystic changes arranged in a stellate configuration
      • May simulate invasive carcinoma mammographically, clinically, grossly and microscopically
      • Atypia or malignancy may arise within or in association with radial scars / complex sclerosing lesions
      • Treatment ranges from observation with clinical and radiologic follow up to surgical excision
      Terminology
      • Radial sclerosing lesion, complex sclerosing lesion (> 1.0 cm)
      ICD coding
      • ICD-10: N64.89 - Other specified disorders of breast
      • ICD-11: GB20.Y - Other specified benign breast disease
      Epidemiology
      Sites
      • Breast parenchyma
      Pathophysiology
      • Pathogenesis uncertain
      • Stromal-epithelial interactions and increased expression for several factors involved in formation of vascular stroma may play a role (Hum Pathol 2002;33:29)
      Etiology
      • Unknown
      • Scar is a misnomer; not related to prior trauma or surgery
      Clinical features
      • Most often incidental microscopic findings in breast tissue removed for other targets / abnormalities; however, some palpable (Clin Radiol 1993;48:319)
      Diagnosis
      • Imaging: ultrasound, mammogram, MRI
      • Invasive procedure: core needle biopsy, fine needle aspiration (not recommended)
      Radiology description
      Radiology images

      Contributed by Kristen E. Muller, D.O.

      Mammogram, spiculated mass

      Ultrasound, hypoechoic mass

      Prognostic factors
      Case reports
      Treatment
      Gross description
      • Stellate, rubbery to firm mass with retracted center; resembles invasive ductal carcinoma
      • Usually 1 cm or less; complex sclerosing lesions > 1 cm
      • May be firm, irregular, with yellow streaks and flecks (due to elastotic stroma)
      Gross images

      AFIP images

      Central sclerosis and elastosis resembling carcinoma

      Microscopic (histologic) description
      • Low power stellate configuration
      • Central sclerotic zone composed of fibrosis and elastosis with ducts and lobules radiating outward between bands of sclerotic tissue
      • Central core frequently contains small entrapped obliterated ductules that may mimic invasive carcinoma; squamous metaplasia may occur in central nidus
      • Variable amounts of usual ductal hyperplasia (may be florid with necrosis), sclerosing adenosis, apocrine metaplasia and cysts
      • Stroma tends to be more cellular in early lesions and more collagenized and sclerotic with abundant elastin in older lesions
      • Atypical hyperplasia (ADH, ALH) observed in 21 - 51% (Breast Cancer Res Treat 2008;107:371, Cancer 2003;97:345)
      • In situ or invasive carcinoma may arise within or in association with radial scar / complex sclerosing lesions
      • Complex sclerosing lesions are larger (typically > 1 cm) and show features of radial scar that combine and converge with intermingling areas of sclerosis, entrapped and distorted glands and fibrocystic changes; however, may not have the well defined configuration of radial scar
      Microscopic (histologic) images

      Contributed by Kristen E. Muller, D.O.

      Stellate architecture

      Central fibroelastotic core


      Central fibroelastotic core

      Squamous metaplasia

      Usual ductal hyperplasia and cysts

      Complex sclerosing lesion


      With atypical ductal hyperplasia

      With atypical lobular hyperplasia

      With invasive ductal carcinoma

      Tubular carcinoma for comparison



      Contributed by Emily S. Reisenbichler, M.D.

      Central sclerosis with fibroelastotic stroma and entrapped glands radiating outward

      Virtual slides

      Images hosted on other servers:

      Complex sclerosing lesion

      Cytology description
      • FNA not reliable for diagnosis; may show usual ductal hyperplasia, apocrine metaplasia and spindle shaped stromal cells (Diagn Cytopathol 1997;16:537)
      Positive stains
      Molecular / cytogenetics description
      • Focally increased numbers of blood vessels and expression of mRNA for collagen type 1, total fibronectin, ED-A+ fibronectin, thrombospondin 1, VPF / VEGF and KDR (Hum Pathol 2002;33:29)
      • Allelic imbalance chromosome 16q and 8p; some areas may be clonal (Hum Pathol 2002;33:715)
      Sample pathology report
      • Right breast, needle core biopsy:
        • Radial scar (see comment)
        • Microcalcifications: associated with radial scar
        • Comment: The radial scar measures 4 mm and appears adequately sampled and completely excised in this biopsy. No atypia or malignancy is identified. Clinical and radiologic correlation is recommended.
      • Right breast, needle core biopsy:
        • Complex sclerosing lesion (see comment)
        • Microcalcifications: not identified
        • Comment: The biopsy shows several tissue cores containing a complex sclerosing lesion comprised of benign glands with varying degrees of fibrocystic changes (usual ductal hyperplasia, cysts and apocrine metaplasia) radiating around a central fibroelastotic core. No atypia or malignancy is identified. The largest contiguous microscopic measurement in a single tissue core is 7 mm. Clinical and radiologic correlation is recommended to ensure adequate sampling of the targeted lesion.
      Differential diagnosis
      • Invasive ductal carcinoma:
        • Lacks myoepithelial cells
        • Infiltration into surrounding fat / breast parenchyma may be a clue
        • Tumor cells show cytologic atypia
      • Tubular carcinoma:
        • Glands show distinctive angulated shapes
        • Lacks myoepithelial cells
        • Infiltration into surrounding fat / breast parenchyma may be a clue
        • Surrounding usual ductal hyperplasia, apocrine metaplasia and cysts commonly seen at the periphery of radial scars typically absent
      • Low grade adenosquamous carcinoma:
        • Irregular glands with varying degrees of squamous differentiation
        • Infiltrative pattern; if arising within a radial scar / complex sclerosing lesion, the malignant cells infiltrate beyond the sclerosing lesion
        • Lymphoid aggregates within and at periphery of the lesion may be a clue
        • Variable expression of myoepithelial and cytokeratin markers
        • Negative for ER, PR, HER2
      Board review style question #1

      Which of the following is true regarding the lesion pictured?

      1. It is frequently associated with concurrent invasive cancer
      2. All the glands lack myoepithelial cells and invade into adipose tissue
      3. Imaging features include irregular spiculated mass with dense center frequently mimicking invasive carcinoma
      4. There is a known association with malignant phyllodes tumors
      Board review style answer #1
      C. This is a radial scar. Imaging features include irregular spiculated mass with dense center frequently mimicking invasive carcinoma

      Comment Here

      Reference: Radial scar
      Board review style question #2
      Which statement is true regarding the treatment and prognosis of radial scars and complex sclerosing lesions?

      1. They require excision since malignancy is found in 50 - 65% of excision specimens
      2. Management options include surgical excision and observation without surgery
      3. They confer a fourfold increased risk for developing breast cancer
      4. Small (< 5 mm) radial scars are more likely to be upgraded on excision
      Board review style answer #2
      B. Management options include surgical excision and observation without surgery

      Comment Here

      Reference: Radial scar

      Radiation therapy & cryoprobe
      Definition / general
      • Whole breast irradiation primarily occurs following breast conserving surgery for breast carcinoma
      • Also patients with Hodgkin lymphoma receiving mediastinal radiation with secondary breast exposure to radiation
      • Radiation can also be delivered intraoperatively to the surgical cavity after removing the tumor but prior to closing the wound
      Essential features
      • Effects of radiation on skin, such as changes in vascularity, pigmentation and fibrosis, may be induced acutely or as a latent reaction years after radiation (Clin Cosmet Investig Dermatol 2016;9:473)
      • Increased risk of secondary radiation induced sarcomas, most commonly angiosarcoma, with an incidence of 0.03 - 0.2% and average latency period of 10 years (Cases J 2008;1:313) (see also Breast angiosarcoma)
      • Radiation may induce cytologic changes in normal breast tissue that can mimic atrophy, recurrent carcinoma or sarcoma
      Clinical features
      • In acute stages, breast skin may become pink or red and peel, as with a sunburn, followed by thickening of the skin in later stages
      • Firmness of the breast may result from fibrosis
      • Lactation may still occur after pregnancy in at least 50% of women who have undergone prior radiation therapy, but production is reduced
      • Lactation is unaffected in the contralateral, untreated breast (Expert Rev Anticancer Ther 2013;13:159)
      • Radiation may induce bullous pemphigoid in the skin (Dermatol Online J 2016;22:13030)
      • In patients undergoing immediate reconstructive surgery, volume loss and fat necrosis may occur due to postmastectomy radiation; however it has not resulted in higher rates of revision surgery in the radiated breast (Ann Plast Surg 2013;71:149)
      Cryoprobe
      • Cryoprobe: uses cryotherapy to makes nonpalpable carcinomas well defined and palpable by creating an ice ball, leading to removal of less tissue (Am J Surg 2006;192:462)
      • Alters tumor morphology making it difficult to grade tumor, distinguish DCIS and invasive tumor, and identify angiolymphatic invasion or mitotic figures (Am J Clin Pathol 2007;128:239)
      • Alternative to excision of fibroadenomas (Radiology 2005;234:63)
      • Normal breast parenchyma in cryozone shows dense fibrosis, fat necrosis, xanthogranulomatous reaction, endovascular fibrosis and hemorrhage (Cryobiology 2007;55:44)
      • Causes reduction in ER / PR receptors, but does not affect Ki67 staining or margin evaluation
      Radiology description
      • Mammographic changes following radiation therapy include increased breast density and skin thickening, both of which are more common following whole breast radiotherapy than with intraoperative radiotherapy (Eur J Surg Oncol 2014;40:163)
      Case reports
      Microscopic (histologic) description
      • Giant tumor cells with bizarre nuclear changes (J Clin Pathol 1990;43:152), naked nuclei and abnormal mitotic figures
      • Extensive tumor necrosis demarcated by thick fibrous walls (when radiating tumor prior to surgery)
      • Changes in nonneoplastic breast take place primarily in the terminal duct lobular unit and include atypia, lobular sclerosis and atrophy (may distort lobular architecture and obscure myoepithelial cells, Hum Pathol 1984;15:545)
      • Intimal thickening in small muscular arteries (Radiother Oncol 2009;92:477) and fat necrosis, but usually no mitotic activity or loss of polarity
      • Epithelial atypia in larger ducts ranges from mild to severe but proliferation of the epithelium is generally seen only with mild cytologic atypia (Am J Surg Pathol 2004;28:47)
      • Atypical stromal fibroblasts
      • Changes do not regress over time (Am J Surg Pathol 2004;28:47)
      Microscopic (histologic) images

      Contributed by
      Emily S. Reisenbichler, M.D.

      Missing Image

      Radiation atypia



       AFIP images

      Thickened basement membranes

      Atypical stromal fibroblasts

      LCIS unchanged
      by radiation
      compared to
      atrophic normal glands

      Collagenization
      of intralobular
      stroma and glandular
      atrophy


      Images hosted on other servers:

      Large stromal
      fibroblasts may
      also occur in
      nonirradiated tissue

      Atypia in normal duct

      Atypia within normal lobule

      Fat necrosis postiridium implant

      Vessel shows
      reduplicated elastic
      lamina and intimal
      thickening (elastin stain)


      Edematous
      lobules with atypia
      of some epithelial
      cells

      Edematous
      lobules with atypia
      of some epithelial
      cells

      Atrophic lobules
      with atypia of some
      epithelial cells

         

      Atypical stromal
      cells, lymphocytic
      infiltrate and
      increased collagen bundles

      Differential diagnosis
      • Recurrent carcinoma (invasive or in situ): cells should be keratin positive and demonstrate similar histologic grade and receptor profile as the previous carcinoma
        • Atypical stromal fibroblasts will be keratin negative
        • Cytologic atypia in a lobule secondary to radiation will typically not fill and distend lumina or demonstrate necrosis and mitotic activity (features suggestive of recurrent high grade DCIS)
      • Sarcoma (radiation induced or primary): radiation induced sarcomas typically present as mass forming lesions rather than microscopic atypical stromal cells seen postradiation
        • Primary sarcoma of the breast would likely be a component of a malignant phyllodes and therefore demonstrate other features of a fibroepithelial lesion
      Board review style question #1
        Which statement about distinguishing radiation induced changes in benign breast tissue from recurrent carcinoma is true?

      1. Cytologically atypical cells scattered in the stroma that are estrogen receptor and keratin positive likely represent fibroblasts with radiation induced atypia.
      2. If the radiation took place more than 10 years previously, any atypia must represent recurrent malignancy.
      3. Low grade cytologic atypia is not seen in benign lobules secondary to radiation effect.
      4. Necrosis in proliferative ductal epithelium with high grade cytologic atypia is indicative of carcinoma in situ.
      5. Proliferative epithelium with high grade cytologic atypia is commonly seen secondary to radiation effect.
      Board review style answer #1
      D. Radiation may induce cytologic atypia of any degree from low to high, but proliferation of epithelium takes place secondary to radiation generally with only low grade atypia. Necrosis, mitoses and proliferative epithelium with high grade atypia together are suggestive of carcinoma rather than radiation atypia. Changes secondary to radiation do not regress over time and can be seen for more than 10 years following radiation therapy. Atypical, postradiation stromal fibroblasts should not be keratin or estrogen receptor positive. This staining pattern is more in keeping with recurrent invasive carcinoma.

      Comment Here

      Reference: Radiation therapy & cryoprobe

      Sclerosing adenosis
      Definition / general
      • Adenosis or lobulocentric processes with increase in glandular elements of terminal duct lobular unit (TDLU) with stromal fibrosis / sclerosis that distorts and compresses glands
      • Preserved 2 cell layer (inner epithelial and outer myoepithelial cells)
      Essential features
      • Enlarged terminal duct lobular unit with distortion by stromal fibrosis / sclerosis
      Terminology
      ICD coding
      • ICD-10:
        • N60.3 - fibrosclerosis, breast
        • N60.8 - other benign mammary dysplasias
      • ICD-11:
        • GB20.Y - other specified benign breast disease
        • GB20.Z - benign breast disease, unspecified
      Epidemiology
      • Most frequent in third to fourth decades but occurs over a wide age range
      • Found in 12 - 28% of all benign and 5 - 7% of malignant biopsies (J Ultrasound Med 2013;32:2029)
      Sites
      • Terminal duct lobular unit; otherwise, no specific location within the breast
      Pathophysiology
      • Unknown
      Etiology
      • Unknown
      Clinical features
      Diagnosis
      • Histologic examination of tissue with or without immunohistochemistry
      Radiology description
      Radiology images

      Contributed by Julie M. Jorns, M.D.
      Calcifications, craniocaudal view

      Calcifications, craniocaudal view

      Calcifications, mediolateral oblique view

      Calcifications, mediolateral oblique view

      Prognostic factors
      Case reports
      Treatment
      • Presence of sclerosing adenosis alone in a core biopsy does not require surgical excision
      • Coexisting atypia will typically prompt surgical consultation
      Gross description
      • Variable depending on extent of involvement and calcifications
      • May be indistinguishable from surrounding breast tissue
      • Multinodular, ill defined, cuts with increased resistance due to fibrosis
      • Gritty due to frequent calcifications but no chalky yellow white foci or streaks as seen in fat necrosis
      • Circumscribed to ill defined white, fibrotic mass if nodular adenosis / adenosis tumor
      • Reference: Schnitt: Biopsy Interpretation of the Breast, 3rd Edition, 2017
      Microscopic (histologic) description
      • Low power: increase in glandular elements plus stromal fibrosis / sclerosis that distorts and compresses glands
      • Maintains lobular architecture at low power with rounded and well defined nodules
      • Centrally is more cellular with distorted and compressed ductules; peripherally has more open or dilated ductules
      • Often has microcalcifications, due to calcification of entrapped secretions
      • Often has associated fibrocystic changes of other types, such as apocrine metaplasia
      • Preservation of luminal epithelium and peripheral myoepithelium (2 cell layer) with surrounding basement membrane
      • Myoepithelial cells may vary from being prominent to indistinct on routine H&E staining
      • Myoepithelial cells are readily apparent via immunohistochemistry, even if difficult to identify on H&E
      • Rarely penetrates walls of blood vessels or perineural spaces
      • No necrosis, no pleomorphism
      • Apocrine adenosis: variant of sclerosing adenosis with prominent apocrine features
      • Epithelium may be involved by proliferative, atypical lesions or in situ carcinoma
      • Can mimic malignancy:
        • If involved by atypia or in situ carcinoma
        • If florid and overtly non-lobulocentric / (pseudo) infiltrative into fat or stroma
        • Conspicuous myoepithelial cells with attenuated epithelial cells can appear like stands of single cells and mimic invasive lobular carcinoma
        • Atypical apocrine metaplasia: nuclear atypia / rare mitosis (Mod Pathol 1991;4:1)
        • (Pseudo) perineural invasion
      Microscopic (histologic) images

      Contributed by Julie M. Jorns, M.D.
      Lobule with distortion Lobule with distortion Lobule with distortion

      Lobule with distortion

      Calcifications and drag

      Calcifications and drag

      Drag mark

      Drag mark


      Calponin

      Calponin

      p63

      p63

      Sclerosing adenosis with DCIS Sclerosing adenosis with DCIS Sclerosing adenosis with DCIS

      Sclerosing adenosis with DCIS


      Sclerosing adenosis with DCIS Sclerosing adenosis with DCIS

      Sclerosing adenosis with DCIS

      Cytology description
      Positive stains
      Videos

      Pathology mini tutorial

      Sample pathology report
      • Left breast, core biopsy:
        • Fibrocystic changes including sclerosing adenosis with microcalcifications
      Differential diagnosis
      • Microglandular adenosis:
        • Haphazardly distributed glands (lacks lobulocentric pattern)
        • Lacks myoepithelium but has intact basement membrane
      • Tubular adenoma:
        • Well circumscribed
        • Nodular growth may mimic nodular adenosis / adenosis tumor
        • Uniform, closely packed tubules (lacks significant distortion by fibrosis)
      • Invasive carcinoma:
        • May be difficult to morphologically distinguish from florid sclerosing adenosis with marked distortion and/or involvement by atypia or DCIS
        • Displays desmoplastic stromal response
        • Lacks myoepithelium
      • Invasive tubular carcinoma:
        • More widely spaced tubules with single epithelial layer
        • Displays desmoplastic stromal response
      Board review style question #1

      A premenopausal woman underwent breast biopsy for screening detected microcalcifications showing the pictured lesion. This lesion typically is associated with

      1. Abundant cytologic atypia
      2. Clinically palpable mass
      3. Lack of myoepithelium
      4. Variable imaging features
      Board review style answer #1
      D. Variable imaging features. The pictured lesion is sclerosing adenosis, a benign breast lesion characterized by expansion of glands (with preserved 2 cell layers: inner epithelial and outer myoepithelial cells) within the terminal duct lobular unit with distortion by fibrosis / sclerosis. Most of the time, sclerosing adenosis lacks cytologic atypia. Sclerosing adenosis frequently has calcifications due to entrapped secretions. However, when abundant, sclerosing adenosis may present as a distortion or mass on imaging. Sometimes sclerosing adenosis clinically presents as a palpable mass, as with nodular adenosis / adenosis tumor. However, sclerosing adenosis can often be an incidental finding when there is another indication for biopsy. Thus, imaging findings are variable, depending on size and extent of the lesion.

      Comment Here

      Reference: Sclerosing adenosis
      Board review style question #2

      Sclerosing adenosis of the breast (without atypia) imparts what risk for future breast cancer?

      1. No increased risk
      2. 1.5 - 2x increased risk
      3. 4 - 5x increased risk
      4. 8 - 10x increased risk bilaterally
      5. 8 - 10x increased risk ipsilaterally
      Board review style answer #2
      B. 1.5 - 2x increased risk. Sclerosing adenosis is considered part of the spectrum of proliferative fibrocystic changes, which imparts a 1.5 - 2x increased risk for future breast cancer. This is opposed to nonproliferative fibrocystic change which has no increased risk and atypical ductal hyperplasia (ADH) with 4 - 5x, ductal carcinoma in situ (DCIS) with 8 - 10x (unilateral: ipsilateral side) and lobular carcinoma in situ (LCIS) with 8 - 10x (bilateral) increased risk for breast cancer.

      Comment Here

      Reference: Sclerosing adenosis

      Secretory
      Definition / general
      • Rare subtype of low grade, translocation associated invasive breast carcinoma
      • Tumor with microcystic, solid and tubular architecture, composed of vacuolated tumor cells producing intracellular and extracellular secretions
      • Generally triple negative with basal-like phenotype (Mod Pathol 2012;25:567)
      • First described by McDivitt and Stewart in 1966 (JAMA 1966;195:388)
      Essential features
      • Rare, < 1% of breast cancers
      • Most common primary pediatric breast cancer but can occur at any age
      • Circumscribed margins, solid nests, cysts and gland formation with PAS+ intraluminal secretions
      • Tumor cells with abundant vacuolated or granular cytoplasm, low nuclear grade
      • Generally triple negative for estrogen receptor, progesterone receptor and HER2
      • Most tumors have translocation yielding ETV6-NTRK3 fusion gene
      • Associated with a favorable prognosis in younger patients
      Terminology
      • Originally named juvenile breast carcinoma, the terminology was changed to secretory carcinoma after reports of the tumor occurring in adults
      ICD coding
      • ICD-O: 8502/3 - secretory carcinoma of breast
      Epidemiology
      Clinical features
      Diagnosis
      • Histologic examination of tissue with immunohistochemistry, demonstration of ETV6-NTRK3 gene fusion by fluorescent in situ hybridization
      Radiology description
      Radiology images

      Images hosted on other servers:
      Missing Image Missing Image

      Hypoechoic lesion on ultrasound

      Case reports
      Treatment
      Clinical images

      Images hosted on other servers:
      Missing Image

      Breast mass involving skin

      Gross description
      Microscopic (histologic) description
      • Well circumscribed with pushing borders but may be infiltrative at periphery
      • Central sclerosis may be observed
      • Architecture is usually microcystic, solid or tubular or admixture of all 3 patterns
      • Can display peripheral papillary architecture
      • Low grade cytologic atypia, bland uniform nuclei, low mitotic rate
      • Histologic hallmark is the presence of tumor cells with vacuolated, foamy cytoplasm and abundant intracellular and extracellular pale blue to dense pink secretions, which are periodic acid Schiff (PAS) positive and diastase resistant (JAMA 1966;195:388, Mod Pathol 2012;25:567)
      • Often an in situ component (Mod Pathol 2009;22:291)
      Microscopic (histologic) images

      Contributed by Gary Tozbikian, M.D. and Case #8

      Glands and solid nests

      Tumor cells with vacuolated cytoplasm


      Low nuclear grade, no mitoses

      Microcystic pattern Microcystic pattern

      Microcystic pattern

      Extracellular secretory material Extracellular secretory material

      Extracellular secretory material


      Missing Image Missing Image

      Solid pattern

      PASD

      S100

      Molecular / cytogenetics description
      Molecular / cytogenetics images

      Images hosted on other servers:
      Missing Image

      Karyogram with t(12;15)(p13;q25)

      Missing Image

      FISH with disrupted ETV6 gene

      Sample pathology report
      • Breast, right, 3:00, ultrasound guided core biopsy:
        • Secretory carcinoma, 0.7 cm in greatest length
        • Estrogen receptor: negative (0%)
        • Progesterone receptor: negative (0%)
        • HER2: negative (score 0)
      Differential diagnosis
      Board review style question #1

      Secretory carcinoma is associated with which cytogenetic abnormality?

      1. 1q chromosomal gains and 16q chromosomal losses
      2. HER2 gene amplification
      3. t(12;15)(p13;q25), resulting in ETV6-NTRK3 fusion gene
      4. t(6;9)(q22-23;p23-24), resulting in fusion of MYB and NFIB
      5. TP53 mutation
      Board review style answer #1
      C. Secretory carcinoma of breast is associated with t(12;15)(p13;q25), resulting in ETV6-NTRK3 fusion gene

      Comment Here

      Reference: Secretory carcinoma of breast
      Board review style question #2
      What histological and immunophenotypic features are most likely to be observed in a breast secretory carcinoma?

      1. Abundant PAS+, diastase sensitive secretions
      2. High grade cytologic atypia with brisk mitotic activity
      3. Irregular clusters of epithelial cells floating in pools of extracellular mucin
      4. Low grade cytologic atypia, triple negative for ER, PR and HER2
      5. Negative for basal-like markers CK5/6 or CK14
      Board review style answer #2
      D. Secretory carcinoma of breast is most likely to show low grade cytologic atypia, triple negative for ER, PR and HER2

      Comment Here

      Reference: Secretory carcinoma of breast

      Secretory change
      Definition / general
      • Pregnancy-like changes in a nonpregnant, nonlactating patient
      Epidemiology
      • Present in 2 - 3% of breast biopsies
      • Identified in needle localization and core biopsies due to calcifications or presence of a mass
      • Women, mean age 44 years, range 38 - 52 years
      • Not lactating, not pregnant (by definition)
      Etiology
      Clinical features
      Treatment
      Gross description
      • No gross lesion
      Microscopic (histologic) description
      • Glands and terminal ducts with little or no secretion
      • Glandular cells are swollen with abundant pale or clear, finely granular or vacuolated cytoplasm
      • Luminal cytoplasmic borders of glandular cells are frayed with small cytoplasmic blebs extending into lumen that may contain nuclei
      • Small, uniform, round and darkly stained nuclei
      Microscopic (histologic) images

      AFIP images

      Expanded acinar glands

      Cells have abundant pale cytoplasm but no secretion

        

      Micropapillary pattern
      of columnar cells with
      luminal cytoplasmic buds



      Images hosted on other servers:

      Cells have finely vacuolated cytoplasm

      Differential diagnosis

      Sentinel lymph nodes
      Definition / general
      • Sentinel lymph nodes (SLN) are the first lymph nodes to receive lymphatic drainage and metastasis from a tumor
      Essential features
      • Blue dye (isosulphan blue or methylene blue) or radioactive isotope labeled colloid (technetium sulfur colloid) is injected to identify the draining SLN
      • Entire breast largely drains to the same few SLN (Arch Surg 2004;139:614)
      • SLN are usually axillary nodes (level I) but may be at level II or III; rarely intramammary, interpectoral (Rotter) or internal mammary node (Eur J Surg Oncol 2009;35:252)
      • Intraoperative frozen section or intraoperative imprint cytology can be performed on the SLN (even after neoadjuvant chemotherapy) to determine need for axillary lymph node dissection (ALND) (Acad Radiol 2009;16:551)
      Terminology
      • At least 1 node with presence or absence of cancer as documented by pathologic examination is required for pathologic N classification in breast cancer staging
      • Axillary lymph node metastases are classified into 3 groups: isolated tumor cell clusters, micrometastasis and macrometastasis
      • AJCC sn modifier: when the number of sentinel and nonsentinel nodes removed is < 6
      • AJCC f modifier: when nodal metastasis is confirmed by FNA or core biopsy only
      ICD coding
      • ICD-10: C77.3 - axillary sentinel lymph node
        • Effective in 2019 edition of ICD-10-CM, starting October 1, 2018
      • ICD-11: 2D60.3 - malignant neoplasm metastasis in axillary lymph nodes
      • CPT codes (surgical pathology) for axillary lymph node:
        • 88305 - axillary tail / axilla, lymph node biopsy
        • 88307 - lymph node, sentinel
        • 88307 - axillary tail / axilla, lymph nodes regional resection
      Epidemiology
      Sites
      • Breast lymphatic drainage (Am J Clin Pathol 2018;150:4):
        • Superficial system: superficial breast and skin to axilla, independent
        • Deep system: breast to axilla, anastomoses with perforating system
        • Perforating system: traverses pectoralis muscles and drains into internal mammary node
      • Axillary lymph nodes are divided into 3 levels:
        • Low axilla: lateral to the lateral border of the pectoralis minor muscle
        • Mid axilla: between the medial and lateral borders of the pectoralis minor muscle, plus the interpectoral (Rotter) lymph nodes
        • Apical axilla or infraclavicular nodes: medial to the medial margin of the pectoralis minor muscle and inferior to the clavicle
      • Intramammary nodes are most commonly present in the upper outer quadrant and are included with axillary nodes in AJCC N classification
      • Internal mammary nodes, supraclavicular nodes and infraclavicular nodes are rarely removed for breast cancer staging
        • If metastases are present in these nodes, there are specific AJCC N categories
        • Medially located tumors are more likely to drain to internal mammary nodes (medial 28%; lateral 15%) (Am J Clin Pathol 2018;150:4)
      • Tumor is more likely at the inflow junction of afferent lymphatic vessels (Am J Surg Pathol 2003;27:385)
      Diagrams / tables

      Images hosted on other servers:

      Sentinel lymph node biopsy

      Clinical features
      • SLN biopsy is successful in > 90% of eligible breast cancer patients
      Diagnosis
      • Gross sectioning of SLN at 2 mm intervals and histological examination of submitted tissue
      Radiology description
      • Axillary ultrasound (US) is the primary modality for evaluating axillary lymph node status prior to surgery
        • Cortical thickening, hilar effacement and non-hilar cortical blood flow, are more important than size criteria (Radiographics 2013;33:1589)
        • Preoperative axillary US and FNA cytology are routine at many breast units, with a sensitivity of 56% (confidence interval: 47 - 64%) and specificity of 90% (84 - 93%) for US alone and 76% (61 - 87%) and 100% (65 - 100%) when combined with FNA cytology (J BUON 2011;16:454)
      • Standard breast MRI is comparable to dedicated axillary US in breast cancer patients; subsequent axillary US can be done on those with suspicious nodal findings (Eur J Radiol 2016;85:2288)
      • CT and radionuclide imaging play a lesser role in imaging the axilla (Radiographics 2013;33:1589)
      Prognostic factors
      • High risk (60%) of tumor in nonsentinel nodes if sentinel node has macrometastatic tumor (≥ 2 mm) versus low risk (3%) if micrometastatic tumor (0.2 - 2 mm) (Mod Pathol 2005;18:762)
      • Extranodal extension was a predictor of nonsentinel node involvement in a single institution study of 266 cases (Sci Rep 2020;10:14684)
      Case reports
      Treatment
      • SLN biopsy (versus ALND) is standard of care in staging clinically node negative T1 / T2 tumors
      • Number of involved lymph nodes and size of deposit (micro / macrometastasis) are important for clinical decision making (Ann Oncol 2018;29:2153)
      • Per current NCCN guidelines, clinically node negative patients with only micrometastasis or with macrometastasis meeting ACOSOG Z11 criteria (T1 / T2 tumor, ≤ 2 positive nodes, breast conserving surgery, whole breast RT planned, no preoperative chemotherapy) may be spared ALND
      • ER+ / HER2- patients with limited involvement of axillary lymph nodes might be spared adjuvant chemotherapy if the tumor biology is favorable (Ann Oncol 2018;29:2153)
      • Role of nodal radiation therapy in patients with 1 - 3 positive lymph nodes is unclear, though there is modest increase in disease free survival (JAMA Oncol 2016;2:991)
        • Current NCCN guidelines recommend strongly considering radiation therapy
      Clinical images

      Images hosted on other servers:

      Blue stained axillary node

      Gross description
      • Axillary SLN may be blue dyed, with or without surrounding adipose tissue
      Frozen section / intraoperative description
      • Axillary SLN are identified by the surgeon by determining uptake of radioisotope technetium 99 sulfur colloid, methylene blue dye or both
      • Intraoperative frozen section or imprint cytology may be performed on the SLN to determine need for axillary lymph node dissection (World J Surg Oncol 2008;6:69, Eur J Surg Oncol 2009;35:16)
      • Frozen section (FS) had an overall mean sensitivity of 73% (macrometastasis: 94%; micrometastasis 40%) and mean specificity of 100% (metaanalysis of 47 studies, Cancer 2011;117:250)
      • SLN FS may be safely omitted in patients who meet ACOSOG Z11 criteria (Clin Breast Cancer 2018;18:276, J Clin Pathol 2016;146:57)
      • FNR of intraoperative frozen section post neoadjuvant chemotherapy was 5.4% in a single institution study of 711 cases (Am J Surg Pathol 2019;43:1377)
      • Atypical cases on FS are usually negative on permanent sections (Mod Pathol 2005;18:58)
      • In most cases, if metastases are present, the SLN will be involved; in rare cases, only nonsentinel nodes contain metastases
      • Metastasis to nonsentinel lymph nodes can occur if the true SLN is completely replaced by tumor (and therefore is not detected by radioactive tracer or dye), if there is unusual lymphatic drainage or if there is failure of the technique to identify the node
      Frozen section images

      Contributed by Sucheta Srivastava, M.D.

      Macrometastasis

      Micrometastasis

      Microscopic (histologic) description / diagnostic criteria
      • See Staging of breast carcinoma
      • Size of the metastatic focus is measured from the largest contiguous cluster of tumor cells
      • Direct extension of primary tumor into a regional node is classified as node positive
        • Tumor nodule with a smooth contour in a regional node area is classified as a positive node
        • Size of the metastasis, not the size of the node, is used for the criterion for the N category
      • Cases with isolated tumor cells only in lymph nodes are classified as pN0(i+)
      • Dispersed pattern of lymph node metastasis can be difficult to categorize
        • Particularly lobular carcinomas metastasize as single cells and do not form cohesive clusters
        • If > 200 tumor cells are present in 1 cross section of the node, then the category isolated tumor cells should not be used
        • If there is difficulty in assigning the N classification, it is recommended that the reason be provided in a note
      • The area of extracapsular extension is included in the overall size of the lymph node metastasis if contiguous
        • Size of the metastasis includes the tumor cells and the desmoplastic response in treatment naïve patients (i.e. the cells do not need to be contiguous but the cells plus fibrosis should be contiguous)
        • Finding of extranodal invasion is also reported
      • Areas of carcinoma invading into the stroma in axillary adipose tissue, without residual nodal tissue, are considered to be positive lymph nodes
      Microscopic (histologic) images

      Contributed by Sucheta Srivastava, M.D. and Jaya Ruth Asirvatham, M.B.B.S.

      Macrometastasis

      Macrometastasis with extranodal extension

      Micrometastasis


      Status post neoadjuvant chemotherapy

      Isolated tumor cells

      Isolated tumor cells

      Isolated tumor cells, cytokeratin

      Isolated tumor cells, pancytokeratin

      Virtual slides

      Images hosted on other servers:

      Metastatic breast ductal adenocarcinoma, lymph node

      Cytology description
      • Fine needle aspiration smears of positive axillary lymph node are characterized by crowded, disorganized groups of cells with enlarged nuclei, nuclear pleomorphism, irregular nuclear membranes and intracytoplasmic vacuoles with or without mucin
      • Necrosis or mucin in the background and single intact malignant cells may be present
      • Imprint cytology had an overall sensitivity of 63% (macrometastasis: 81%, micrometastasis: 22%) and specificity of 99% (meta analysis of 31 studies, Br J Surg 2005;92:1068)
      • Factors that can affect the outcome of fine needle aspiration are size of the metastasis in the lymph node, experience of the person performing image guided fine needle aspiration and pathologist evaluation of the sample for adequacy (at the time of fine needle aspiration)
      Cytology images

      Contributed by Sucheta Srivastava, M.D.

      Touch prep

      Diff-Quik

      Papanicolaou

      Cell block

      Positive stains
      Molecular / cytogenetics description
      • Some studies have reported usefulness of molecular assays, usually reverse transcriptase polymerase chain reaction (RT-PCR) (J Clin Oncol 2008;26:3338)
        • False positive and false negative results can occur with RT-PCR
        • Significance of a positive RT-PCR result for a histologically negative lymph node is unknown (J Clin Oncol 2008;26:3338)
        • Current AJCC cancer staging manual (8th edition) includes positive molecular findings by RT-PCR; no isolated tumor cell clusters detected as pN0 (mol+)
      Sample pathology report
      • Right axilla, sentinel lymph node #1, excision:
        • One lymph node negative for metastatic carcinoma (0/1)
      • Right axilla, sentinel lymph node #1, excision:
        • One lymph node with macrometastatic carcinoma (12 mm deposit) with extranodal extension (measuring 2 mm) (1/1)
      Differential diagnosis
      Additional references
      Board review style question #1

      The finding in the image above of a pancytokeratin immunostain of an axillary sentinel lymph node from a breast cancer patient is best described as

      1. pN0(i+)
      2. pN1a
      3. pN1mi
      4. pNX
      Board review style answer #1
      A. pN0(i+): isolated tumor cell clusters (malignant cell clusters ≤ 0.2 mm)

      Comment Here

      Reference: Sentinel lymph nodes
      Board review style question #2
      Which of the following is true for axillary sentinel lymph nodes?

      1. Evaluation requires grossing at 2 mm intervals and routine use of immunohistochemistry
      2. Isolated tumor cell clusters are clusters of cells ≤ 0.2 mm and ≤ 200 cells in a single cross section
      3. Most commonly, sentinel lymph node is in level II
      4. Only the first excised lymph node is a sentinel lymph node
      Board review style answer #2
      B. Isolated tumor cell clusters are clusters of cells ≤ 0.2 mm and ≤ 200 cells in a single cross section

      Comment Here

      Reference: Sentinel lymph nodes
      Board review style question #3
      Which of the following supports a diagnosis of metastatic carcinoma in sentinel lymph nodes?

      1. Bland nests located within lymph node capsule that are positive for MelanA
      2. Cell clusters within lymph node parenchyma with significant nuclear atypia
      3. Glands with ciliated cells that are positive for PAX8
      4. Glandular structures with peripheral myoepithelial cells
      Board review style answer #3
      B. Cell clusters within lymph node parenchyma with significant nuclear atypia

      Comment Here

      Reference: Sentinel lymph nodes

      Silicone leak / lymphadenopathy
      Definition / general
      • Foreign body inflammatory reaction induced by silicone
        • Diffuse in the breast → silicone mastitis
        • Nodular in the breast → silicone granulomas
        • Leakage to regional lymph nodes → silicone lymphadenitis
      Essential features
      • Complication of leakage or rupture of silicone containing breast prosthesis or as a reaction to silicone injection
      • Foreign body inflammatory reaction induced by silicone
      • Most common sites: breast, regional lymph nodes
      Terminology
      • Silicone granuloma
      • Silicone mastitis
      • Silicone lymphadenopathy
      ICD coding
      • ICD-10: T85.49XA - other mechanical complication of breast prosthesis and implant, initial encounter
      • ICD-11: PK9B.2 - general or plastic surgery devices associated with injury or harm, prosthetic or other implants, materials or accessory devices
      Epidemiology
      Sites
      • Breast and regional lymph nodes
      Pathophysiology
      Etiology
      • Leakage of silicone gel contained in prostheses
      Clinical features
      Diagnosis
      • Can be suspected clinically and radiologically
      • Histopathology is the gold standard for a definitive diagnosis
      Radiology description
      • Mammography: radiopaque silicone outside the implant shell in the breast or lymph nodes with or without irregular and coarse calcifications
      • Ultrasound: extracapsular silicone in the breast or lymph nodes (snowstorm sign) (Indian J Radiol Imaging 2016;26:216)
      • Magnetic resonance imaging: high signal intensity deposits in the breast or lymph nodes
      Radiology images

      Images hosted on other servers:
      Rupture of silicone implant (mammography)

      Rupture of silicone implant (mammography)

      Snowstorm sign (ultrasound)

      Snowstorm sign (ultrasound)

      Extracapsular free silicone conglomerates (MRI)

      Extracapsular free silicone conglomerates (MRI)

      Case reports
      Treatment
      Gross description
      • Firm to hard nodular surface, gritty if there is calcification (Insights Imaging 2018;9:59)
      • Cystic spaces containing thick pale yellow or white material
      Gross images

      Contributed by Debra L. Zynger, M.D.

      Breast mass



      Images hosted on other servers:
      Fibrous capsule

      Fibrous capsule

      Silicone lymphadenopathy

      Silicone lymphadenopathy

      Microscopic (histologic) description
      • Features are not specific:
        • Chronic inflammatory cells, foreign body giant cell reaction and fibrosis
        • Fat necrosis → histiocytes containing clear, refractile, nonpolarizable material within cytoplasmic vacuoles
        • Extracellular silicone can also be seen as nonbirefringent crystals or particles within empty spaces
      • Later features include:
        • Fibrous capsule or bursa: well defined band of collagenized fibrous tissue and mixed chronic inflammatory cells
        • Calcifications in the capsule or around: globular aggregates, may have bone formation
      Microscopic (histologic) images

      Contributed by Syed Hoda, M.B.B.S. and Julie M. Jorns, M.D.
      Capsule with silicone

      Capsule with silicone

      Silicone and synovial metaplasia

      Silicone and synovial metaplasia

      Foamy histiocytes Foamy histiocytes

      Foamy histiocytes


      Clinically ruptured silicone implant Clinically ruptured silicone implant

      Clinically ruptured silicone implant

      Silicone lymphadenitis Silicone lymphadenitis Silicone lymphadenitis

      Silicone lymphadenitis


      Implant with associated seroma Implant with associated seroma

      Implant with associated seroma

      Silicone lymphadenitis Silicone lymphadenitis

      Silicone lymphadenitis

      Virtual slides

      Images hosted on other servers:

      Breast, silicone granuloma

      Lymph node, silicone lymphadenopathy

      Cytology description
      Positive stains
      Negative stains
      Electron microscopy description
      • Calcifications are composed of hydroxyapatite crystals
      Sample pathology report
      • Left breast, core needle biopsy:
        • Benign breast tissue with fibrous capsule with associated foreign body giant cell reaction to silicone, chronic inflammation and fat necrosis
      Differential diagnosis
      • Sinus histiocytosis with massive lymphadenopathy:
        • Fever, leukocytosis, anemia
        • Usually no history of breast implant
        • Large histiocytes with emperipolesis, plasma cells
        • No foreign body giant cell reaction
        • No foreign material
      • Fat necrosis:
        • Usually secondary to injury or previous procedures
        • No history of breast implant
        • May have foreign material from prior procedure(s) but is morphologically different from silicone
      • Abscess / acute mastitis:
        • Associated with lactation in most cases
        • Neutrophils are predominant
        • Bacteria are usually present (gram positive)
        • No foreign material
      • Cystic neutrophic granulomatous mastitis:
        • Fever, leukocytosis, anemia
        • Granulomatous inflammation with cystic spaces rimmed by neutrophils
        • Coryneform bacteria is present in cystic spaces (gram positive, culture or molecular testing)
        • No foreign material
      Board review style question #1

      A 34 year old nulliparous female presented with a unilateral breast mass 8 years after bilateral breast augmentation with silicone implants. On examination, the patient was afebrile but her breast was tender with a focal deformity. Which of the following histomorphological features is associated with her condition?

      1. Caseating granulomas
      2. Cystically dilated ducts with inspissated material
      3. Gram positive bacteria
      4. Histiocytes with refractile material
      Board review style answer #1
      D. Histiocytes with refractile material

      Comment Here

      Reference: Silicone leak / lymphadenopathy
      Board review style question #2
      Which of the following features is seen in silicone lymphadenitis?

      1. Crystals
      2. Emperipolesis
      3. Fungal hyphae
      4. Langhans giant cells
      Board review style answer #2

      SMOLD
      Definition / general
      • Keratinizing squamous metaplasia of lactiferous ducts (SMOLD) in association with acute and chronic inflammation including foreign body giant cells
      Essential features
      • Retroareolar inflammatory lesion due to lactiferous duct blockage and rupture secondary to keratinizing squamous metaplasia
      • Strong association with smoking
      • Characteristic microscopic findings include a mixed inflammatory infiltrate including keratinaceous debris within giant cells and lactiferous ducts with squamous metaplasia or intraluminal keratin
      • Recurrence is common without definitive treatment, including excision of the affected duct and smoking cessation
      Terminology
      • Zuska disease, recurrent subareolar abscess, periductal mastitis, subareolar fistula
      ICD coding
      • ICD-10: N61 - inflammatory disorders of breast
      Epidemiology
      Sites
      • Subareolar, breast (bilateral)
      Pathophysiology
      • Accumulation of keratin secondary to squamous metaplasia in deeper lactiferous ducts with subsequent blockage and rupture
      • Rupture of duct with extrusion of keratinaceous debris incites a mixed inflammatory response
      • Secondary infection or abscess may occur
      • Repeat recurrences may lead to fistula formation
      • Fistula tract forms beneath smooth muscle of areola, which is a path of least resistance
      • Reference: Schnitt: Biopsy Interpretation of the Breast, 3rd Edition, 2017
      Etiology
      • Strong correlation with smoking
        • Relative vitamin A deficiency related to smoking may be a causative factor in squamous metaplasia
      • Less common associations
      Clinical features
      • Painful, warm, erythematous swelling or mass near nipple
      • Often misinterpreted as an abscess
      • Less common features include purulent nipple discharge, skin thickening or ipsilateral axillary lymphadenopathy
      • Inverted nipple can be seen
      • Fistula tract opening at the edge of the areola is characteristic
      • Reference: Schnitt: Biopsy Interpretation of the Breast, 3rd Edition, 2017
      Diagnosis
      • Integrative diagnosis utilizing clinical findings, radiology and pathology
      Radiology description
      • Mammogram: retroareolar localized increased stroma / asymmetric density; less common findings include stromal distortion, solitary mass or normal imaging (Clin Radiol 2012;67:e42)
      • Ultrasound: medial periareolar hypoechoic lesion, possibly with discreet sinus or abscess with adjacent dilated ducts (Clin Radiol 2012;67:e42)
      Case reports
      • 29 year old woman with 8 month history of nipple cellulitis and inversion (BMJ Case Rep 2020;13:e237568)
      • 56 year old woman with recurrent left subareolar breast abscesses and fistula associated with Prevotella bivia infection (Cureus 2022;14:e28904)
      • 82 year old woman with diagnosis of SMOLD in association with primary squamous cell carcinoma (Breast Cancer 2018;25:365)
      Treatment
      • Excision of affected duct is most effective
      • Antibiotics or incision and drainage procedures result in recurrence
      • Ultrasound guided percutaneous needle electrolysis (PNE) was effective treatment in a small series (J Clin Med 2020;9:649)
      • Smoking cessation is recommended
      • Reference: BMJ Case Rep 2020;13:e237568
      Clinical images

      Contributed by Nicolas Ajkay, M.D.
      SMOLD with acute abscess

      Acute abscess

      SMOLD with single fistula

      Single fistula

      Gross description
      • Portion (or entire) nipple - areola complex and underlying breast tissue containing possible fistula tract or abscess
      Microscopic (histologic) description
      • Squamous metaplasia past the normal transition point to ductal epithelium, which is 1 - 2 mm beneath the skin surface
      • Ducts filled or expanded by keratin debris
      • Duct rupture with keratin debris inciting acute and chronic inflammatory reaction
      • Keratin debris within giant cells is characteristic
      • Reference: Schnitt: Biopsy Interpretation of the Breast, 3rd Edition, 2017
      Microscopic (histologic) images

      Contributed by Katie C. Hall, M.D., Anna Biernacka, M.D., Ph.D. and Mary Ann Gimenez Sanders, M.D., Ph.D.
      Extension of squamous epithelium

      Extension of squamous epithelium

      Squamous metaplasia of lactiferous duct

      Squamous metaplasia of lactiferous duct

      Squamous metaplasia

      Squamous metaplasia

      Keratin accumulation, duct distension

      Keratin accumulation, duct distension

      Squamous and glandular lining

      Squamous and glandular lining

      Duct rupture

      Duct rupture


      Inflammatory response

      Inflammatory response

      Keratin and acute inflammation

      Keratin and acute inflammation

      Foreign body giant cells

      Foreign body giant cells

      Giant cells engulfing keratin

      Giant cells engulfing keratin

      Inflammation, giant cells, keratin

      Inflammation, giant cells, keratin

      Keratin, slit-like spaces

      Keratin, slit-like spaces


      Periductal inflammation

      Periductal inflammation

      Metaplastic squamous epithelium

      Metaplastic squamous epithelium

      Plasma cell rich infiltrate

      Plasma cell rich infiltrate

      Surface epithelium

      Surface epithelium

      Involved duct Involved duct

      Involved duct


      Keratin debris

      Keratin debris

      Lactiferous ducts of the nipple Lactiferous ducts of the nipple

      Lactiferous ducts of the nipple

      Deep in nipple

      Deep in nipple


      Normal transition Normal transition

      Normal transition

      Normal transition, CK7

      Normal transition, CK7

      Normal transition, p63

      Normal transition, p63

      Cytology description
      • Metaplastic squamous cells, anucleated squamous cells and mixed inflammation including macrophages and giant cells (Clin Radiol 2012;67:e42)
      Cytology images

      Contributed by Mary Ann Gimenez Sanders, M.D., Ph.D.
      Pap smear Pap smear

      Pap smear

      Negative stains
      Sample pathology report
      • Breast, left, retroareolar, ultrasound guided biopsy:
        • Skin and breast parenchyma with chronic active inflammation including foreign body giant cells (see comment)
        • Comment: There are rare keratin flakes associated with the inflammatory infiltrate. Given the patient's history of recurrent painful subareolar abscesses and chronic tobacco use, these findings could be compatible with squamous metaplasia of lactiferous ducts (SMOLD). Clinical correlation is recommended.
      Differential diagnosis
      • Mammary duct ectasia:
        • Major and minor ducts affected with lymphocyte predominant inflammatory reaction
        • Histiocytes within and surrounding duct wall in response to lipid or secretory material, not keratin
        • Periductal fibrosis may be present
      • Retroareolar abscess secondary to infection:
        • Specific microorganism(s) identified via special stains or culture
        • Squamous metaplasia or keratin debris not present in stroma
      • Epidermal inclusion cyst:
        • Present within superficial dermis and not continuous with lactiferous duct
      • Cystic neutrophilic granulomatous mastitis or idiopathic granulomatous mastitis:
        • Lobulocentric granulomatous inflammation
        • Peripheral location
        • Sinus tracts may open to skin surface without associated squamous metaplasia or central location
      Board review style question #1

      A 39 year old woman presents with a retroareolar mass. She has a history of recurrent retroareolar abscesses previously treated with antibiotics and incision / drainage. An ultrasound guided biopsy of the mass shows similar findings to those in the photo. What is the definitive treatment?

      1. Central excision to include entire nipple
      2. Excision of affected duct
      3. Incision / draining and antibiotics
      4. Smoking cessation
      Board review style answer #1
      B. Excision of affected duct. This is the correct answer and definitive treatment; squamous metaplasia of lactiferous ducts (SMOLD) is likely to recur without excision of the affected duct. Answer D is incorrect because while smoking cessation is suggested, it is not the definitive treatment. Answer A is incorrect because excision is the definitive treatment; however, the entire nipple does not necessarily need to be included. Answer C is incorrect because antibiotics and incision / drainage is the treatment for an abscess.

      Comment Here

      Reference: SMOLD
      Board review style question #2
      What finding differentiates squamous metaplasia of lactiferous ducts (SMOLD) from other inflammatory breast conditions?

      1. Duct rupture with associated acute and chronic inflammation
      2. Lack of microorganisms on special stains and culture
      3. Lobulocentric granulomatous inflammation with neutrophils
      4. Squamous metaplasia with keratinaceous debris, possibly with foreign body giant cells
      Board review style answer #2
      D. Squamous metaplasia with keratinaceous debris, possibly with foreign body giant cells. These findings in the appropriate clinical context and location are diagnostic for SMOLD. Answer B is incorrect because this is not the only inflammatory breast disease that can lack microorganisms on special stains or culture. Answer C is incorrect because a lobulocentric pattern of inflammation would not be expected in SMOLD. This is pattern of inflammation can be seen in cystic neutrophilic granulomatous mastitis and idiopathic granulomatous mastitis. Answer A is incorrect because duct rupture can be seen in SMOLD as well as mammary duct ectasia.

      Comment Here

      Reference: SMOLD

      Solid papillary carcinoma
      Definition / general
      • Subtype of ductal carcinoma characterized by solid cellular nodules punctuated by thin fibrovascular cores
      • Frequently demonstrates neuroendocrine differentiation
      • May be either in situ or invasive
      Essential features
      • Circumscribed solid growth pattern with interspersed delicate fibrovascular cores
      • Low or intermediate grade nuclear atypia
      • Neuroendocrine differentiation is common
      • In situ carcinoma is diagnosed when the tumor is composed of well circumscribed solid nests in a distribution pattern consistent with an intraductal process; myoepithelial cells may or may not be demonstrable at the periphery
      • Invasive carcinoma is diagnosed when there is a lack of myoepithelium and a convincing pattern of infiltrative growth
      • ER+ / PR variable / HER2-
      • Associated with an excellent prognosis (Breast 2016;26:67)
      Terminology
      • In situ or invasive nature of the tumor should be specified
      • Also called neuroendocrine breast carcinoma in situ, spindle cell ductal carcinoma in situ, neuroendocrine ductal carcinoma in situ, endocrine ductal carcinoma in situ
      ICD coding
      • ICD-O: 8509/2 - solid papillary carcinoma in situ
      • ICD-O: 8509/3 - solid papillary carcinoma with invasion
      • ICD-10: C50 - malignant neoplasm of breast
      • ICD-11: 2E65.Y & XH0134 - other specified carcinoma in situ of breast & solid papillary carcinoma in situ
      • ICD-11: 2C64 - solid papillary carcinoma of breast with evidence of invasion
      Epidemiology
      Sites
      • Any part of the breast but the central / subareolar region is most common
      Pathophysiology
      • Unknown
      Etiology
      • No specific etiologic factors
      Clinical features
      Diagnosis
      • Diagnosed by histologic examination of tissue removed by biopsy or surgical excision
      Radiology description
      • On mammography, solid papillary carcinoma can appear as a lobulated, circumscribed mass, with or without calcifications
      • Margins may be indistinct in invasive cases (AJR Am J Roentgenol 2012:198;264)
      • Ultrasonography may demonstrate a hypoechoic or heterogeneous solid mass (AJR Am J Roentgenol 2012:198;264)
      • Magnetic resonance imaging findings include mass enhancement with circumscribed margins, oval or irregular shapes, heterogeneous signal intensity and rapid enhancement in initial contrast phases (BMC Cancer 2017;17:525)
      Radiology images

      Contributed by Melinda Lerwill, M.D.
      Invasive solid papillary carcinoma

      Invasive solid papillary carcinoma

      Prognostic factors
      • Excellent prognosis
      • Axillary lymph node metastasis (6%) and distant metastasis are rare and appear limited to cases with an overt invasive component (Breast 2016;26:67)
      Case reports
      Treatment
      • Solid papillary carcinoma in situ is a form of ductal carcinoma in situ and is managed as such
        • Surgical excision
        • Radiation therapy in the setting of breast conserving surgery
        • Consideration of endocrine therapy for risk reduction
      • Invasive solid papillary carcinoma is managed as an invasive breast carcinoma
        • Surgical excision and sentinel node evaluation
        • Radiation therapy in the setting of breast conserving surgery
        • Endocrine therapy is typically recommended due to the strong ER positivity of these tumors
        • Systemic therapy decisions should factor in the favorable prognosis of this histologic subtype
      Gross description
      • Mass forming examples demonstrate a well circumscribed, firm to soft tumor with tan-white to tan-pink cut surfaces (Am J Surg Pathol 1995;19:1237)
      • In situ examples with less confluent growth may demonstrate scattered firm nodularity
      Gross images

      Contributed by Melinda Lerwill, M.D.
      Invasive solid papillary carcinoma

      Invasive solid papillary carcinoma

      Frozen section description
      Microscopic (histologic) description
      • Expansile nodules composed of a solid epithelial proliferation punctuated by delicate fibrovascular cores
      • Cytologic features (Am J Surg Pathol 1995;19:1237)
        • Monomorphic cytologic appearance
        • Round, oval, plasmacytoid or spindle shaped cells
        • Low to intermediate grade nuclear atypia
          • Mildly enlarged, round to oval nuclei with fine chromatin and indistinct nucleoli
          • Some examples have slightly more open, granular chromatin and small nucleoli
          • Nuclear grooves are occasionally conspicuous
        • Variable mitotic activity
        • Cytoplasm ranges from pale and amphophilic to eosinophilic and granular
        • Intracellular and extracellular mucin are common and signet ring cells are occasionally observed
      • Architectural features (Am J Surg Pathol 1995;19:1237)
        • Solid epithelial growth pattern with interspersed thin fibrovascular cores
        • Cellular palisading around fibrovascular cores
        • Streaming growth pattern is common in cases with spindle cells
        • Microcystic spaces are occasionally present
        • Fibrovascular cores can demonstrate hyalinization
      • Solid papillary carcinoma in situ
        • In situ carcinoma is diagnosed when the nodules have smooth rounded contours and a distribution pattern consistent with involvement of the underlying glandular tree
        • Myoepithelial cells may be retained, attenuated or absent around the perimeter of the nodules
          • If the morphology is consistent with an intraductal process, in situ carcinoma (pTis) is diagnosed even if no myoepithelial cell layer is seen
          • Careful histologic assessment of tumor distribution and borders is required in cases without an identifiable myoepithelial cell layer
        • Myoepithelial cells are often attenuated or absent along the internal fibrovascular cores
        • Graded according to nuclear features
      • Solid papillary carcinoma with invasion
        • Invasive carcinoma arising in association with solid papillary carcinoma in situ may take one of several forms:
          • Invasive solid papillary carcinoma
          • Mucinous carcinoma
          • Conventional pattern of invasive carcinoma (most often invasive ductal carcinoma, NOS)
          • Admixture of the above patterns
        • Invasive solid papillary carcinoma is diagnosed when a solid papillary carcinoma shows evidence of destructive growth within extralobular stroma
          • Tumor nests have angulated or ragged contours or form irregularly anastomosing islands
          • Jigsaw-like appearance
          • Distribution is too confluent and haphazard to represent involvement of the underlying glandular tree
          • Myoepithelial cells are absent
          • Additional clues to infiltrative growth:
            • Desmoplastic stroma
            • Engulfment of normal glands or fat
            • Vascular invasion
          • In rare examples, the invasive nests may have more rounded shapes but their geographic confluence, expansile mass forming growth, random distribution and association with reactive stroma are indicative of growth beyond the confines of the ductal tree
        • Nottingham grading and ER, PR and HER2 studies are applied to the invasive component only
        • Tumor is staged according to the size of the invasive component only
      Microscopic (histologic) images

      Contributed by Melinda Lerwill, M.D.
      Multiple nodules

      Multiple nodules

      Solid growth

      Solid growth

      Cytologic atypia Cytologic atypia

      Cytologic atypia

      Spindle cells

      Spindle cells

      Mucin

      Mucin


      Invasive pattern

      Invasive pattern

      Jigsaw-like pattern

      Jigsaw-like pattern

      Irregular invasive nests

      Irregular invasive nests

      Ragged contours

      Ragged contours

      Solid papillary carcinoma with mucinous carcinoma

      Solid papillary carcinoma with mucinous carcinoma

      Chromogranin

      Chromogranin


      Cytokeratin 14

      Cytokeratin 14

      p63

      p63

      Virtual slides

      Images hosted on other servers:

      Solid papillary carcinoma

      Cytology description
      • Hypercelluar specimen
      • Small to large discohesive epithelial clusters and many single isolated cells with a low nuclear cytoplasmic ratio (Diagn Cytopathol 2007;35:417)
      • Small round to oval, bland nuclei with finely granular chromatin, eccentrically placed nuclei and inconspicuous nucleoli
      • Lack of naked myoepithelial cell nuclei in the background
      • Mucin and capillary vascular structures are infrequently noted (Diagn Cytopathol 2007;35:417)
      Cytology images

      Contributed by Melinda Lerwill, M.D.
      Granular chromatin

      Granular chromatin

      Capillary vessel

      Capillary vessel

      Positive stains
      Negative stains
      Electron microscopy description
      Molecular / cytogenetics description
      • Luminal phenotype with relatively simple genomes and few copy number aberrations (J Pathol 2012;226:427)
      • 16q loss, 16p gains and 1q gains (J Pathol 2012;226:427)
      • Genes related to neuroendocrine differentiation, including RET, ASCL1 and DOK7, are expressed at higher levels in solid papillary carcinomas than in encapsulated papillary carcinomas (Mol Oncol 2014;8:1588)
      Videos

      Papillary lesions of the breast (Dr. Collins)

      Papillary lesions of the breast
      (Dr. Collins)

      Sample pathology report
      • Left breast, excision:
        • Solid papillary carcinoma in situ, grade 1, spanning at least 1.0 cm
        • Alternative terminology: ductal carcinoma in situ, solid papillary type
      • Right breast, excision:
        • Invasive solid papillary carcinoma (1.2 x 1.0 cm), grade 2
        • Alternative terminology: invasive ductal carcinoma, solid papillary type
      Differential diagnosis
      Board review style question #1

      Which of the following statements most accurately characterizes the illustrated breast lesion?

      1. Solid papillary carcinoma in situ frequently demonstrates neuroendocrine differentiation
      2. Solid papillary carcinoma in situ is associated with a poor prognosis
      3. Solid papillary carcinoma in situ is positive for high molecular weight cytokeratins
      4. Usual ductal hyperplasia involving a papilloma is associated with a 7 fold increase in risk for subsequent breast cancer
      5. Usual ductal hyperplasia involving a papilloma is most commonly observed in women in the eighth decade of life
      Board review style answer #1
      A. Solid papillary carcinoma in situ frequently demonstrates neuroendocrine differentiation. The illustrated lesion is an example of solid papillary carcinoma in situ. Solid papillary carcinoma is characterized by a solid cellular proliferation of monomorphic, atypical ductal cells with interspersed delicate fibrovascular cores. It demonstrates neuroendocrine differentiation in approximately half of cases and is associated with an excellent prognosis. Lack of expression of high molecular weight cytokeratins can aid in distinguishing it from usual ductal hyperplasia.

      Comment Here

      Reference: Solid papillary carcinoma
      Board review style question #2
      Invasive solid papillary carcinoma of the breast demonstrates

      1. Branching filiform papillae within a cystic space
      2. ER-, PR- , HER2- immunophenotype
      3. Frequent lymph node metastases
      4. Irregular tumor nests in a jigsaw-like pattern
      5. Myoepithelial cells along fibrovascular cores but not along the periphery of the tumor
      Board review style answer #2
      D. Irregular tumor nests in a jigsaw-like pattern. Invasive solid papillary carcinoma is diagnosed when there is destructive growth of the tumor within extralobular stroma, most readily recognized by the presence of irregularly shaped nests in a confluent, jigsaw-like pattern. Myoepithelial cells are absent. Invasive papillary carcinoma is rarely associated with lymph node metastases and is typically ER / PR+ and HER2-.

      Comment Here

      Reference: Solid papillary carcinoma

      Sparganosis
      Definition / general
      • Sparganosis is a parasitic zoonosis caused by spargana, the plercercoid tapeworm larvae of the genera Spirometra (Emerg Infect Dis 2017;23:665)
      Essential features
      ICD coding
      • ICD-10: B89 - unspecified parasitic disease
      • ICD-11: 1G2Y - other specified parasitic diseases
      Epidemiology
      Sites
      • Common sites are the superficial soft tissue and muscle (Parasitol Int 2020;75:102036)
      • Less commonly involves internal sites, such as the breast, abdominal wall, pericardium, liver, bone, eye and central nervous system
      Pathophysiology
      • Dogs; cats are the primary hosts of the adult tapeworms and when eggs are shed in their feces, they mature into coracidia in water and are ingested by copepods or crustaceans (the first intermediate hosts) (Lancet Infect Dis 2015;15:1226)
      • Copepods / crustaceans are consumed by amphibians, reptiles or fish (second intermediate hosts) and the coracidia develop into plerocercoids (larvae)
      • Cycle is completed when the primary hosts consume the secondary and the larvae mature into adult tapeworms in the hosts intestinal tracts
      • Humans may consume the first or second intermediate hosts or use them as part of a poultice and the larvae migrate into tissue sites outside of the gastrointestinal tract
      • Once in a site, the larvae form a nodular cyst or mass
      • Sparganum proliferum larvae may proliferate and present with multiple plerocercoids in a single site
      Etiology
      • Consumption of contaminated water, raw snakes / frogs and cultural practices such as placing raw poultices of snakes / frogs in wounds (Lancet Infect Dis 2015;15:1226)
      Diagrams / tables

      Images hosted on other servers:

      Life cycle of Spirometra

      Clinical features
      • Incubation period and clinical presentation depend on the route of infection and final location of the migrating larvae (Lancet Infect Dis 2015;15:1226)
      • Incubation period is typically 6 - 11 days but may take several years to present symptoms
      • Presents with localized erythema or slowly growing, itchy, nodules in the breast (Clin Infect Dis 2020 Jul 23 [Epub ahead of print])
      • Patients infected by Sparganum proliferum show symptoms associated with the spread of proliferating larvae to various organs in the body over the course of years (Lancet Infect Dis 2015;15:1226)
      Diagnosis
      • Histologic evaluation of surgically removed sparganum
      • Laboratory testing with polymerase chain reaction (PCR) for species level identification (Lancet Infect Dis 2015;15:1226)
      • Imaging with CT, MRI and ultrasonography depending on the site
      Laboratory
      Radiology description
      • Well defined serpiginous hypoechoic radio-opaque tubular structures are present, usually in subcutaneous fat (World J Surg 2011;35:573)
      • Can mimic malignancy on mammogram: multiple, lobular, marginated, amorphic and solid masses without calcifications, which may mimic circumscribed breast cancer or fibroadenoma
      • Migration of larvae induces inflammation and increased eosinophil response, which can cause increased F-18 fluorodeoxyglucose (FDG) uptake
      Radiology images

      Images hosted on other servers:

      Mammography with nodular and tortuous structure

      Ultrasound with nodular tubular hypoechoic lesion

      Mammography with tubular elongated structure

      Prognostic factors
      • Prognosis is dependent on the site of infection
      • Infection of the breast has an excellent prognosis and is curative with surgery
      Case reports
      Treatment
      • Primary prevention by avoidance of contaminated food, water and poultices
      • Complete surgical removal of superficial sparganum is the most effective therapy (Lancet Infect Dis 2015;15:1226)
      • Localized chemotherapy or anthelmintic drugs if the site is not suitable for surgery
      Clinical images

      Images hosted on other servers:

      Scolex of the sparganum

      Gross description
      • Sparganum larvae are white, ribbon-like parasites that typically range from 3 - 30 cm in length and 1 - 2 mm in width (Lancet Infect Dis 2015;15:1226)
      Gross images

      Images hosted on other servers:

      White, ribbon-like sparganum

      Microscopic (histologic) description
      • Cestode shows a pale myxoid matrix with longitudinal smooth muscle fibers and calcareous corpuscles (mineralized concretions), surrounded by a noncellular eosinophilic tegument (Breast J 2014;20:92)
      • Thin longitudinal smooth muscle fibers within myxoid matrix of the cestode larva area is the distinguishing feature for organism identification
      • Scolex is absent (in contrast to cysticerci)
      • Surrounding tissue may show chronic inflammation, cyst formation, granulomatous inflammation
      Microscopic (histologic) images

      Contributed by Bobbi Pritt, M.D.
      Sparganum in breast tissue

      Sparganum in breast tissue

      Spargana

      Spargana

      Longitudinal smooth muscle fibers

      Longitudinal smooth muscle fibers

      Calcareous corpuscles

      Calcareous corpuscles

      Positive stains
      Sample pathology report
      • Left breast, excisional biopsy:
        • Foreign body reaction to larval organisms showing a myxoid matrix with longitudinal grooves and calcareous corpuscles, consistent with sparganosis
      Differential diagnosis
      • Cysticercosis:
        • Clinically presents with multiple lesions
        • Classically involves the brain, skeletal and cardiac muscle
        • Caused by larval cysts of the tapeworm Taenia solium
        • Larvae with similar stroma but containing a scolex, hooks and surrounding fluid filled bladder
        • Absence of longitudinal smooth muscle
      • Acute mastitis:
        • Acute phase shows variable foci of acute inflammatory, necrotic and proteinaceous debris
        • May show granulation tissue and chronic inflammatory infiltration in chronic phase
        • Multinucleated foreign body cells may be present
      • Granulomatous mastitis:
        • Shows well formed granulomas within lobules or adjacent to ducts
        • May show Langhans giant cells and chronic inflammation
        • Microabscesses may be present with lipid vacuoles
      Board review style question #1
      Sparganosis TOPICNAME


      A 36 year old Chinese woman presents with a pruritic, slowly growing erythematous nodule on her left breast over the past 2 weeks. The nodule is surgically removed and shows the images above on review. What histological finding is most diagnostic of sparganosis of the breast?

      1. Eosinophilic tegument
      2. Longitudinal smooth muscle fibers
      3. Presence of a scolex
      4. Myxoid stroma
      Board review style answer #1
      B. Longitudinal smooth muscle fibers. Cestodes (tapeworms), including Eichanococcus, Taenia and Spirometra, show calcareous corpustles and a tegument (the outer covering) on histologic evaluation. However, these are not specific for Spirometra. Sparganum are also known for their absence of a scolex. A key histologic feature supportive of sparganosis is the longitudinal smooth muscle fibers.

      Comment Here

      Reference: Sparganosis

      Spread and metastases
      Definition / general
      • See also Staging topic
      • Breast carcinoma spreads by axillary, interpectoral and internal mammary lymphatic and vascular channels first to regional lymph nodes
      • Regional lymph nodes for breast carcinoma include ipsilateral intramammary, internal mammary, axillary and supraclavicular lymph nodes for staging purposes (pN)
      • Metastases to any other lymph nodes, including cervical or contralateral internal mammary or contralateral axillary lymph nodes, are considered distant metastases for staging purposes (pM)
      • Additional sites of distant metastases vary depending on tumor type and receptor status but most often include bone, lung, brain and liver
      • Circulating tumor cells or incidental tumor deposits (≤ 0.2 mm) called microscopic tumor cells do not constitute metastatic disease
      Essential features
      ICD coding
      • C77.3: Secondary and unspecified malignant neoplasm of axilla and upper limb lymph nodes
      • C77.9: Secondary and unspecified malignant neoplasm of lymph node, unspecified
      • C78.00: Secondary malignant neoplasm of unspecified lung
      • C79.31: Secondary malignant neoplasm of brain
      • C79.51: Secondary malignant neoplasm of bone
      Sites
      • Cervical, contralateral internal mammary and contralateral axillary lymph nodes are distant metastases (pM1)
      • Ipsilateral axillary, intramammary, internal mammary and supraclavicular lymph nodes are considered regional nodes, not distant, and are part of pathologic nodal staging of the primary tumor (pN)
      • HER2+ and triple negative (ER- / PR- / HER2-) breast tumors typically recur within 5 years of primary diagnosis (J Cancer Res Clin Oncol 2018 Apr 19 [Epub ahead of print])
      • Basal-like tumors have higher rates of brain, lung and distant nodal metastases than liver and bone (J Clin Oncol 2010;28:3271)
      • ER+ tumors often recur after long periods and have a propensity to first metastasize to bone (Medicine (Baltimore) 2016;95:e2909)
      • Malignant phyllodes tumors and primary sarcomas of the breast have a propensity to metastasize to lungs, bones and liver (Br J Cancer 2004;91:237)
      Radiology description
      • Imaging studies are utilized to screen for regional and distant metastases, focusing on organs most frequently involved in breast carcinoma
      • Axillary ultrasound or MRI may be used for preoperative evaluation of regional lymph nodes to determine clinical stage
      • National Comprehensive Cancer Network (NCCN) guidelines version 1.2018 for preoperative workup (NCCN: NCCN Guidelines [Accessed 24 May 2018]):
        • For clinical stage I - IIB: additional studies only if clinical signs or symptoms (i.e. bone scan if bone pain or elevated alkaline phosphatase; abdominal CT or MRI if abnormal liver function tests or abdominal symptoms)
        • For clinical stage IIIA (T3, N1, M0): strongly consider chest, abdominal and pelvic CT with contrast or MRI, bone scan, FDG PET / CT
      Prognostic factors
      • ER status and lymph node involvement are significant prognostic factors for late skeletal recurrence (Medicine (Baltimore) 2016;95:e2909)
      • Patients with bone metastases at site of first relapse have better overall survival than patients with visceral metastases at site of first relapse (Breast Cancer Res Treat 2000;59:271)
      • Conversion from negative to positive ER or PR or remaining receptor negative is significantly worse than patients remaining receptor positive (Cancer 2012;118:4929)
      • ER or PR conversion from positive in the primary breast tumor to negative in distant metastases is a poor prognostic indicator (Cancer 2012;118:4929)
      Case reports
      • 47 year old woman with breast cancer metastases to the gastrointestinal tract, mimicking hyperplastic polyps (Surg Case Rep 2018;4:23)
      • 51 year old woman with menorrhagia, found to have an undiagnosed invasive ductal carcinoma of the breast, metastatic to the endometrium (Am J Case Rep 2018;19:494)
      • 65 year old woman with lobular breast carcinoma, metastatic to anorectum (BMC Res Notes 2018;11:268)
      Microscopic (histologic) description
      • Metastatic tumors often resemble the morphology of the primary tumor
      • Only the stromal component of a malignant phyllodes tumor will metastasize and morphologically resembles a primary sarcoma
      • Receptor status in distant metastases often changes from the original primary tumor receptor status (J Natl Cancer Inst 2018 Jan 5 [Epub ahead of print]):
        • Estrogen receptor (ER):
          • 22.5% positive primary to negative in the metastasis
          • 21.5% negative primary to positive in the metastasis
        • Progesterone receptor (PR):
          • 49.4% positive primary to negative in the metastasis
          • 15.9% negative primary to positive in the metastasis
        • HER2:
          • 21.3% positive primary to negative in the metastasis
          • 9.5% negative primary to positive in the metastasis
      Microscopic (histologic) images

      Contributed by Emily S. Reisenbichler, M.D.

      Poorly differentiated breast carcinoma, metastatic to the liver (H&E and GATA3 immunostain)




      Case #125

      Breast cancer metastases - to endometrial polyp

      Breast cancer metastases - to endometrial polyp
      (left to right): ER, PR, GCDFP-15


      Breast cancer metastases - to cervix

      Positive stains
      Differential diagnosis
      Board review style question #1
      What is the most likely combination of the site of breast carcinoma metastasis, tumor type and timing of metastases from primary diagnosis?

      1. ER+ / PR+ / HER2- metastasis to bone 10 years after primary diagnosis
      2. ER- / PR- / HER2- metastasis to bone 10 years after primary diagnosis
      3. ER+ / PR+ / HER2- metastasis to brain 2 years after primary diagnosis
      4. ER- / PR- / HER2- metastasis to ovary 10 years after primary diagnosis
      Board review style answer #1
      A. ER+ / PR+ / HER2- metastasis to bone 10 years after primary diagnosis. Hormone receptor+ (estrogen receptor+ / progesterone receptor+) tumors are more likely to metastasize after long periods and the most frequent site of first distant metastases is the bone. Triple negative (estrogen receptor- / progesterone receptor- / HER2-) tumors typically metastasize within 5 years of primary diagnosis.

      Comment Here

      Reference: Spread and metastases

      Squamous cell carcinoma
      Definition / general
      • Breast neoplasm with greater than 90% squamous differentiation with variable degrees of keratinization (Eur J Surg Oncol 2003;29:386, Acta Cytol 1989;33:201)
      • The following criteria are also required to establish a firm diagnosis:
        • Tumor is centered in the breast parenchyma, independent of the overlying skin and nipple of adnexal elements (exclusion of primary cutaneous neoplasm)
        • Exclusion of metastasis from an extramammary primary squamous cell carcinoma (SCC) (e.g., lung, cervix, bladder, head and neck)
      Essential features
      • All or at least 90% of tumor must be composed of malignant squamous cells
      • Etiology uncertain; thought to arise as a result of extensive squamous metaplasia in benign conditions (e.g., cysts, chronic inflammatory conditions, abscess, intraductal papillomas)
      • Associated with a lower rate of lymph node metastasis and a significant rate of distant metastasis without lymph node involvement
      • Aggressive and often refractory to treatment
      Terminology
      • Pure metaplastic squamous cell carcinoma of the breast
      • Primary squamous cell carcinoma of the breast
      • Pure squamous cell carcinoma of the breast
      ICD coding
      • ICD-O: 8575/3 - metaplastic carcinoma, NOS
      • ICD-11: 2C6Y & XH0RD4 - other specified metaplastic carcinoma of breast & metaplastic carcinoma, NOS
      Epidemiology
      • First described by Troell in 1908
      • Very rare and constitutes < 0.1% of primary invasive breast cancer (Hum Pathol 1984;15:526)
      • Prevalence is uncertain due to rarity of the disease; a limited number of cases have been reported thus far
      • Usually known to affect postmenopausal females
      Sites
      • Can affect any anatomical area of breast
      Pathophysiology
      • Histogenesis remains unclear and many theories have been postulated
      • Commonly proposed theories are:
      Etiology
      • No single specific etiological factor identified or described; no association with familial cancer syndromes reported
      • Chronic inflammation resulting in squamous metaplasia may represent a predisposing factor (e.g., implant associated squamous cell carcinoma) (Hum Pathol 2017;67:94, Cancer 1994;73:1449)
      Clinical features
      Diagnosis
      • Squamous component can be diagnosed on cytology or biopsy; however, to diagnose pure squamous cell carcinoma, histopathological evaluation of the excised lesion is recommended
      • Primary cutaneous or metastatic squamous cell carcinomas from other sites must be ruled out
      Radiology description
      • No specific mammography findings
      • Solid hypoechogenic masses with complex cystic components (Am Surg 1999;65:1153)
      Prognostic factors
      • Prognostic information is limited; however, reports suggest an aggressive clinical course, with outcomes comparable to poorly differentiated ductal carcinoma of the breast
      • 5 year survival has been reported to be 52% and 63% (Anticancer Res 2007;27:547, Cancer 1990;65:272)
      Case reports
      • 43 year old Moroccan woman presenting with an infected mass in the left breast associated with ipsilateral axillary lymphadenopathy (Pan Afr Med J 2015;20:152)
      • 46 year old woman with unusual case of implant associated SCC of the breast post silicone gel breast implant (Cureus 2018;10:e3405)
      • 54 year old woman with primary pure squamous cell carcinoma of the breast in the form of an intracystic tumor (Cancer Rep (Hoboken) 2021;4:e1391)
      • 57 and 60 year old women with acantholytic variant of squamous cell carcinoma (Cesk Patol 2011;47:184)
      • 61 year old woman with breast abscess as the initial manifestation of primary pure squamous cell carcinoma (Breast Dis 2011;33:125)
      • 68 year old woman with primary squamous cell carcinoma of the breast presenting as a right breast mass (J Surg Case Rep 2019;2019:rjz182)
      • 72 year old woman with primary squamous cell carcinoma of the breast with pathologic complete response after neoadjuvant chemotherapy (Curr Probl Cancer 2019;43:308)
      Treatment
      Gross description
      Gross images

      AFIP images
      Partially cystic tumor

      Partially cystic tumor



      Images hosted on other servers:
      Presenting as a cystic mass

      Presenting as a cystic mass

      Cystic area with necrosis (upper), gray-white solid area (below)

      Cystic area with
      necrosis (upper);
      gray-white solid
      area (below)

      Microscopic (histologic) description
      • Considered pure squamous cell carcinoma when squamous component comprises > 90% of the tumor
      • Usually presents as a cystic lesion with cavity lined by squamous cells with variable nuclear atypia (Int J Surg Case Rep 2011;2:194)
      • Infiltration in surrounding parenchyma as sheets, cords and nests
      • Often elicits a significant stromal reaction with prominent inflammatory infiltrates (Cancer 1990;65:272)
      • Variable keratinization and spindle cell transformation may be seen
      • Acantholytic subtype has pseudovascular or pseudoglandular appearance due to tumor cell discohesiveness, with lining composed of squamous epithelium and spaces with necrotic debris and keratin (Cesk Patol 2011;47:184)
      Microscopic (histologic) images

      Contributed by Chanchal Rana, M.D.
      Cystic lesion with infiltration

      Cystic lesion with infiltration

      Desmoplastic stromal reaction

      Desmoplastic stromal reaction

      Keratin pearls

      Keratin pearls

      High Ki67 proliferation index

      High Ki67 proliferation index

      Cytology description
      Cytology images

      Images hosted on other servers:
      Polygonal tumor cells with abundant eosinophilic cytoplasm and nucleus hyperchromasi Polygonal tumor cells with abundant eosinophilic cytoplasm and nucleus hyperchromasi

      Polygonal tumor cells with abundant eosinophilic cytoplasm and nuclear hyperchromasia

      Positive stains
      Negative stains
      Electron microscopy description
      Molecular / cytogenetics description
      • Displays a basal-like molecular phenotype and usually shows poor prognostic features; lack of BRCA1 expression is expected (Anticancer Res 2007;27:547)
      Sample pathology report
      • Left breast, upper quadrant, core needle biopsy:
        • Squamous cell carcinoma (see comment)
        • Comment: The tumor shows extensive squamous differentiation. The differential diagnosis includes
          • Metaplastic carcinoma of the breast with squamous cell differentiation, primary squamous cell carcinoma breast and metastatic squamous cell carcinoma from a non mammary primary site. Additional clinical and radiologic correlation is recommended.

      • Left breast, upper outer quadrant, wide local excision:
        • Primary squamous cell carcinoma breast
      Differential diagnosis
      Board review style question #1

      A 65 year old woman presented with well defined swelling of the upper inner quadrant of the right breast. Histological details are shown in the figure above. Which of the following statements is true of this entity?

      1. Commonly presents as a solid homogenous mass
      2. Has a low proliferative index
      3. Is never more than 4 cm in greatest dimension
      4. May metastasize without locoregional lymph node involvement
      Board review style answer #1
      D. May metastasize without locoregional lymph node involvement. The microphotograph shows a neoplasm with tumor cells arranged in islands. These cells are polygonal and have pleomorphic hyperchromatic nuclei with a moderate to abundant amount of dense eosinophilic cytoplasm. No other type of invasive component is seen. These features are consistent with primary squamous cell carcinoma of the breast.

      Primary squamous cell carcinoma of the breast is less likely to be associated with lymph node metastasis and may metastasize even without axillary lymph node involvement. It is usually of larger size at the time of presentation and has a high proliferative index.

      Comment Here

      Reference: Squamous cell carcinoma of breast
      Board review style question #2
      Which of the following statements about primary squamous cell carcinoma (SCC) of the breast is true?

      1. Can always be diagnosed on core needle biopsy
      2. Can arise at any anatomical site in the breast
      3. Is more commonly encountered in perimenopausal women
      4. Is very responsive to conventional chemotherapy and radiotherapy
      Board review style answer #2
      B. Can arise at any anatomical site in the breast. Primary squamous cell carcinoma (SCC) of the breast is commonly encountered in postmenopausal women. It does not have any specific site involvement but can arise at any anatomical location in a breast. All or at least > 90% of the tumor cells should be squamous for diagnosis, which requires evaluation of the entirety of the neoplasm; hence, SCC cannot be diagnosed on core needle biopsy alone. It is an aggressive tumor and is often refractory to conventional treatment options.

      Comment Here

      Reference: Squamous cell carcinoma of breast

      Staging
      Definition / general
      • All carcinomas of the breast are covered by this staging system
      • Breast sarcomas, phyllodes tumor and breast lymphomas are not staged using this system
      Essential features
      • AJCC 7th edition staging was sunset on December 31, 2017; as of January 1, 2018, use of the 8th edition is mandatory
      ICD coding
      • ICD-10:
        • C50.0 - nipple
        • C50.1 - central portion of breast
        • C50.2 - upper inner quadrant
        • C50.3 - lower inner quadrant
        • C50.4 - upper outer quadrant
        • C50.5 - lower outer quadrant
        • C50.6 - axillary tail
        • C50.8 - overlapping lesion of breast
        • C50.9 - breast, not otherwise specified
      Primary tumor (pT)
      • pTX: cannot be assessed
      • pT0: no evidence of primary tumor
      • pTis: ductal carcinoma in situ, Padget disease, encapsulated papillary carcinoma and solid papillary carcinoma
        • pTis (DCIS): ductal carcinoma in situ without invasive carcinoma
        • pTis (Paget): Paget disease without invasive carcinoma
      • pT1mi: tumor ≤ 1 mm
      • pT1a: tumor > 1 mm but ≤ 5 mm
      • pT1b: tumor > 5 mm but ≤ 10 mm
      • pT1c: tumor > 10 mm but ≤ 20 mm
      • pT2: tumor > 20 mm but ≤ 50 mm
      • pT3: tumor > 50 mm
      • pT4a: extension to chest wall (not including pectoralis muscle)
      • pT4b: edema (including peau d'orange), ulceration of skin or ipsilateral satellite skin nodules
      • pT4c: both T4a and T4b
      • pT4d: inflammatory carcinoma (involves > 1/3 of the breast skin, primarily a clinical diagnosis)

      Notes:
      • Lobular carcinoma in situ (LCIS) is no longer classified as Tis and is now considered a risk factor, not a malignancy
      • For invasive tumors, do not include in situ tumor in the tumor measurement used to determine pT category
      • Round invasive tumor size to the nearest millimeter, except if between 1.0 and 1.4 mm, then round up to 2.0 mm to avoid classifying as pT1mi
      • Do not add tumor dimensions from the needle biopsy to the excision; use the maximum dimension in either the needle biopsy or excision for pT categorization (invasive tumor is larger in the needle biopsy than subsequent excision in 12% of cases, Am J Surg Pathol 2013;37:739)
      • If multiple excisions, may want to report "at least pT_, a more accurate estimate may be based on imaging studies"
      • If there are multiple simultaneous, macroscopically measurable, ipsilateral invasive tumors that are grossly and histologically not connected, use largest individual size, do not sum sizes; can use (m) suffix, e.g., pT1b(m)
      • Contiguous tumor within the pectoralis muscle should be included in the tumor measurement to determine pT category
      • In a postneoadjuvant specimen, measure the largest single contiguous focus; do not include the fibrous tumor bed without viable tumor
      Regional lymph nodes (pN)
      • pNX: cannot be assessed
      • pN0: no regional lymph node metastasis histologically
      • pN0(i-): no regional lymph node metastasis by histology or immunohistochemistry
      • pN0(i+): isolated tumor cells (cluster ≤ 0.2 mm and < 200 cells)
      • pN0(mol+): RT-PCR positive but negative by light microscopy
      • pN1mi: micrometastasis (tumor deposit > 0.2 mm and ≤ 2.0 mm or ≤ 0.2 mm and > 200 cells)
      • pN1a: metastasis in 1 - 3 axillary lymph nodes with at least 1 tumor deposit > 2.0 mm
      • pN1b: metastasis in internal mammary sentinel lymph node with tumor deposit > 2.0 mm
      • pN1c: pN1a and pN1b
      • pN2a: metastasis in 4 - 9 axillary lymph nodes with at least 1 tumor deposit > 2.0 mm
      • pN2b: metastasis in clinically detected internal mammary nodes with pathologically negative axillary nodes
      • pN3a: metastasis in ≥ 10 axillary lymph nodes with at least 1 tumor deposit > 2.0 mm or metastasis to infraclavicular lymph node
      • pN3b: positive internal mammary node by imaging with pN1a or pN1b
      • pN3c: metastasis in ipsilateral supraclavicular lymph node

      Notes:
      • Lymph nodes, including sentinel lymph nodes, should be bisected along the long axis, not serially sectioned along the short axis; if the bisected halves are thick enough (> 2 mm), they should be further longitudinally sectioned (CAP: Cancer Protocol Templates [Accessed 31 March 2022], Mod Pathol 2010;23 Suppl 2:S26, APMIS 2011;119:868)
      • Regional lymph nodes include axillary, internal mammary, supraclavicular and intramammary
      • Isolated tumor cells = cluster ≤ 0.2 mm and < 200 cells
      • Micrometastasis = deposit > 0.2 mm and ≤ 2.0 mm or ≤ 0.2 mm and > 200 cells
      • Macrometastasis = deposit > 2.0 mm
      • Count cells or measure tumor within a single lymph node cross section
      • Measure only the largest contiguous focus of metastatic tumor cells; adjacent satellites are not included
      • Extranodal extension should be included in the tumor deposit measurement
      • The number of nodes with isolated tumor cells does not change pN category (e.g., 3 nodes with macrometastases plus 1 node with isolated tumor cells is pN1a, not pN2a)
      • Direct extension of tumor into an intramammary lymph node is included as a positive regional lymph node
      • Rounded tumor nodules without nodal tissue present in a nodal drainage area should be considered lymph nodes completely replaced with tumor, unless a vascular wall is present
      • "Clinically detected" is defined as detected by imaging studies (excluding lymphoscintigraphy) or by clinical examination and having characteristics highly suspicious for malignancy or a presumed pathologic macrometastasis based on fine needle aspiration biopsy with cytologic examination
      • In a postneoadjuvant node, measure only the largest contiguous metastatic deposit; do not add separate tumor deposits or include fibrosis without viable tumor
      Distant metastasis (M)
      • pM1: distant metastasis histologically proven > 0.2 mm
      Prefixes
      • y: preoperative radiotherapy or chemotherapy
      • r: recurrent tumor stage
      AJCC prognostic stage groups
      • pTNM, tumor grade, ER, PR and HER2 status are incorporated into prognostic stage groups to refine prognosis
      • Refer to the AJCC 8th edition for the stage group definitions
      Registry data collection variables
      Histologic grade (G)
      • GX: cannot be assessed
      • G1: low grade (score 3 - 5)
      • G2: intermediate grade (score 6 - 7)
      • G3: high grade (score 8 - 9)

      Notes:
      • To assign a histologic grade, assess and combine values for tubule formation (1 - 3), nuclear pleomorphism (1 - 3) and mitotic count (1 - 3) into a score
      Gross images

      Contributed by Debra L. Zynger, M.D.

      Bone metastasis (pM1)

      Microscopic (histologic) images

      Contributed by Sucheta Srivastava, M.D.

      Macrometastasis

      Macrometastasis with extranodal extension


      Micrometastasis

      Isolated tumor cells, pancytokeratin

      Additional references
      Board review style question #1
      What is the pN category for a patient with 1 macrometastasis, 2 micrometastases and 2 nodes with isolated tumor cells?

      1. pNX
      2. pN0(i+)
      3. pN1a
      4. pN2a
      5. pN3a
      Board review style answer #1
      C. pN1a. The number of nodes with isolated tumor cells does not increase the pN category.

      Comment Here

      Reference: Breast - Staging
      Board review style question #2
      Which is the correct pT for a primary breast carcinoma that is 4 mm?

      1. pTis
      2. pTmi
      3. pT1a
      4. pT1b
      5. pT1c
      Board review style answer #2
      C. pT1a. pT1a is defined as a tumor that is > 1 mm but ≤ 5 mm.

      Comment Here

      Reference: Breast - Staging

      Subareolar sclerosing duct hyperplasia
      Definition / general
      • A sclerosing papillary variant of radial sclerosing lesion in the subareolar region, first described in 1987 (Cancer 1987;59:1927)
      • Beneath areola, without involvement of surface of nipple (called nipple adenoma if nipple is involved)
      • Note: this is the only article describing this lesion in the literature
      Clinical features
      • 26 to 73 years, mean 46 years
      • A mass located beneath the nipple or areola in the breast
      • Left and right breast affected equally
      • Erosion or ulceration of the nipple are absent
      • Nipple retraction may occur
      • Several patients had blood discharge
      • Nonspecific mammographic findings, may mimic carcinoma
      Treatment
      • Excision through circumareolar incision, sparing the nipple
      • Recurrence may occur after incomplete excision
      • Benign, no evidence that this condition is premalignant
      Gross description
      • Firm to hard, round to oval tumor with indistinct margins
      • Mean 1.2 cm, range 0.6 to 2.0 cm
      • Yellow streaks may be noted
      Microscopic (histologic) description
      • Prominent central elastosis and sclerosis in the center of the tumor, duct hyperplasia is more prominent in the periphery, causing distortion of the ductal pattern
      • Cartilaginous metaplasia may occur in the sclerotic core
      • Varying amounts of papillary ductal proliferation
      • Papillary epithelial hyperplasia within ducts may exhibit considerable atypia
      • Generally no cystic change, no papilary apocrine change, no squamous metaplasia
      Microscopic (histologic) images

      AFIP images

      Papillary duct proliferation

      Solid foci in ducts

      Papillary duct hyperplasia

      Differential diagnosis

      Supernumerary / accessory tissue
      Definition / general
      • Persistent epidermal thickenings along milk line from axilla to perineum / vulva, due to clusters of primordial breast cells that fail to involute
      • Ectopic breast tissue (nipple, areola and glandular breast tissue) may arise from anywhere along mammary ridge (midaxilla to inguinal area)
      • Reference: eMedicine: Supernumerary Nipple [Accessed 5 May 2023]
      Essential features
      • Supernumerary or accessory breast tissue can arise anywhere along mammary ridge
      • Mostly benign but reported with an increased risk of genitourinary neoplasms (renal adenocarcinoma, urinary bladder carcinoma, testicular and prostate cancers) (Dermatol Online J 2013;19:4)
      • May develop breast carcinomas because it contains breast tissue
      Terminology
      • Polymastia classification system (Eplasty 2012;12:ic5)
        • Class I: consists of a complete breast including glandular tissue, nipple and areola
        • Class II: consists of only glandular tissue and nipple, without areola
        • Class III: consists of only glandular tissue and areola, without nipple
        • Class IV: consists of only glandular tissue
        • Class V (pseudomamma): consists of only nipple and areola without glandular tissue
        • Class VI (polythelia): accessory nipple (no glands or areola)
        • Class VII (polythelia areolaris): accessory areola (no nipple or glands)
        • Class VIII (polythelia pilosa): consists only of hair
        • Polymastia: accessory breast including nipple, areola and glandular breast tissue
      ICD coding
      • ICD-10: Q83.3 - accessory nipple
      • ICD-11:
        • LB62 - supernumerary breasts
        • LB63 - accessory nipple
      Epidemiology
      • ~2 - 6% of females and 1 - 3% of males (Eplasty 2012;12:ic5)
      • Occurrence rates vary widely on the basis of ethnicity and gender, ranging from 0.6% in Caucasians to 5% in Japanese females (Eplasty 2012;12:ic5)
      Sites
      • Found along the thoracoabdominal region of the milk line (the embryologic mammary streak)
      • Accessory nipples: thorax or abdomen (65%) and axilla (20%) are most common; back, shoulder, buttock, face and neck are less common
      • Accessory glandular tissue: axilla is most common (may occur bilaterally)
      • Reference: S D J Med 1996;49:149
      Pathophysiology
      Etiology
      Diagrams / tables

      Images hosted on other servers:

      Extramammary tissue may arise anywhere along mammary ridge

      Clinical features
      • Higher incidence of urinary tract malformations reported in patients with supernumerary nipples; this may be explained by ethnic or geographic variations (Eur J Pediatr 1998;157:821)
      • Polythelia is associated with an increased risk of genitourinary malignancies such as testicular seminoma (Dermatol Online J 2013;19:4)
      • Supernumerary nipple can be considered a cutaneous marker of mitral valve prolapse with chest pain, palpitations, precordial pulsations and cardiomegaly in patients of Asian Indian ethnicity (Am J Cardiol 2000;86:695)
      Diagnosis
      • Usually made by self examination
      • Mostly similar to brown freckle
      Radiology description
      • Radiographically the accessory tissue resembles normal glandular tissue as a high density lesion resembling normal breast parenchyma but is separate from normal breast tissue (Radiol Case Rep 2021;16:2804)
      • The mean radiographic dimension of the accessory tissue is 3.9 cm and on mammography, clearly separated from glandular parenchyma but located in the axilla (Radiology 1987;163:709)
      • Accessory tissue is best visualized on oblique and exaggerated craniocaudal views in mammography (Radiology 1987;163:709)
      • Ultrasound can help to differentiate accessory breast tissue from lipoma, lymphadenopathies, sebaceous cysts and other pathologic entities in the axilla (J Ultrasound Med 2017;36:1469)
      • All the benign and malignant diseases of the breast also occur in accessory breast tissue with the same imaging features as the main breast tissue; the identification of normal breast tissue adjacent to a mass strongly suggests that the lesion arose from accessory breast tissue (J Ultrasound Med 2017;36:1469)
      Radiology images

      Images hosted on other servers:

      Proliferation of glandular tissue, without areola or nipple

      Normal appearance of accessory breast tissue on mammography Normal appearance of accessory breast tissue on mammography

      Normal appearance of accessory breast tissue on mammography

      Accessory breast tissue on ultrasound

      Accessory breast tissue on ultrasound

      Prognostic factors
      • Usually benign
      • If containing breast glands and stroma, breast malignancies can arise in the breast tissue underlying supernumerary nipples
      Case reports
      Treatment
      • Usually no surgery
      • Surgery performed for cosmetic reasons or if a lesion has developed in the breast tissue
      Clinical images

      Images hosted on other servers:

      Supernumerary nipple in a neonate

      Supernumerary nipple in an adolescent boy

      Supernumerary nipple (bilateral) in an adolescent girl

      Ectopic supernumerary nipple on the shoulder

      Nipple on foot


      Large axillary supernumerary breast

      Hyperpigmented papule shape skin elevation

      Cutaneous fibroadenoma in ectopic breast tissue

      Supernumerary nipple on abdomen

      Gross description
      • Well demarcated, hyperpigmented or skin colored papule, with central elevation or opening
      Microscopic (histologic) description
      • Same histologic features as normal nipple, including hyperpigmentation, slight hyperkeratosis, pilosebaceous structure of Montgomery tubercles, smooth muscle, Toker cells and possibly breast lobules and ducts (J Cutan Pathol 2003;30:256)
      Microscopic (histologic) images

      Contributed by Sara Salehiazar, M.D. and Gary Tozbikian, M.D.
      Normal appearing mammary glands Normal appearing mammary glands Normal appearing mammary glands

      Normal appearing mammary glands

      Mammary type glands in the deep dermis Mammary type glands in the deep dermis Mammary type glands in the deep dermis

      Mammary type glands in the deep dermis


      Glands in the deep dermis Glands in the deep dermis Glands in the deep dermis

      Glands in the deep dermis

      Mammary glands in the dermis

      Mammary glands in the dermis

      Positive stains
      Videos

      Extra nipple under the microscope

      Sample pathology report
      • Skin lesion, punch biopsy / excision:
        • Skin with benign mammary glandular tissue in deep dermis suggestive of accessory breast tissue (see comment)
        • Comment: IHC stains of the glands are positive for BRST2, mammaglobin and GATA3, consistent with mammary glands. All controls showed appropriate reactivity.
      Differential diagnosis
      • Lentigo and melanocytic nevi:
        • Show no smooth muscle proliferation and papillomatosis or acanthosis
        • Have melanocytic nests with no pilosebaceous structure of Montgomery tubercles
        • Lack breast glandular tissue
      • Dermatofibroma or fibrous histiocytoma:
        • Show fascicular proliferation of fibroblastic cells with bland spindled to histiocytoid shaped cells
        • Lack breast glandular tissue
        • Positive for factor XIIIa and CD163
      • Becker nevus:
        • Epidermal acanthosis with increased basal cell layer pigmentation due to benign melanocyte hyperplasia
        • Lacks breast glandular tissue
      • Smooth muscle hamartoma:
        • Increased, irregular, dermal smooth muscle bundles that are not associated with hair follicles or sebaceous glands
        • Lacks breast glandular tissue
      Board review style question #1
      Which of the following statements is true regarding supernumerary breast?

      1. Develops during the fifteenth week of gestation along the milk line
      2. Ectopic breast tissue never demonstrates breast cancer
      3. Mammary ridge involutes during the fifth week of gestation
      4. Varies by ethnicity with highest prevalence in Japanese population
      Board review style answer #1
      D. The accessory breast tissue varies by ethnicity with highest prevalence in Japanese population and the lowest in white populations. Answer A is incorrect because the milk line forms during the fifth week of gestation. Answer B is incorrect because ectopic breast tissue can give rise to malignancy. Answer C is incorrect because the mammary ridge involutes during the fifteenth week of gestation.

      Comment Here

      Reference: Supernumerary / accessory tissue
      Board review style question #2

      Patients with the lesions shown in the image above have increased risk of which of the following malignancies?

      1. Dermatologic malignancies
      2. Gastrointestinal malignancies
      3. Genitourinary malignancies
      4. Gynecologic malignancies
      5. Sarcomas of the bone
      Board review style answer #2
      C. Genitourinary malignancies. Supernumerary nipples have been reported to be associated with increased incidence of genitourinary malignancies and pediatric hematologic disorders (Pediatr Hematol Oncol 2004;21:461). Answers A, B, D and E are incorrect because these conditions can occur in the supernumerary breast tissue, just as in the general population. This means that supernumerary breast tissue does not increase the risk of malignancies other than genitourinary malignancies.

      Comment Here

      Reference: Supernumerary / accessory tissue

      Syringomatous tumor
      Definition / general
      Terminology
      • Also called syringomatous adenoma of nipple or infiltrating syringomatous adenoma of nipple
      Epidemiology
      • Rare
      • Age at diagnosis ranges from 11 to 67 years (average 40 years)
      • Mostly females; at least 2 cases reported in males
      Etiology
      • May originate from eccrine sweat ducts of the nipple
      Clinical features
      • Presents as a mass in the nipple or subareolar region
      • May be associated with pain, skin changes (crusting or ulceration), nipple discharge or nipple retraction
      Case reports
      Treatment
      • Excision with adequate margins (may require nipple areolar resection)
      • May recur if incompletely excised
      Clinical images

      Images hosted on other servers:

      Firm subareolar mass, 17 mm in diameter

      Gross description
      • Firm, discrete mass, 1 - 3 cm, present in the nipple or subareolar region
      Gross images

      Contributed by Dr. Mark R. Wick:

      Syringomatous tumor

      Microscopic (histologic) description
      • Situated in the dermis of the nipple areolar complex
      • May focally involve the subcutaneous tissue; does not involve the epidermis
      • Infiltrative haphazard proliferation of elongated ductules and tubules lined by a two layer epithelium (luminal and basal)
      • Ducts have teardrop, comma or branching shapes
      • Identical to cutaneous syringoma
      • Tumor cells are bland with absent or rare mitoses
      • Background of hyalinized or myxoid fibrous stroma
      • Infiltrates around smooth muscle bundle and lactiferous ducts of nipple
      • Perineural invasion may be present
      • Variable squamous metaplasia with small keratinizing cysts with calcifications
      Microscopic (histologic) images

      Contributed by Emily S. Reisenbichler, M.D.

      Syringomatous tumor of nipple


       AFIP images

      Keratinization in syringomatous duct


       Contributed by Dr. Mark R. Wick

      Syringomatous tumor

      Perineurial invasion



      Images hosted on other servers:

      Acanthotic epidermis

      Small, solitary, evenly spaced duct-like structures

      Layers of cuboidal or squamoid epithelial cells

      Small, dark tumor cells

      Keratotic cysts

      Positive stains
      Differential diagnosis
      Board review style question #1
      A 35 year old woman presents with a nipple mass. An excisional biopsy is performed and histomorphology is consistent with syringomatous tumor. Of the following, which histologic description best describes what was seen on the histologic slide of the nipple mass?

      1. Angulated tubules lined by a single layer of epithelial cells
      2. Bland tumor cells in a background of mucin
      3. Comma or teardrop shaped ducts infiltrating stroma
      4. Ducts with squamous metaplasia communicating with a fistula track to the skin surface
      5. High grade tumor cells with perineural invasion
      Board review style answer #1
      C. Comma or teardrop shaped ducts infiltrating stroma. Syringomatous tumor (SyT) of the nipple is identical to syringoma of the skin. These tumors are characteristically described as having comma or teardrop shaped ducts. Although perineural invasion can be seen with SyT, the tumor cells are bland. Option A is a description of tubular carcinoma. Tubules of SyT have more than 1 epithelial layer. Option B is a description of mucinous carcinoma. SyT has a background stroma that can be either myxoid or sclerotic. Option D is a description of squamous metaplasia of lactiferous ducts (SMOLD). Although SyT can have keratin cysts, fistula tracks out to the skin surface is not a feature.

      Comment here

      Reference: Syringomatous tumor of nipple

      Tall cell carcinoma with reverse polarity
      Definition / general
      • Rare subtype of papillary carcinoma of breast that histologically resembles tall cell variant of papillary thyroid carcinoma and frequently shows distinct IDH2 R172 hotspot mutations
      Essential features
      • Histologic subtype of papillary carcinoma of breast, first described in 2003 (Am J Surg Pathol 2003;27:1114)
      • Solid nodules of columnar epithelial cells, many with thin fibrovascular cores, leading to solid papillary architecture
      • Epithelial cells contain abundant glassy eosinophilic cytoplasm and show abnormal apically located nucleus giving the impression of reverse nuclear polarity
      • Frequent IDH2 R172 hotspot mutations
      • Triple negative phenotype with presumed low malignant potential
      Terminology
      • Not recommended per WHO:
        • Solid papillary carcinoma with reverse polarity (SPCRP)
        • Breast tumor resembling tall cell variant of papillary thyroid carcinoma
        • Solid papillary breast carcinoma resembling tall cell variant of papillary thyroid neoplasm
      ICD coding
      • ICD-O: 8509/3 - tall cell carcinoma with reversed polarity
      • ICD-11: 2C6Y - other specified malignant neoplasms of breast
      Epidemiology
      Sites
      • Breast
      Pathophysiology
      Clinical features
      • Typically presents as a mammographic or palpable breast mass
      Diagnosis
      • Tissue biopsy or surgical excision specimen showing the key histologic and immunohistochemical features
      • Demonstration of IDH2 or TET2 mutation
      • Exclusion of metastatic disease (i.e., thyroid carcinoma)
      Radiology description
      Prognostic factors
      Case reports
      Treatment
      • Surgical excision is mainstay treatment (Am J Surg Pathol 2017;41:887)
      • Lack of evidence for sentinel lymph node procedure, radiation or systemic therapy
      Gross description
      Microscopic (histologic) description
      • Solid circumscribed nodules of epithelial cells with thin, petite papillae in a dense fibrous stroma (Am J Surg Pathol 2017;41:887)
      • Cuboidal, columnar or tall columnar tumor cells with abundant eosinophilic cytoplasm; bland nuclear features with nuclei located at the apical pole rather than the basal pole, giving the impression of reverse nuclear polarity
      • Variable presence of nuclear grooves with intranuclear pseudoinclusions
      • True papillae and cystic spaces with amphophilic colloid-like secretions may be seen
      • Foamy histiocyte aggregates often present within fibrovascular cores
      • Mitotic figures rare
      Microscopic (histologic) images

      Contributed by Koorosh Haghayeghi M.D., Sonja Chen M.D. and Yihong Wang M.D.

      Solid epithelial nodules

      Solid papillae

      Papillae with histiocytes

      Reverse polarity

      Papillae with reverse polarity

      Petite papillae


      CK5/6

      p63

      SMMHC

      E-cadherin

      MUC1

      ER

      Cytology description
      Positive stains
      Negative stains
      Molecular / cytogenetics description
      Sample pathology report
      • Right breast, 9 o'clock, excisional biopsy:
        • Tall cell carcinoma with reverse polarity (see comment)
        • Comment: Sections show circumscribed nests of a solid, papillary and partially cystic neoplasm comprised of low to intermediate nuclear grade columnar epithelial cells with abundant cytoplasm, irregular nuclear contours and scattered nuclear grooves. The columnar cells appear to show apical localization, so called reverse polarization. Myoepithelial immunostains are negative. The lesional cells are strongly and diffusely immunoreactive for cytokeratin 5 and negative for ER, PR and HER2. Concurrent molecular analysis revealed IDH2 and PIK3CA mutations. Overall, the morphologic, immunohistochemical and molecular findings are compatible with a tall cell carcinoma with reverse polarity.
      Differential diagnosis
      Board review style question #1

      A 59 year old woman had a 1.0 cm spiculated breast mass, represented above. Tumor cells express GCDFP-15 and mammaglobin. The tumor cells are negative for ER, PR and HER2. Myoepithelial markers (p63 and SMMHC) are also negative. Other negative markers include TTF1 and thyroglobulin. What is the most common molecular alteration seen in these tumors?

      1. BRAF mutation
      2. CDH1 mutation
      3. IDH2 R172 hotspot mutations
      4. RET mutation
      5. TET2 truncating mutation
      Board review style answer #1
      C. IDH2 R172 hotspot mutations

      Comment here

      Reference: Tall cell carcinoma with reverse polarity
      Board review style question #2
      Which of the following IHC panels would differentiate solid papillary carcinoma from recently described tall cell carcinoma with reverse polarity?

      1. CK5/6 and ER
      2. CK7, CK18 and KIT
      3. E-cadherin and p120
      4. ER, PR and HER2
      5. GATA3 and TTF1
      Board review style answer #2
      A. CK5/6 and ER. Tall cell carcinoma with reverse polarity is usually ER negative and may show variable loss of keratin 5/6. In contrast, solid papillary carcinoma shows negative CK5/6 and strong ER staining. Both tumors show loss of myoepithelial cells by p63 and smooth muscle myosin heavy chain. ER, PR and HER2 (biomarkers), E-cadherin and p120 (ductal versus lobular), GATA3 and TTF1 (primary breast versus metastasis), and CK4, CK14, CK18 and KIT (adenoid cystic carcinoma) do not help in distinguishing solid papillary carcinoma and tall cell carcinoma with reverse polarity.

      Comment here

      Reference: Tall cell carcinoma with reverse polarity

      Transgender breast pathology
      Definition / general
      • Transgender individuals have a gender identity that differs from their assigned sex; the term includes gender nonbinary individuals who do not identify as exclusively male or female (Clin Breast Cancer 2019;19:e12)
      • This article is focused on breast pathology in transmasculine (TM) people, transgender men and gender nonbinary individuals who were assigned female at birth (AFAB) but their gender identity / expression falls in the masculine range (i.e., female to male [FtM])
      • TM people frequently pursue gender affirming testosterone therapy to induce masculinization, with or without subsequent chest contouring surgery or other gender affirming surgery (Clin Breast Cancer 2019;19:e12)
      Essential features
      • Gender affirming surgery in the TM subgroup of transgender individuals includes top surgery (i.e., chest contouring surgery, mastectomy, reduction mammoplasty)
      • Testosterone therapy is often taken before and after gender affirming surgeries and has an impact on breast histopathology, particularly when taken for at least 12 months (Mod Pathol 2021;34:85)
      • Most commonly observed histologic alteration seen in patients taking testosterone is lobular atrophy
      • As top surgery does not necessarily remove all identifiable breast tissue, long term follow up studies are needed to better understand the risk of breast cancer in this population
      Terminology
      • Gender dysphoria, gender incongruence, gender nonconforming individual (GNCI), transsexual, assigned female at birth (AFAB), female to male (FtM)
      • Gender affirming surgery, gender reassignment surgery, sex reassignment surgery, top surgery
      ICD coding
      • ICD-10
        • F64.0 - gender dysphoria in adolescents and adults and gender identity disorder in adolescence and adulthood
        • F64.9 - gender identity disorder, unspecified
        • Z87.890 - personal history of sex reassignment
      Epidemiology
      • Estimated 1.4 million (0.6%) adults in the United States identify as transgender or gender nonconforming (Plast Reconstr Surg Glob Open 2018;6:e1738)
      • Increasing social acceptance, greater health insurance policy coverage of gender affirming health care services and the implementation of the Affordance Care Act Section 1557 in 2016 has resulted in an increasing number of TM people pursuing gender affirming hormone therapy and gender affirming surgeries (Fed Regist 2016;81:31375, Transgend Health 2019;4:326, Transgend Health 2019;4:131)
      • Gender affirming surgeries in the TM subgroup of transgender individuals may include top surgery (i.e., chest contouring surgery, mastectomy, reduction mammoplasty), as well as facial masculinization and genital surgery (Plast Reconstr Surg 2019;143:857e)
      • In the United States, TM individuals tend to undergo chest contouring surgery to affirm their masculine identity at a relatively young age (< 30 years old) (Mod Pathol 2021;34:85)
      Sites
      • Breast
      Pathophysiology
      Etiology
      • Gender affirming testosterone therapy use
      Clinical features
      Diagnosis
      • Diagnosed with gender dysphoria by a mental health professional
      Laboratory
      • TM people receiving gender affirming testosterone therapy have circulating testosterone levels comparable to cisgender men of 300 - 1,000 ng/dL (Curr Opin Endocrinol Diabetes Obes 2013;20:553)
      • TM people receiving gender affirming testosterone therapy have circulating estradiol levels of < 50 pg/mL, lower than cisgender premenopausal women (30 - 400 pg/mL)
      Radiology description
      • Screening recommendations for TM people are evolving and may include the following (AJR Am J Roentgenol 2014;202:1157)
        • Breast examination and screening mammography as for cisgender women for TM people who have undergone no chest surgery or reduction mammoplasty only
        • Annual chest wall and axillary examination for TM people following bilateral mastectomy
        • Preoperative mammography for TM people who meet standard screening criteria for cisgender women
      • Breast cancer may develop in residual breast tissue following top surgery and is most frequently identified as a palpable mass with similar radiologic appearances to tumors arising in cisgender men and women (AJR Am J Roentgenol 2014;202:1149)
      Case reports
      Clinical images

      Images hosted on other servers:
      Pre and postoperative photographs Pre and postoperative photographs Pre and postoperative photographs

      Pre and postoperative photographs

      Gross description
      • Breast parenchyma is typically without macroscopic abnormality
      • If macroscopically within normal limits, representative sections of fibrous breast tissue, skin and nipple areolar complex (if present) may be sampled for microscopic evaluation
      • As guidelines for the gross examination and subsequent tissue submission have not been standardized, one's institutional protocol for reduction mammoplasty specimens may be followed with additional tissue evaluated if atypical proliferations are identified on the initially submitted tissue (Mod Pathol 2021;34:85, Arch Pathol Lab Med 2020;144:888)
      Gross images

      Contributed by Kelsey McGinley PA (ASCP) and Gabrielle M. Baker, M.D.
      Mastectomy specimen; gender dysphoria, trans man

      Mastectomy
      specimen

      Microscopic (histologic) description
      • Full spectrum of benign and malignant histologic alterations seen in cisgender breast tissue may be seen in the transgender setting
      • Same diagnostic criteria are applied in breast tissue from cisgender and transgender patients
      • Most common histologic alteration observed in breast specimens from TM people taking testosterone is a variable degree of lobular atrophy (Mod Pathol 2021;34:85)
      • Gynecomastoid change is also commonly seen in breast tissue from TM people taking testosterone (Histopathology 2017;71:859, Mod Pathol 2021;34:85)
      • Singly dispersed Toker cells and Toker cell hyperplasia (clusters of 3 or more Toker cells or Toker cells in acinar formation) is common in the nipple areolar complex of TM people who take testosterone therapy (Mod Pathol 2023;36:100121)
      Microscopic (histologic) images

      Contributed by Yaileen Guzman, M.D. and Gabrielle M. Baker, M.D.
      DCIS; gender dysphoria, trans man

      Ductal carcinoma in situ

      29 year old transgender man with ADH

      Atypical ductal hyperplasia

      Toker cells

      Toker cells

      Toker cell hyperplasia

      Toker cell hyperplasia

      Immunofluorescence images

      Contributed by Gabrielle M. Baker, M.D. and Yu Jing Jan Heng, Ph.D.
      Toker cell hyperplasia Toker cell hyperplasia

      Toker cell hyperplasia

      Positive stains
      • Toker cell CK7 (> 90%)
      • Toker cell AR (> 90%)
      • Toker cell ER (very variable, 0 - 90%)
      • Note that Toker cells with or without intraepidermal gland formation may demonstrate some degree of HER2 expression in order to avoid making an erroneous diagnosis of Paget disease
      • Reference: Mod Pathol 2023;36:100121
      Negative stains
      Sample pathology report
      • Right breast, mastectomy:
        • Breast tissue, portion of nipple areolar complex and skin with no significant pathologic change (see comment)
        • Comment: The lobules appear atrophic, consistent with the patient's reported use of testosterone therapy.
      Differential diagnosis
      • Gynecomastia (Endocrine 2017;55:37, Am J Clin Pathol 1972;57:431):
        • May affect 1 or both breasts
        • Increase in the number of ducts, with or without associated dilation
        • Lobule formation may occur
        • Pseudoangiomatous stromal hyperplasia (PASH) is common and may be conspicuous
        • May be divided into early and late stages
          • Early stage
            • Cellular, loose, vascular periductal stroma
            • Epithelial hyperplasia may be florid and commonly exhibits tapered tips
          • Late stage
            • Periductal fibrosis
            • Stroma may be hyalinized
            • Epithelial atrophy
      • Normal physiologic alterations in postmenopausal women (Maturitas 2004;49:2):
        • Variable degrees of lobular atrophy
        • Relative increase in adipose tissue with relative decrease in collagenous stroma and glandular components
      • Paget disease:
        • Single or cluster of cells with pale cytoplasm, large irregular nuclei with prominent nucleoli present throughout the epidermis
        • May display gland formation
        • Underlying carcinoma is commonly of high grade invasive carcinoma or DCIS
      Board review style question #1
      Which of the following is the most commonly observed histologic alteration in breast specimens from transmasculine individuals taking testosterone?

      1. Collagenous spherulosis
      2. Intraductal papillomas
      3. Invasive ductal carcinoma
      4. Lobular atrophy
      Board review style answer #1
      D. Lobular atrophy. The most common histologic alteration observed in breast specimens from transmasculine people taking testosterone is a variable degree of lobular atrophy. Longer duration of testosterone therapy is associated with higher degrees of lobular atrophy, independent of age at surgery, ethnicity, BMI and presurgical oophorectomy. Answers A - C are incorrect because although collagenous spherulosis, papilloma and invasive carcinoma can occur with testosterone therapy, they are not most common.

      Comment Here

      Reference: Transgender breast pathology
      Board review style question #2
      Which of the following is true regarding the risk of breast cancer in transmasculine individuals who are taking testosterone and have undergone top surgery?

      1. Long term follow up studies are needed to better understand the risk of breast cancer in this population
      2. These individuals are known to have a higher risk of developing breast cancer than cisgender females
      3. These individuals are known to have a lower risk of developing breast cancer than cisgender males
      4. These individuals have no risk of developing subsequent breast cancer
      Board review style answer #2
      A. Long term follow up studies are needed to better understand the risk of breast cancer in this population. There is a need for long term follow up as well as culturally sensitive and appropriate screening algorithms to determine risk.  Answers B - D are incorrect because data remains limited regarding the influence of exogenous testosterone on hormone sensitive tissues, such as the breast and the risk of subsequent malignancy; therefore, risk under these conditions has yet to be definitively determined.

      Comment Here

      Reference: Transgender breast pathology
      Board review style question #3

      What is the most likely finding depicted in this image taken from the nipple areolar complex of a transgender man on long term testosterone therapy, now undergoing chest contouring surgery?

      1. Invasive ductal carcinoma
      2. Nipple adenoma
      3. Paget disease
      4. Toker cell hyperplasia with gland formation
      Board review style answer #3
      D. Toker cell hyperplasia with gland formation. The image demonstrates intraepidermal gland formation composed of cytologically bland epithelial cells predominantly located at the base of the epidermis, findings that support the diagnosis of Toker cell hyperplasia with gland formation. Answer C is incorrect because Paget disease is defined as a proliferation of malignant glandular epithelial cells within the epidermis of the nipple areolar complex; the neoplastic cells of Paget disease are typically of high nuclear grade. Although Paget disease is most often characterized by the presence of singly dispersed cells or clusters of malignant epithelial cells with variable upward (i.e., pagetoid) scatter throughout the epidermis, gland formation may occur. Answer A is incorrect because the intraepidermal gland formation depicted here is cytologically benign; additionally, while invasive ductal carcinoma involving the breast parenchyma proper may secondarily involve the dermis and epidermis, there is no evidence of an underlying carcinoma in the provided image. Answer B is incorrect because in contrast to the image shown here, a nipple adenoma is generally characterized by a well demarcated but variably complex proliferation of epithelial elements in the dermis of the nipple areolar complex; of note, in some cases the glandular proliferation may involve the overlying epidermis and Toker cell hyperplasia has been reported to arise in association with an underlying nipple adenoma.

      Comment Here

      Reference: Transgender breast pathology

      Trastuzumab
      Definition / general
      • Humanized IgG1 kappa monoclonal antibody targets HER2 / neu receptor (human epidermal receptor 2)
      • Discovered by scientists including Dr. Axel Ullrich and Dr. H. Michael Shepard at Genentech, Inc., South San Francisco, CA
      • Synonym: anti-HER2 (N Engl J Med 2007;357:39)
      Trade name
      • Herceptin®
      Clinical information
      • Approved by U.S. Food and Drug Administration on September 25, 1998 (Oncology (Williston Park) 1998;12:1727)
      • Biosimilars (Ther Adv Med Oncol 2019;11:1758835919887044):
        • Trastuzumab-dkst (Ogivri): FDA approved, 2017
        • Trastuzumab-pkrb (Herzuma): FDA approved, 2018
        • Trastuzumab-dttb (Ontruzant): FDA approved, 2019
        • Trastuzumab-qyyp (Trazimera): FDA approved, 2019
        • Trastuzumab-anns (Kanjinti): FDA approved, 2019
      • Indications and usage (National Cancer Institute: Trastuzumab [Accessed 9 February 2022]):
        • Breast cancer with HER2 positive and hormone receptor negative: combination therapy or alone after combination chemotherapy including anthracycline
        • Metastatic breast cancer: combined with paclitaxel as first line treatment or alone in patients with previous chemotherapy
        • Gastric adenocarcinoma or gastroesophageal junctional adenocarcinoma with HER2 positive and metastasis: combination therapy with cisplatin and capecitabine or fluorouracil
      • Clinical pharmacokinetics (Oncologist 2011;16:800):
        • Route of administration: intravenous
        • Elimination: predominantly in epithelial cells; renal elimination very low
        • Half life: 28 days
      • Side effects (Ann Pharmacother 2019 Oct 9 [Epub ahead of print]):
        • Infusion related reactions, if occurring < 24 hours after first infusion: fever, chills, rash, headache, dizziness; (severe) angioedema, respiratory distress syndrome, severe hypotension, anaphylaxis (Oncologist 2007;12:601)
        • Cardiotoxicity: hypotension, congestive heart failure, ventricular dysfunction
        • Respiratory: dyspnea, bronchospasm, hypoxia, asthma
        • Gastrointestinal: diarrhea, vomiting, dyspepsia
        • Hematologic: anemia, leukopenia, thrombocytopenia
      • Mechanism of drug resistance (Front Oncol 2012;2:62):
        • Loss of phosphatase and tensin homologue (pTEN)
        • Activation of the phosphoinositide 3 kinase pathway
        • Overexpression of other surface receptors (insulin-like growth factor)
      • Trastuzumab costs about U.S. $70,000 for a full course of treatment, U.S. $1,800 - $1,955 per 440 mg vial (Wikipedia: Trastuzumab [Accessed 9 February 2022])
      Pathophysiology
      • HER2, a member of HER family, does not have a receptor specific ligand binding site
      • HER2 / neu signaling in cancer cells:
        • HER2 heterodimerizes with HER1, HER3 or HER4, phosphorylates and activates intracellular tyrosine kinase domain
        • Activated tyrosine kinase on HER2 activates PI3K-Akt pathway and induces cellular survival
        • Activated tyrosine kinase activates SOS, induces a cascade of activation of RAS-RAF-MAPK-MEK and MAPK, eventually promotes cellular proliferation
        • Cleavage of HER2 extracellular domain produces phosphorylated P95 that could activate downstream signal transduction
      • Mechanism of action:
        • Upon binding the extracellular domain of HER2, trastuzumab reduces cleavage of HER2 receptor, blocks the activation ability of P95 residue and eventually decreases signaling
        • Binding of trastuzumab to extracellular domain of HER2 inhibits homo or heterodimerization of HER2 to HER1, HER2, HER3 or HER4 and reduces signaling
        • Trastuzumab binds to immune effector cells and activates antibody dependent cell mediated cytotoxicity, eventually leads to tumor cell lysis
        • Endocytosis of HER2 is increased and leads to intracellular HER2 degradation
      • Reference: N Engl J Med 2007;357:39, see Diagrams / tables, figure 3
      Diagrams / tables

      Images hosted on other servers:

      HER2 ISH in invasive component of breast cancer

      HER2 IHC
      testing in gastric &
      gastroesophageal
      adenocarcinoma

      Mechanism of action

      Uses by pathologists - general
      • Identify HER2 positive tumors to identify candidates for treatment
        • Primary breast carcinomas
        • Gastric and gastroesophageal adenocarcinomas
        • Metastatic diseases (test performed in a metastatic site)
      Uses by pathologists - breast carcinoma
      • HER2 immunohistochemistry in breast carcinoma:
        • ASCO-CAP HER2 Test 2013 and 2018 Guideline Recommendation (Arch Pathol Lab Med 2014;138:241, Arch Pathol Lab Med 2018;142:1364)
          • Negative (score 0):
            • No staining
            • Incomplete faint membrane staining in ≤ 10% invasive tumor cells
          • Negative (score 1+):
            • Incomplete faint membrane staining in > 10% invasive tumor cells
          • Equivocal (score 2+): perform HER2 ISH
            • Incomplete, weak / moderate membrane staining in > 10% invasive tumor cells
            • Complete, intense membrane staining in ≤ 10% invasive tumor cells
          • Positive (score 3+):
            • Complete, intense circumferential membrane staining in > 10% invasive tumor cells
      • HER2 in situ hybridization (ISH) (see Diagrams / tables, figure 1):
        • The Panel recommends that concomitant IHC review should become part of the interpretation of single probe ISH results and the Panel preferentially recommends the use of dual probe instead of single probe ISH assays
        • Positive:
          • Single probe average HER2 copy number ≥ 6.0 signals/cell
          • Dual probe HER2/CEP17 ratio ≥ 2.0 and an average HER2 copy number ≥ 4.0
        • Additional workup required:
          • If a case has a HER2/CEP17 ratio ≥ 2.0 but the average HER2 signals/cell is < 4.0, a definitive diagnosis will be rendered based on additional workup
          • If a case has an average of ≥ 6.0 HER2 signals/cell with a HER2/CEP17 ratio of < 2.0, formerly diagnosed as ISH positive for HER2, a definitive diagnosis will be rendered based on additional workup
          • If the case has an average HER2 signals/tumor cell of ≥ 4.0 and < 6.0 HER2 signals/cell and HER2/CEP17 ratio is < 2.0, formerly diagnosed as ISH equivocal for HER2, a definitive diagnosis will be rendered based on additional workup
        • Additional workup steps:
          • IHC testing for HER2 should be performed using sections from the same tissue sample used for ISH
            1. If the IHC result is 3+, diagnosis is HER2 positive
            2. If the IHC result is 2+, recount ISH by having an additional observer, blinded to previous ISH results, count at least 20 cells that include the area of invasion with IHC 2+ staining:
              • If reviewing the count by the additional observer changes the result into another ISH category, the result should be adjudicated per internal procedures to define the final category
              • If the count remains an average of < 4.0 HER2 signals/cell and HER2/CEP17 ratio is ≥ 2.0, the diagnosis is HER2 negative with a comment
              • If the HER2/CEP17 ratio remains < 2.0 with ≥ 6.0 HER2 signals/cell, the diagnosis is HER2 positive
              • If the count remains an average of ≥ 4.0 and < 6.0 HER2 signals/cell with HER2/CEP17 ratio < 2.0, the diagnosis is HER2 negative with a comment
            3. If the IHC result is 0/1+, diagnosis is HER2 negative with comment
      Uses by pathologists - gastric and gastroesophageal adenocarcinomas
      • HER2 immunohistochemistry testing in gastric and gastroesophageal adenocarcinoma (see Diagrams / tables, figure 2):
        • 2017 CAP / ASCP / ASCO guidelines (J Clin Oncol 2017;35:446, Virchows Arch 2010;457:299):
          • Representative surgical samples or at least 6 to 8 biopsy samples
          • Score 0: negative
            • No membranous staining or staining in < 10% of tumor cells (surgical specimen) or < 5 cohesive tumor cells (biopsy)
          • Score 1+: negative
            • Weak staining in only one part of the membrane in ≥ 10% of tumor cells (surgical specimen) or at least 5 cohesive tumor cells (biopsy)
          • Score 2+: equivocal, perform HER2 ISH
            • Moderate / weak complete or basolateral membranous staining in ≥ 10% of tumor cells (surgical specimen) or at least 5 cohesive tumor cells (biopsy)
          • Score 3+: positive
            • Strong or complete or basolateral membranous staining in ≥ 10% of tumor cells (surgical specimen) or at least 5 cohesive tumor cells (biopsy)
      • HER2 ISH in gastric and gastroesophageal adenocarcinoma (Mod Pathol 2012;25:637):
        • Positive: HER2/CEP17 ratio ≥ 2.0
        • Positive: HER2 copy number > 6.0 (using single probe)
        • HER2 copy number 4 to 6: use dual probe testing and recount 20 cells
      Board review style question #1
      Which of the following drugs target HER2 / neu on cancer cells?

      1. Dabrafenib
      2. Larotrectinib
      3. Trametinib
      4. Trastuzumab
      Board review style answer #1
      D. Trastuzumab

      Comment Here

      Reference: Trastuzumab
      Board review style question #2
      Which of the following results is interpreted as positive staining (score 3+) for HER2 in gastric and gastroesophageal adenocarcinoma?

      1. Intense nuclear and cytoplasmic staining > 10% of tumor cells
      2. Intense luminal membranous staining > 10% of tumor cells
      3. Intense basolateral membranous staining > 10% of tumor cells
      4. Intense luminal and lateral membranous staining > 10% of tumor cells
      Board review style answer #2
      C. Intense basolateral membranous staining > 10% of tumor cells

      Comment Here

      Reference: Trastuzumab

      Triple negative breast cancer
      Definition / general
      • Breast cancers with absence of estrogen receptor (ER), progesterone receptor (PR) and HER2 immunohistochemistry (IHC) expression
      Essential features
      • Defined by absence of ER, PR and HER2 expression by IHC
      • Overlaps with basal-like breast cancers, which are defined by gene expression profiling
      • Generally associated with high stage on presentation, increased risk of recurrence and shorter survival
      • Highly variable histology, including invasive ductal, metaplastic and medullary
      • A small subset is low grade, comprising mostly rare salivary gland type tumors and variants of metaplastic carcinomas (fibromatosis-like metaplastic carcinoma, low grade adenosquamous carcinoma); this subset is associated with good prognosis concordant to low grade carcinomas
      Terminology
      • Considerable overlap with basal-like breast cancers but not interchangeable
        • Triple negative breast cancers are defined by absence of ER, PR and HER2 expression by IHC
        • Basal-like breast cancers are defined by gene expression profiling
        • ~80% of triple negative breast cancers belong to basal-like breast cancers (Adv Anat Pathol 2020;27:27)
        • Triple negative staining has been advocated as IHC surrogate for molecular basal-like breast cancer in clinical practice
      ICD coding
      • No specific ICD code for triple negative breast cancer
      Epidemiology
      Sites
      • Breast parenchyma
      Etiology
      • Reproductive / hormonal factors may play a role
      • BRCA1/2 (Breast Dis 2010;32:25)
        • Majority (71%) of breast cancers in BRCA1 carriers
        • 25% of breast cancers in BRCA2 carriers
        • Triple negative breast cancer represents 17% of breast cancers in non-BRCA1/2 carriers
      • Precursor lesion
        • Microglandular adenosis (MGA) or atypical MGA has been postulated to be a potential precursor lesion for some triple negative breast cancers
        • Evidence that some atypical MGA shared similar genomic changes with the associated triple negative breast cancer (more commonly acinic cell carcinoma) (J Pathol 2016;238:677)
      Clinical features
      Diagnosis
      • Microscopic examination of resected tissue that is then tested for ER, PR and HER2
      Radiology description
      • Mammography (Acta Radiol 2013;54:889)
        • Hyperdense, oval or lobular masses
        • Indistinct or circumscribed margins
      • Ultrasound (Ann Oncol 2012;23:vi23)
        • Distinct masses with well circumscribed margins
        • Posterior acoustic enhancement occasionally identified
      • Magnetic resonance imaging (Breast J 2013;19:643)
        • Rim and mass enhancement
        • Areas of high intratumoral T2 signal intensity
        • Highest sensitivity among all modalities (Ann Oncol 2012;23:vi23)
      Radiology images

      Images hosted on other servers:

      Enhancement on MRI

      Prognostic factors
      Case reports
      Treatment
      • Complete surgical excision remains mainstay of curative treatment
      • Neoadjuvant treatment is given for locally advanced disease but also recommended for early (up to pT1cN0M0) disease in triple negative breast cancer (Ann Oncol 2015;26:v8, Eur J Cancer 2020;135:66)
      • Metastatic / advanced / recurrent disease
      Gross description
      • Similar gross features to nontriple negative breast cancer
      Microscopic (histologic) images

      Contributed by Joshua J.X. Li, M.B.Ch.B. and Gary M. Tse, M.B.B.S.

      High grade ductal carcinoma

      Low grade adenosquamous carcinoma

      Squamous cell carcinoma

      Carcinoma with medullary features


      Triple negative immunophenotype

      Virtual slides

      Images hosted on other servers:

      Fibromatosis-like metaplastic carcinoma

      Secretory carcinoma

      Medullary carcinoma

      Adenosquamous carcinoma

      Squamous cell carcinoma

      Cytology description
      • Features more likely seen in triple negative breast cancers (Cancer Cytopathol 2012;120:401)
        • Necrotic background, lymphocytes
        • Syncytial clusters
        • Ill defined cell borders
      Cytology images

      Contributed by Joshua J.X. Li, M.B.Ch.B. and Gary M. Tse, M.B.B.S.

      Abundant lymphocytes

      Ill defined cell borders

      Positive stains
      Negative stains
      Molecular / cytogenetics description
      Sample pathology report
      • Left breast, 3 o’clock, needle core biopsy:
        • Invasive ductal carcinoma, grade 3 (glandular differentiation 3, nuclear pleomorphism 3, mitotic rate 3)
        • ER negative (0%), PR negative (0%), HER2 negative (1+) for expression
      Differential diagnosis
      Board review style question #1

      Which of the following is a poor prognostic factor in triple negative breast cancers?

      1. Complete pathologic response to neoadjuvant chemotherapy
      2. Negative nodal status
      3. Presence of rich tumor infiltrating lymphocytes
      4. Presence of central necrosis
      5. Small tumor size
      Board review style answer #1
      D. Presence of central necrosis. Tumors rich in tumor infiltrating lymphocytes are associated with better survival. Complete pathologic response to neoadjuvant chemotherapy and lower tumor / node stage are all favorable prognostic factors. Centrally necrotizing breast carcinomas have poor prognosis (BMC Cancer 2015;15:282).

      Comment Here

      Reference: Triple negative breast cancer
      Board review style question #2
      Which of the following types of invasive breast carcinoma most likely displays a triple negative immunophenotype?

      1. Encapsulated papillary carcinoma
      2. Invasive cribriform carcinoma
      3. Invasive lobular carcinoma
      4. Invasive papillary carcinoma
      5. Low grade adenosquamous carcinoma
      Board review style answer #2
      E. Low grade adenosquamous carcinoma. Encapsulated papillary, invasive cribriform and invasive lobular carcinoma commonly and invasive micropapillary carcinoma less commonly show hormone receptor expression. Low grade adenosquamous carcinomas are invariably triple negative.

      Comment Here

      Reference: Triple negative breast cancer
      Board review style question #3
      Which of the following type of triple negative breast cancer is associated with a more aggressive course of disease?

      1. Adenoid cystic carcinoma
      2. Fibromatosis-like metaplastic carcinoma
      3. Low grade adenosquamous carcinoma
      4. Secretory carcinoma
      5. Squamous cell carcinoma
      Board review style answer #3
      E. Squamous cell carcinoma. Adenoid cystic carcinoma, secretory carcinoma, fibromatosis-like metaplastic carcinoma and low grade adenosquamous carcinoma are considered low grade triple negative breast cancers. Squamous cell carcinoma of the breast is morphologically of a higher grade and associated with a worse prognosis (Hum Pathol 2010;41:679).

      Comment Here

      Reference: Triple negative breast cancer

      Tuberculosis
      Terminology
      • Also called tuberculous mastitis
      Epidemiology
      Clinical features
      Case reports
      Treatment
      Clinical images

      Images hosted on other servers:

      Resembles breast carcinoma

      Costochondral junction TB

      Discoloration of overlying skin multiple sinuses

      Gross description
      • Multiple sinuses or fistulas; may have focal discoloration or mass
      Microscopic (histologic) description
      • Granulomas with Langhans giant cells and caseous necrosis (often)
      Microscopic (histologic) images

      AFIP images

      Granulomatous inflammation and Langhans giant cells

      Granulomatous
      inflammation of
      major lactiferous duct
      with epithelial necrosis


       Contributed by Dr. Mark R. Wick

      Tuberculosis

      Ziehl-Neelsen stain+



      Images hosted on other servers:

      Granuloma with central necrosis and epithelioid histiocytes

      Suppurative granuloma and inflammatory infiltrate

      TB and infiltrating ductal carcinoma

      Cytology description
      • Foamy histiocytes, neutrophils, necrotic debris

      Tubular
      Definition / general
      • Special subtype of invasive breast carcinoma with favorable prognosis
      • Composed of distinct, well differentiated angular tubular structures (> 90%) with open lumina, lined by a single layer of low grade malignant epithelial cells, with a fibrous / desmoplastic stromal response
      Essential features
      • > 90% of the tumor is composed of ovoid or angular tubules with open lumina lined by a single layer of epithelial cells with low grade nuclei and sparse mitoses (grade 1)
      • ER positive and HER2 negative
      Terminology
      • Tubular carcinoma
      • Invasive tubular carcinoma
      ICD coding
      • ICD-O: 8523/3 - infiltrating duct mixed with other types of carcinoma
      • ICD-10:
        • C50.812 - malignant neoplasm of overlapping sites of left female breast
        • C50.912 - malignant neoplasm of unspecified site of left female breast
      • ICD-11: 2C60 - carcinoma of breast, specialized type
      Epidemiology
      • 1.6% of invasive breast carcinomas
      • Postmenopausal women, median age at presentation is 63 years
      • Slightly higher incidence in white women than black women in the U.S.
      Sites
      • Breast, with no specific site of predilection
      Pathophysiology
      Etiology
      Clinical features
      Diagnosis
      • Histologic examination of tissue
      Radiology description
      • Mammography: may present as a discrete or ill defined mass with well demarcated or spiculated, irregular margins, respectively or as an architectural distortion, with variable calcifications (Radiol Med 2006;111:773)
      • Ultrasonography: hypoechoic mass with poorly defined margins and posterior acoustic shadowing (Tumori 2003;89:417)
      • MRI: enhancing mass, enhancing focus or non mass enhancement (Egyptian J Radiol Nuc Med 2018;49:281)
      Radiology images

      Contributed by Julie M. Jorns, M.D.
      Diagnostic mammogram

      Diagnostic mammogram

      Prognostic factors
      • Well differentiated variant with very favorable prognosis (low rates of recurrence and metastasis) (Am J Surg 2009;197:674)
      • Longer disease free and breast cancer specific survival with tubular carcinoma (n = 102) as compared to grade 1 invasive ductal carcinoma of no special type (n = 212) (J Clin Oncol 2010;28:99)
      • Higher disease free survival was seen with pure tubular histology as well as if 3 criteria were met (≥ 70% tubules, pure grade 1 nuclei and no / rare mitoses) in a cohort of grade 1 - 2 tubular (n = 32) and invasive ductal (n = 115) carcinomas (Am J Clin Pathol 2004;122:728)
      • Mixed with invasive ductal carcinoma of no special type has a worse prognosis (Hum Pathol 1983;14:694)
      • Cause specific survival of 97% at 10 years (Eur J Surg Oncol 2005;31:9)
      • 10 - 27% have axillary metastases, often micrometastases but still have excellent prognosis (Eur J Surg Oncol 2006;32:488, Breast J 2003;9:298)
      Case reports
      Treatment
      Gross description
      • Spiculated tumor with a firm consistency; may have gritty texture if there are calcifications
      • Mean size is 1.2 cm
      • White, tan or gray in color
      Gross images

      Images hosted on other servers:
      Yellow-white sclerotic mass

      Yellow-white sclerotic mass

      Microscopic (histologic) description
      • Invasive breast cancer with infiltrative growth pattern, often with invasion into adipose tissue and with an associated fibrous or desmoplastic stromal response
        • No myoepithelium surrounding the tubules
      • > 90% of the tumor is composed of small, ovoid or angulated tubules with open lumina
      • Tubules are lined by a single layer of epithelial cells with low grade nuclei and sparse mitoses (grade 1)
        • Tumor cells often have apical cytoplasmic tufting or snouts
        • Cells lining the neoplastic tubules are cuboidal to columnar with uniform, small to intermediate sized nuclei
        • No significant cytologic atypia, multilayering or high mitotic activity (the presence of these would rule out tubular carcinoma)
        • There may be luminal secretions or calcifications
      • Tumor cells are ER positive and HER2 negative
      • Recent study shows that the actual structure is not tubular, instead it simulates a beaded necklace
        • 2 dimensional microscopy and 3 dimensional modeling studies revealed that structure resembled a necklace formed by a string of beads with the tubules visualized as blebs bridged together by solid cords of cells; continuous or branching tubules were not seen (Histopathology 2006;48:556)
      • Desmoplastic or fibroelastotic stroma
      • Frequent presence of associated nonobligate precursors:
      Microscopic (histologic) images

      Contributed by Julie M. Jorns, M.D., Mark R. Wick, M.D. and AFIP
      Tubular carcinoma

      Infiltrative angulated tubules

      Angulated tubules

      Angulated tubules

      Apical tufting

      Apical tufting

      Desmoplasia

      Desmoplasia

      Associated ADH, FEA and ALH

      Associated ADH, FEA and ALH


      Associated atypical lobular hyperplasia

      Associated atypical lobular hyperplasia

      Associated atypical ductal hyperplasia / flat epithelial atypia

      Associated atypical ductal hyperplasia / flat epithelial atypia

      Open tubules

      Angular glands with cytoplasmic snouts

      Angular glands with cytoplasmic snouts

      Virtual slides

      Images hosted on other servers:
      Invasive tubular carcinoma

      Invasive tubular carcinoma

      Cytology description
      • Increased cellularity, somewhat angular or irregular or small tubular epithelial clusters and single epithelial cells
      • Cells are bland and orderly with variable atypia and a variable number of prominent myoepithelial cells
      • Features of malignancy such as nuclear atypia, absence of myoepithelial cells and cellular dissociation are subtle and hence diagnosis on cytology can be challenging (Acta Cytol 1997;41:1139, Am J Clin Pathol 1994;101:488)
      Positive stains
      Negative stains
      Electron microscopy description
      • Ductal differentiation, no myoepithelial cells and no basement membrane
      Molecular / cytogenetics description
      Sample pathology report
      • Right breast, ultrasound guided core biopsy:
        • Invasive mammary carcinoma with tubular features, preliminary modified Bloom-Richardson grade 1 (1+2+1) (see comment)
        • Estrogen receptor: (91 - 100%, strong) positive
        • Progesterone receptor: (91 - 100%, strong) positive
        • HER2 / neu: (1+) negative for overexpression
        • Comment: The carcinoma in this biopsy has entirely tubular features. If there is similar morphology in > 90% of the tumor on resection it would be best classified as an invasive tubular carcinoma.
      Differential diagnosis
      • Sclerosing adenosis:
        • Overall lobular architecture but with distortion by fibrosis, compression of glands with 2 layers (inner epithelial and outer myoepithelial)
        • Positive for myoepithelial markers (Appl Immunohistochem Mol Morphol 2006;14:71)
      • Radial scar:
        • Desmoplastic / elastotic stroma, bland tubules
        • Does not infiltrate at the periphery
        • Often has intermixed fibrocystic changes such as cysts and usual ductal hyperplasia
        • Retained myoepithelial cell layer
      • Microglandular adenosis:
        • More rounded (not angulated) tubules with intraluminal colloid-like secretory material (Am J Surg Pathol 1982;6:401)
        • Lacks myoepithelium but basement membrane material is present
        • ER negative and S100 positive
      • Mixed tumors with a combination of tubular carcinoma and another subtype of cancer:
        • Should be reported when the tumor mass shows 10 - 90% tubular carcinoma (common) or ≥ 90% is tubular carcinoma and remaining tumor shows nuclear grade 3 (rare)
      • Tubulolobular carcinoma:
        • Mixed tubular and lobular components
        • Not an entity in WHO
        • Thought to be a variant of tubular carcinoma as they share similar architectural and growth patterns (tubulolobular carcinoma also resembles a beaded necklace but with longer connecting strands of single cells)
        • Both show E-cadherin positivity
        • Both show indolent clinical behavior
      • Low grade adenosquamous carcinoma:
        • Squamous differentiation
        • Less uniformity (open and compressed tubules, nests)
        • Often ER negative
      • Low grade invasive ductal carcinoma of no special type:
        • Similar low grade cytologic features to tubular carcinoma, with or without prominent apical cytoplasmic tufting but with more complex architecture
        • Tumor shows multilayering, stratification, glandular fusion or branching and may have > 10% ribbons or cords
        • ER expression is often high but may be of lower intensity in tubular carcinoma (i.e., third quartile or > 50 - 75%, expression) than in well differentiated invasive ductal carcinoma (i.e., fourth quartile or > 75 - 100%, expression) (Arch Pathol Lab Med 2014;138:1507)
      Board review style question #1

      A postmenopausal woman presented with a breast mass measuring 0.8 cm. Biopsy and resection showed the morphology seen in the image above in > 90% of the tumor. What is the expected immunoprofile?

      1. ER / PR positive, HER2 negative
      2. ER / PR positive, HER2 positive
      3. ER / PR negative, HER2 positive
      4. ER / PR negative, HER2 negative
      Board review style answer #1
      A. ER / PR positive, HER2 negative. The tumor consists of infiltrative small, angulated tubules with a single layer of low grade neoplastic epithelium with prominent apical tufting, features diagnostic of tubular carcinoma. Tubular carcinoma is a luminal A breast carcinoma and thus is ER / PR positive and HER2 negative.

      Comment Here

      Reference: Tubular carcinoma of breast
      Board review style question #2
      A core biopsy of a breast mass shows an invasive carcinoma with tubular features. What are the essential criteria to diagnose this tumor as tubular carcinoma?

      1. > 90% tubules lined by highly pleomorphic epithelial cells and brisk mitosis
      2. > 90% tubules lined by low grade epithelial cells with sparse mitosis
      3. < 90% tubules lined by highly pleomorphic epithelial cells and brisk mitosis
      4. < 90% tubules lined by low grade epithelial cells with sparse mitosis
      Board review style answer #2
      B. > 90% tubules lined by low grade epithelial cells with sparse mitosis. The diagnosis of tubular carcinoma or any special subtype of invasive breast carcinoma, requires that > 90% of the tumor show the specific diagnostic features of the special subtype. Thus, in the case of tubular carcinoma, > 90% of the tumor must have small, angulated tubules lined by a single layer of low grade malignant epithelium and sparse to absent mitoses. Because the patient underwent core biopsy, which is only a sampling of the tumor, the designation as tubular carcinoma should be reserved until the tumor is fully evaluated on subsequent resection. In such cases, a note or comment can be helpful (please see Sample pathology report for an example).

      Comment Here

      Reference: Tubular carcinoma of breast

      Tubular adenoma
      Definition / general
      • Benign fibroepithelial tumor of the breast composed of compact bilayered tubules with sparse intervening stroma
      Essential features
      • Circumscribed / nodular benign tumor of the breast
      • Closely packed tubules with sparse intervening stroma
      Terminology
      • Fibroadenomas variant (pericanalicular subtype), not recommended by WHO
      ICD coding
      • ICD-O: 8211/0 - tubular adenoma, NOS
      • ICD-11: 2F30.0 - tubular adenoma of breast
      • ICD-11: XH7SY6 - tubular adenoma, NOS
      Epidemiology
      Sites
      Pathophysiology
      • Arises from terminal duct lobular units
      Etiology
      • Unknown
      • May be associated with reproductive hormones due to high incidence in women of reproductive age
      Clinical features
      Diagnosis
      • Histology recommended for definitive diagnosis (N Am J Med Sci 2014;6:219)
        • Radiology and cytology usually allow identification of the benign nature of lesion but may not be sufficient in excluding differential diagnoses
      Radiology description
      • Well defined and well circumscribed lesion (AJR Am J Roentgenol 2000;174:757)
      • Hypoechoic on ultrasound
        • Posterior acoustic enhancement sometimes seen
      • Tiny, punctate and irregular microcalcifications may be present on mammography in older patients
      Radiology images

      Images hosted on other servers:
      Ultrasound

      Ultrasound

      Prognostic factors
      Case reports
      Treatment
      • Curable by complete surgical excision
      • Observation acceptable for asymptomatic, low risk patients
      Clinical images

      Images hosted on other servers:
      Lesion in accessory breast

      Lesion in accessory breast

      Gross description
      Gross images

      Images hosted on other servers:
      Well defined borders

      Well defined borders

      Tan cut surface

      Tan cut surface

      Microscopic (histologic) description
      • Well defined borders
      • Sparse fibrovascular stroma intervening tubules
      • Small, uniform, closely packed round tubules (Clin Med Insights Pathol 2018;11:1179555718757499)
        • Lined by an inner layer of luminal epithelial cells and an outer layer of myoepithelial cells
          • Rare mild atypia and mitotic figures does not exclude diagnosis
        • Occasional luminal eosinophilic secretion
      Microscopic (histologic) images

      Contributed by Joshua J.X. Li, M.B.Ch.B. and Gary M. Tse, M.B.B.S.
      Well defined borders

      Well defined borders

      Closely packed tubules

      Closely packed tubules

      CK5/6

      CK5/6

      p63

      p63

      Eosinophilic luminal secretions

      Eosinophilic luminal secretions

      Sparse intervening stroma

      Sparse intervening stroma



      Contributed by Jijgee Munkhdelger, M.D., Ph.D. and Andrey Bychkov, M.D., Ph.D.
      Missing Image

      Well defined lesion

      Missing Image

      Mimicking invasion

      Missing Image

      Myoepithelial layer on CD10

      Missing Image

      Breast tubular adenoma immunoprofile

      Virtual slides

      Images hosted on other servers:
      Biopsy

      Biopsy of tubular adenoma

      Cytology description
      Cytology images

      Contributed by Joshua J.X. Li, M.B.Ch.B. and Gary M. Tse, M.B.B.S.
      Background naked nuclei

      Background naked nuclei

      3 dimensional cell balls

      3 dimensional cell balls

      Bland nuclear features

      Bland nuclear features

      Positive stains
      Negative stains
      Molecular / cytogenetics description
      • Recently, next generation sequencing demonstrated that tubular adenomas and fibroadenomas show different mutation profiles (J Cell Biochem 2019;120:182)
      Sample pathology report
      • Left breast, excisional biopsy:
        • Tubular adenoma (see comment)
        • Comment: Sections show a proliferation of small, crowded tubules with minimal intervening stroma, composed of bland epithelial cells with an intact myoepithelial cell layer. Occasional eosinophilic luminal secretions are noted.
      Differential diagnosis
      • Fibroadenoma:
        • Expanded stromal component in between ducts
        • Ducts distorted and compressed by stroma (intracanalicular pattern)
        • Presence of MED12 mutations
      • Lactating adenoma:
        • Luminal epithelium shows secretory changes (hobnail changes, granular or vacuolated cytoplasm)
        • Luminal secretions positive for α-lactalbumin
      • Nipple adenoma:
        • Haphazardly distributed tubular ducts
        • Lacks circumscription
        • Located at the superficial aspect of nipple, often with epidermal connection
      • Adenomyoepithelioma:
        • Biphasic lesion with multilayered or solid nests of myoepithelial cells
        • Mixed nontubular patterns, including lobular and spindle patterns
      • Sclerosing adenosis:
        • Hyalinized and occasionally expanded stroma
        • Ducts compressed and distorted by stroma
      • Collagenous spherulosis:
        • Intraductal proliferation with cribriform pattern
        • Spherules containing basement membrane material enclosed by myoepithelial cells
        • Usually associated with benign proliferative lesions
      • Microglandular adenosis:
        • Infiltrative, haphazardly distributed small round glands
        • Lacks myoepithelial cells
        • Lacks circumscription
      • Tubular carcinoma:
        • Infiltrative angulated tubules with tapering ends
        • Lacks myoepithelial cells
        • Stroma shows desmoplastic reaction
      Board review style question #1

      Which of the following is a common clinical feature of tubular adenoma?

      1. Attachment to the nipple
      2. Irregular and fixed on palpation
      3. Located at upper outer quadrant of the breast
      4. Presence of axillary lymphadenopathy
      5. Presenting in a postmenopausal woman
      Board review style answer #1
      C. Tubular adenoma is most commonly but not exclusively located at the upper outer quadrant of the breast and presents in young women. Attachment to the nipple is a feature of nipple adenoma. Axillary lymphadenopathy, an irregular border and fixation are malignant features not commonly associated with tubular adenomas.

      Comment Here

      Reference: Tubular adenoma
      Board review style question #2
      Which of the following features favors the diagnosis of tubular adenoma over fibroadenoma?

      1. Detection of MED12 mutation
      2. Minimal intervening fibrous stroma between tubules
      3. Presence of myoepithelial layer
      4. Presence of rare mitotic figures
      5. Tubules arranged in an intracanalicular pattern
      Board review style answer #2
      B. Minimal intervening fibrous stroma between tubules. Fibroadenomas often demonstrate expansion of fibrous stroma, which compresses tubules and produces an intracanalicular pattern. MED12 mutation is associated with fibroepithelial lesions of the breast, in particular fibroadenomas. Myoepithelial layer is present in both lesions and rare mitotic figures are allowed in both lesions.

      Comment Here

      Reference: Tubular adenoma

      Tubulolobular carcinoma
      Definition / general
      Essential features
      • Carcinoma with areas of tubular carcinoma and invasive lobular carcinoma
      • Most are estrogen receptor (ER) positive and HER2 negative
      • Not synonymous with invasive carcinoma with ductal and lobular features
      Terminology
      • In the 5th edition of the WHO Classification of Breast Tumours, tubulolobular carcinoma is considered a histologic variant of invasive lobular carcinoma; however, many authors consider this histological pattern a variant of invasive breast carcinoma of NST (Diagnostics (Basel) 2022;12:2658)
      ICD coding
      • ICD-11: XH3RK9 - tubulolobular carcinoma
      Epidemiology
      Sites
      Etiology
      • Etiology is multifactorial: genetics, hormones, diet, age and reproductive factors are risk factors (WHO 5th edition)
      • ER positive pathway of breast cancer development characterized by gains of 1q and losses of 16q (WHO 5th edition)
      • See etiologies associated with low grade, ER positive invasive breast cancer of no special type (NST)
      • Expression of E-cadherin supports ductal differentiation, despite a dominant lobular growth pattern (Am J Surg Pathol 2004;28:1587)
      Clinical features
      Diagnosis
      • Diagnosis can be made on core needle biopsy or resection specimens
      Radiology description
      Prognostic factors
      Case reports
      Treatment
      • Localized excision / mastectomy
      • Choice of treatment is based on the size and the tumor stage: localized excision, radical or modified radical mastectomy, hormone therapy and adjuvant therapy (Mod Pathol 2007;20:130)
      Gross description
      • Ill defined, firm, gray tumor, usually measuring < 2 cm in greatest dimension, ranging in size from 0.5 cm to 2.5 cm (median, 1.4 cm) (Am J Surg Pathol 2004;28:1587)
      Microscopic (histologic) description
      • Invasive tumor with mixed pattern of small tubules with a lobular pattern of infiltration within a dense collagenous stroma often with prominent elastosis
        • Tubular component: small, round to angulated tubules with low grade nuclei and variable apical snouts
        • Lobular component: single cells, single file cords of cells, often displaying targetoid growth around benign ducts
        • Cells are uniform and display low grade nuclear atypia (nuclear grade 1 or 2) with inconspicuous nucleoli and minimal amphophilic cytoplasm
        • Cells with lobular growth pattern may show occasional intracytoplasmic lumina and rare signet ring morphology
      • Can have variable proportion of tubular and lobular components
      • Usually well differentiated (grade 1); can be moderately differentiated
        • Nuclear score can be grade 1 - 2
      • May have associated ductal carcinoma in situ (DCIS) or lobular carcinoma in situ (LCIS)
      • Lacks surrounding myoepithelial cell layer
      • References: Mod Pathol 2007;20:130, Am J Surg Pathol 2004;28:1587, Breast Care (Basel) 2008;3:423
      Microscopic (histologic) images

      Contributed by S. Emily Bachert, M.D. and AFIP
      Haphazard arrangement

      Haphazard arrangement

      Infiltrative pattern

      Infiltrative pattern

      Tubular and lobular components

      Tubular and lobular components

      Fibrous stroma

      Fibrous stroma


      Predominantly lobular component

      Predominantly lobular component

      Infiltrative tubules

      Infiltrative tubules

      Single file cells

      Single file cells

      ER+

      ER+

      Cytology description
      • Diagnosis can be suggested but not definitively made on cytologic preparations
      • Single filing of cells and tubular structure formation
      • Low nuclear grade
      • Low mitotic rate
      • Intracytoplasmic vacuoles and rare nucleoli (Acta Cytol 1996;40:465)
      Positive stains
      Negative stains
      Sample pathology report
      • Right breast, excision:
        • Tubulolobular carcinoma, well differentiated, measuring 1.5 cm
        • Nottingham grade I / III: tubule score = 1, nuclear score = 1, mitoses score = 1
      Differential diagnosis
      Board review style question #1

      Which of the following is true regarding the histologic subtype of the breast carcinoma shown above?

      1. The tumor will most likely be ER negative and HER2 positive
      2. The tumor will most likely be ER positive and HER2 negative
      3. The tumor will most likely be negative for E-cadherin
      4. The tumor will most likely be positive for SMMHC and p63
      Board review style answer #1
      B. The tumor will most likely be ER positive and HER2 negative. The picture depicts an invasive carcinoma with small round tubules and single filing of cells, compatible with a tubulolobular carcinoma of the breast. Answer A is incorrect because the vast majority of these tumors are ER positive and HER2 negative. Answer C is incorrect because E-cadherin is usually positive in these tumors as they have tubule formation. Answer D is incorrect because these tumors will be negative for myoepithelial markers, compatible with invasive carcinomas.

      Comment Here

      Reference: Tubulolobular carcinoma

      Usual ductal hyperplasia
      Definition / general
      • Benign intraductal proliferation of progenitor epithelial cells with varying degrees of solid or fenestrated growth
      Essential features
      • Component of fibrocystic changes
      • Mild cytologic variability
      • Streaming growth pattern with fenestrated spaces and lack of cellular polarity
      • Immunoreactive for high molecular weight cytokeratins
      • Associated with slight increase in subsequent breast cancer risk (1.5 - 2 times)
      Terminology
      • Also called epithelial hyperplasia, intraductal hyperplasia, hyperplasia of usual type, ductal hyperplasia without atypia, epitheliosis
      ICD coding
      • ICD-10: N60.99 - unspecified benign mammary dysplasia of unspecified breast
      • ICD-11: GB20.Y - other specified benign breast disease
      Epidemiology
      Sites
      • Terminal duct lobular units
      • Occasionally, extralobular ducts
      Pathophysiology
      • Proliferation of CK5+ progenitor cells that can differentiate along glandular or myoepithelial lineages; glandular progenitor cells appear to predominate and show intermediate levels of differentiation (J Pathol 2002;198:458)
      Etiology
      • No specific etiologic factors
      Clinical features
      • No specific clinical findings
      Diagnosis
      • Diagnosis by histologic examination of tissue removed via biopsy or surgical excision
      Radiology description
      • No specific mammographic findings; occasional examples are associated with microcalcifications
      • Can involve an underlying lesion (e.g. radial scar or papilloma) that is identified on imaging
      • May show enhancement on magnetic resonance imaging (Arch Pathol Lab Med 2017;141:1513)
      Prognostic factors
      Case reports
      Treatment
      • No treatment necessary
      Gross description
      • No macroscopic findings
      Microscopic (histologic) description
      • Proliferation of cells of luminal and myoepithelial lineages, occasionally with intermixed apocrine cells
      • Cytologic features (Semin Diagn Pathol 2004;21:10)
        • Mild variation in cellular and nuclear size and shape
        • Relatively small ovoid nuclei with frequent elongated or asymmetrically tapered (pear shaped) forms
        • Lightly granular euchromatic chromatin and small nucleoli
        • Frequent longitudinal nuclear grooves (coffee bean-like) and occasional nuclear pseudoinclusions
        • Many examples demonstrate cellular maturation, where the cells shrink as they progress from a basal location to the center of the proliferation, becoming small and nearly pyknotic
        • Eosinophilic, nonabundant cytoplasm with indistinct cell borders
      • Architectural features (Semin Diagn Pathol 2004;21:10)
        • Cohesive proliferation with haphazard, jumbled cell arrangement or streaming growth pattern
        • Fenestrated, solid and occasional micropapillary patterns
        • Irregular slit-like fenestrations are common, especially along periphery
        • Cells run parallel to the edges of secondary spaces and do not exhibit a polarized orientation (this contrasts with the cells of atypical ductal hyperplasia and ductal carcinoma in situ, which have apical-basal polarity and radially orient their apical poles toward the spaces)
      • Variant patterns and features
        • Micropapillary
          • Typically focal in a background of conventional pattern usual ductal hyperplasia
          • Mild duct dilation
          • Short stubby papillae of roughly uniform height
          • Cytologic features of usual ductal hyperplasia
          • Cellular maturation present, with tips of papillae formed by tight knots of mature cells
          • Lack of polarization
        • Immature
          • Uncommon variant
          • Larger immature basal hyperplastic cells predominate or are increased beyond their usual 1 - 2 cell layers and are instead several cell layers thick
          • Cellular maturation is still present
          • Most often encountered in fibroepithelial lesions with cellular stroma
        • Necrosis
          • Florid usual ductal hyperplasia can rarely demonstrate central necrosis
          • Typically occurs within a radial scar / complex sclerosing lesion, nipple adenoma or juvenile papillomatosis
        • Mild nuclear enlargement in radial scars
          • Florid usual ductal hyperplasia within radial scars / complex sclerosing lesions can occasionally have more active appearing nuclei with mild nuclear enlargement
          • Other cytologic and architectural features of usual ductal hyperplasia remain intact
      Microscopic (histologic) images

      Contributed by Melinda Lerwill, M.D.
      Missing Image Missing Image Missing Image

      Cytologic features

      Missing Image Missing Image

      Cellular maturation

      Missing Image

      Merging with apocrine metaplasia


      Missing Image

      Fenestrated spaces

      Missing Image

      Lack of cellular polarity

      Missing Image

      Micropapillary usual ductal hyperplasia

      Missing Image

      Immature usual ductal hyperplasia

      Missing Image

      Florid usual ductal hyperplasia in radial scar

      Missing Image

      Cytokeratin 5/6


      Missing Image

      Estrogen receptor

      Cytology description
      • Sample may be moderately to highly cellular
      • Sheets and cohesive clusters of bland ductal cells with regular spacing and associated myoepithelial cells (Am J Clin Pathol 1995;103:438)
      • Lack of significant nuclear overlap / crowding
      • Ductal cell nuclei with finely granular chromatin and inconspicuous small nucleoli
      • Lack of ductal cell discohesion
      • Naked myoepithelial cell nuclei in the background may be present
      Cytology images

      Contributed by Amy Ly, M.D.
      Missing Image

      Fine needle aspiration

      Positive stains
      Molecular / cytogenetics description
      Videos

      High yield breast pathology cases

      Sample pathology report
      • Left breast, needle core biopsy:
        • Usual ductal hyperplasia
      Differential diagnosis
      Board review style question #1

        What is the risk for subsequent breast cancer associated with the illustrated lesion?

      1. 1.5 - 2 times increased risk
      2. 2 - 3 times decreased risk
      3. 4 - 5 times increased risk
      4. 8 - 11 times increased risk
      5. No change in risk compared to control populations
      Board review style answer #1
      A. 1.5 - 2 times increased risk

      This is usual ductal hyperplasia. Usual ductal hyperplasia is associated with a slight increase in risk (1.5 - 2 times) for subsequent breast cancer. Risk appears to be slightly higher in those patients with a positive family history of breast cancer.

      Comment Here

      Reference: Usual ductal hyperplasia
      Board review style question #2
        What is the typical high molecular weight cytokeratin / estrogen receptor (HWMCK / ER) immunoprofile for usual ductal hyperplasia of the breast?

      1. HMWCK negative / ER diffusely positive
      2. HMWCK negative / ER negative
      3. HMWCK mosaic positive / ER diffusely positive
      4. HMWCK mosaic positive / ER heterogeneously positive
      5. HMWCK mosaic positive / ER negative
      Board review style answer #2
      D. HMWCK mosaic positive / ER heterogeneously positive

      Usual ductal hyperplasia is positive for HMWCK in a mosaic to occasionally diffuse pattern and demonstrates heterogeneous positivity for ER.

      Comment Here

      Reference: Usual ductal hyperplasia
      Board review style question #3
        Which of the following is a feature of usual ductal hyperplasia that aids in distinguishing it from low grade ductal carcinoma in situ?

      1. Cellular maturation
      2. Monomorphic hyperchromatic nuclei
      3. Palisading around fibrovascular cores
      4. Polarization around secondary spaces
      5. Red macronucleoli
      Board review style answer #3
      A. Cellular maturation

      Many examples of usual ductal hyperplasia demonstrate cellular maturation, where the cells shrink as they progress from a basal location to the center of the proliferation, becoming small and nearly pyknotic. Cellular maturation is not a feature of low grade ductal carcinoma in situ or atypical ductal hyperplasia.

      Comment Here

      Reference: Usual ductal hyperplasia

      WHO classification
      Definition / general
      • Tumor classification is a dynamic process that integrates multiple sources of recent information
      • Based on current 5th edition, published in 2019
      Major updates
      • Includes new, renamed and removed entities from the 4th edition of WHO classification along with changes to diagnostic criteria
      • Carcinomas with medullary features have been subsumed into a combined morphological subset under the category of invasive carcinoma, no special type (NST) with basal-like and medullary pattern
      • Mucinous cystadenocarcinoma has been recognized as a new entity
        • Characterized by cystic structures lined by tall columnar cells with abundant intracytoplasmic mucin, resembling pancreatobiliary or ovarian mucinous cystadenocarcinoma
      • Tall cell carcinoma with reversed polarity has been recognized as a new entity
      • Neuroendocrine neoplasms (NENs) harmonized with those of other organ systems incorporating uniform classification network (Mod Pathol 2018;31:1770)
      • Removal of well differentiated liposarcoma as a histologic criterion for malignancy in phyllodes tumor in the absence of additional supporting features
        • Abnormal adipocytes within phyllodes tumor lack MDM2 or CDK4 amplifications in contrast to extramammary well differentiated liposarcoma (Histopathology 2016;68:1040)
      • Updated information on molecular pathology, expression profiling and molecular classification of breast tumors; however, focus remains on morphologic classification
      • Conversion of mitotic count from a common denominator of 10 high power fields to a defined area expressed as mm2
      WHO (2019)
      Microscopic (histologic) images

      Contributed by Indu Agarwal, M.D., Mirna B. Podoll, M.D. and Julie M. Jorns, M.D.
      Low grade DCIS

      Low grade DCIS

      LCIS, classic type

      LCIS, classic type

      Invasive carcinoma, NOS Invasive carcinoma, NOS

      Invasive carcinoma, NOS

      Invasive and in situ carcinoma

      Invasive and in situ carcinoma

      Metastasis of lobular carcinoma

      Metastasis of lobular carcinoma


      Immunostain cytokeratin AE1 / AE3

      Immunostain cytokeratin AE1 / AE3

      Invasive carcinoma, NOS Metaplastic carcinoma

      Metaplastic carcinoma

      Encapsulated papillary carcinoma Encapsulated papillary carcinoma

      Encapsulated papillary carcinoma


      Missing Image Missing Image

      Adenoid cystic carcinoma

      Tall cell carcinoma with reverse polarity

      Tall cell carcinoma with reverse polarity

      Invasive ductal carcinoma with medullary pattern

      Invasive ductal carcinoma with medullary pattern

      Phyllodes tumor with liposarcomatous differentiation

      Phyllodes tumor with liposarcomatous differentiation

      Board review style question #1
      Which of the following is a new entity in the WHO Classification of Breast Tumours, 5th edition, published in 2019?

      1. Malignant phyllodes tumor with well differentiated liposarcomatous differentiation alone
      2. Medullary carcinoma
      3. Mucinous cystadenocarcinoma
      4. Solid papillary carcinoma
      Board review style answer #1
      C. Mucinous cystadenocarcinoma. Tall cell carcinoma with reversed polarity (TCCRP) and mucinous cystadenocarcinoma, NOS are 2 new entities. TCRRP is a rare subtype of invasive breast carcinoma characterized by tall columnar cells with reversed nuclear polarity, arranged in solid and solid papillary patterns and most commonly associated with IDH2 p.Arg172 hotspot mutations. Mucinous cystadenocarcinoma of the breast is an invasive breast carcinoma characterized by cystic structures lined by tall columnar cells with abundant intracytoplasmic mucin, resembling pancreatobiliary or ovarian mucinous cystadenocarcinoma.

      Answer A is incorrect because liposarcomatous differentiation has been removed as a histologic criterion for malignancy in phyllodes tumor in the absence of additional supporting features. Answer B is incorrect because tumors previously called as medullary carcinoma, atypical medullary carcinoma and invasive carcinoma with medullary features have been subsumed into a combined morphologic subset under the category of invasive carcinoma, NST with basal-like and medullary pattern, regarding them as a part of a spectrum of tumor infiltrating lymphocytes rich breast carcinomas. Answer D is incorrect because encapsulated papillary carcinoma and solid papillary carcinoma have been previously present in prior WHO editions.

      Comment Here

      Reference: Breast - WHO classification
      Board review style question #2

      Tall cell carcinoma with reversed polarity (TCCRP) is a new entity in the WHO Classification of Breast Tumours, 5th edition, published in 2019, that has what typical prognosis and estrogen receptor (ER), progesterone receptor (PR) and HER2 / neu (HER2) profile?

      1. Good prognosis, ER / PR negative, HER2 negative
      2. Good prognosis, ER / PR positive, HER2 negative
      3. Poor prognosis, ER / PR negative, HER2 negative
      4. Poor prognosis, ER / PR positive, HER2 negative
      Board review style answer #2
      A. Good prognosis, ER / PR negative, HER2 negative. Tall cell carcinoma with reversed polarity (TCCPR) is a rare subtype of invasive breast carcinoma characterized by tall columnar cells with reversed nuclear polarity, arranged in solid and solid papillary patterns and most commonly associated with IDH2 p.Arg172 hotspot mutations. Despite the good prognosis and indolent course of TCCRP, this tumor typically expresses low and high molecular weight keratins, including CK5/6 and is ER / PR and HER2 negative, thus part of a group of rare good prognosis triple negative breast carcinoma (TNBC), which also includes some salivary gland type tumors (e.g., adenoid cystic carcinoma) and variants of metaplastic carcinoma (e.g., low grade fibromatosis-like carcinoma). Answers B and D are incorrect because TCCRP is not ER / PR positive. Answers C and D are incorrect because it is not a poor prognosis breast cancer subtype.

      Comment Here

      Reference: Breast - WHO classification

      Yokohama system, malignant
      Definition / general
      • A malignant diagnosis is an unequivocal statement that the sample is malignant; if possible, the type of malignancy should be specified
      • Positive predictive value (PPV) of a malignant diagnosis has a range of 92 - 100% in the literature; however, using the Yokohama criteria, it is 99 - 100%
      • Perform ER / PR / HER2 on cell block for treatment planning (if validated)
      • Biopsy of axillary lymph nodes can be done
      • If a malignant diagnosis is not consistent with the clinical and imaging findings, a core needle biopsy or surgical biopsy should be done
      Essential features
      • For a malignant diagnosis, the fine needle aspiration biopsy (FNAB) should show multiple malignancy features, since individual features can be seen in other lesions; features of malignancy are:
        • Marked cellularity
        • Prominent single cells
        • Crowded, small, discohesive epithelial tissue fragments with nuclear overlapping
        • Enlarged, pleomorphic nuclei with irregular nuclear membranes, coarse chromatin and large nucleoli
        • Lack of bare bipolar nuclei
        • Lack of myoepithelial cells
      • Softer criteria for malignancy include:
        • Tissue fragments with tubular architecture
          • Can also be seen in fibroadenoma
        • Intracytoplasmic vacuoles in cells with atypical nuclei
        • Elastoid fragments
        • Multinucleated tumor giant cells
      CPT coding
      No information provided
      Sites
      • Breast
      Diagrams / tables
      None
      Laboratory
      Radiology description
      Radiology images
      None
      Case reports
      None
      Cytology description
      • Invasive carcinoma of no special type (ductal carcinoma)
        • Low power:
          • High cellularity
            • However, desmoplastic stroma may lead to a hypocellular sample
          • Epithelial tissue fragment size varies with grade:
            • Low grade:
              • Larger, more cohesive fragments
            • High grade:
              • Smaller tissue fragments, more single cells
              • Tissue fragments fray at the edges
              • Nuclei are crowded, overlapping and disorganized
          • Micro biopsies of stromal fragments with infiltrating atypical cells
          • In liquid based cytology, the 3D tissue fragments appear more flattened than in smears
        • Background:
          • No myoepithelial cells or bare bipolar nuclei
          • Necrosis with calcifications suggests a high grade in situ component
        • High power:
          • Nuclear debris in tissue fragments
          • Eccentric intracytoplasmic vacuoles
          • Nuclear features vary with grade:
            • Low grade:
              • Dispersed cells are monotonous with moderately enlarged nuclei
              • Nuclei are round to pleomorphic
              • Mild to moderate hyperchromasia
              • Small nucleoli
            • High grade:
              • Dispersed cells have denser cytoplasm than low grade
              • High N:C ratio
              • Pleomorphic, enlarged nuclei
              • Dense, coarse chromatin
              • Prominent large, irregular nucleoli with perinucleolar clearing
      • Invasive lobular carcinoma
        • Low power:
          • Variable cellularity but can be low
          • Mostly dispersed single cells, short linear (single file) strands
          • Loosely cohesive fragments show windows between cells
        • Background:
          • No bare bipolar nuclei
          • Stripped nuclei, chromatin smearing
        • High power:
          • Uniform, intermediate sized cells
          • Nuclei are round to angulated, with subtle nuclear envelope indentations, bland chromatin, inconspicuous nucleoli
          • Cells have eccentric, relatively dense cytoplasm
          • Some cells have intracytoplasmic vacuoles with mucin droplets (targetoid or magenta bodies)
      • Tubular carcinoma
        • Low power:
          • Variable cellularity but typically moderate to high
          • Small, rigid, angulated, comma shaped, narrow and cohesive tubular epithelial tissue fragments
          • > 90% of tumor is made up of open tubules lined by a single layer of low columnar cells
          • Cribriform or micropapillary 3D fragments suggest an in situ component
        • Background:
          • Very few to no bare bipolar nuclei
        • High power:
          • Subtle nuclear indentations and grooves
          • Mild hyperchromasia
          • Apoptotic debris may mimic myoepithelial cells
      • Carcinoma with medullary features
        • Low power:
          • High cellularity
          • Small, syncytial sheets of epithelial cells infiltrated by lymphocytes
          • Variable number of dispersed single cells
        • Background:
          • Stripped, bizarre nuclei
          • Lymphocytes, plasma cells, cellular debris, lymphoglandular bodies
        • High power:
          • Large epithelial cells with a high N:C ratio
          • Nuclei have coarse chromatin and macronucleoli
          • Many mitotic figures
      • Mucinous carcinoma
        • Low power:
          • Variable cellularity but may be high
          • Variably sized tissue fragments that may be balled up, rounded, cribriform, papillary or tubular
          • Tissue fragments do not have myoepithelial nuclei
          • Naked branching capillaries
        • Background:
          • Fibrillary mucin that is blue to purple on Giemsa and pale green to orange on Pap
          • On liquid based preparations, the mucin is reduced or lost
          • No bare bipolar nuclei
        • High power:
          • Mild to moderate nuclear atypia but occasionally high grade
          • Occasional intracytoplasmic vacuoles, signet ring cells
          • Occasional calcifications
      • Invasive papillary carcinoma
        • Low power:
          • Slender and complex papillae with thin fibrovascular cores
          • Dissociated tall columnar cells
          • No myoepithelial cells
        • Background:
          • No bare bipolar nuclei
        • High power:
          • Variable nuclear atypia
      • Invasive micropapillary carcinoma
        • Low power:
          • Moderate to high cellularity
          • Predominantly small epithelial tissue fragments and dispersed cells
          • Jigsaw pattern of tissue fragments that are close together and may show molding
          • Mucinous blush on the outside of the tissue fragments
        • Background:
          • Necrosis
          • Calcifications
          • May show mucin
        • High power:
          • Large, atypical, hyperchromatic nuclei with prominent nucleoli
          • Moderate to high N:C ratio
          • Dense apocrine-like cytoplasm or eccentric columnar cytoplasm
      • Metaplastic carcinoma
        • Low power:
          • Moderately to high cellularity
          • Variable patterns depending on the cell differentiation
            • Poorly differentiated carcinoma show small, discohesive tissue fragments and dispersed single cells
            • Squamous cell carcinoma will show sheets and tissue fragments with keratinous debris
            • Low grade adenosquamous carcinoma shows small glands and tubules with focal keratinization and a spindle cell component
            • Sarcomatous differentiation will show plump spindle cells in small tissue fragments and dispersed single cells
            • Can see fibromatosis-like spindle cells or chondroid matrix
        • Background:
          • Necrosis and debris
          • Neutrophils, histiocytes, hemosiderin laden macrophages
        • High power:
          • Depends on cell differentiation
      • High grade phyllodes
        • Low power:
          • Crowded tissue fragments with frequent single cells
          • Scant epithelial fragments
        • Background:
          • Necrosis, foamy histiocytes
        • High power:
          • Large, plump spindle cells
          • High N:C ratio
          • Large prominent nucleoli
          • Frequent mitoses, some atypical
          • Magenta stroma may be seen between the pale blue cytoplasm of the malignant cells
      • Secretory carcinoma
        • Low power:
          • Moderate to high cellularity
          • Cohesive or loosely cohesive sheets and small tissue fragments
        • Background:
          • Abundant intensely staining bubbly secretory material
          • Stripped mildly atypical nuclei
        • High power:
          • Round to polygonal cells with abundant cytoplasm
          • Round, mildly atypical nuclei with fine to moderately coarse chromatin
          • Prominent intracytoplasmic vacuoles
          • Occasional signet ring cells or plasmacytoid cells
      • Glycogen rich clear cell carcinoma
        • Low power:
          • Moderate to high cellularity
          • Small, loosely cohesive syncytial sheets and single cells
        • Background:
          • Finely granular tigroid background
        • High power:
          • Moderate to marked nuclear pleomorphism
          • Finely granular, cleared, eccentric cytoplasm
          • Well defined cell membranes
          • Koilocyte-like cells
          • Focal apocrine change
      • Adenoid cystic carcinoma
        • Low power:
          • Moderate cellularity
          • Variably sized epithelial sheets and 3D tissue fragments
        • Background:
          • Many dispersed epithelial cells
          • Stripped nuclei
        • High power:
          • Basaloid epithelial cells with poorly defined, scant cytoplasm
          • Mildly atypical nuclei with single small nucleoli
          • Rounded hyaline globules that are magenta with Giemsa and pale gray-green with Pap stain
      • Apocrine carcinoma
        • Low power:
          • Moderate to high cellularity
          • Crowded, small, discohesive tissue fragments and dispersed single cells
        • Background:
          • Necrosis
          • Calcifications (intraductal component)
        • High power:
          • Large pleomorphic cells with granular, well defined cytoplasm
          • Nuclei are hyperchromatic, with coarse chromatin and large nucleoli
      • Carcinoma with neuroendocrine features
        • Low power:
          • High cellularity
          • Dispersed single cells, larger fragments with branching, thin fibrovascular cores
          • Poorly differentiated neuroendocrine carcinoma will resemble small cell carcinoma
          • Mucinous and solid papillary carcinomas can show neuroendocrine differentiation
        • Background:
          • No bare bipolar nuclei
          • No lymphoglandular bodies
        • High power:
          • Monotonous cells with bland, small, rounded nuclei, salt and pepper chromatin
          • Plasmacytoid cells with eccentric, pale, granular cytoplasm
      • Non-Hodgkin lymphoma
        • Diffuse large B cell lymphoma is most common
          • About 67% of primary breast lymphomas
          • Dispersed single cells
          • Atypical large lymphoid cells resembling centroblasts or immunoblasts
            • Eccentric basophilic cytoplasm
          • Lymphoglandular bodies
        • Marginal zone lymphoma involving mucosa associated lymphoid proliferations
          • Heterogeneous lymphoid population
            • Plasmacytoid cells, lymphocytes, centrocytes, plasma cells, scattered larger lymphoid cells
          • Difficult to differentiate from reactive process
          • Core needle biopsy usually needed for ancillary studies like clonality
        • Anaplastic large cell lymphoma
          • Textured implants
          • New peri-implant fluid collection
          • Large lymphoid cells with large, atypical, hyperchromatic, with 1 or 2 nucleoli and small rim of cytoplasm
          • Hallmark horseshoe shaped cells
          • CD30 positive
      • Angiosarcoma
        • Low power:
          • Usually low cellularity but can be high
          • Small, loosely cohesive spindle cell fragments
          • Larger tissue fragments containing capillary-like structures
        • Background:
          • Hemorrhagic
        • High power:
          • Cell shape can be spindle and sarcomatous, or rounded and epithelioid
          • Variable but often marked nuclear atypia
          • Nuclei show nuclear indentations and folds and 1 or 2 nucleoli
          • Intracytoplasmic hemosiderin, vacuoles
      Cytology images

      Contributed by Hillary Zalaznick, M.D.
      Carcinoma NST Carcinoma NST

      Carcinoma NST

      Carcinoma NST

      Carcinoma NST

      Carcinoma NST Carcinoma NST

      Carcinoma NST


      Mucin in mucinous carcinoma Mucin in mucinous carcinoma

      Mucin in mucinous carcinoma

      Molecular / cytogenetics description
      No information provided
      Molecular / cytogenetics images
      None
      Sample pathology report
      • Invasive carcinoma of no special type (ductal carcinoma) (direct quotes from Field: The International Academy of Cytology Yokohama System for Reporting Breast Fine Needle Aspiration Biopsy Cytopathology, 1st Edition, 2020)
        • Malignant (see comment)
          • "These highly cellular smears show plentiful dispersed cells and small epithelial tissue fragments consisting of similar cells, with large pleomorphic hyperchromatic nuclei."
          • "Comment: The features are those of carcinoma of the breast."
        • Malignant (see comment)
          • "Scattered large atypical cells are seen singly and in small tissue fragments with occasional sclerotic stromal fragments infiltrated by similar atypical cells, in a background of a large amount of necrosis with occasional calcifications."
          • "Comment: Carcinoma of the breast is present with features suggestive of a high grade intraductal component."
        • Malignant (see comment)
          • "These moderately cellular smears show large pleomorphic cells with large atypical nuclei and single dispersed atypical cells. Benign epithelial material is also present."
          • "Comment: The features are those of carcinoma of the breast."
        • Malignant (see comment)
          • "These moderately cellular smears show small tissue fragments and dispersed large atypical cells in a background of focal necrosis. Calcifications are seen."
          • "Comment: The features are those of carcinoma with features suggestive of a high grade intraductal component. Core needle biopsy is recommended to confirm that invasive carcinoma is present."
      • Invasive lobular carcinoma (direct quote from Field: The International Academy of Cytology Yokohama System for Reporting Breast Fine Needle Aspiration Biopsy Cytopathology, 1st Edition, 2020)
        • Malignant (see comment)
          • "These mildly cellular smears show dispersed small epithelial cells with small to intermediate sized atypical nuclei, eccentric cytoplasm and occasional vacuoles, some of which contain mucin droplets."
          • "Comment: The features are those of carcinoma of the breast and suggest lobular carcinoma."
      • Mucinous carcinoma (direct quote from Field: The International Academy of Cytology Yokohama System for Reporting Breast Fine Needle Aspiration Biopsy Cytopathology, 1st Edition, 2020)
        • Malignant (see comment)
          • "These moderately cellular smears show moderately atypical epithelial cells in a background of abundant fibrillary mucin."
          • "Comment: The features are those of mucinous carcinoma."
      • High grade phyllodes (direct quote from Field: The International Academy of Cytology Yokohama System for Reporting Breast Fine Needle Aspiration Biopsy Cytopathology, 1st Edition, 2020)
        • Malignant (see comment)
          • "These highly cellular smears show large, hypercellular stromal fragments and marked nuclear atypia of dispersed plump spindle cells with focal necrosis."
          • "Comment: The features suggest a high grade phyllodes tumor but a high grade spindle sarcoma or metaplastic spindle cell carcinoma should also be considered. Core needle biopsy is recommended."
      Differential diagnosis
      • Invasive carcinoma of no special type (ductal carcinoma):
      • Tubular carcinoma:
        • Fibroadenoma:
          • Tubules in fibroadenoma lack a rigid architecture
          • Myoepithelial cells and bare bipolar nuclei
        • Cribriform carcinoma:
          • Can be admixed with tubular carcinoma
        • Sclerosing adenosis:
          • Small, cohesive ductular tissue fragments
          • Usually has myoepithelial cells
          • Some bare bipolar nuclei
      • Carcinoma with medullary features:
        • High grade lymphoma:
          • Almost completely dispersed cells
          • Cells are smaller than cells of carcinoma with medullary features
          • Lymphoglandular bodies
        • Carcinoma metastatic to an intramammary or axillary tail lymph node:
          • Must correlate with imaging
        • Lymphoepithelioma-like carcinoma:
          • Rare
      • Mucinous carcinoma:
      • Invasive papillary carcinoma:
        • Intraductal papilloma:
          • Epithelium is hyperplastic and has bland nuclear features
          • Fewer dissociated columnar cells
      • Invasive micropapillary carcinoma:
      • Metaplastic carcinoma:
      • High grade phyllodes:
      • Secretory carcinoma:
      • Glycogen rich clear cell carcinoma:
      • Adenoid cystic carcinoma:
      • Apocrine carcinoma:
        • Apocrine metaplasia:
          • Retains an ordered arrangement of nuclei and low N:C ratio
          • Nuclei have minimal pleomorphism and smaller nucleoli
      • Carcinoma with neuroendocrine features:
        • Metastatic neuroendocrine tumors / carcinomas:
          • Immunostains and correlation with patient history
      • Angiosarcoma:
        • Granulation tissue:
          • May have similar low power features but without nuclear atypia
        • Spindle cell lesions
      Board review style question #1

      Which of the following is a malignant feature demonstrated in this breast fine needle aspiration (FNA) smear?

      1. Bare bipolar nuclei
      2. Centrally located nucleus
      3. Discohesion and single cells
      4. Monomorphic nuclei
      5. Salt and pepper chromatin
      Board review style answer #1
      C. This image shows marked discohesion and single cells, which is a feature of malignancy in breast carcinoma. A few cells with intracytoplasmic lumens are also present. Answers A and B are not features of malignancy. D and E are not demonstrated in this image.

      Comment Here

      Reference: Yokohama system, malignant
      Board review style question #2
      A breast fine needle aspiration biopsy (FNAB) smear shows many uniform single cells, moderately enlarged nuclei, eccentric cytoplasm and scattered intracytoplasmic vacuoles with targetoid mucin droplets. Which subtype of carcinoma is most likely?

      1. Lobular carcinoma
      2. Metaplastic carcinoma
      3. Mucinous carcinoma
      4. Neuroendocrine tumor
      5. Secretory carcinoma
      Board review style answer #2
      A. This description best fits lobular carcinoma

      Comment Here

      Reference: Yokohama system, malignant
      Back to top
      Recent Breast Pathology books

      Bhargava: 2015

      Cimino-Mathews: 2024

      Dabbs: 2016

      Damjanov: 2017

      Field: 2020

      Hoda: 2017

      Hoda: 2020

      Husain: 2021

      IARC: 2019

      Jiang: 2023

      Lester: 2021

      Mody: 2022

      Schnitt: 2017



      Find related Pathology books: breast, cytopathology, gynecologic, pediatric
      Image 01 Image 02